con law

Pataasin ang iyong marka sa homework at exams ngayon gamit ang Quizwiz!

Standing to Sue requires...

injury, causation, and redressability.

Expressive Conduct

lot of conduct may be symbolic of some expression. What to do about that? Such laws upheld if regulation furthers an important interest. Laws regulating expressive conduct are upheld if: 1. They further an important interest; 2. That interest is unrelated to the suppression of expression; and 3. The burden on expression is no greater than necessary. Key is whether govts interest is unrelated to suppression of expression, in which case law will likely be upheld or whether govts law is actually aimed at suppressing speech in which case law will be struck down.

Even though federal powers are superior...

most federal powers are concurrent with those of the states. (On most topics, both Congress and the states have regulatory powers. If there is a conflict, Congress wins.) concurrent means most things congress can regulate can also be regulated by states. If conflict congress wins, but if no conflict, both state and national govt have concurrent power over most matters.

Accommodation of religious belief...

not constitutionally required but in one context it is. Exception: "Ministerial exception"—in 2012, the Supreme Court held that the First Amendment requires a ministerial exception to employment laws.

Redressability

o A court can remedy or redress the injury. o If the injury is in the past, the remedy is damages. o If future injury is threatened, the remedy is an injunction to prevent future harm. o Past injury does not necessarily give automatic standing to seek an injunction for future injury. To get an injunction, you must show that it will happen again.

Causation

o A defendant's conduct has caused or will cause the injury.

Delegation of Powers

o Congress can delegate its power to administrative agencies, so long as there are intelligible standards governing the exercise of that delegated power. o Not a demanding test—almost all delegations of legislative powers are upheld.

The Commerce Power

o Almost anything can be regulated as interstate commerce. o Congress can regulate: § The channels of interstate commerce (highways, seaways, airways, etc.); § The instrumentalities of interstate commerce (cars, trucks, railroads, ships, etc.); and § Intrastate (occurring within one state) (and interstate) activity (economic or commercial) that has a substantial effect on interstate commerce. • Substantial effect is judged in the aggregate. The question is not whether ur activity in isolation would have a substantial effect on interstate commerce, but whether the aggregate activity of everyone doing the same thing has a substantial effect on interstate commerce, and the answer is almost always yes. Posing question in aggregate means answer is almost yes. Almost impossible to think of economic activity that wouldn't have substantial effect on interstate commerce If all of us did it at once. But for non economic activity or non commercial activity, substantial effect on interstate commerce must be proved. • Not within the Commerce Clause: For non-economic, non-commercial activity, Congress can regulate intrastate actions only by actually demonstrating a substantial effect on interstate commerce (substantial effect is presumed for economic or commercial activity). Thus, Congress cannot regulate purely intrastate, non-commercial, non-economic activity for which a substantial effect on interstate commerce has not been shown. Example 9: Wickard v. Filburn (wheat growing case where growing crops for sale was judged in the aggregate and a substantial effect was demonstrated) - congress said it was illegal to grow wheat beyond certain amount - farmers said no im not doing interstate commerce, I don't sell the wheat. We just grow wheat, and harvest and anything done is done right here - we don't send it anywhere. How is it interstate commerce? Sc said no matter, substantial effect is determined in the aggregate - all farmers in America growing wheat will affect price of wheat nationwide and that's good enough for us. Congress can regulate production of agricultural products. Example 10: Gonzales v. Raich (where the cultivation and medical use of marijuana was judged in the aggregate and a substantial effect was demonstrated). Question about congress having power to regulate prodcutin of marijuana nationwide and congress said yes effect of sale of marijuana would have economic effect so congress can regulate it. Example 11: United States v. Lopez (a statute prohibited possession of a firearm within 1000 feet of a school—it could not be proved that it was an economic or commercial activity, and it could not be demonstrated that there was a substantial effect on interstate commerce). This is non economic activity - sc said no one showed that possessing firearm within 1000 feet of school has substantial effect on interstate commerce. Example 12: United States v. Morrison (a statute provided a federal remedy for gender-motivated violence—it could not be shown that, in the aggregate, the activity had a substantial effect on interstate commerce) - sc said gender violence is not commercial or economic activity - its not buying or selling stuff so gender violence may have substantial effect insome areas but not sufficiently proved. Example 13: 2012 Affordable Health Care case (the individual mandate required individuals to buy health insurance or to pay a penalty for not doing so—the Court said that forcing individuals who are not engaged in commercial or economic activities to buy health insurance they do not want cannot be sustained as a regulation of interstate commerce.) - key question concerned so called individual mandate - what mandate does is forces individuals to buy health insurance or pay penalty if failing to do so. Sc said that forcing individuals not engaged in commercial activities to buy health insurance they didn't want couldn't be sustained as regulation of interstate commerce. If lines between cases seem thin and not really clear, that's correct. Most imp thing is to remember way to think of it - for economic and commercial activity, substantial effect on interstate commerce is easy, presumed, and always found. For noneconomic and noncommercial activity, must actually demonstrate substantial effect on interstate commerce. sometimes can be done and sometimes not.

religious conduct

o Protected qualifiedly § Laws regulating religious conduct because of its religious significance are unconstitutional (i.e., laws aimed at religion). § Neutral regulation of conduct: Neutral, generally applicable laws (laws not aimed at religion but simply regulating conduct) must be obeyed despite religious objections. Ex: must pay taxes even if u believe in taxes or not. No constitutional right to demand government accommodate ur religious belief. • No right to accommodation. (accommodation means what? Accommodations means you are demanding that the government adjust the laws to accommodate ur religious belief - that u have right not to obey a law everyone else would have to follow because it offends your religious scruples - there is no constitutional right of accommodation). • No constitutional right to exemption from neutral, generally applicable regulations of conduct (e.g., Peyote case). Peyote case - state had statute against consuming hallucinogens. Peyote is a hallucinogen. Statute can be enforced across board as an anti drug law and can even be enforced against native americans claiming they have religious motivation for peyote use. Neutral, generally applicable laws aimed at conduct can be enforced against everyone on the same terms. Example 33: Religious Freedom Restoration Act (RFRA) overruled the Peyote decision: Even neutral, generally applicable laws could not be enforced against religious objectors unless the government had a compelling interest. Insofar as RFRA applied to state and local governments, the Supreme Court struck it down as beyond federal legislative authority (Congress cannot force states to accommodate religious beliefs). RFRA would not be beyond federal legislative authority insofar as it applied to the federal government's own actions. Can states and localities require a religious accommodation for themselves? Yes. Rfra went much farther than protecting against discrimination - it required accommodatin of religious beliefs and scfound that was beyond fed legislative power insofar as it addressed states and localities - insofar as rfra addresses fed entitites, its still good law. That's because fed govt has undoubted power over army navy and district of Colombia and its own employment practices ,etc. so wherever fed govt has legislative power, like army navy and district of Colombia, it can require accommodation of religious belief and states can require accommodation of religious beleif by states and localities and many of them have done so. Accommodation of religious belief not constitutionally required but in one context it is. • Exception: "Ministerial exception"—in 2012, the Supreme Court held that the First Amendment requires a ministerial exception to employment laws. o Non-discrimination employment laws cannot be applied to ministers. Constitutional right not to have nondiscrimination laws apply to ministers. o Plaintiff was a commissioned minister working as a teacher in a religious school. The school fired her in a plain violation of the Americans with Disabilities Act. She sought reinstatement as provided by the statute. o The Supreme Court held unanimously that the First Amendment precludes litigation of claims concerning the employment relationship between a religious institution and its ministers. Note term minister not limited to people who really are ministers - it was construed broadly to include a religious school teacher, most of whos duties were secular. So at least in context of employment laws, ministerial exception that is that employment laws cant be applied to ministers, ministerial exception does require that religious believers be treated special. o The federal government cannot regulate employment relations between a religious institution and its ministers. o The term "minister" was construed broadly. § Campus Access: A state university that allows student groups to meet on campus must allow student religious groups equal access.

Broadcasters

o The only special case is broadcasters. o Traditionally, because of early limits on the broadcast spectrum, the government had greater regulatory authority over broadcasters than over print media or other unlimited technologies like the Internet. Frankly not clear just how much govt can regulate broadcast but it is clear they have some additional authority to regulate broadcast because spectrum is limited and that one fact is probably worth noting.

Article III lists

Article III lists the kinds of cases that come within the judicial power of the United States— the most important are diversity jurisdiction (i.e., disputes between citizens of different states) and federal question jurisdiction (i.e., cases arising under federal law).

Delegation of Powers

Congress can delegate its power to administrative agencies, so long as there are intelligible standards governing the exercise of that delegated power. - Not a demanding test—almost all delegations of legislative powers are upheld.

Regulation of the Time, Place, or Manner of Expression (Content-neutral)

Content neutral regulations are called time place or manner. Its not content that's being regulated but time, place, or manner. You absolutely can count on being tested on this rule. • Apply principally in a public forum. • A public forum is a place traditionally reserved for speech activities. These places include streets, parks, and public sidewalks around public buildings (but not airports). • Only time, place, and manner may be regulated in a public forum. To have a viable or constitutional time place or manner regulations, there are three requirements (must absolutely memorize this, most important thing in con law): 1) Content neutral: Must be content neutral on its face and neutral as applied. If a facially neutral law is applied to favor one speaker or one point of view or one political party its not content neutral. Content neutral also means law must not allow executive discretion - if an executive officer has discretion, if exec officer has power to pick and choose who can speak risk is that the discretion will be administered in a non-neutral way. The supreme court has tried to guard against that possibility by saying that a law must be content neutral on its face, it must be content neutral as it has been applied, and it must not allow executive discretion which would threaten content neutrality. Example 37: A local D.C. ordinance prohibited picketing within 500 feet of a foreign embassy (but it didn't just stop there - it didn't say all picketing within 500 feet of a forein embassy, it said only if...) if the picketing sign brought the foreign government into odium or disrepute (that's not time place or manner regulation, that's regulation based on content - whether signs were hostile to foreign govt.). This is plainly content-based and is thus, unconstitutional. Example 38: Parade permit laws (getting permit before parading on street or using public property) giving permits to first come, first served (like first group to apply for a permit gets one) are constitutional because content neutral. Parade permit laws giving the chief of police discretion to decide whether it was in public interest to grant someones permit are unconstitutional because the law may not be applied in a content-neutral way - if chief of police has discretion to pick and choose among permit applicants, the risk is that he will grant the applicaiotns for those he agrees with. And deny permits to those he finds reprehensible. That risk of content based administration makes the law unconstitutional. 2) Alternative channels of communication must be left open: Time, place, or manner law must be a guideline for speech, not a flat prohibition of speech. Example 39: Laws against amplified sound trucks during the night-time hours are fine - its valid, content neutral, and leaves lot of other times and places where u can sound. Laws against amplified sound during all times and all places would be struck down. Whats the difference? One is a guideline for speech leaving alternative channels open, other is a prohibition of speech attempting to shut down a certain method of speaking. 3) Must narrowly serve a significant state interest: Under this test, most content-neutral time, place, or manner regulations are upheld. • Does not require a compelling interest - Note absence of buzz word compelling. Content neutral regulation of time place and manner doesn't require a compelling state interest. Doesn't require a national emergency we cant live without it kind of justification. It requires a signficiant state interest which is basically anything that makes sense to a judge and for that reason most content neutral regulations of time place and manner are upheld - not all by nay menas - but most content neutral regulations of time place and manner are upheld - Theyre extremely common in American cities and theyre routinely enforced. • Nonpublic forum o This includes all kinds of government property that is not a public forum and not traiditonally open to speech activities (e.g., government offices, jails, power plants, military bases, etc. - not typically places where you can go and parade and protest and speak). Here, the government has great power over what u do in nonpublic forums. Basically, any reasonable regulation of speech will be upheld. Some things that are not reasonable - it would not be reasonable to favor republicans over democrats, etc. viewpoint discrimination is invalid - but absent that kind of viewpoint discrimination which u rarely see, govt can regulate speech activities in a nonpublic forum almost any way it likes. You cant go to govt office building and march up and down inside corridors shouting slogans - that would disrupt govt functions. but where can u go? You can go to public sidewalk outside govt building as long as u obey neutral time place and manner regulations and protest to your hearts content. That's difference between a public forum like streets, sidewalks and public parks, and other govt property such as an office building. § Viewpoint discrimination is invalid: One clearly unreasonable kind of regulation would be to discriminate based on viewpoint (e.g., between members of different political parties). § Disruption of the functions of government: One should go outside to the public sidewalk surrounding the building since that is a public forum. • Limited public forum - idea is its some place that's not public forum but govt has chosen voluntarily to open to all comers. o Describes a place that is not a traditional public forum, but that the government chooses to open to all comers (e.g., a municipal theater that anyone can rent). Municipal theater - govt doesn't have to build it or have to have one but if it does have one and it rents it out to all kinds of plays, then it can only enforce content neutral regulations on what happens there. Cant disapprove a particular play because of its point of view. Don't get carried away with limited public forum - it's a very limited idea. School auditorium for example is not a limited public forum. Now schools have auditoriums - they open them to various speech activities but only on appropriate topics. So a student who makes a profane or sexually explicit speech in a school assembly absolutely may be disciplined because the speech was inppropriate in the school auditotorium. Student could have gone outside to the public sidewalk, down the street to the public park, or student could have gone to public forum and said exactly the same profane and sexually explicit things and been home free. Difference is a public forum vs other property. o In such areas, only time, place, or manner regulations are allowed, but this is a narrow category. Example 40: A school auditorium is not a public forum, limited or otherwise. Just because a school auditorium is open to some speech does not mean it has to be open to all.

fundamental rights

D. Fundamental Rights Some fundamental rights almost always come up under equal protection. Fundamental rights trigger same strict scrutiny test under both due process and equal protection. General rule of thumb - law denying fundamental right to everyone violates due process. law denying fundamental right to some people and not to others violates equal protection but some fundamental rights arise in the context of equal protection, including: 1. Right to travel 2. Right to vote—one person, one vote o Requires districts of approximately equal size, i.e., approximately the same number of voters in each o Applies whenever you elect representatives by district. If one district had 100k voters and one had only 50k voters, every voter in second district would have twice the impact of voter in first district. So districts ofequal size or population is how to achieve one person, one vote. That rules applies whenever u elect representatives by district including us house of reps - within each state each congressiaonal district should be approximately equal size. § Examples include: U.S. House of Representatives; both houses of a state legislature (even if one of them called state senate still must have senatorial district of approximately equal size); local governments when they elect representatives by district. • Very narrow Exception: Special purpose governments—A highly specialized government (e.g., for distribution of water rights) can have a franchise based on that special purpose (e.g., acreage or water entitlements). If highly specialized government` with narrow purpose, franchise can be restricted to that narrow purpose. 3. Gerrymandering - Comes in two varieties—racial and political - gerrymandering - districts of equal size with same number of people in each district but arrange them to achieve advantage for one side rather than the other. Can be different types of gerrymandering: a. Racial gerrymandering - contemplated and often required by the voting rights act. § Vote Dilution: Drawing districts to scatter minorities all around so that they are not a critical mass in any one district. If done with a discriminatory purpose, it's unconstitutional. § Voting Rights Act: Requires racial gerrymandering to ensure minority success by creating majority-minority districts. Gerrymandering in favor of minorities. More or less required by voting rights act. How to do it? Bring minorities together in a majority minority district, collect racial or ethnic minorities in a majority minority district and that's supposed to ensure minority political success. • Rule: Race may be a factor in drawing district lines so as to advantage racial minorities as voting rights act wishes to do - race may be a factor in drawing district lines, but not the predominant or only factor. • Other factors include compactness and observing/respecting local, political subdivisions. • A bizarrely shaped district which wheels goods in and out with long fingers and stretches all over state - very bizarrely shaped district will be taken as evidence that race was the predominant purpose, predominant racial purpose. But really no easy way to tell. b. Political gerrymandering (drawing districts to hurt one party) - occurs if one party controls the legislature and can screw the other party. Gerrymandering is very important in California and other states where u can draw districts with aide of computers that really advantages one political party at expense of the other. § political gerrymandering Can, in theory, violate equal protection. In practice, it is never struck down. If asked whether constitutional problem with political gerrymandering, or asked whats strongest argument that political gerrymandering is unconstitutional, equal protection is answer. In theory political gerrymandering can violate equal protection but in practice it never does because the Supreme Court has not found any judicially manageable standards for implementing that guarantee. § A political question (non-justiciable)

Obscenity

Defined by the rule of "S" - most imp category of speech that can be regulated because of govts interest. o obscenity must be Sexy: Must be erotic; must appeal to the prurient interest. Must make you itch or long with desire. (Gore and violence are not legally obscene. May be disgusting but don't make you itch or long, theyre not sexy.) o obscenity also must make Society sick: Must be patently offensive to the average person in the society, so not to most refined and delicate amongst us but to the average person. The society may be the nation as a whole, or can be a particular state, or a major metropolitan area. But wherever you are, material must simultaneously make you itch or long and be sick. o obscenity must be defined by the proper Standards - if a law doesn't have right standards it will be invalid as vague and overbroad: Must be defined by the proper standards for determining what is obscene, not vague and/or overbroad. For example, full frontal nudity. Full frontal nudity might be obscene but some of its not the least bit sexy. So law banning all nudity is overbroad - goes too far and is unconstitutional for that reason. Example 41: Tax on films displaying frontal nudity. Not a valid law because frontal nudity is not a good enough standard. o obscenity must lack Serious value: The material must lack serious value. If material has serious value (artistic, scientific, educational, or political), it cannot be held legally obscene. This determination is made by the court, not the jury, and it must be based on a national standard, not a local one. Juries cant speculate on whether something has serious value. Example - film which was mainstream Hollywood one - maybe because of name or title of film "carnal knowledge" it was prosecuted against as being obscene in the state of Georgia. Supreme court said the film is not obscene, it was a mainstream Hollywood film, got great reviews, and it has artistic value so it has serious value. So good reviews probably means no obscenity. 4 s's - sexy, making society sick, must be defined by proper standards, lacking serious value - that defines law of obscenity but there are a few additional points to know below: o Footnotes on obscenity: § Minors: A lesser legal standard can be applied to minors, but the government cannot ban adult speech simply because it would be inappropriate for minors. A lesser legal standard can be applied to minors but only if the minors can be dealt with separately - issue cant restrict materials to everyone because it might be inappropriate to minors. But you can have special standards for materials distributed to minors. § Child pornography: Can be prohibited whether or not it is legally obscene, and possession can be punished even if it is in the privacy of your own home. Child porn is special case - all bets off for child porn. Child porn is sexual representation of children - it can be prohibited whether or not it is legally obscene - child porn can be prohibited even if it does have some artistic value. It can be prohibited whether or not it is legally obscene and even if u possess it in the privacy of your own home. Even in the privacy of your own home, child porn can be banned. § Land use restrictions: Narrowly drawn ordinances can regulate the zoning of adult theaters, but cannot ban them entirely. But they can regulate where they are placed. Noones quite sure whether its better to scatter them all around so no one neighborhood is overrun or whether its better to bring all the adult theaters into one neighborhood and just close it off. but whatever it is, u can zone adult theaters so long as u allow some place for them to exist. Note 6: Courts have recently begun to distinguish legally obscene speech from pornography. Merely establishing that speech constitutes pornography is generally insufficient to establish that the speech is obscene. Second category of speech that can be punished because of its content is incitement.

example of congruence and proportionality

Example 17: The Religious Freedom Restoration Act (RFRA) Overview: Neutral, generally applicable regulations of conduct can validly be applied despite religious objections. Everyone can be made to obey the law whether they believe in it or not. Background: Religious individuals often claimed a right of accommodation of religious belief. They claimed that laws valid against everyone else could not be enforced against religious objectors unless the government showed a compelling interest. Called accommodation of religious belief because claim is that govt is required to accommodate my religious belief and treat me specially because of that belief. That claim was rejected by the Supreme Court, which held that neutral regulations of conduct could be enforced against anyone, including religious objectors. Congress disagreed with the Supreme Court and passed RFRA, which declared that religious believers have the right not to obey otherwise valid laws, unless the government can show a compelling interest in enforcing the law against them. The Supreme Court struck down RFRA, as it went beyond federal legislative power, because it was not really remedial. It was not narrowly aimed at the kinds of laws the Supreme Court had declared unconstitutional—laws aimed at religion. Congress could have made it a crime for the government to discriminate against religious belief. Congress could have said that any government or government official who discriminated against religious belief had to pay money damages. Congress could give the Department of Justice special enforcement powers to look into religious discrimination and make sure it is rooted out. All of these actions would have been reasonable remedies against a recognized constitutional violation of discrimination against religious belief. In RFRA, Congress said something different - it said even non-discriminatory laws, even laws showing no history or trace of anti religious motivation but that incidentally restricted religiously motivated conduct, were invalid unless the government showed a compelling interest. That, said the Supreme Court, was not an attempt to enforce the Free Exercise right against religious discrimination as it had been defined by the court. Rather, it was an attempt to overturn the Court's decision about what the Constitution did and did not protect. Therefore, RFRA was struck down as beyond the federal legislative power. Note very important point: RFRA was unconstitutional only as it applied to states and localities. Requiring states and localities to accommodate religious belief was found to be beyond Congress's power under the 14th Amendment. However, Congress is free to accommodate religious belief in the exercise of its undoubted federal powers (e.g., Congress's powers over the Army and the Navy, the District of Columbia, etc.). When Congress exercised its federal powers, it may choose to accommodate religious believers, but Congress has no power to force that policy on the states. Difference between remedy reasonably related to judicially recognized right and statute which goes beyond that. Distinction between legislation that remedies violations of individual rights as rights have been defined by the courts - that's okay. And legislation seeking to define individual rights in ways not embraced by the courts - which is not okay. That's only hard topic in entire subject of legislative powers. For questions on this topic most likely something that looks like RFRA. Remember these characteristics of that paradigmatic example - congress tried to reverse a supreme court interpretation of individual rights, congress's attempt to adopt a broader interpretation of individual rights was struck down as it applied to states and localities because congress has no power to legislate religious policy for states and localities but same legislation is sustained for federal govt because of congress's undoubted power over federal matters.

Standing Example

Example 3: Corporation A applies for a business license, but is turned down. Corporation B applies for the same license and gets it. Corporation A sues to enjoin the government from licensing Corporation B. Does Corporation A have standing? No. Denying the license to Corporation B would not redress the wrong to Corporation A. The wrong to Corporation A was a wrongful denial of its license. Corporation A should sue for its own license. Must have standing to seek particular relief sought - if u don't seek that particular relief youll be kicked out of court even if you have standing for some other relief.

Fifteenth Amendment

Fifteenth Amendment (specifically focused on voting) - Congress has the power to ensure there is no racial discrimination in voting.

Standing to Sue requires

Injury, causation, and redressability.

Procedural due process

Procedural Due Process—the Right to Notice and a Hearing 1. Consists of two questions: 1) Is life, liberty, or property being taken? 2) If life, liberty, or property is being taken, what process is due? Some process must be due. 2. Breakdown of loss of life, liberty, and property o Life: Death penalty requires procedural due process o Liberty: Physical confinement, probation and parole, physical injury (such as a spanking in school), any restriction on legal rights (including being punished for free speech). Even someone on parole or probation has limitations on where they can go, etc. so limitations on them so procedural due process is required. Physical injury is also loss of liberty - if school child is spanked he has loss of liberty - not much procedural due process but physical injury is still loss of liberty. Any restriction on ur legal righs is also loss of liberty - if ur punished for exercising free speech, if some adverse action taken against u by govt because u belong to wrong party or u wrote letter to edior or some other exercise of free speech, uve lost liberty. Injury to reputation is not a loss of liberty. Without more, it's not loss of liberty. If prosecutor says were really keeping our eye on you because we think ur engaged in illict activities, that statement may damage ur reputation, conceivably maybe defamation claim, but its not loss of liberty and doesn't trigger procedural due process. o Property Exam Tip 10: Out of the three, property is the most heavily tested. Most of the questions in this area concern government jobs or benefits. Most property is clear - owning house or car, etc. if they take that stuff, they must give u procedural due process. what about govt jobs and benefits? Can u have property right in govt employment or some kind of govt benefit? Answer is yes. § You have a property interest in your government job or benefit whenever you have a legitimate entitlement to continued enjoyment of the job or benefit. § A mere expectation of continued employment or benefit does not suffice. § Most government benefits, such as welfare, Medicaid, social security, are entitlements, and hence property. Statutes say if ur this old or this income level , or meet this criteria ur entitled to this benefit - that's property interest and due process is required if they take it away. Jobs however are different - u may have entitlement to job in which case u ave property interest but mere expectation of continued employment - like ur expectation that u keep working for govt isn't enough. How to tell whether its entitlement to govt job or mere expectation? Entitlement is property, mere expectation is nothing. § How can you tell whether you have an entitlement to a government job or whether you have a mere expectation of government employment? • Government jobs are entitlements only when the government says so—such as by providing a contractual term or discharge only "for cause." Example 25: You are hired as the city planner and the boss says, "This job has a term of five years. You will not be fired during that five-year term except for cause." You have a property right in that job for five years. You have entitlement to job for 5 yr term. But if u go for job interview and boss says this is political appointment - you'll be my special assistant and u serve at will. If they tell you that you can be fired at any time and for any reason, u have no entitlement to hold position, u can be fired any time and for any reason. Example 26: Barnes was an assistant professor at a state community college. He had a three-year teaching contract and under state law, five years was required for tenure. At the end of three years, his contract was not renewed, and he sued. What is his strongest argument that he should get notice and a hearing? A. He was the best teacher in the school. (no ur not entitled to job just because youre good at it) B. He was the only teacher not rehired. (so what? Youre not entitled to job just because your friends have one. If barnes can show though that he wasn't rehired because of race or religion or gender, then he woud have claim. U have constitutional right not to be discriminated against based on race, religion, or gender, but if just that other teachers were rehired and he was not, that doesn't do it.) C. He was so sure of success that he moved his elderly parents to town to care for them. (that's not property, doesn't create entitlement) D. He had an oral promise of re-employment from the president of the college. (maybe not winning argument, but right answer because strongest argument) Answer choice D is the correct answer because the president is acting as the government, and arguably Barnes had an entitlement to continued employment. President, someone in charge, told him hed be reemployed and that would tend to show legitimate entitlement. What is the strongest argument against Barnes receiving notice and a hearing? Answer: He had a three-year contract that ended. School said 5 years required for tenure, you've only been here for 3, so you cant be entitled before you've been here for 5 years. o Deprivation: Notice and a hearing are not required when there is an accident. Random negligence by a state employee does not constitute a deprivation of life, liberty, or property (e.g., death by a municipal garbage truck). Deprivation requires the intentional taking away of life, liberty, or property. What is deprivation? Maybe youre run over by city owned garbage truck - Garbage truck might kill you in which case you lost life, garbage truck might injure you in which case you've lost liberty, garbage truck might just tear up clothes in which case you lost property. Are you entitled to notice and hearing to protect against accidents with garbage truck? Defs not, random negligence by a state employee does not constitute a deprivation under due process. if accidentally injured by govt employee may have a tort claim. But you don't have constitutional violation unless govt intentionally takes life, liberty, or property from you. 3. If life, liberty, or property is being taken by the government, what process is due? o Procedural due process is variable, and the types of hearings can range from casual to very elaborate. Due process can range from nothing so very minimal and casual procedures all the way to the elaborate procedures of criminal trial. procedural due process can mean a little or a lot. o To decide what kind of process is due, the courts balance three factors: 1) The individual interest at stake (life, liberty, property) - what kind of interest will it be? Life, liberty, or property- theyre the only ones that count.; 2) The value of the procedure in protecting that interest - whatever procedure being claimed or whatever person wants to have, u assess individual interest at stake and value of procedure in protecting that interest against the governments interest in efficiency; and 3) The governments interest in efficiency and cost - which usually means cost. Sometimes balance requires highly elaborate procedures as in criminal trials or imposing a fine. Sometimes it allows minimal procedures like disciplining a school child - not much in terms of a hearing required. o Timing of the hearings § Sometimes, a hearing must occur before the deprivation or something taken away. So pre-deprivation hearing. But sometimes evidetinary hearings may be allowed after something taken away so long as they are prompt and fair. Pre deprivation hearings apply to terminating welfare benefits, and to non emergency revocations of drivers licenses. But for many other govt actions, hearings can come after action taken so long as hearing is prompt and fair. • Examples include: Terminating welfare benefits; non-emergency revocations of driver's licenses. § Sometimes, the hearing can occur after the action, so long as the hearing is prompt and fair. Usually okay to act first and give hearing later but hearing must be prompt and fair - cant be hearing down the road several months and cant be hearing not giving u fair shot at prevailing. • Examples of major actions for which post deprivation hearings are allowed so long as they are prompt and fair include: Terminating disability benefits; disciplinary suspension from a public secondary school. Question of pre or post deprivation hearing is resolved by same 3 factor balancing test above -weigh individual interest at stake, and the value of a pre termination hearing - value of having hearing before termination - against govts interest in efficiency. o Public employees who can be fired only "for cause" must be given some opportunity to be heard prior to discharge, unless there is a signficiant reason not to keep the employee on the job. Public employees who can be fired only for cause have tenure - doesn't apply to just university profesors, but to every public employee who has job security. People who have job security typically msut be given some oppornity to heard - so not necessarily full scale trial hearing but some opportunity to be heard prior to discharge. However if signficiant reason not to keep employee on job employee can be immediately suspended without pay. Signficiant reason not to keep employee on job - employee can be immediately suspended without pay, so long as theres a prompt post suspension hearing and so long as there is reinstatement with back pay to make employee whole. If there is a significant reason not to keep a person on the job, then the discharge can come first with a subsequent hearing that is prompt and provides reinstatement with back pay (fair). Example 27: A police officer charged with a crime can be suspended immediately because there is a significant reason not to keep criminally charged officer on the job. The hearing can come afterwards, but it must be prompt and provide reinstatement with back pay if the charges are ultimately unfounded. May be union agreements that limit police department freedom of action but only about constitutional law for purposes of the problem.

main powers of congress

Taxing, spending, and commerce.

Privacy concepts

a. Marriage - There are all sorts of requirements for marriage (e.g., age or restriction on marrying close relatives), but substantial interference with the marriage—including same-sex marriage—is unconstitutional. b. Contraception - It is a fundamental right for everyone, whether married or not, to purchase contraceptives. Everyone has fundamental right to buy contraceptives - true for unmarried and married persons, and old and young people. c. Sexual intimacy - Perhaps not technically a fundamental right—the Supreme Court found that the government has no legitimate interest in regulating non-commercial sexual intimacy between consenting adults, including same-sex couples. Not quite same as sayig it's a fundamental right, but saying even if not fundamental right, govt regulation fails because govt has no legitimate interest in regulating non-commercial sexual intimacy between consenting adults and that includes same sex couples. d. Abortion § Roe v. Wade: A woman has a right to terminate her pregnancy until viability of the fetus. Viability means the fetus can live outside the womb. After that stage, restrictions can apply so long as there are exceptions to preserve the life and health of the mother. § States can regulate abortion in a variety of reasonable ways, but they cannot impose an undue burden on the woman's right to terminate her pregnancy. • Informed-consent requirements are allowed. That is you can require before person has abortion, that person be informed about opportunities available for carrying child to term, and having it adopted. Long lines of people who wish to adopt babies, etc. you can require certain info to be communicated, the informed consent requirement is allowed. • Twenty-four hour waiting periods are allowed - we don't want abortions to be performed upon spur of moment so can have a system requiring woman sign consent form and wait 24 hours for procedure incase she changes her mind. • Parental notification requirements (for minors) are allowed. Obviously not for adult females, but for minors. • Parental consent requirements are generally not allowed. Govt can require underaged minor to notify her parent of intention to have abortion - maybe gives parent opportunity to voice an opinion. But parent cant be given veto over girls decision to have abortion. Parents consent cannot be required. o Narrow exception for requirement that an underage female get the consent of a parent or a judge, but requires the judge to give consent if the underage female understands the nature of the act. Very elaborate way of allowing judicial override of parental veto. So Parental notification can be required, but parental consent cannot be required - must be way around a parental no. • Spousal consent requirements are not allowed. No law requiring consent of spouse has been upheld. • Government financing of abortion is not required. Govt never has to pay for abortions, even if it provides funding for live births. Govt can refuse to pay for abortions and can refuse to allow abortions to be performed in facilities it funds. Govt has to allow abortion but doesn't have to pay for it. e. Parental rights § Parents have a fundamental right to raise their children as they see fit, including the choice of religious or private schools. § Can lose their rights through abandonment, abuse, or neglect - until parents' rights are severed because of some kind of misconduct, parents have a fundamental right to raise kids as they see fit. f. Family relations - fundamental right to live with close relatives if they want you to. § Includes the right to live together with close relatives City adopted an extremely narrow single family zoning ordinance - nothing odd about having zoning ordinance limiting habitation to single family - most places have law like that. But city adopted a single family zoning ordinance so narrow that if you wanted to take ur nephew into household, u had to kick your grandmother out - they both couldnt fit in single family. Supreme court said that violates due process, u have a right to live with close relatives if they want you to. g. Obscene material § Fundamental right to read obscene material in the privacy of one's own home. § However, no fundamental right to purchase, sell, import, or distribute such material. § Right to possess obscenity in ones home does not apply to child porn. For child porn completely banned. h. Refusal of medical treatment § Not clear whether this is a fundamental right, but there is a liberty interest in refusing medical treatment § No fundamental right to commit suicide

supremacy clause

federal statute governs by virtue of the Supremacy Clause.

Exceptions to no discrimination against out-of-state interests:

• State as Market Participant—When a state is buying or selling goods or services on its own or for itself, it can choose to deal with only in-state persons. o Examples include: Sale of cement produced by state-owned plant only to instate purchasers; garbage stored in state-owned landfill limited to in-state garbage; law requiring 50% local workforce on state-financed construction projects. For example, if state owned cement plant it can sell only to in state persons. If state owned landfill it can refuse to accept garbage from elsewhere. If state financed construction project it can require certain percentage of workforce be local. When state acting as a market participant, it can deal only with its own. It's the state as a government that's barred from discrimination. • Subsidies—A state can always choose to subsidize only its own citizens (for example, welfare benefits or in-state college tuition). Doesn't have to provide same benefits to citizens from other states. Can restrict welfare benefits, for example, to in-state citizens. same with in state tuition costs. Why is it fine? when u attend public university ur not paying all cost of education - ur education is being subsidized by state appropriation. And when state is spending money and subsidizing activity, it can choose to subsidize only its own. • Federal Approval—Remember, the Dormant Commerce Clause applies only in the absence of federal action. If Congress authorizes or consents to state regulation of commerce, nothing the state does will violate the Commerce Clause, even if it discriminates against out-of-state interests. Remember - the entire section about no discrimination against out of state interests applies in the absence of federal action. When congress has not legislated on point - but if congress authorizes or consents to state regulation of commerce, nothing the state does will violate the commerce clause - even if it discriminates against out of state interests. Common question - congress passed statute, statute authorizes states to regulate particular industry like insurance any way they like, a state then turns around and taxes out of state insurance providers at a higher rate than in state competitors - that's discrimination, it would be struck down in a heartbeat if state did it on its own but when congress authorizes or consents to it, then its okay under the commerce clause. Exam Tip 9: Commonly tested on the MBE b. No undue burdens on interstate commerce - non discriminatory regulation of commerce simply means in state interests and out of state interests are treated alike - that's non discriminatory. § Non-discriminatory state regulation of commerce is almost always upheld. § Only when it is so outrageously costly relative to the benefits of the regulation is a non-discriminatory state regulation struck down as an undue burden on interstate commerce. § This is a balancing test, but non-discriminatory regulations are rarely struck down. c. No regulation of wholly extraterritorial activity § A state may not regulate conduct occurring wholly beyond its borders. Example 21: Connecticut tried to legislate that beer sold in Connecticut could not be sold at a price different from the price of beer sold in other states. Connecticut can legislate price of beer in its own state, but it cant make the lawfulness of beer sold in its state depend on the prices in new York. Supreme court said that's attempt to pass extraterritorial regulation.

campaign finance

• The use of money to support a political campaign is political speech and the regulation of campaign finance raises First Amendment issues. as regulations have become more elaborate constitutional restictions on those regulations have also become more complicated and to some extent unstable. Key points - distinguish campaign contributions from campaign expenditures. • Contributions versus expenditures: o Contributions to political campaigns and candidates can be regulated, provided that the limits are not unreasonably low. o Direct expenditures in support of a candidate, a campaign, or a political issue cannot be regulated. They can be made regardless of amount. § The rationale is to prevent corruption: a candidate could theoretically act in a certain way in exchange for a larger, direct contribution. Diff between contribution and expenditure? Different is in their relation to corruption. Large contributions where u give money to a campaign or candidate may create the reality or at least appearance of corruption as candidate would presumably be willing to do things to get the contributions but direct expenditure of ones own money doesn't pose same risk of corruption. Now lots of people who doubt whether the distinction between contributions and expenditures makes much sense on the ground - but its deeply embedded in constitutional doctrine. Campaign contributions can be limited, campaign expenditures cannot be. If you distinguish between contributions and expenditures, must also distinguish between independent expenditures and coordinated expenditures. • Independent expenditures (person spends her own money to support a political candidate maybe by buying tv ads, that's fine) versus coordinated expenditures (a disguised campaign contribution - when a supporter spends money as the campaign directs, for example campaign says pls pay bills for tv ads and they do it. May be structured to look like an expenditure by you but its actually a contribution): So expenditures coordinated with or controlled by the campaign - these are treated as contributions and can be regulated as all other contributions can be regulated. Expenditures that are coordinated with or controlled by camapgin - they tell u how to spend ur money to support them. Coordinated expenditurs are treated as contributins and can be regulated as all contributins can be. o Independent expenditures cannot be regulated. o A coordinated expenditure is a disguised contribution (the campaign is in control) and be regulated as contributions can be regulated. Law banning judicial candidates from personally soliciting campaign contributions - in many states, judges are elected and there was statute prohibiting candidates for judgeships from personally soliciting campaign contributions - the law was upheld based on the appearance of corruption. Citizens united case - The constitutional protection of direct independent expenditures (that is you can make direct expenditures to support your point of view or the people who agree with you in amounts that are unlimited) applies not only to individuals, but also to corporations, including nonprofits, and unions. Citizens united case involved ideological group organized as a corporation that promoted mocumentary against a presidential candidate at the time - overtly political debate and court said it was constitutionally protected. • The Supreme Court has consistently rejected equalization of campaign resources as a valid rationale for restricting campaign expenditures. Most of people who want to put limits on campaign expenditures have as their essential reason - equalization - preventing person with lot of money from drowning out those who have less. Court has considered that and time and time again, court has rejected equalization of campaign resources as a rationale for restricting campaign expenditures. Idea that you want to restrict wealthy speakers in order to increase relative voice of others - that's consistently been a losing argument. Can see that in the corporate free speech case and it temps to give advantages to poorer candidates. Example 45: Congress passed the Millionaires' Amendment, which basically said that if your opponent spends a lot of his or her own money, then your contribution limit is automatically raised. So telling rich people if you spend your own money above a certain amount, then the rules for your opponent are changed. If you spend your own money above a certain figure, then contribution limits that are normally applicable don't apply to your opponent. That's restricting how much you can spend or making something depend on how much you spend in an effort to equalize the voice of the rich and poor. The Supreme Court struck it down. Court also struck down another scheme that said that public candidates who accept public financing can get extra money if their opponents outspend them. Point is equalization is a rationale that will appeal to many in society. But under case law, equalization is not a sufficient reason to restrict campaign expenditures. What is a sufficient reason? Corruption or the appearance of corruption. Equalization is not.

Redressability

1. A court can remedy or redress the injury. 2. If the injury is in the past, the remedy is damages. 3. If future injury is threatened, the remedy is an injunction to prevent future harm. 4. Past injury does not necessarily give automatic standing to seek an injunction for future injury. To get an injunction, you must show that it will happen again.

non-suspect classifications

1. Age discrimination in employment is barred by statute, but it is not a suspect or quasi-suspect classification under the Equal Protection Clause. Constitutional test for age requirements is rational basis. o Triggers rational basis 2. Wealth is not a suspect or quasi-suspect classification, - Govt can and often does charge a price or sets price for particular license or something and usually paying price is much harder for poor people than people who are well off. govt can generally charge a price. But what happens if person is indigent and just doesn't have means and cant pay? When does govt have to waive fees for indigent? the government has to waive filing fees for indigents when charging the fees would deny a fundamental right - divorce, transcript for appeal from criminal conviction, transcript for appeal for the termination of parental rights.

Establishment of Religion

1. Three-Part Test of Lemon v. Kurtzman - test has never been formally overruled: o Does the law have a secular purpose - non religious purpose? o Does the law have a primary effect that neither advances nor inhibits religion? o Does the law avoid excessive government entanglement with religion?

Injury

1. Almost anything can be injury, especially if Congress says so (either past or future). 2. Must be concrete (not abstract), particular in some sense, but need not be economic. - If your freedom of movement or enjoyment of public space is impaired, that constitutes injury. Even if no economic consequence to you at all. 3. Mere ideological objection is not injury. Cannot sue to stop govt from doing something because you don't like it. Must harm you in some concrete way. 4. An organization has standing if its members have standing.

Free Exercise of Religion

1. Religious Belief Protected absolutely (entitled to hold any belief or none at all) - absolutely entitled to have any religious belief you like or none at all. Public office cannot be limited to those with particular religious beliefs or those with religious belief of any sort. People who have religious belief, including clericks, cannot be denied public office. Simply nothing govt can do to restrict ur religious belief. Religiously motivated conduct though is a different matter. Of course you are protected against laws that prohibit religious conduct because its religious. Youre protected from laws aimed at religion. If law prohibits religious conduct because conduct is religious or prohibits religious conduct in order to suppress religion, then of course its unconstitutional but that's not something that ever really comes up. Whats not constitutionally required is accommodation of religious belief. 2. Religious Conduct o Protected qualifiedly § Laws regulating religious conduct because of its religious significance are unconstitutional (i.e., laws aimed at religion). § Neutral regulation of conduct: Neutral, generally applicable laws (laws not aimed at religion but simply regulating conduct) must be obeyed despite religious objections. Ex: must pay taxes even if u believe in taxes or not. No constitutional right to demand government accommodate ur religious belief. • No right to accommodation. (accommodation means what? Accommodations means you are demanding that the government adjust the laws to accommodate ur religious belief - that u have right not to obey a law everyone else would have to follow because it offends your religious scruples - there is no constitutional right of accommodation). • No constitutional right to exemption from neutral, generally applicable regulations of conduct (e.g., Peyote case). Peyote case - state had statute against consuming hallucinogens. Peyote is a hallucinogen. Statute can be enforced across board as an anti drug law and can even be enforced against native americans claiming they have religious motivation for peyote use. Neutral, generally applicable laws aimed at conduct can be enforced against everyone on the same terms. Example 33: Religious Freedom Restoration Act (RFRA) overruled the Peyote decision: Even neutral, generally applicable laws could not be enforced against religious objectors unless the government had a compelling interest. Insofar as RFRA applied to state and local governments, the Supreme Court struck it down as beyond federal legislative authority (Congress cannot force states to accommodate religious beliefs). RFRA would not be beyond federal legislative authority insofar as it applied to the federal government's own actions. Can states and localities require a religious accommodation for themselves? Yes. Rfra went much farther than protecting against discrimination - it required accommodation of religious beliefs and sc found that was beyond fed legislative power insofar as it addressed states and localities - insofar as rfra addresses fed entitites, its still good law. That's because fed govt has undoubted power over army navy and district of Colombia and its own employment practices ,etc. so wherever fed govt has legislative power, like army navy and district of Colombia, it can require accommodation of religious belief and states can require accommodation of religious belief by states and localities and many of them have done so. Accommodation of religious belief not constitutionally required but in one context it is. • Exception: "Ministerial exception"—in 2012, the Supreme Court held that the First Amendment requires a ministerial exception to employment laws. o Non-discrimination employment laws cannot be applied to ministers. Constitutional right not to have nondiscrimination laws apply to ministers. o Plaintiff was a commissioned minister working as a teacher in a religious school. The school fired her in a plain violation of the Americans with Disabilities Act. She sought reinstatement as provided by the statute. o The Supreme Court held unanimously that the First Amendment precludes litigation of claims concerning the employment relationship between a religious institution and its ministers. Note term minister not limited to people who really are ministers - it was construed broadly to include a religious school teacher, most of whos duties were secular. So at least in context of employment laws, ministerial exception is that employment laws cant be applied to ministers, ministerial exception does require that religious believers be treated special. o The federal government cannot regulate employment relations between a religious institution and its ministers. o The term "minister" was construed broadly. § Campus Access: A state university that allows student groups to meet on campus must allow student religious groups equal access.

broadcasters

1. The only special case is broadcasters. 2. Traditionally, because of early limits on the broadcast spectrum, the government had greater regulatory authority over broadcasters than over print media or other unlimited technologies like the Internet. Frankly not clear just how much govt can regulate broadcast but it is clear they have some additional authority to regulate broadcast because spectrum is limited and that one fact is probably worth noting.

In order to discourage the transfer of electrical products that could threaten the security of the United States as well as to raise revenue, Congress enacted a statute that imposed a tax on the export of specific electrical products to countries that were determined to be hostile to the United States.Is this statute constitutional? A No, because of the Export Taxation Clause of Article I, Section 9 of the United States Constitution. B No, because Congress has no general police power to legislate for the health, safety, welfare, or morals of citizens. C Yes, because of the taxing power granted to Congress in Article I, Section 8 of the United States Constitution. D Yes, because of the Commerce Clause of Article I, Section 8 of the United States Constitution.

Answer choice A is correct. Congress may not tax goods exported to foreign countries. Under the Export Taxation Clause, a tax or duty that falls on goods during the course of exportation or on services or activities closely related to the export process is prohibited. Answer choice B is incorrect. While Congress, unlike a state legislature, does not enjoy a general police power, Congress can generally exercise its power to tax for any public purpose, such as national security. However, the Export Taxation Clause specifically prohibits Congress from taxing exports. Answer choice C is incorrect. Although Article I, Section 8 of the Constitution gives Congress the plenary (i.e., exclusive) power to raise revenue through the imposition of taxes, and the General Welfare Clause of this section has been interpreted as permitting Congress to exercise its power to tax for any public purpose, the Export Taxation Clause specifically prohibits Congress from taxing exports. Answer choice D is incorrect. Although the Commerce Clause grants Congress power over commerce, including international commerce, the Export Taxation Clause specifically prohibits Congress from taxing exports.

Congress may delegate authority to...

Congress may delegate authority to the Department of the Interior so long as Congress specifies intelligible standards for the delegatee to follow. If a statute lays out guidelines for how the funds will be allocated, that is constitutional.

The Dormant Commerce Clause does NOT apply in which of the following situations? A The state regulation discriminates against out-of-state commerce with Congressional approval. B The state unduly burdens interstate commerce. C The state discriminates against out-of-state commerce in the absence of federal action. D The state regulates wholly extraterritorial activity.

Correct Answer: A

The Free Exercise Clause protects from governmental restriction not only the freedom to hold any religious belief but also the freedom not to hold a religious belief. Is this statement true? A Yes, absolutely. B Yes, unless the governmental restriction passes the strict scrutiny test. C Yes, unless the governmental restriction passes the intermediate scrutiny test. D Yes, unless the governmental restriction passes the rational basis test.

Correct Answer: A

A student joined a small national organization during her freshman year of college after several of her friends, who were active in the organization, told her about the organization's annual all-expense paid ski trip for its card-carrying members. The student attended a recruitment drive, signed a pledge of loyalty to the party, paid her annual dues, and received an organization pin. The student never attended a meeting or read the organization's literature she was given. The student later joined other members of the organization at the ski resort. During a meeting around the ski lodge fireplace, the student learned for the first time that the organization was a radical organization. The organization's members were preparing to use subversive means to achieve their objective of installing the organization's spiritual leader as Supreme Dictator of the United States. To this end, the organization was stockpiling mind control serum and planned to poison the nation's water supply. After returning home, the student consciously avoided members of the organization and never participated in the organization's activities again, but the student's name remained on the organization's active member roster. Three years later, the student neared graduation and applied for federal employment. Can the student be denied public employment based upon her membership in this organization? A No, because the student did not personally participate in the organization's subversive plot. B No, because the student did not intend to install the organization's spiritual leader as Supreme Dictator. C Yes, because the student continues to be listed as an active member of a subversive organization. D Yes, because the student has knowledge of the organization's illegal objectives and failed to report them to the authorities.

Answer choice B is correct because the student lacked the specific intent to further the organization's illegal objectives; this intent is necessary in order to punish her based upon her association with the group. The First Amendment's freedom of association protects the right to form or participate in any group, club, or other organization virtually without restriction, but the right is not absolute. The government may justifiably infringe the right of association to advance a compelling state interest, such as preventing discrimination or subversive activities designed to undermine the democratic process. A person may only be punished or deprived of public employment based on their political affiliation if that individual (i) is an active member of a subversive organization, (ii) has knowledge of the organization's illegal activities, and (iii) has a specific intent to further those illegal objectives. Answer choice A is incorrect because a person does not have to personally or physically participate in a subversive organization's overt activities to be punished if the person is a member of that organization, aware of the organization's illicit goals, and intends that those subversive objectives be furthered. Answer choice C is incorrect because her inclusion on the active member list is insufficient for purposes of denying her public employment. Answer choice D is incorrect because an individual cannot be denied public employment based simply upon knowledge of the organization's objectives. She would also need the intent to further those objectives.

A tenured professor at a state university was summarily dismissed by the university president after it came to light that she had engaged in plagiarism. Shortly following her dismissal, the professor contested her termination in a post-termination evidentiary hearing, at which her termination was upheld. The professor then filed an action in federal court, asserting that she was denied due process of law.Were the professor's due process rights violated? A Yes, because the professor was not granted a full evidentiary hearing prior to termination. B Yes, because the professor was not provided with a pre-termination opportunity to respond. C No, because the professor could be terminated without cause, so no due process was required. D No, because the professor received notice of her dismissal and a post-termination hearing.

Answer choice B is correct. A public employee who may be discharged only for cause has a property interest in her job and therefore is entitled to notice of termination and a pre-termination opportunity to respond. A formal hearing is not required, as long as there is pre-termination notice, an opportunity to respond to the decision maker, and a post-termination evidentiary hearing. In this case, the professor at the state university could only be fired for cause because she was tenured. Accordingly, due process required that the professor be provided with a pre-termination opportunity to respond. Because such an opportunity was not provided, the professor's due process rights were violated. Answer choice A is incorrect because the professor was only entitled to a pre-termination opportunity to respond, not a pre-termination evidentiary hearing. Answer choice C is incorrect because the professor's public employment at the state university could only be terminated for cause. Moreover, a public employee who may be fired with or without cause is still entitled to notice and a post-termination hearing. Answer choice D is incorrect because the professor should also have been given a pre-termination opportunity to respond.

Regulation of the Time, Place, or Manner of Expression (Content-neutral)

Content neutral regulations are called time place or manner. Its not content that's being regulated but time, place, or manner. - Apply principally in a public forum. - A public forum is a place traditionally reserved for speech activities. These places include streets, parks, and public sidewalks around public buildings (but not airports).

Regulation of the Time, Place, or Manner of Expression (Content-neutral)

Content neutral regulations are called time place or manner. Its not content that's being regulated but time, place, or manner. You absolutely can count on being tested on this rule. • Apply principally in a public forum. • A public forum is a place traditionally reserved for speech activities. These places include streets, parks, and public sidewalks around public buildings (but not airports). • Only time, place, and manner may be regulated in a public forum. To have a viable or constitutional time place or manner regulations, there are three requirements (must absolutely memorize this, most important thing in con law) 1) Content neutral: Must be content neutral on its face and neutral as applied. If a facially neutral law is applied to favor one speaker or one point of view or one political party its not content neutral. Content neutral also means law must not allow executive discretion - if an executive officer has discretion, if exec officer has power to pick and choose who can speak risk is that the discretion will be administered in a non-neutral way. The supreme court has tried to guard against that possibility by saying that a law must be content neutral on its face, it must be content neutral as it has been applied, and it must not allow executive discretion which would threaten content neutrality. Example 37: A local D.C. ordinance prohibited picketing within 500 feet of a foreign embassy (but it didn't just stop there - it didn't say all picketing within 500 feet of a forein embassy, it said only if...) if the picketing sign brought the foreign government into odium or disrepute (that's not time place or manner regulation, that's regulation based on content - whether signs were hostile to foreign govt.). This is plainly content-based and is thus, unconstitutional. Example 38: Parade permit laws (getting permit before parading on street or using public property) giving permits to first come, first served (like first group to apply for a permit gets one) are constitutional because content neutral. Parade permit laws giving the chief of police discretion to decide whether it was in public interest to grant someones permit are unconstitutional because the law may not be applied in a content-neutral way - if chief of police has discretion to pick and choose among permit applicants, the risk is that he will grant the applicaiotns for those he agrees with. And deny permits to those he finds reprehensible. That risk of content based administration makes the law unconstitutional. 2) Alternative channels of communication must be left open: Time, place, or manner law must be a guideline for speech, not a flat prohibition of speech. Example 39: Laws against amplified sound trucks during the night-time hours are fine - its valid, content neutral, and leaves lot of other times and places where u can sound. Laws against amplified sound during all times and all places would be struck down. Whats the difference? One is a guideline for speech leaving alternative channels open, other is a prohibition of speech attempting to shut down a certain method of speaking. 3) Must narrowly serve a significant state interest: Under this test, most content-neutral time, place, or manner regulations are upheld. • Does not require a compelling interest - Note absence of buzz word compelling. Content neutral regulation of time place and manner doesn't require a compelling state interest. Doesn't require a national emergency we cant live without it kind of justification. It requires a significant state interest which is basically anything that makes sense to a judge and for that reason most content neutral regulations of time place and manner are upheld - not all by any means - but most content neutral regulations of time place and manner are upheld - Theyre extremely common in American cities and theyre routinely enforced.

Congress enacted legislation that made funding available to the states to provide services for children with special needs. The act conditions the funding on the states' meeting a variety of conditions designed to ensure that children with special needs received adequate services. In addition, the act grants parents of a child with special needs a right of action against a local school board for the failure to provide adequate services. The act permits the parents to recover the cost of tuition at a private school that meets the child's needs, together with "attorney fees and costs." The act is silent as to whether expert fees are considered costs. The parents of a child with special needs successfully pursued an action against the local school board under this statute. The court awarded the parents the appropriate damages, as well as attorney's fees and costs, including expert fees.Of the following, which is the most likely constitutional basis on which the local school board can challenge the court's award? A The act itself is unconstitutional because the Tenth Amendment prohibits federal involvement in areas, such as elementary education, that have been traditionally reserved to the States. B The awarding of expert fees is unconstitutional because conditions imposed on states by Congress in the exercise of its spending power must be set out unambiguously. C The conditions imposed on the states to receive funding are unconstitutional, because Congress can only exercise its spending power to carry out another enumerated power. D Providing the parents the right to bring this action is unconstitutional, because the Eleventh Amendment prohibits suits by private citizens against local governmental entities for money damages.

Answer choice B is correct. While Congress has the power under the spending power to spend for the general welfare and may impose conditions on the receipt of an appropriation by a state, such conditions must be set out unambiguously to be enforceable. Since expert fees were not unambiguously included in the amounts that may be recovered by the parents of a child with special needs under this statutory cause of action, these fees cannot be recovered. Answer choice A is incorrect. The Tenth Amendment provides that all powers not assigned by the Constitution to the federal government are reserved to the states, or to the people. However, the federal government has very broad authority under the spending power to spend for the general welfare, making state power rarely exclusive. Therefore, because states do not have exclusive power over education, this answer choice is incorrect. Answer choice C is incorrect. The spending power has been interpreted very broadly. Congress has the power to spend for the "general welfare"—i.e., any public purpose—not just to pursue its other enumerated powers. Therefore, this answer choice misstates the law. Answer choice D is incorrect. The Eleventh Amendment is a jurisdictional bar that prohibits the citizens of one state from suing another state in federal court. While the Supreme Court has expanded the amendment's reach to preclude citizens from suing their own state in federal court as well, this amendment does not apply to suits brought against local governments and their entities, such as school boards. Consequently, the provision in the act providing parents the right to sue their local school board does not violate the Eleventh Amendment.

Regulation of the Content of Expression

Content-based regulations of speech trigger strict scrutiny and are usually struck down - few categories of speech that can be regulated because of content but quite exceptional.

FILL IN THE BLANK. The ________________ test requires that a law be substantially related to an important government interest. A strict scrutiny B suspect classification C rational basis D intermediate scrutiny

Correct Answer: D

Due to a rising concern for children with undisclosed food allergies, as well as preserving a controlled environment in school cafeterias, a city enacted an ordinance stating that only lunches made without a specific list of ingredients commonly causing food allergies could be prepared and served in school cafeterias. In addition, to avoid any lunches that could cause food allergies, the ordinance stated that only students who ordered lunch in the cafeteria could eat there; students who brought their lunches from home were not permitted inside of the cafeteria during lunch time. Instead, these students were required to eat their lunches in a different room. A student who only ate certain foods in accordance with his religious beliefs brought his lunch from home. The lunch did not contain any of the prohibited ingredients listed in the ordinance. All of the student's friends ate their lunch in the cafeteria, but the student was forced to eat his lunches in another room.If the student's parents challenge the constitutionality of the ordinance under the Equal Protection Clause of the Fourteenth Amendment, are they likely to succeed? A Yes, because the ordinance is not narrowly tailored to address the dangers posed by food allergies. B Yes, because the ordinance has a disparate impact on children with religious dietary restrictions. C No, because the ordinance is rationally related to a legitimate governmental interest. D No, because the ordinance applies to all students.

Answer choice C is correct. The Equal Protection Clause of the Fourteenth Amendment provides that states are generally prohibited from passing legislation that treats similarly situated persons differently. To trigger strict or intermediate scrutiny, there must be discriminatory intent on the part of the government or a classification based on a suspect or quasi-suspect class. Strict scrutiny also applies if a fundamental right is involved. The rational basis standard is used in all cases in which one of the higher standards does not apply. A law passes the rational basis standard of review if it is rationally related to a legitimate governmental interest. This is a test of minimal scrutiny. Here, students who bring their lunches from home are being treated differently from students who eat in the cafeteria because they must eat in a separate room. However, the ordinance has a rational basis that is related to the legitimate government interest of preventing complications due to food allergies. Because this ordinance does not target a suspect class or involve a fundamental right and has no discriminatory intent, it passes rational basis review. Therefore, the parents' challenge is likely to fail. Answer choice A is incorrect because this ordinance only needs to meet rational basis scrutiny, and need not be narrowly tailored to its purpose. Answer choice B is incorrect because the fact that legislation has a disparate effect on people of different races, genders, or religions, without discriminatory intent, is insufficient to trigger strict or intermediate scrutiny. Answer choice D is incorrect. The fact that the ordinance applies to all students does not mean it is constitutional on that basis because it still differentiates between students who bring their lunches from home and those who buy their lunches in the cafeteria, requiring the former to eat lunch in a separate room.

FILL IN THE BLANK. Under the _________ Amendment, most individual constitutional rights are specifically applicable to actions by the state and local governments. A Thirteen B Fifth C Tenth D Fourteenth

Correct Answer: D

An unmarried man and woman lived together for two years in State A and had a child. Two years after the child was born, the couple ended their relationship and the woman took full custody of the child. However, the man refused to provide any financial support for the child. The woman properly sued him in State A court. The State A court issued a final judgment on the merits ordering the man to pay the woman $500 per month in child support until the child turned 18. After the judgment was issued, the man immediately moved to State B and refused to pay any child support to the woman. The woman and child both remained in State A. The woman subsequently filed an enforcement proceeding in State B to enforce the State A judgment.Which of the following would best support the enforcement of the State A judgment regarding child support in State B? A The Tenth Amendment B The Eleventh Amendment C The Full Faith and Credit Clause D The Supremacy Clause

Answer choice C is correct. The Full Faith and Credit Clause of Article IV, Section 1 provides that full faith and credit shall be given in each state to the public acts, records, and judicial proceedings of every other state. In order to be given full faith and credit, a decision must meet three requirements: (i) the court that rendered the judgment must have had jurisdiction over the parties and the subject matter; (ii) the judgment must have been on the merits rather than on a procedural issue; and (iii) the judgment must be final. Here, State A had jurisdiction over the man and woman, and the judgment was final and based upon the merits of the case. Thus, the Full Faith and Credit Clause is applicable to this situation. Answer choice A is incorrect. The Tenth Amendment provides that all powers not assigned by the Constitution to the federal government are reserved to the states or to the people. This amendment does not apply in this situation because there is no state versus federal government issue. Answer choice B is incorrect. The Eleventh Amendment is a jurisdictional bar that prohibits the citizens of one state from suing another state in federal court. Here, the woman is not suing State B in federal court, thus it does not apply. Answer choice D is incorrect. It is possible (and common) for the federal and state governments to legislate in the same area. When this happens, the Supremacy Clause (Article VI, paragraph 2) provides that federal law supersedes conflicting state law. Here, there is no conflict between federal and state legislation in the same area. Therefore, the Supremacy Clause is inapplicable.

The Judiciary Committee of the U.S. House of Representatives initiated impeachment proceedings against a federal district court judge. The President, a lifelong friend of the judge, considered the grounds for impeachment that were being discussed to be politically motivated and without substantial merit. Prior to any hearing on matter by the House committee, the president pardoned the judge.What effect does this pardon have on the impeachment proceedings against the judge? A The proceedings must stop, because the President's power to pardon is plenary. B The proceedings must stop, because the President acted in good faith in granting the pardon. C The pardon has no effect on the proceedings because a President's power to pardon does not extend to impeachment. D The pardon has no effect on the proceedings because a President may not pardon a person until that person has been convicted.

Answer choice C is correct. The President's power to pardon does not extend to impeachment of a federal official. Answer choice A is incorrect. While the President's power to pardon with regard to federal crimes is often referred to as plenary, it does not extend to impeachment of a federal official. Answer choice B is incorrect because there is no good faith restriction imposed by the Constitution on the President's exercise of the power to pardon. Answer choice D is incorrect because, while a President may not pardon a person with respect to future acts, a President may pardon a person at any time after the offense has been committed.

The Judiciary Committee of the U.S. House of Representatives initiated impeachment proceedings against a federal district court judge. The President, a lifelong friend of the judge, considered the grounds for impeachment that were being discussed to be politically motivated and without substantial merit. Prior to any hearing on matter by the House committee, the president pardoned the judge.What effect does this pardon have on the impeachment proceedings against the judge? A The proceedings must stop, because the President's power to pardon is plenary. B The proceedings must stop, because the President acted in good faith in granting the pardon. C The pardon has no effect on the proceedings because a President's power to pardon does not extend to impeachment. D The pardon has no effect on the proceedings because a President may not pardon a person until that person has been convicted. SUBMIT ANSWER

Answer choice C is correct. The President's power to pardon does not extend to impeachment of a federal official. Answer choice A is incorrect. While the President's power to pardon with regard to federal crimes is often referred to as plenary, it does not extend to impeachment of a federal official. Answer choice B is incorrect because there is no good faith restriction imposed by the Constitution on the President's exercise of the power to pardon. Answer choice D is incorrect because, while a President may not pardon a person with respect to future acts, a President may pardon a person at any time after the offense has been committed.

FILL IN THE BLANKS. The Eleventh Amendment generally prohibits a suit against a _________ for _________. A state official; money damages B state official; injunctive relief C local government; money damages D state; money damages

Correct Answer: D

Can there by regulatory taking where govt does owe u money even without physical occupation?

Can there by regulatory taking where govt does owe u money even without physical occupation? Yes it can happen but only where regulation leaves no economically viable use for the property.

The District of Columbia government has the power to levy an income tax. Congress has recently enacted a statute that prohibits the District of Columbia from imposing a tax on the income of individuals who work in the District of Columbia but live elsewhere. Congress was seeking to provide an incentive for nonresidents to do business within the District of Columbia.Is this statute likely to be found constitutional? A No, because it violates the Due Process Clause of the Fifth Amendment to the U.S. Constitution. B No, because it violates the Uniformity Clause of Article I, Section 8 of the U.S. Constitution. C Yes, under the Enclave Clause in Article I, Section 8 of the U.S. Constitution. D Yes, under the Sixteenth Amendment to the U.S. Constitution.

Answer choice C is correct. Under Article I, Section 8, Clause 17 of the U.S. Constitution, also known as the Enclave Clause, Congress has the general police and regulatory powers over the District of Columbia that a state enjoys over persons and things within its boundaries. Among those powers is the power to tax or not to tax income earned within its boundaries. Answer choice A is incorrect. There is an equal protection component of the Due Process Clause of the Fifth Amendment that corresponds to the Equal Protection Clause of the Fourteenth Amendment. However, while the statute does treat residents of the District of Columbia who earn income there differently from the nonresidents of the District of Columbia who earn income there, the residents of the District of Columbia are not members of a suspect class. Consequently, the statute needs only a rational basis. The provision of an incentive for nonresidents to do business within the District of Columbia constitutes a rational basis for the statute. Answer choice B is incorrect because the Uniformity Clause applies to indirect taxes, such as excise taxes and import duties, not to direct taxes on income. Answer choice D is incorrect because the Sixteenth Amendment gave Congress the power to impose an income tax without apportioning the tax among the states based on population. Here, the statute does not impose an income tax, but prohibits the District of Columbia from imposing one on certain income.

In response to a highly publicized incident involving a local sonographer who failed to correctly read a sonogram, and at the urging of a state association of medical technicians, the state legislature enacted a bill that lengthened the period of education required to receive a license to be employed as a sonographer from one year to two years. Prior to the enactment of this law, an obstetrician with a private practice had promised a clerk who worked in her office that she would employ him as sonographer if he obtained the necessary state license. Three months before the passage of the law, the clerk had begun his coursework in order to satisfy the then one-year educational requirement to become a sonographer. Although discussed at the hearing that the legislative committee had held on this legislation, the statute did not contain a provision that delayed its effective date for those already enrolled in an educational program to become a sonographer. The clerk has filed suit challenging the constitutionality of the state statute as applied to him due to its failure to grant him an exemption from the two-year educational requirement. How is the court likely to rule? A For the clerk, because employment is a fundamental right and the state cannot satisfy the strict scrutiny test. B For the clerk, because the statute violates the Contract Clause of Article I, Section 10. C For the state, because the legislation is a regulatory taking of intangible personal property, and therefore the Takings Clause of the Fifth Amendment does not apply. D For the state, because the state legislature could rationally believe that the statute, as passed, was in the best interest of the public.

Answer choice D is correct. Because the statute does not discriminate against a suspect or quasi-suspect class, and does not infringe upon a fundamental right, the statute is subject to the rational basis test. Under this standard, the statute is presumed valid until the plaintiff establishes that the law is arbitrary or irrational. Here, the clerk is effectively asserting that the statute's failure to apply the one-year educational requirement to students enrolled in an educational program at the time of the enactment of the statute is arbitrary or irrational. However, the legislature did consider such an exception, and could have concluded that applying the two-year educational requirement to current students would better serve to protect the public from mistakes made by sonographers who were not adequately trained. Even if this reason had not been expressly discussed when the legislation was passed, any legitimate justification for a law, even one not demonstrably relied upon by the legislature, will suffice under the rational basis test. Answer choice A is incorrect. Although substantive due process does require the application of the strict scrutiny test to governmental action that infringes upon a fundamental right, the right to be employed in any profession is not a fundamental right. Answer choice B is incorrect. Although the Contract Clause does apply to state legislative action, it does not preclude any interference with a person's freedom to contract, but only prevents the substantial impairment of a contract between private parties if the government cannot demonstrate that the interference was reasonable and necessary to serve an important governmental interest. Here, it is unclear whether the clerk can enforce his employer's promise to employ him as a sonographer due to his detrimental reliance on the promise, but even assuming that he can, the statute does not prevent the clerk from enforcing that promise. Therefore, it does not impair a contract between private parties, and does not violate the Contract Clause. Answer choice C is incorrect because the Takings Clause does apply to a regulatory taking of intangible personal property. However, even assuming that the clerk has an enforceable contract with his employer that would give rise to an intangible personal property interest, the regulation does not take away his right to be a sonographer or permanently deprive his education of economic value, but merely delays the clerk's employment as a sonographer.

A state legislature passed a law requiring employers to provide their employees with health insurance that covered certain prescription drugs. Violation of this statute was considered a crime that subjected the offender to fines, which were described in detail in the statute. The law was effective immediately. An employer did not provide its employees with insurance that covered the required drugs, and argued that such drugs were prohibited by the religion practiced by the employer. The employer filed a complaint in federal court asserting that the law was unconstitutional and asked for a preliminary injunction against the attorney general to prevent him from enforcing the statute while the case was heard. The attorney general filed a motion to dismiss, asserting that the federal court did not have jurisdiction to hear the case.How should the federal court rule on the attorney general's motion to dismiss? A Grant the motion based on prudential grounds. B Grant the motion based on the doctrine of abstention. C Deny the motion, because the employer has taxpayer standing due to the imposition of fines. D Deny the motion, because the employer's injury is imminent.

Answer choice D is correct. In order to bring a claim in federal court, a plaintiff must have standing and the issue must be ripe for litigation. To have standing, a plaintiff must establish (1) a particularized injury, which must have actually occurred or be imminent; (2) that defendant caused this injury; and (3) that the relief requested is likely to redress the injury. First, the employer can show an imminent injury—loss of money from the threatened fines, which can be imposed immediately. Second, the government caused this injury by forcing the employer to comply with the law or pay a fine. Third, the requested injunction will redress that injury by removing the burden of compliance unless and until a court rules that the law is constitutional. Furthermore, the case is ripe because the threat of enforcement is imminent and the legal issues are sufficiently developed for a court to decide the case. Answer choice A is incorrect because neither of the categories of prudential standing applies here: The court is not being asked to adjudicate the claims of third parties or "generalized grievances" (i.e., injuries that are widely shared, as contrasted with the employer's individualized injury here). Answer choice B is incorrect because the two abstention doctrines—the Younger abstention doctrine, which applies only to pending state criminal cases or administrative proceedings, and the Pullman abstention doctrine, in which constitutional claims depend on resolving an unsettled issue of state law—are inapplicable. Answer choice C is incorrect because the doctrine of taxpayer standing, which allows a taxpayer standing to file a federal lawsuit challenging a specific legislative appropriation for violation of the Establishment Clause, is also inapplicable.

Regarding standing, which of the following is FALSE? A A taxpayer may challenge any federal action on the basis of a violation of the Establishment of Religion Clause. B A federal taxpayer always has standing to challenge his own tax liability. C A litigant generally does not have standing to bring a lawsuit based on the legal claims of a third party. D An organization may have standing if its members have standing.

Correct Answer: A

Regarding the Freedom of Association Clause, which of the following is FALSE? A A state can require a political party to hold an open primary. B A state generally cannot deny bar admission to a candidate on the basis of political association. C A public employee can be required to take a loyalty oath. D A loyalty oath is often struck down for vagueness and overbreadth.

Correct Answer: A

Thirteenth Amendment

Congress has broad power to legislate against racial discrimination, whether public or private. - Includes purely private racial discrimination. This can be reached by congress under 13th amendment power.

A private secular school rented out its auditorium to the public for various uses on a first come, first serve basis. A local religious group sought to rent the auditorium for weekly services. The school refused because it did not want to be seen as endorsing a particular religion. The group sued the school, arguing that, by refusing to allow it to rent the auditorium, the school was violating its constitutional rights.Is the religious group likely to succeed? A Yes, because the school's denial violated the religious group's First Amendment right of free speech. B Yes, because the religious group is entitled to rent the auditorium under the First Amendment Free Exercise Clause. C No, because permitting the religious group to use the auditorium for services would violate the First Amendment Establishment Clause. D No, because the school was a private entity.

Answer choice D is correct. The First Amendment applies only to federal or state action. A private person's action may constitute state action if a private person performs a traditional governmental function or there is significant state involvement. Here, the school is identified as a private entity. Consequently, the First Amendment does not apply to its actions. Answer choice A is incorrect. A public school that opens its facilities for use by the general public violates a religious group's First Amendment right of free speech by prohibiting the group from using the facilities because the group would discuss religious topics on the premises. However, the school in question is a private entity and therefore is not subject to First Amendment constraints. Answer choice B is incorrect because, although the school's refusal to rent to the religious organization does impact the group's ability to exercise its religious beliefs, the school in question is a private entity and therefore is not subject to First Amendment constraints. Answer choice C is incorrect because the school in question is a private entity and therefore is not subject to First Amendment constraints. Moreover, a public school that allows student groups or organizations to use its facilities does not violate the Establishment Clause by allowing a religious organization to use its facilities.

State Regulation and Taxation of Commerce - Privileges and Immunities of State Citizenship under Article IV (Comity Clause)

Comity clause of article 4 guarantees privileges and immunities of state citizenship. o Technically, a subject of individual rights, but functionally belongs here because of its similarity to the next subject (Dormant Commerce Clause). o Forbids serious discrimination against out of state individuals, absent substantial justification. § Does not protect out-of-state corporations but do protect nonresident individuals against serious discrimination. § "Serious discrimination" typically involves employment. o Rule: There can be no legal requirement of residency for private employment. States cannot require that you live or reside in the state to work in the state. However, public employment can require residency requirements. Example 18: A law that requires all cab drivers to live in the city is not valid because if u have to reside in city u cannot reside in another state, and state or city cannot make law requiring residency for access to private job market, including for cab drivers.. § Examples of private employment requirements (can't depend on residency): Admission to the bar - cannot require person live somewhere to pass bar there, why? Getting license to practice law is prerequisite for private employment as lawyer. States cannot require residency for access to the private job market; other occupational licenses. § Examples of public employment which is different (can depend on residency): A city hiring only city residents or requiring a certain percentage of city residents on city construction projects. If a city wants to only hire city residents for its own jobs that's fine. Or if a city wants to require city building contactors to hire certain percentage of city residents, that's fine. Its public employment and government can always choose to spend its own money on its own residents. But whereas a city can require city contractors to have a certain percentage of local workers, a city cannot require all contractors to have a certain percentage of local workers - that's private employment, unconstitutional discrimination against out of city and therefore out of state individuals. o Non-serious discrimination: States can discriminate with regard to recreational opportunities, such as hunting licenses or state park access. Can charge out of state persons more money for hunting license than in state person. Or higher fee for using state park for out of state residents. That's all fine. Serious discrimination though is employment - no requirement of residency for private employment.

In response to recent unusual earthquake activity, a large city has enacted an ordinance requiring the permanent placement of seismographic equipment in the basement of 20 randomly selected commercial buildings throughout the city as part of the creation of an early warning network. Although the owners of the buildings will not be compensated, the cost of the purchase and installation of the equipment is to be borne by a private university that will operate the early warning network. The owner of one building that has been randomly selected as a site for this equipment has challenged this law as unconstitutional.Should the court rule in favor of the owner? A No, because the economic impact on the owner would be slight. B No, because the owner's building was randomly selected. C Yes, because the city will not operate the network. D Yes, because the equipment will reside permanently in the building.

Answer choice D is correct. The placement of the seismographic equipment in the building would result in the permanent physical occupation of the building by the city. Consequently, this placement would constitute a taking. Since the owner would not be compensated for the taking, it is unconstitutional. Answer choice A is incorrect because the Fifth Amendment Takings Clause, which is applicable to state and local governments through the Due Process Clause of the Fourteenth Amendment, requires the payment of just compensation. The fact that the economic impact on the building owner would be slight would affect the amount of compensation, but not the requirement that compensation be paid. Answer choice B is incorrect because the fact that the building was randomly selected does not prevent the government's action from being an unconstitutional taking. Answer choice C is incorrect because the Takings Clause applies when the property taken by the government is given to a private person and when the government retains the property.

In response to recent unusual earthquake activity, a large city has enacted an ordinance requiring the permanent placement of seismographic equipment in the basement of 20 randomly selected commercial buildings throughout the city as part of the creation of an early warning network. Although the owners of the buildings will not be compensated, the cost of the purchase and installation of the equipment is to be borne by a private university that will operate the early warning network. The owner of one building that has been randomly selected as a site for this equipment has challenged this law as unconstitutional.Should the court rule in favor of the owner? A No, because the economic impact on the owner would be slight. B No, because the owner's building was randomly selected. C Yes, because the city will not operate the network. D Yes, because the equipment will reside permanently in the building.

Answer choice D is correct. The placement of the seismographic equipment in the building would result in the permanent physical occupation of the building by the city. Consequently, this placement would constitute a taking. Since the owner would not be compensated for the taking, it is unconstitutional. Answer choice A is incorrect because the Fifth Amendment Takings Clause, which is applicable to state and local governments through the Due Process Clause of the Fourteenth Amendment, requires the payment of just compensation. The fact that the economic impact on the building owner would be slight would affect the amount of compensation, but not the requirement that compensation be paid. Answer choice B is incorrect because the fact that the building was randomly selected does not prevent the government's action from being an unconstitutional taking. Answer choice C is incorrect because the Takings Clause applies when the property taken by the government is given to a private person and when the government retains the property.

Fifteenth Amendment (specifically focused on voting)

Congress has the power to ensure there is no racial discrimination in voting. Exam Tip 7: On exam day, be on the lookout for Congressional acts that mirror RFRA. Remember that such acts are generally valid as to Congress, but invalid as applied to the states and localities.

To address a shortfall in the judiciary's budget and to comply with a provision in the state constitution requiring a balanced budget, a state increased, by statute, various court fees, including the fee to file an appeal, and eliminated any waiver of these fees except for a defendant convicted of a capital crime and sentenced to death. A defendant imprisoned after his conviction of a non-capital offense filed an appeal and a request for a waiver of the fee because of his inability to pay. Citing the statute, the state appellate court ruled that the defendant was not entitled to the waiver, as he was not convicted of a capital offense and sentenced to death.The defendant properly challenged this ruling in federal court. Should the federal court uphold the state appellate court's ruling? A Yes, because the fee is rationally related to the goal of adequately funding the state's judicial system. B Yes, because state law required payment of the fee. C No, because a state may not prohibit a prisoner from exercising a fundamental right. D No, because the defendant was indigent.

Answer choice D is correct. While most state actions that discriminate against the poor are subject only to rational basis scrutiny, the availability of appeal in a criminal case cannot hinge on ability to pay a filing fee. To deny an indigent the right to appeal through the imposition of a fee violates the Due Process and Equal Protection Clauses of the Fourteenth Amendment. Answer choice A is incorrect because, although there is a rational basis for the state's increase in the various court fees, an indigent accused has a constitutional right to waiver of a fee that prevents him from enjoying a fundamental right, such as access to the courts. Answer choice B is incorrect because, even though the state court decision was based on state law, the state court cannot enforce a state law that conflicts with the federal constitution. Answer choice C is incorrect because, while a prisoner has a right of reasonable access to the court, the law in question did not deny the defendant such access on the basis of his status as a prisoner, but due to his failure to pay the necessary filing fee.

If you can't sue a state, then whom can you sue?

Answer: A state officer. • Injunctive relief—you can always get injunctive relief simply by enjoining the appropriate state officer (e.g., sue the state attorney general). If you claim state statute is unconstitutional, you simply sue state attorney general to enjoin its enforcement. • Money damages—you can also get money damages, but only from the officer personally for some kind of constitutional tort • Damages from the state treasury are barred (unless the state consents or Congress expressly says so to enforce individual rights).

Privileges or Immunities Clause of the Fourteenth Amendment

Applies only to a state's treatment of its own citizens, and is thus irrelevant to situations where a state is dealing with out-of-staters. Further, it has a limited application, and is usually only successful in cases involving the right to travel.

direct tax rules by fed govt

Article I, Section 9 provides that "no . . . direct tax shall be laid, unless in proportion to the Census[.]" Although there is dispute as to the types of taxes that are encompassed within the definition of a direct tax, a tax on real property interests is undisputedly a direct tax. To satisfy the apportionment rule, a state with twice the population of another state would have to pay twice the tax, even if the more populous state's share of the national tax base were smaller. Here, because the real property tax is not in proportion to the population of each state, it would run afoul of Article I, Section 9.

A citizen of the United States sued a citizen of a foreign country in the courts of that country and won a judgment, but was unable to enforce that judgment. Based on the terms of the provisions of a self-executing treaty between the United States and the foreign country that had been ratified by the Senate, a valid, proper judgment by a court of the foreign country is enforceable in a federal district court. Subsequently, the defendant moved to the United States and became a U.S. citizen residing in the same state as the plaintiff.Does Article III of the U.S. Constitution permit the plaintiff to bring an action in the federal court for his home state to enforce the foreign judgment? A No, because the defendant is not a citizen of a foreign country. B No, because the plaintiff and the defendant are citizens of the same state. C Yes, because the defendant is now a United States citizen. D Yes, because the case arises under a treaty of the United States.

Article III provides that federal courts, which have been created by Congress pursuant to Article III, only have jurisdiction over specific cases and controversies. Among those cases and controversies are cases that arise under the treaties of the United States. In this case, because the right of the plaintiff to collect on the foreign judgment obtained against the defendant is based on a treaty provision, Article III permits the plaintiff to maintain this action in federal district court.

Regarding the right to privacy, which of the following statements is FALSE? A There is no fundamental right for related family members to reside in the same household. B There is a right to marriage without substantial governmental interference by an age-qualified man and woman. C There is a right to contraception for unmarried as well as married couples. D The government has no legitimate interest in regulating noncommercial sexual intimacy between consenting adults.

Correct Answer: A

STATE TAXATION OF INTERSTATE COMMERCE, PREEMPTION, AND RELATIONS AMONG STATES - relations among states

C. Relations Among States • Interstate compacts are agreements among states. States can make interstate compacts, but if the compact affects federal rights, Congress must approve. • Full Faith and Credit Clause: States do not have to follow other states' laws, but they do have to give full faith and credit to judgments rendered by other states' courts, so long as the rendering court had jurisdiction to render a final judgment on the merits.

Commercial Speech

Commercial Speech - 4 part test but most imp point is first one: o Most regulations of commercial speech are struck down - generally as a rule, regulations of commercial speech are struck down. So long as the advertising is truthful and informational, it must be allowed. o Test for commercial speech is version of intermediate scrutiny: Regulation of commercial speech must directly advance a substantial government interest and be narrowly tailored to that interest. Key thing is that test is substantial state interest, not compelling which is the usual test for free speech. Not merely significant, which is the test for content neutral time place and manner - its an in between standard, substantial state interest. Also note misleading commercial speech point: o Misleading speech: Misleading commercial speech (unlike political speech) may be prohibited. If commercial advertising would cause you to think as a reasonable consumer, that the product had qualities it doesn't have, if it would cause you to believe that the product can cure disease that its useless for - misleading commercial speech can be suppressed. Almost all nonmisleading commercial advertising must be allowed. And that includes attorney advertising.

Corporations

Corporations have the same First Amendment right of free speech to speak as individuals. Citizens united - supreme court held govt could not suppress polticial speech merely because speaker was a corporation.

endorsement

Endorsement o It is a violation of the Establishment Clause for the government to endorse one religion over another and also to endorse religion over non-religion. You not only have freedom of religion - freedom to choose any one you want without govt pressure - you also have freedom from religion, freedom to choose none of the above if that's your choice. endorsement idea sounds good but number of very obvious endorsements that we tolerate without much concern. Like in god we trust below. § But, many endorsements are upheld, such as "In God We Trust" on currency. o The Supreme Court really mainly wants to prevent coercive endorsement of religion (one that might override individual choice). that's the real nub of the problem - coercive endorsement of religion. Whats unconstitutional is govt endorsement of religion in a context that might prove coercive on an individuals conscience. That explaisn why school prayer is unconstitutional - any officially sponsored school prayer is unconstitutional, no matter how general the prayer or how nondenominational the phrasing. There will be some kids who don't believe it, and they may feel intimidated or coerced when everyone else stands up to pray. Officially sponsored school prayer is a coercive endorsement of religion. § The Establishment Clause prohibits government endorsement of religion in a context that might prove coercive on an individual's conscience. § Examples: • Officially-sponsored school prayer is unconstitutional. • Officially-sponsored graduation prayer is unconstitutional. However its phrased, there will be someone who doesn't want to say it - and that person may feel in the context of graduation ceremony, coerced. But truly voluntary student prayer - one not sponsored by govt - but simply volunteered by individual - that's okay. • Bible reading is permissible, but cannot be inspirational (e.g., as literature or poetry). In public schools can read bible as literature and poetry - as maybe extended genology. As maybe fouhdnataionl document of western culture. Can read bible for all those purposes but cannot read it for inspiration. The minute kids begin to take bible seriously as religious text, u have to cut out rreading in public schools. Its likehearsay rule - bible can be admitted to classroom for any purpose other than its truth. • Display of the Ten Commandments is sometimes okay. It can be displayed for secular purposes (e.g., historical or promoting morals), but not to inspire religious belief. Lot of cases about whether displays of ten commandments are endorsements of religion. Ppl argue theyre not because they have historical or secular purpose to promote morals rather than advance religion. Some uses of ten commandments have been upheld and others have not. You could teach ten commandments though as early legal code. You can say look how short they were then - no ban on having ten commandments as topic of discussion. But Cannot post the Ten Commandments in a classroom and leave it there every day of the school year— that's designed to inspire religious belief in ten commandments and would be struck down. o Can teach the Ten Commandments in school as an example of an early legal code. o Cannot post the Ten Commandments in a classroom and leave it there every day of the school year—designed to inspire religious belief. o Cannot post the Ten Commandments in a courthouse if the context makes plain that the purpose is to endorse religious belief. Main idea is that - if ten commandments are posted to endorse their religious content, that's an establishment of religion - posted by the government to endorse their religious content -thats an establishment of religion. If theyre posted for some other secular purpose, that's okay. • Laws prohibiting teaching evolution have been struck down. Probably because they fail test of mere rationality. • Legislative prayer is okay for historical practices. Supreme court said its okay for state legislature to have a chaplain say a prayer at the beginning of each session - very commonly done in state legislatures and in US congress. Its been done forever and court said historical practice seemed to matter. • Nativity scenes are okay on public property if there is something else there to dilute the religious message (e.g., Hanukkah symbols, Rudolph the red-nosed reindeer). Suprem court has wrestled with this - first time sc had cresh case, they said nativity scene okay because Christmas used to be religiously significant but no logner is - itd becomg shopping season. Even court grew ashamed of saying that and backed up to try again. Second time Court said u can have cresh on public property at Christmas with all the religious symbolism it includes if u have something else there with it - some other thigns there that dilute religious message. so if just nativity scene with jesus and mary and shephards, that would be struck down as endorsement of chritsianity. But if nativity scene and additions like Rudolph the rednose raindeer, the augmented nativity scene augmented by non Christian elements would be allowed.

example

Example 45: Congress passed the Millionaires' Amendment, which basically said that if your opponent spends a lot of his or her own money, then your contribution limit is automatically raised. So telling rich people if you spend your own money above a certain amount, then the rules for your opponent are changed. If you spend your own money above a certain figure, then contribution limits that are normally applicable don't apply to your opponent. That's restricting how much you can spend or making something depend on how much you spend in an effort to equalize the voice of the rich and poor. The Supreme Court struck it down. Court also struck down another scheme that said that public candidates who accept public financing can get extra money if their opponents outspend them.

example of properly remedial legislation

Example 17: The Religious Freedom Restoration Act (RFRA) Overview: Neutral, generally applicable regulations of conduct can validly be applied despite religious objections. Everyone can be made to obey the law whether they believe in it or not. Background: Religious individuals often claimed a right of accommodation of religious belief. They claimed that laws valid against everyone else could not be enforced against religious objectors unless the government showed a compelling interest. Called accommodation of religious belief because claim is that govt is required to accommodate my religious belief and treat me specially because of that belief. That claim was rejected by the Supreme Court, which held that neutral regulations of conduct could be enforced against anyone, including religious objectors. Congress disagreed with the Supreme Court and passed RFRA, which declared that religious believers have the right not to obey otherwise valid laws, unless the government can show a compelling interest in enforcing the law against them. The Supreme Court struck down RFRA, as it went beyond federal legislative power, because it was not really remedial. It was not narrowly aimed at the kinds of laws the Supreme Court had declared unconstitutional—laws aimed at religion. Congress could have made it a crime for the government to discriminate against religious belief. Congress could have said that any government or government official who discriminated against religious belief had to pay money damages. Congress could give the Department of Justice special enforcement powers to look into religious discrimination and make sure it is rooted out. All of these actions would have been reasonable remedies against a recognized constitutional violation of discrimination against religious belief. In RFRA, Congress said something different - it said even non-discriminatory laws, even laws showing no history or trace of anti religious motivation but that incidentally restricted religiously motivated conduct, were invalid unless the government showed a compelling interest. That, said the Supreme Court, was not an attempt to enforce the Free Exercise right against religious discrimination as it had been defined by the court. Rather, it was an attempt to overturn the Court's decision about what the Constitution did and did not protect. Therefore, RFRA was struck down as beyond the federal legislative power. Note very important point: RFRA was unconstitutional only as it applied to states and localities. Requiring states and localities to accommodate religious belief was found to be beyond Congress's power under the 14th Amendment. However, Congress is free to accommodate religious belief in the exercise of its undoubted federal powers (e.g., Congress's powers over the Army and the Navy, the District of Columbia, etc.). When Congress exercised its federal powers, it may choose to accommodate religious believers, but Congress has no power to force that policy on the states. Difference between remedy reasonably related to judicially recognized right and statute which goes beyond that. Distinction between legislation that remedies violatiosn of idnviidual rights as rights have been defind by the courts - that's okay. And legislation seeking to define individual rights in ways not embraced by the courts - which is not okay. That's only hard topic in entire subject of legislative powers. For questions on this topic most likely something that looks like RFRA. Remember these characteristics of that paradigmatic example - congress tried to reverse a supreme court interpretation of individual rights, congress's attempt to adopt a broader interpretation of individual rights was struck down as it applied to states and localities because congress has no power to legislate religious policy for states and localities but same legislation is sustained for federal govt because of congress's undoubted power over federal matters. NOTE - Be on the lookout for Congressional acts that mirror RFRA. Remember that such acts are generally valid as to Congress, but invalid as applied to the states and localities.

federal taxpayers always...

Federal taxpayers always have standing to challenge their own tax liability. However, taxpayers do not have standing to challenge government expenditures. Govt expenditures don't directly affect ur liability, no injury. - Narrow exception under the Establishment Clause: An establishment of religion challenge to specific congressional appropriations can be raised by any taxpayer. (Note also that state courts often allow municipal taxpayers to challenge a municipality's expenditures.) extremely narrow - must be establishment clause challenge - no other constitutional right will do and must be specific congressional appropriation under spending clause - no other kind of government benefit will do.

No state discrimination against out-of-state interests

How might state discriminate? By forbidding out of state products to be sold here - not allowed to do that. Other is by taxing out of state products at higher rate than local products. § Examples include: forbidding sale of only out-of-state products; taxing out-of-state products at a higher rate; requiring that manufacturing be performed in-state. Example 19: A state law said that metal ore mined in the state must be processed in the state (i.e., refined) before it could be shipped out of the state. The state law is unconstitutional because it discriminates against out-of-state metal refiners or processors. Cant have rule only allowing in state and forbidding out of state interests. Example 20: A law limiting landfills to in-state garbage is unconstitutional because it discriminates against out-of-state garbage. If landfills allowed to operate and receive garbage they must be allowed to receive in state and out of state garbage on same terms. § Hypothetically ok if there is a substantial state interest and no other non-discriminatory means is available; this is incredibly rare. 99% of time state discrimination against out of state interests is struck down under dormant commerce clause, even if only garbage. § 3 Exceptions to no discrimination against out-of-state interests: • State as Market Participant—When a state is buying or selling goods or services on its own or for itself, it can choose to deal with only in-state persons. o Examples include: Sale of cement produced by state-owned plant only to instate purchasers; garbage stored in state-owned landfill limited to in-state garbage; law requiring 50% local workforce on state-financed construction projects. For example, if state owned cement plant it can sell only to in state persons. If state owned landfill it can refuse to accept garbage from elsewhere. If state financed construction project it can require certain percentage of workforce be local. When state acting as a market participant, it can deal only with its own. It's the state as a government that's barred from discrimination. • Subsidies—A state can always choose to subsidize only its own citizens (for example, welfare benefits or in-state college tuition). Doesn't have to provide same benefits to citizens from other states. Can restrict welfare benefits, for example, to in-state citizens. same with in state tuition costs. Why is it fine? when u attend public university ur not paying all cost of education - ur education is being subsidized by state appropriation. And when state is spending money and subsidizing activity, it can choose to subsidize only its own. • Federal Approval—Remember, the Dormant Commerce Clause applies only in the absence of federal action. If Congress authorizes or consents to state regulation of commerce, nothing the state does will violate the Commerce Clause, even if it discriminates against out-of-state interests. Remember - the entire section about no discrimination against out of state interests applies in the absence of federal action. When congress has not legislated on point - but if congress authorizes or consents to state regulation of commerce, nothing the state does will violate the commerce clause - even if it discriminates against out of state interests. Common question - congress passed statute, statute authorizes states to regulate particular industry like insurance any way they like, a state then turns around and taxes out of state insurance providers at a higher rate than in state competitors - that's discrimination, it would be struck down in a heartbeat if state did it on its own but when congress authorizes or consents to it, then its okay under the commerce clause. Exam Tip 9: Commonly tested on the MBE

independent vs. coordianted expenditures

If you distinguish between contributions and expenditures, must also distinguish between independent expenditures and coordinated expenditures. 1. Independent expenditures (person spends her own money to support a political candidate maybe by buying tv ads, that's fine) versus coordinated expenditures (a disguised campaign contribution - when a supporter spends money as the campaign directs, for example campaign says pls pay bills for tv ads and they do it. May be structured to look like an expenditure by you but its actually a contribution): So expenditures coordinated with or controlled by the campaign - these are treated as contributions and can be regulated as all other contributions can be regulated. Expenditures that are coordinated with or controlled by camapgin - they tell u how to spend ur money to support them. Coordinated expenditurs are treated as contributins and can be regulated as all contributins can be. 2. Independent expenditures cannot be regulated. 3. A coordinated expenditure is a disguised contribution (the campaign is in control) and be regulated as contributions can be regulated. Law banning judicial candidates from personally soliciting campaign contributions - in many states, judges are elected and there was statute prohibiting candidates for judgeships from personally soliciting campaign contributions - the law was upheld based on the appearance of corruption. 4. Citizens united case - The constitutional protection of direct independent expenditures (that is you can make direct expenditures to support your point of view or the people who agree with you in amounts that are unlimited) applies not only to individuals, but also to corporations, including nonprofits, and unions. Citizens united case involved ideological group organized as a corporation that promoted mocumentary against a presidential candidate at the time - overtly political debate and court said it was constitutionally protected. 5. The Supreme Court has consistently rejected equalization of campaign resources as a valid rationale for restricting campaign expenditures. Most of people who want to put limits on campaign expenditures have as their essential reason - equalization - preventing person with lot of money from drowning out those who have less. Court has considered that and time and time again, court has rejected equalization of campaign resources as a rationale for restricting campaign expenditures. Idea that you want to restrict wealthy speakers in order to increase relative voice of others - that's consistently been a losing argument. Can see that in the corporate free speech case and it temps to give advantages to poorer candidates.

In general, a state can...

In general, a state is free to spend the revenue that it collects in any manner that is not arbitrary or irrational, but the state may not expend funds in a manner that violates the Constitution.

Naturalization Clause of Article I, Section 8

Naturalization Clause of Article I, Section 8, has been interpreted as a source of Congressional power over immigration and aliens. If anything, it would provide support for the constitutionality of this statute rather than support for a challenge to its constitutionality.

obscenity

Obscenity: Defined by the rule of "S" - most imp category of speech that can be regulated because of govts interest. 1. obscenity must be Sexy: Must be erotic; must appeal to the prurient interest. Must make you itch or long with desire. (Gore and violence are not legally obscene. May be disgusting but don't make you itch or long, theyre not sexy.) 2. obscenity also must make Society sick: Must be patently offensive to the average person in the society, so not to most refined and delicate amongst us but to the average person. The society may be the nation as a whole, or can be a particular state, or a major metropolitan area. But wherever you are, material must simultaneously make you itch or long and be sick. 3. obscenity must be defined by the proper Standards - if a law doesn't have right standards it will be invalid as vague and overbroad: Must be defined by the proper standards for determining what is obscene, not vague and/or overbroad. For example, full frontal nudity. Full frontal nudity might be obscene but some of its not the least bit sexy. So law banning al nudity is overbroad - goes too far and is unconstitutional for that reason. Example 41: Tax on films displaying frontal nudity. Not a valid law because frontal nudity is not a good enough standard. 4. obscenity must lack Serious value: The material must lack serious value. If material has serious value (artistic, scientific, educational, or political), it cannot be held legally obscene. This determination is made by the court, not the jury, and it must be based on a national standard, not a local one. Juries cant speculate on whether something has serious value. Example - film which was mainstream Hollywood one - maybe because of name or title of film "carnal knowledge" it was prosecuted against as being obscene in the state of Georgia. Supreme court said the film is not obscene, it was a mainstream Hollywood film, got great reviews, and it has artistic value so it has serious value. So good reviews probably means no obscenity. 4 s's - sexy, making society sick, must be defined by proper standards, lacking serious value - that defines law of obscenity but there are a few additional points to know below: - Footnotes on obscenity: 1. Minors: A lesser legal standard can be applied to minors, but the government cannot ban adult speech simply because it would be inappropriate for minors. A lesser legal standard can be applied to minors but only if the minors can be dealt with separately - issue cant restrict materials to everyone because it might be inappropriate to minors. But you can have special standards for matierlas distributed to minors. 2. Child pornography: Can be prohibited whether or not it is legally obscene, and possession can be punished even if it is in the privacy of your own home. Child porn is special case - al bets off for child porn. Child porn is sexual representation of children - it can be prohibited whether or not it is legally obscene - child porn can be prohibited even if it does have some artistic value. It can be prohibited whether or not it is legally obscene and even if u possess it in the privacy of your own home. Even in the privacy of your own home, child porn can be banned. 3. Land use restrictions: Narrowly drawn ordinances can regulate the zoning of adult theaters, but cannot ban them entirely. But they can regulate where they are placed. Noones quite sure whether its better to scatter them all around so no one neighborhood is overrun or whether its better to bring all the adult theaters into one neighborhood and just close it off. but whatever it is, u can zone adult theaters so long as u allow some place for them to exist. Note 6: Courts have recently begun to distinguish legally obscene speech from pornography. Merely establishing that speech constitutes pornography is generally insufficient to establish that the speech is obscene.

states cannot shield...

States cannot shield state officers from federal liability for violating federal constitutional rights. Exception: The anti-commandeering principle - States cannot be directly forced to carry out federal programs - state officers cannot be conscripted for federal duties - violates anti commandeering principle. The federal government can always use the spending power to bribe states to comply though, by making money available only if you do something and fed govt can adopt direct federal regulations administered by federal agents.

If life, liberty, or property is being taken by the government, what process is due?

Procedural due process is variable, and the types of hearings can range from casual to very elaborate. Due process can range from nothing so very minimal and casual procedures all the way to the elaborate procedures of criminal trial. procedural due process can mean a little or a lot. - To decide what kind of process is due, the courts balance three factors: 1) The individual interest at stake (life, liberty, property) - what kind of interest will it be? Life, liberty, or property- theyre the only ones that count.; 2) The value of the procedure in protecting that interest - whatever procedure being claimed or whatever person wants to have, u assess individual interest at stake and value of procedure in protecting that interest against the governments interest in efficiency; and 3) The governments interest in efficiency and cost - which usually means cost. Sometimes balance requires highly elaborate procedures as in criminal trials or imposing a fine. Sometimes it allows minimal procedures like disciplining a school child - not much in terms of a hearing required.

Standards of Review

Same for Due Process and Equal Protection • Strict scrutiny, i.e., is the law necessary for a compelling interest? • Intermediate scrutiny, i.e., is the law substantially related to an important government interest? • Rational basis, i.e., is the law rationally related to a legitimate government interest?

Necessary and Proper Clause - treaties

The Necessary and Proper Clause enables Congress to enact laws that carry into execution a treaty that's been negotiated by the President and ratified by the Senate in the exercise of their constitutional powers. Such laws are valid regardless of whether Congress has some other enumerated power that might be relevant—such as the Spending Power (here, asserted to fund activities that will promote the treaty's aims).

presidents power to pardon....

The President's power to pardon does not extend to impeachment of a federal official. However, there is no good faith restriction imposed by the Constitution on the President's exercise of the power to pardon. While a President may not pardon a person with respect to future acts, a President may pardon a person at any time after the offense has been committed.

racial discrimination - whats required to trigger strict or intermediate scrutiny?

To trigger strict or intermediate scrutiny, there must be discriminatory intent on the part of the government. The fact that legislation has a disparate effect on people of different ethnic minorities, without intent, is insufficient. Discriminatory intent can be shown facially, as applied, or when there is a discriminatory motive. A law that is neutral on its face and in its application may still result in a disparate impact. By itself, however, a disparate impact is not sufficient to trigger strict or intermediate scrutiny; proof of discriminatory motive or intent is required to show a violation of the Equal Protection Clause. Here, there is clearly a disparate impact, but there is no evidence of a discriminatory motive or intent. Therefore, rational basis would apply.

fundamental rights

Triggers the strict scrutiny test under both due process and equal protection 1. Travel o We have a fundamental right of interstate travel and settlement. Right to go to other states in country and right to settle there and become full fledged citizen. o States can impose reasonable residency requirements for political participation (requiring u live there for 30 days for example before you vote) or for government benefits. - Most reasonable residency requirements are 30-90 days - One year is too long for everything except in-state tuition and jurisdiction to issue a divorce. 2. Voting and Ballot Access a. Voting is a fundamental right to all citizens age 18 and over. Not right for noncitizens though. - Poll taxes are unconstitutional because they burden the fundamental right to vote. Taxes on exercise of ballot - tax paid to vote. - Short-term (e.g., 30 days) residency requirements are permitted. Anything over 90 days runs into trouble. b. Ballot access - has looser rules. - States can impose residency requirements for candidates to be listed on a ballot, such as longer residency, filing fees, and nomination petitions, so long as serious candidates can reasonably comply. Don't want ballot to be so long or confusing or with so many fake names that no one knows what it means. So states can do what they want as long as requiremnets allow reasonable, serious candidates can comply. - If the requirements become so onerous that they effectively bar access to the ballot, then they are unconstitutional. 3. Privacy - amorphous concept - includes whats below: a. Marriage - There are all sorts of requirements for marriage (e.g., age or restriction on marrying close relatives), but substantial interference with the marriage—including same-sex marriage—is unconstitutional. b. Contraception - It is a fundamental right for everyone, whether married or not, to purchase contraceptives. Everyone has fundamental right to buy contraceptives - true for unmarried and married persons, and old and young people. c. Sexual intimacy - Perhaps not technically a fundamental right—the Supreme Court found that the government has no legitimate interest in regulating non-commercial sexual intimacy between consenting adults, including same-sex couples. Not quite same as sayig it's a fundamental right, but saying even if not fundamental right, govt regulation fails because govt has no legitimate interest in regulating non-commercial sexual intimacy between consenting adults and that includes same sex couples. d. Abortion - Roe v. Wade: A woman has a right to terminate her pregnancy until viability of the fetus. Viability means the fetus can live outside the womb. After that stage, restrictions can apply so long as there are exceptions to preserve the life and health of the mother. - States can regulate abortion in a variety of reasonable ways, but they cannot impose an undue burden on the woman's right to terminate her pregnancy. - Informed-consent requirements are allowed (you can require before person has abortion, that person be informed about opportunities available for carrying child to term, and having it adopted), Twenty-four hour waiting periods are allowed (we don't want abortions to be performed upon spur of moment so can have a system requiring woman sign consent form and wait 24 hours for procedure incase she changes her mind), Parental notification requirements (for minors) are allowed (obviously not for adult females, but for minors), Parental consent requirements are generally not allowed (Govt can require underaged minor to notify her parent of intention to have abortion - maybe gives parent opportunity to voice an opinion. But parent cant be given veto over girls decision to have abortion. Parents consent cannot be required), Spousal consent requirements are not allowed (No law requiring consent of spouse has been upheld), Government financing of abortion is not required (Govt never has to pay for abortions, even if it provides funding for live births. Govt can refuse to pay for abortions and can refuse to allow abortions to be performed in facilities it funds. Govt has to allow abortion but doesn't have to pay for it). e. Parental rights - Parents have a fundamental right to raise their children as they see fit, including the choice of religious or private schools. - Can lose their rights through abandonment, abuse, or neglect - until parents' rights are severed because of some kind of misconduct, parents have a fundamental right to raise kids as they see fit. f. Family relations - fundamental right to live with close relatives if they want you to. - Includes the right to live together with close relatives g. Obscene material - Fundamental right to read obscene material in the privacy of one's own home. - However, no fundamental right to purchase, sell, import, or distribute such material. - Right to possess obscenity in ones home does not apply to child porn. For child porn completely banned. h. Refusal of medical treatment - Not clear whether this is a fundamental right, but there is a liberty interest in refusing medical treatment - No fundamental right to commit suicide

Preempting the Field

When Congress determines that there should be no state law of any sort in a particular field, then any state law in that area is inconsistent with the federal statute and is preempted. This is rare.

Congress limiting the jurisdiction of the federal judiciary?

While Congress may limit the jurisdiction of the federal judiciary, Congress may not exercise this power in a manner that violates the Constitution.

Congress's power under the Commerce Clause extends to....

environmental laws such as an auto-recycling statute.

General rule of thumb -

law denying fundamental right to everyone violates due process. law denying fundamental right to some people and not to others violates equal protection.

Only time, place, and manner may be....

regulated in a public forum.

The War and Defense Powers

o Congress has the power to declare war and the power to maintain the Army and Navy. o Congress has the power to provide for military discipline of U.S. military personnel. o Congress can provide for military trial of enemy combatants and enemy civilians. o Congress cannot provide for military trials of U.S. citizens who are civilians. They must be tried in article 3 courts.

Deficiency of Lemon Test

o Trouble is its Exceedingly difficult to apply - hard to know what is and what is not constitutional under the test. o It once was interpreted to condemn aid to religious primary and secondary schools, but neutral aid is now allowed (goes to everyone). As long as neutral and goes to everyone - all private schools, all religious, all non religious schools - and govt isn't making decision about who gets aid and who doesnt - then its fine. All govt has to do today is say we will give each parent money that the parent can use to send her child to a private school, whether religious or not. That's neutral and today it passes muster. § The government gives aid to parents and the parents are allowed to send their children to the schools of their choice. § The parents are making the decision as to whether a religious school gets the money. • The government is not picking and choosing recipients. In recent years court has spoken more about endorsement and less about lemon test.

IMPORTANT ANALYSIS - the Privileges or Immunities Clause of the Fourteenth Amendment

the Privileges or Immunities Clause of the Fourteenth Amendment does not apply to aliens but only to United States citizens; further, this clause applies only to the states, not the federal government.

city regulating ballot access

while it is true that a city has a legitimate interest in regulating ballot access, there are limits to this regulation. For example, a city cannot require that a candidate own property and restrictions cannot be discriminatory.

No regulation of wholly extraterritorial activity

§ A state may not regulate conduct occurring wholly beyond its borders. Example 21: Connecticut tried to legislate that beer sold in Connecticut could not be sold at a price different from the price of beer sold in other states. Connecticut can legislate price of beer in its own state, but it cant make the lawfulness of beer sold in its state depend on the prices in new York. Supreme court said that's attempt to pass extraterritorial regulation.

Relations Among States

• Interstate compacts are agreements among states. States can make interstate compacts, but if the compact affects federal rights, Congress must approve. • Full Faith and Credit Clause: States do not have to follow other states' laws, but they do have to give full faith and credit to judgments rendered by other states' courts, so long as the rendering court had jurisdiction to render a final judgment on the merits.

STATE TAXATION OF INTERSTATE COMMERCE, PREEMPTION, AND RELATIONS AMONG STATES - State Taxation of Interstate Commerce

• Generally, the requirements are the same as for any other state regulation of commerce: Discriminatory taxation will be struck down unless Congress consents to it, and non-discriminatory taxation (simply means in state and out of state taxpayers are treated alike) will be upheld unless it is unduly burdensome which is rare. o Sometimes not obvious what constitutes discrimination in field of taxation - so two particular requirements for non-discirminatory taxation of out of state interests. Non-discriminatory taxation is valid if the following two requirements are met: 1) Must be Substantial nexus between the taxing state and the property or activity to be taxed Substantial nexus previously was interpreted to require the physical presence of the seller in state. Until recently a state could levy a sales tax, one that the seller is forced to collect, only if the seller had some physical presence in the state beyond merely delivering goods there. That meant interstate marketers, like amazon, unless happening to have physical facilities in the state, could not be forced to collect sales tax on goods sold over the internet. Supreme court changed its mind and said old rule was invalid because court did not then have before it the present realities of the interstate marketplace. Because court had not foreseen in 1992 just how large internet based commerce would become. Under new rule substantial nexus still required for imposing sales tax collected by seller but substantial nexus no longer requires physical presence. At least where the out of state seller does some nontrivial amount of business in state - it can be required to collect a sales tax on in state purchases even if all the physical facilities are located elsewhere. • "Substantial nexus" does not require any kind of physical presence of seller in state. In general interstate sales by internet marketers can be subjected to sales tax whether or not theres physical activity in the state. 2) other requirement - Must be a fair apportionment of tax liability among states. Imagine international corporation doing business all over US and all over world. Example 22: A state may tax the in-state portion of a corporation's revenue because theres a substantial nexus between taxing state and corporation doing business there but it cannot tax the entire world-wide activity of the corporation - that would be piggy and would interfere with taxing interest of other jx. Instead the state must adopt some form of fair apportionment of tax liability among states - fair apportionment of tax liability among states - can tax portion of corps revenue that is equivalent to the in-state activity of revenue. Now technically in real world state taxation of out of state businesses can become very complicated because of exactly how u administer idea of fair apportionment. But on bar exam - simple - just substantial nexus which now includes delivering goods in state and some scheme of fair apportionment among taxing states. • Ad Valorem Property Taxes (value-based property taxes) - for ad valorem taxes on personal property key is to distinguish between commodities in interstate commerce and the instrumentalities of interstate commerce - they are taxed differently. Commodities are goods moving from state to state - instrumentalities are transportation moving goods like railroads, trucks, airplanes, etc. o Levied on personal property o Distinguish between two kinds of personal property because they are taxed differently: § Commodities are the goods that move from state to state. • States usually tax all commodities within their borders on a specified date (called tax day), but not goods that are merely in transit—the commodities have to come to rest in the state. Cannot tax goods merely in transit - merely on their way through the state to another destination. • Rule: for commodities in interstate commerce, pay the full tax to every state where goods are stopped for a business purpose on tax day. No taxes are due where they are merely passing through. § Instrumentalities are the transportation equipment that moves commodities (railroads, trucks, airplanes, etc.). for instrumentalities, each state can tax its fair share of an instrumentality used in that state.taxpayers domicile can tax entire value of instrumentality but domicile then has to give credit for taxes paid elsewhere. So end up with scheme of fair apportionment of tax liability among those states with a substantial nexus to the instrumentality of interstate commerce. • Fair apportionment of tax liability among states with a substantial nexus to the instrumentality. • Each state in which an instrumentality is used can tax the value of that instrumentality. Example 23: A long-haul trucker drives his rig from Maine to California. Every state through which the truck passes can tax the truck itself but each state can only tax a portion of the truck's value, roughly equal to the portion of in-state use. The state of the taxpayer's domicile taxes the full value of the truck, then turns around and gives a credit to every state that asserts taxation when the truck uses its highways. You end up with a scheme of fair apportionment of tax liability among those states with a substantial nexus to the truck. The commodities in the truck cannot be taxed if they are merely passing through; they must stop for a business purpose on tax day.

Takings

• Private property shall not be taken for public use without just compensation. simply property govt doesn't own yet. o Public use—doesn't have to be good use or something specifically benefiting public, just is basically anything the government wants to do with the property. It need only be rationally related to a concievable public purpose. So argument that public use isn't good idea will go nowhere. It doesn't have to be good idea, just has to be rationally related to a conceivable public purpose.This includes taking private property to resell to another private owner for purposes of economic development. Anything govt wants to do with ur property is a public use. o Just compensation—fair market value at the time of the taking. • Taking—This is where the controversy often occurs. Not $200 ur grandparents paid for farm, but 2 million its worth now. Defining issue is whether theres been a taking - which is event for which compensation is required - or whether theres merely regulation of property - for which compensation is generally not required. o Taking versus regulation—If there is a taking of property, compensation is required; if there is a mere regulation on property, compensation is not required, even if the regulation reduces the value of the property. o Economic impact—The adverse economic impact of the government's action does not necessarily mean there has been a taking (e.g., a new prison built next door to a beautiful, countryside home - value of ur property plummets but no taking so they don't owe u any compensation). Many regulations can dramatically affect the value of property but that does not trigger a right to compensation. o Physical occupation—This is the key question. If the government physically occupies a private owner's property, then a taking has occurred and it owes just compensation. Key is physical occupation - if govt physically occupies your property they owe you compensation because it's a taking. § If the government physically occupies only a tiny portion of your property, it is still a taking and they owe you compensation. o No physical occupation—Generally, no physical occupation means that no taking has occurred. Even if they restrict use of ur property in way making it less valuable. • Zoning - govt doesnt physically occupy property or move in but they pass zoning laws restricting use—generally no compensation required for zoning decisions - Not a taking and no compensation is required, so long as the zoning advances legitimate interests and does not extinguish a fundamental attribute of ownership. • Regulatory Taking—A zoning regulation can be considered a taking when it leaves no economically viable use for the property (rare). Can there be taking without physical occupation? Can there by regulatory taking where govt does owe u money even without physical occupation? Yes it can happen but only where regulation leaves no economically viable use for the property. If govt passes regulation leaving u no economically viable use for ur property that can be taking and they owe u compensation but that's extreme and rarely happens. Example 32: Two adjacent beachfront lots were purchased for the purpose of building a vacation home. After the lots were purchased, a new zoning law was passed, which stated that no permanent structure could ever be built on the property for environmental reasons. The Supreme Court said that no economically viable use was left for the owner, so this constitutes a taking and compensation is required. Not just that he lost money or would make less money now - but he had been wiped out in that possession. But note this is exceptional case - most of time only physical occupation triggers taking and just compensation. Usually no physical occupation means no taking and no right to compensation. • Development permits—Development is often conditioned on "concessions" by the developer, such as building an access road or donating land to a park. Such extractions are valid so long as they can be seen as offsetting the adverse impact of the development. Generally okay to extract concessions from developer if what is demanded from the developer can be seen as offsetting the adverse impact of the development. As long as govts conditions seen as merely forcing the developer to bear cost of development, that's okay and that's usually true. Only if govt coerced u to give up land completely unrelated to impact of ur development might there be a taking.

PRIVILEGES & IMMUNITIES, TAKINGS, AND PROHIBITED LEGISLATION - Takings

• Private property shall not be taken for public use without just compensation.simply property govt doesn't own yet. o Public use—doesn't have to be good use or something specifically benefiting public, just is basically anything the government wants to do with the property. It need only be rationally related to a concievable public purpose. So argument that public use isn't good idea will go nowhere. It doesn't have to be good idea, just has to be rationally related to a conceivable public purpose.This includes taking private property to resell to another private owner for purposes of economic development. Anything govt wants to do with ur property is a public use. o Just compensation—fair market value at the time of the taking. • Taking—This is where the controversy often occurs. Not $200 ur grandparents paid for farm, but 2 million its worth now. Defining issue is whether theres been a taking - which is event for which compensation is required - or whether theres merely regulation of propert - for which compensation is generally not required. o Taking versus regulation—If there is a taking of property, compensation is required; if there is a mere regulation on property, compensation is not required, even if the regulation reduces the value of the property. o Economic impact—The adverse economic impact of the government's action does not necessarily mean there has been a taking (e.g., a new prison built next door to a beautiful, countryside home - value of ur proeprtu plummets but no taking so they don't owe u any compensation). Many regulations can dramatically affect the value of property but that does not trigger a right to compensation. o Physical occupation—This is the key question. If the government physically occupies a private owner's property, then a taking has occurred and it owes just compensation. Key is physical occupation - if govt physically occupies your property they owe you compensation because it's a taking. § If the government physically occupies only a tiny portion of your property, it is still a taking and they owe you compensation. o No physical occupation—Generally, no physical occupation means that no taking has occurred. Even if they restrict use of ur property in way making it less valuable. • Zoning - govt doesnt physically occupy property or move in but they pass zoning laws restricting use—generally no compensation required for zoning decisions - Not a taking and no compensation is required, so long as the zoning advances legitimate interests and does not extinguish a fundamental attribute of ownership. • Regulatory Taking—A zoning regulation can be considered a taking when it leaves no economically viable use for the property (rare). Can there be taking without physical occupation? Can there by regulatory taking where govt does owe u money even without physical occupation? Yes it can happen but only where regulation leaves no economically viable use for the property. If govt passes regulation leaving u no economically viable use for ur property that can be taking and they owe u compensation but that's extreme and rarely happens. Example 32: Two adjacent beachfront lots were purchased for the purpose of building a vacation home. After the lots were purchased, a new zoning law was passed, which stated that no permanent structure could ever be built on the property for environmental reasons. The Supreme Court said that no economically viable use was left for the owner, so this constitutes a taking and compensation is required. Not just that he lost money or would make less money now - but he had been wiped out in that possession. But note this is exceptional case - most of time only physical occupation triggers taking and just compensation. Usually no physical occupation means no taking and no right to compensation. • Development permits—Development is often conditioned on "concessions" by the developer, such as building an access road or donating land to a park. Such exactions are valid so long as they can be seen as offsetting the adverse impact of the development. Generally okay to extract concessions from developer if what is demanded from the developer can be seen as offsetting the adverse impact of the development. As long as govts conditions seen as merely forcing the developer to bear cost of development, that's okay and that's usually true. Only if govt coerced u to give up land completely unrelated to impact of ur development might there be a taking.

EQUAL PROTECTION (RACE, ETHNICITY, AND NATIONAL ORIGIN) - Suspect Classifications (Trigger Strict Scrutiny and Arise under Equal Protection)

• Race, Ethnicity, or National Origin—laws that disadvantage minorities will be struck down. o However, discriminatory purpose is required: For constitutional purposes, a law is a racial classification only if the plaintiffs show it has a discriminatory purpose (not enough to show a disproportionate impact). Rule is quite diff under many anti discrimination statutes - they usually only require showing of disproportionate impact - that law affects whites and blacks differently, but constitution requires showing of discriminatory purpose. Example 28: Economically restrictive zoning in Arlington Heights—a developer wanted to come in and build a large multi-family housing complex. Developer argued that the two-acre, single-family zoning ordinance has the effect of restricting the town to wealthy white people; he argued that discriminated against minorities. The Supreme Court said the zoning law was not unconstitutional as racial discrimination unless you can show it was done for the purpose of discriminating against minorities. Developer came to court and said ur honor the effect of this 2 acre minimum zoning law - effect of very restrictive zoning law is to keep town all white - u should strike it down as racial classification. Sc said that may or may not be effect but it doesn't matter - the zoning law is not unconstitutional as racial discrimination unless done for purpose of excluding minorities. U have to show discriminatory purpose to show racial classification under the constitution. § How to show Discriminatory purpose: May be explicit on the face of the statute, or may be proved by a history of discriminatory application (If apparently neutral law only invokved against racial minorities, if demonstrated history of discriminatory application that can revela a discriminatory purpose even tho face of statute is clear), or by extrinsic evidence about the purposes of those who passed the law (so can use all kinds of evidence to show discriminatory purpose exists, but u must show that discriminatory purpose exists in order to show a racial classification). § School desegregation: De jure (by law) segregation is unconstitutional. De facto segregation is not (usually by residential housing patterns). Just another way of saying discriminatory purpose is required - de jure segregation is segregation by law, segregation on purpose - that's what makes it de jure and its unconstitutional. De facto segregation is separation of races where no discriminatory purpose can be proved. De facto segregation is not unconstitutional - just another way of saying discriminatory purpose is required. o Affirmative action - used to describe acts designed to benefit a racial or ethnic minority. § It is a racial classification; an effort to benefit a racial or ethnic minority. § Triggers strict scrutiny and requires a compelling interest - for affirmative action u usually find compelling interest. For example, Specific past discrimination - Affirmative action is valid when it specifically corrects past discrimination by the specific department or agency now engaged in affirmative action. Example - police dept has long history of discriminating against african American candidates for employment and now forced to set plan to set history right and to prefer African American applicants until such time as police force has racial and ethnic balance it should have had all along. That's fine, that affirmative action is fine - because it specifically corrects past discrimination by that particular institution or agency. Affirmative action is always allowed when it specifically corrects past discrimination by that institution or agency. But must be quite specific as seen with example below. Example 29: New Haven Firefighters case—Nineteen firefighters sued the city of New Haven when they had all scored high enough on a standardized test to be promoted to management positions. City officials abandoned the test results because minority firefighters had not scored high enough to be considered for these positions, and then the city hired minorities anyway for these spots. The Supreme Court said the city's actions looked like a racially motivated quota and struck it down as being unconstitutional. Lack of specific connection to past misconduct of that agency was responsible for that decision. Affirmative action not valid to correct general societal discrimination. • General societal discrimination does not justify affirmative action. Not good enough to say our nation has a bad past and we want to do something about it. Affirmative action has to correct specific past discrimination by that institution or agency. § Affirmative action is allowed in the context of preferential admissions to colleges and universities. • Preferential admissions based on race, ethnicity, ethnic group, etc. are allowed if necessary to achieve a diverse student body and diversity is essential to the education. o school has to show - Must be a strong showing that racial preferences are essential to achieving a diverse class. o Racial preferences must be "holistic" (can be built into an evaluation if every student is evaluated individually in a holistic way) and flexible. means if race taken into account along with other factors. o Rigid Quotas are not allowed. o Separate admissions tracks or procedures for minority applicants are not allowed. • Preferential admissions not allowed for secondary schools (though schools may be located and attendance zones created to maximize diversity).

Suspect Classifications

(Trigger Strict Scrutiny and Arise under Equal Protection) 1. Race, Ethnicity, or National Origin—laws that disadvantage minorities will be struck down. However, discriminatory purpose is required: For constitutional purposes, a law is a racial classification only if the plaintiffs show it has a discriminatory purpose (not enough to show a disproportionate impact). 2. Affirmative action - used to describe acts designed to benefit a racial or ethnic minority. a. It is a racial classification; an effort to benefit a racial or ethnic minority. b. Triggers strict scrutiny and requires a compelling interest - for affirmative action u usually find compelling interest. Affirmative action is always allowed when it specifically corrects past discrimination by that institution or agency. But must be quite specific -Affirmative action is valid when it specifically corrects past discrimination by the specific department or agency now engaged in affirmative action, not valid for general societal discrimination. 3. Alienage—Requirement of U.S. Citizenship - Classifications based upon U.S. citizenship are generally suspect classifications that require a compelling interest, but two important exceptions to strict scrutiny for alienage apply: a. Federal government • Congress has plenary power over citizenship, immigration, and naturalization. • Federal classifications based on U.S. citizenship do not trigger strict scrutiny. • Federal classifications are valid unless arbitrary and unreasonable. Otherwise theyre upheld if not arbitrary and unreasonable. b. State and local participation in government functions • These are jobs that have a particular relevance to the role of government and non-U.S. citizens can be barred from these jobs. • States and localities may require U.S. citizenship for participation in government functions, including voting, serving on a jury, and working in any kind of government law enforcement position (including probation and parole officers), or as a public school teacher. o However, states and localities cannot require U.S. citizenship for access to private employment or for government benefits - 2 contexts where alienage treated as suspect classification and law restricting private employment or govt benefits will be struck down.

Speech by Government Employees

- General rule: Government employees generally cannot be hired or fired based on political party, political philosophy, or any act of expression such as letter to the editor. This is true for everyone including run of the mill govt workers like police offices, public school teahcers, transit workers, etc. - they can be fired for not doing their jobs - fact that u disagree with govt policy doesn't give u the right not to follow it. That is u can be fired if u disrupt the work place. You can be fired if you dotn carry out your emplokymetn instructions. But cant be fired simply because you belong to the wrong party or endorse the wrong views. None of this applies to confidential or policy making employees - these are upper level employees - policy makers are like cabinet officials, agency heads - confiddental employees are their immediate advsors. And confidential and policy making employees can be hired and fired based on loyalty and politics. - Can be fired for disrupting the workplace or not doing their jobs. Exception: This general rule does not apply to confidential advisors or policy-making employees (e.g., the President's cabinet officers).

Significant state involvement:

- Government cannot facilitate private discrimination. - Government cannot profit (or be financial partner profiting from) from private discrimination. - Government cannot enforce a private agreement to discriminate. - But, government is not required to prevent private discrimination. Generally, the government acts constitutionally so long as its own conduct is neutral and even-handed (e.g., trespass laws, liquor licenses for private clubs).

Breakdown of loss of life, liberty, and property

- Life: Death penalty requires procedural due process - Liberty: Physical confinement, probation and parole, physical injury (such as a spanking in school), any restriction on legal rights (including being punished for free speech). Even someone on parole or probation has limitations on where they can go, etc. so limitations on them so procedural due process is required. Physical injury is also loss of liberty - if school child is spanked he has loss of liberty - not much procedural due process but physical injury is still loss of liberty. Any restriction on ur legal righs is also loss of liberty - if ur punished for exercising free speech, if some adverse action taken against u by govt because u belong to wrong party or u wrote letter to edior or some other exercise of free speech, uve lost liberty. Injury to reputation is not a loss of liberty. Without more, it's not loss of liberty. If prosecutor says were really keeping our eye on you because we think ur engaged in illict activities, that statement may damage ur reputation, conceivably maybe defamation claim, but its not loss of liberty and doesn't trigger procedural due process. - Property: Out of the three, property is the most heavily tested. Most of the questions in this area concern government jobs or benefits. Most property is clear - owning house or car, etc. if they take that stuff, they must give u procedural due process. what about govt jobs and benefits? Can u have property right in govt employment or some kind of govt benefit? Answer is yes. - You have a property interest in your government job or benefit whenever you have a legitimate entitlement to continued enjoyment of the job or benefit. Government jobs are entitlements only when the government says so—such as by providing a contractual term or discharge only "for cause." - A mere expectation of continued employment or benefit does not suffice. - Most government benefits, such as welfare, Medicaid, social security, are entitlements, and hence property. Statutes say if ur this old or this income level , or meet this criteria ur entitled to this benefit - that's property interest and due process is required if they take it away.

commerce power

1. Almost anything can be regulated as interstate commerce. 2. Congress can regulate: - The channels of interstate commerce (highways, seaways, airways, etc.); - The instrumentalities of interstate commerce (cars, trucks, railroads, ships, etc.); and - Intrastate (occurring within one state) (and interstate) activity (economic or commercial) that has a substantial effect on interstate commerce. 3. Substantial effect is judged in the aggregate. The question is not whether ur activity in isolation would have a substantial effect on interstate commerce, but whether the aggregate activity of everyone doing the same thing has a substantial effect on interstate commerce, and the answer is almost always yes. Posing question in aggregate means answer is almost yes. Almost impossible to think of economic activity that wouldn't have substantial effect on interstate commerce If all of us did it at once. But for non economic activity or non commercial activity, substantial effect on interstate commerce must be proved. 4. Not within the Commerce Clause: For non-economic, non-commercial activity, Congress can regulate intrastate actions only by actually demonstrating a substantial effect on interstate commerce (substantial effect is presumed for economic or commercial activity). Thus, Congress cannot regulate purely intrastate, non-commercial, non-economic activity for which a substantial effect on interstate commerce has not been shown. 5. Most important thing is to remember way to think of it - for economic and commercial activity, substantial effect on interstate commerce is easy, presumed, and always found. For noneconomic and noncommercial activity, must actually demonstrate substantial effect on interstate commerce. sometimes can be done and sometimes not.

Powers of the President—Foreign Affairs

1. Commander in Chief: The President has control over military decisions, although Congress has exclusive power to declare war. - Congress can cut off funding to military actions. Congress can reign in foreign involvement by president by cutting off funding for military activities. 2. Treaties and Executive Agreements - Treaties are negotiated by the President, but require approval by a 2/3 vote of the Senate. Once a treaty is ratified (approved), it has the same authority as a statute. Treaties should be distinguished from executive agreements - Executive agreements are presidential negotiations not submitted for approval by the Senate. They can be authorized, precluded, or overridden by statute, but they take precedence over conflicting state laws. They do not have the binding status of a treaty.

Anti-Commandeering

1. Congress cannot force states to adopt or enforce regulatory programs. It cannot commandeer state and local agencies to implement federal programs. Example 16: The Brady Gun-Control Act required background checks for people wanting to purchase firearms, and it required that state and local law enforcement carry out those background checks. The Supreme Court said this was unconstitutional. Congress cant force states to adopt federal regulatory programs. 2. What can Congress do to enforce regulatory programs? a. It can bribe states through use of the spending power. Congress can say were going to make a lot available money for law enforcement and u get it only if you do the thing we want about background checks. b. It can adopt its own regulatory program and enforce it with federal officers. All congress cannot do is force the state to carry out federal programs.

The War and Defense Powers

1. Congress has the power to declare war and the power to maintain the Army and Navy. 2. Congress has the power to provide for military discipline of U.S. military personnel. 3. Congress can provide for military trial of enemy combatants and enemy civilians. 4. Congress cannot provide for military trials of U.S. citizens who are civilians. They must be tried in article 3 courts.

Fourteenth Amendment

1. Congress has the power to remedy violations of individual rights by the government, but only as those rights have been defined by the courts. - Does not enable Congress to redefine constitutional rights by legislation - Only permits Congress to adopt remedies reasonably designed to enforce individual rights as they have been defined by the courts. 2. To be properly remedial, the legislation must have "congruence" and "proportionality." That is, there has to be a reasonable fit or reasonable closeness between the remedial law enacted by Congress and the constitutional right as defined by the Supreme Court. this is subtle concept and not either simple nor obvious - best way to explain is with example below.

Three-Part Test of Lemon v. Kurtzman - test has never been formally overruled:

1. Does the law have a secular purpose - non religious purpose? 2. Does the law have a primary effect that neither advances nor inhibits religion? 3. Does the law avoid excessive government entanglement with religion?

Few categories of speech that can be regulated because of content but quite exceptional.

1. Expressive Conduct (a.k.a. Symbolic Speech) - lot of conduct may be symbolic of some expression. What to do about that? Such laws upheld if regulation furthers an important interest. o Laws regulating expressive conduct are upheld if: § They further an important interest; § That interest is unrelated to the suppression of expression; and § The burden on expression is no greater than necessary. Key is whether govts interest is unrelated to suppression of expression, in which case law will likely be upheld or whether govts law is actually aimed at suppressing speech in which case law will be struck down. Example 34: Laws against flag desecration are unconstitutional because they suppress an expressive type of speech to show disapproval. Theyre aimed at a particular message - ppl were allowed to use flag as symbol to express patriotism or admiration for country. They were allowed to use flag as a symbol to express affirmative ideas, but they were not allowed to use the flag to express their hatred or contempt for the country. Thus a law that prohibits any desecration of American flag was aimed at suppressing a particular kind of message and was therefore struck down. Flag desecration laws apply to your flag, obviously you have no right to do something with someone elses flag, including the govt. but laws against flag desecration apply to all flags, including those u have bought and paid for and they were struck down. Contrast public nudity - ban on public nudity is constitutional. Does ban on public nudity incidentally burden a certain kind of expression? Maybe but the law against public nudity was aimed at public order effects of nudity in public places - not at trying to suppress a particular message. Example 35: Laws against public nudity are constitutional because public order must be kept, which is unrelated to speech. o The key: If the government is trying to suppress a particular message by regulating some kind of expressive conduct, then the law will be struck down; if the government is trying to pursue an interest unrelated to the suppression of expression, then the law will be upheld. 2. Vagueness and Overbreadth - technically terms are distinct. o Vagueness: Vague laws are ones that give no clear notice of what is prohibited and thus violate due process. o Overbreadth: Overbroad laws are ones that go too far in regulating speech. These laws burden substantially more speech than is necessary to protect a compelling interest and thus violate the First Amendment. Example 36: Laws that prohibit all nudity in drive-in movies are overbroad. Some nudity in drive in movies is legally obscene and obscenity can be banned. But much nudity in drive in movies is not legally obscene, but merely very large so law banning all nudity in drive in movies is overbroad - it goes substantially beyond a compelling interest and is therefore invalid. Exam Tip 12: Vagueness and overbreadth are almost always seen together on the bar exam. That is its almost inconceivable that you'll be required to distinguish between an answer saying vague and an answer saying overbroad. More likely one of 4 answers will say law Is vague and overbroad - in fact they overlap. On ground and in real world, vagueness and overbreadth overlap and on bar exam typically seen together. 3. Prior Restraints - even when speech might be constitutionally punished after the fact, prior restraints are especially disfavored. Means almost impossible to get an injunction agasint publication. Really has to be extraordinarily strong case to get injunction against publication because prior restraints are disfavored. o Are especially disfavored and will be struck down even when other forms of regulation might be upheld. o Injunctions against speech are almost impossible to get.

Defamation

1. False statements of fact (not opinion) damaging to a person's reputation can be prohibited. Critical that defamation concern a false statement of fact. An opinion, however unpleasant, cannot be defamatory. If u said someone is worst teacher ever, that cannot be defamation because its ur opinion. If u say I saw someone stealing money from till somewhere, that's a factual statement which if not true can lead to liability for defamation. 2. Public officials (Self defining category - those who hold govt office) and public figures (less clear category - people who have standing and reputation and prominence in the community) can recover for defamation only on proof of knowing or reckless falsity. Reckless means newspaper or publisher consciously recognized risk that material was false and published it anyways. 3. Private plaintiffs (people who don't hold office and who aren't terribly prominent) can recover on proof of negligent falsity. Defamation involves false statements of fact - many contexts in which false statements of facts are punished - perjury is one example - lie under oath. Any false of statement of fact cannot be punished though. Example 42: United States v. Alvarez: The Supreme Court struck down the Stolen Valor Act. The Stolen Valor Act made it a crime to falsely claim receipt of military declaration or medals. The Act was struck down by the Supreme Court as applied to a local politician who lied about receiving the Congressional Medal of Honor. The Justices held that the fact that the statement was a blatant self-serving lie did not exempt it from First Amendment protection. Stolen valor act was content based - not one of 5 exceptions - so it was unconstitutional. The prohibition was content-based and was not supported by one of the five recognized categories of unprotected speech. The statute was unconstitutional. Not the whole category of false statements of fact that are outside the first amendment - but it's the narrower category of defamation that's outside the first amendment.

preemption

1. Federal law preempts (overrides) inconsistent state law. Any kind of federal law can pre-empt - can be constitution itself or can be fed statute. Can be administrative regulation or even executive order - any valid federal law overrides state law if inconsistent with or incompatible with federal law. 2. State law is not preempted simply because it addresses the same subject matter or topic as a federal statute. Its preempted only if inconsistent with fed statute - There must be incompatibility or conflict.

Regulation of Association

1. Freedom of association: generally means persons cannot be punished or disadvantaged because of political associations. 2. Loyalty oaths: Public employees can be required to take a loyalty oath to the Constitution, but most loyalty oaths that say anything else besides constitituion are struck down as vague and/or overbroad. Loyalty oaths are very likely is invalid because of vagueness and overbreadth. 3. Bar membership: State bar can investigate good character, but they cannot deny admission based on political affiliations. Cant deny because ur member of communist party or because ur member of some alt right organization. Ur political views cannot be used to deny bar membership. But they can refuse bar membership if u refuse to answer relevant questions or give them false information. 4. Political parties: political parties have rights of asociaiton too as well as indiivduals. Political aprties have rights of association that limit how states can regulate them - specifically States cannot require open primaries (i.e., one in which you do not have to be registered as a member of the particular party to participate in the primary). In open primary all voters can participate in either primary or whatever primary they like - lot of states have open primaries now but state cannot demand open primaries - party itself gets to decide how it chooses its nominee.

The Powers of the President—Domestic

1. Has the power to enforce the law, not to make it or break it 2. The power to enforce is greatest when authorized by statute. Generally speaking, the President's powers are subject to control by statute.

Congressional Limits on the Executive

1. Impeachment - Applies to executive officers (e.g., President, Vice President, cabinet officers, federal judges) can be removed from office by impeachment - An accusation passed by the House of Representatives requiring a majority vote. - Once impeached, the person is tried in the Senate. - Conviction requires a two-thirds vote of the Senate. - Impeachment and conviction lead to removal from office. No other penalty applies. 2. Impoundment - If a statute gives the President discretion to spend or withhold funds, he may do so. - But, if a statute unambiguously requires that certain funds be spent on certain purposes, then the President cannot refuse to do so. There is no power to impound (withhold) funds simply because he disagrees with congress's decisions. 3. Legislative Veto - It is unconstitutional; - It arises when Congress passes a law reserving to itself the right to disapprove future executive actions by simple resolution. That's unconstitutional - why? If Congress wants to override executive actions, it must change the law (so that the President has an opportunity to veto the new legislation). To change the law, it must enact a new statute. And how to enact new statute? Must be passed by house of reps and passed by senate and presented to president for his veto. And if president doesn't like the new law he can veto it and congress has to override by 2/3 of each house. So problem with so called legislative veto - problem with congress trying to set up scheme where it can override executive actions by simple resolution, problem is it cuts out constitutionally guaranteed veto opportunity of the president. And avoiding constitutionally guaranteed veto opportunity of president makes it unconstitutional. - Congress cannot evade the President's guaranteed veto opportunity by passing a law saying that in the future it plans to govern by resolution.

Endorsement

1. It is a violation of the Establishment Clause for the government to endorse one religion over another and also to endorse religion over non-religion. You not only have freedom of religion - freedom to choose any one you want without govt pressure - you also have freedom from religion, freedom to choose none of the above if that's your choice. endorsement idea sounds good but number of very obvious endorsements that we tolerate without much concern. Like in god we trust below. - But, many endorsements are upheld, such as "In God We Trust" on currency. 2. The Supreme Court really mainly wants to prevent coercive endorsement of religion (one that might override individual choice). that's the real nub of the problem - coercive endorsement of religion. Whats unconstitutional is govt endorsement of religion in a context that might prove coercive on an individuals conscience. That explaisn why school prayer is unconstitutional - any officially sponsored school prayer is unconstitutional, no matter how general the prayer or how nondenominational the phrasing. There will be some kids who don't believe it, and they may feel intimidated or coerced when everyone else stands up to pray. Officially sponsored school prayer is a coercive endorsement of religion. - The Establishment Clause prohibits government endorsement of religion in a context that might prove coercive on an individual's conscience.

Legislative standing

1. Legislators do not have standing to challenge laws that they voted against. When legislator votes against law, that's merely ideological objection - voting against it because you don't like it - so ideological objection doesn't give standing to anyone. 2. In some circumstances, the legislature itself as a body may have standing, if the claim concerns its institutional functions. Supreme court occasionally allowed legislature standing, but not legislator standing. Occasional legislature standing, but no legislator standing.

dormant commerce clause

1. More important than Privileges and Immunities of Article IV because it protects out-of-state businesses as well as out-of-state individuals. Therefore has much broader application. 2. "Dormant" describes what the Commerce Clause means in the absence of federal regulation (when the federal commerce power is unexercised). A state has no power to regulate commerce in the face of federal regulation - state has no power to regulate commerce in a way that congress has forbidden. Question is what happens when congress does not act? What happens when no federal law on point? What happens in absence of fed regulation? 3. Rule: In the absence of federal regulation, state regulation of commerce is valid so long as: 1) There is no discrimination against out-of-state interests; how might state discriminate? By forbidding out of state products to be sold here - not allowed to do that. Other is by taxing out of state products at higher rate than local products. 2) The regulation does not unduly burden interstate commerce; and non discriminatory regulation of commerce simply means in state interests and out of state interests are treated alike - that's non discriminatory. Non-discriminatory state regulation of commerce is almost always upheld; only when it is so outrageously costly relative to the benefits of the regulation is a non-discriminatory state regulation struck down as an undue burden on interstate commerce. This is a balancing test, but non-discriminatory regulations are rarely struck down. 3) The regulation does not apply to wholly extraterritorial activity. A state may not regulate conduct occurring wholly beyond its borders. State regulation of commerce in the absence of federal regulation is okay so long as it meets three requirements above.

REGULATION OF THE MEDIA, REGULATION OF ASSOCIATION, SPEECH BY GOVERNMENT EMPLOYEES, AND CAMPAIGN FINANCE - regulation of the media

1. No Special Privileges The press and media have no special privileges. They have the same rights as everyone else. No more, but of course no less. If question asked about freedom of press, some law that restricts media or newspaper or television and like don't need to know a lot about that subject - just ask urself could govt do this to me? If govt could restrict ur speech they can restrict the press in the same way. Freedom of press is same rights the rest of us have - no more but no less. Exam Tip 13: When asked about a law on the media, ask whether that law could be applied to you. Example 43: Confidential sources for the media. Does a reporter have a constitutional right to refuse to answer grand jury questions about a confidential source? Think of it in terms of yourself: You will not answer questions because you promised that you would not reveal a confidential source. Can the government force you to talk? Yes, you can be put in jail (held in contempt). The same is true of the media and press. Does report have const right to refuse to give testimony to grand jury? Ask would I have const right to refuse to testify before grand jury? Can u go before grand jury and say sorry I promised not to tell? Of course not. U have to answer grand jury questions unless pleading fifth amendment and if u don't they can hold u in contempt. Its true of normal people and newspaper reports too. Example 44: Open trials—you have the right to attend public traisl and so does The press - it has the right of access to open trials. They have same rights you do. 2. Broadcasters o The only special case is broadcasters. o Traditionally, because of early limits on the broadcast spectrum, the government had greater regulatory authority over broadcasters than over print media or other unlimited technologies like the Internet. Frankly not clear just how much govt can regulate broadcast but it is clear they have some additional authority to regulate broadcast because spectrum is limited and that one fact is probably worth noting.

Few powers that are exclusively executive and, therefore, not subject to statutory control:

1. Pardon Power: The President can pardon or commute punishment for any and all federal offenses. (Governors have a similar power for state crimes.) This power cannot be limited by Congress. Congress cannot deny or overrule effect of presidential pardon. 2. Veto Power: The President has 10 days to veto legislation. The President can veto for any reason or no reason, but cannot veto specific provisions in the legislation and accept others. Veto power is all or nothing. - Overriding a veto requires a 2/3 majority vote of each house for congress to override veto. 3. Appointment and Removal of Executive Officers: Only the President (or his appointees) can hire or fire executive officers. Some senior officers (e.g., cabinet officers, ambassadors, federal judges) require the advice and consent of the Senate to be appointed. The Senate has a power of rejection though - can refuse to appoint or confirm a fed judge whos been nominated or refuse to confirm cabinet nominee but it cannot appoint judges itself nor can it fire executive officers save by elaborate process of impeachment. The Senate's approval power does not translate into a power of appointment. - Executive officers are anyone who takes action on behalf of the U.S. - executive officers must be hired and fired within executive branch. Who does that not include? Whom can congress hire and fire? People who don't act. Congress can hire and fire [eople who investigate, advise, and recommend. All kinds of commissions and monitors, people who do studies and recommend action - but if person takes action binding the U.S., that person is executive officer and must be hired and fired (impeachment aside) within executive branch. - Just as Congress cannot hire or fire an executive officer, it cannot give executive power to any officer it can hire or fire. Example: The question tells you that a certain officer is appointed by Congress (e.g., the Speaker of the House), and is therefore under legislative control. Then, someone proposes legislation that gives the officer executive power to act on behalf of US. Once the officer is under legislative control, any legislation that attempts to give the officer executive power is unconstitutional. Congress cannot hire or fire executive officers, also cannot give executive power to any officer it can hire or fire.

general rules for the bar exam:

1. Pick the taxing power when the law involves a tax - if fed tax and asking why they can do it, yes they can do it because they have the taxing power; 2. Pick the spending power whenever federal money is spent or disbursed - if fed spending appropriation or fed expenditure of fed money, spending is the right argument; 3. When in doubt, pick the commerce power - if no tax and no money pick commerce.

3 key errors to avoid for "congressional powers"

1. Promoting the general welfare is not a power of Congress - never pick this answer, its always wrong; 2. The federal government, unlike states, does not have a general police power - doenst refer narrowly to law enforcement but refers broadly to any govt action serving the public good. But states have a general police power which means they have unlimited legislative power but the national government doesn't have general police power; 3. also necessary and proper is not a free-standing power of Congress. Its an addon - addition to some other legislative power. It works only as an add-on to some other legislative power. Never pick necessary and proper by itself. Right answer might by necessary and porper to regulate interstate commerce or necessary and proper to protet individual rights or to coin money or to maintain post office or to have army and navy, but never just necessary and proper by itself.

Religious Conduct

1. Protected qualifiedly - Laws regulating religious conduct because of its religious significance are unconstitutional (i.e., laws aimed at religion). - Neutral regulation of conduct: Neutral, generally applicable laws (laws not aimed at religion but simply regulating conduct) must be obeyed despite religious objections. Ex: must pay taxes even if u believe in taxes or not. No constitutional right to demand government accommodate ur religious belief. - No right to accommodation. (accommodation means what? Accommodations means you are demanding that the government adjust the laws to accommodate ur religious belief - that u have right not to obey a law everyone else would have to follow because it offends your religious scruples - there is no constitutional right of accommodation). - No constitutional right to exemption from neutral, generally applicable regulations of conduct (e.g., Peyote case). Peyote case - state had statute against consuming hallucinogens. Peyote is a hallucinogen. Statute can be enforced across board as an anti drug law and can even be enforced against native americans claiming they have religious motivation for peyote use. Neutral, generally applicable laws aimed at conduct can be enforced against everyone on the same terms.

Free Exercise of Religion

1. Religious Belief Protected absolutely (entitled to hold any belief or none at all) - absolutely entitled to have any religious belief you like or none at all. Public office cannot be limited to those with particular religious beliefs or those with religious belief of any sort. People who have religious belief, including clericks, cannot be denied public office. Simply nothing govt can do to restrict ur religious belief. 2. Religiously motivated conduct though is a different matter. Of course you are protected against laws that prohibit religious conduct because its religious. Youre protected from laws aimed at religion. If law prohibits religious conduct because codncut is religious or prohibits religious conduct in order to suppress religion, then of course its unconstitutional but that's not something that ever really comes up. Whats not constitutionally required is accommodation of religious belief.

Suspect Classifications (Trigger Strict Scrutiny and Arise under Equal Protection)

1. Race, Ethnicity, or National Origin—laws that disadvantage minorities will be struck down. o However, discriminatory purpose is required: For constitutional purposes, a law is a racial classification only if the plaintiffs show it has a discriminatory purpose (not enough to show a disproportionate impact). Rule is quite diff under many anti discrimination statutes - they usually only require showing of disproportionate impact - that law affects whites and blacks differently, but constitution requires showing of discriminatory purpose. Example 28: Economically restrictive zoning in Arlington Heights—a developer wanted to come in and build a large multi-family housing complex. Developer argued that the two-acre, single-family zoning ordinance has the effect of restricting the town to wealthy white people; he argued that discriminated against minorities. The Supreme Court said the zoning law was not unconstitutional as racial discrimination unless you can show it was done for the purpose of discriminating against minorities. Developer came to court and said ur honor the effect of this 2 acre minimum zoning law - effect of very restrictive zoning law is to keep town all white - u should strike it down as racial classification. Sc said that may or may not be effect but it doesn't matter - the zoning law is not unconstitutional as racial discrimination unless done for purpose of excluding minorities. U have to show discriminatory purpose to show racial classification under the constitution. § How to show Discriminatory purpose: May be explicit on the face of the statute, or may be proved by a history of discriminatory application (If apparently neutral law only invoked against racial minorities, if demonstrated history of discriminatory application that can reveal a discriminatory purpose even tho face of statute is clear), or by extrinsic evidence about the purposes of those who passed the law (so can use all kinds of evidence to show discriminatory purpose exists, but u must show that discriminatory purpose exists in order to show a racial classification). § School desegregation: De jure (by law) segregation is unconstitutional. De facto segregation is not (usually by residential housing patterns). Just another way of saying discriminatory purpose is required - de jure segregation is segregation by law, segregation on purpose - that's what makes it de jure and its unconstitutional. De facto segregation is separation of races where no discriminatory purpose can be proved. De facto segregation is not unconstitutional - just another way of saying discriminatory purpose is required. 2. Affirmative action- used to describe acts designed to benefit a racial or ethnic minority. § It is a racial classification; an effort to benefit a racial or ethnic minority. § Triggers strict scrutiny and requires a compelling interest - for affirmative action u usually find compelling interest. For example, Specific past discrimination - Affirmative action is valid when it specifically corrects past discrimination by the specific department or agency now engaged in affirmative action. Example - police dept has long history of discriminating against african American candidates for employment and now forced to set plan to set history right and to prefer African American applicants until such time as police force has racial and ethnic balance it should have had all along. That's fine, that affirmative action is fine - because it specifically corrects past discrimination by that particular institution or agency. Affirmative action is always allowed when it specifically corrects past discrimination by that institution or agency. But must be quite specific as seen with example below. Example 29: New Haven Firefighters case—Nineteen firefighters sued the city of New Haven when they had all scored high enough on a standardized test to be promoted to management positions. City officials abandoned the test results because minority firefighters had not scored high enough to be considered for these positions, and then the city hired minorities anyway for these spots. The Supreme Court said the city's actions looked like a racially motivated quota and struck it down as being unconstitutional. Lack of specific connection to past misconduct of that agency was responsible for that decision. Affirmative action not valid to correct general societal discrimination. • General societal discrimination does not justify affirmative action. Not good enough to say our nation has a bad past and we want to do something about it. Affirmative action has to correct specific past discrimination by that institution or agency. § Affirmative action is allowed in the context of preferential admissions to colleges and universities. • Preferential admissions based on race, ethnicity, ethnic group, etc. are allowed if necessary to achieve a diverse student body and diversity is essential to the education. o school has to show - Must be a strong showing that racial preferences are essential to achieving a diverse class. o Racial preferences must be "holistic" (can be built into an evaluation if every student is evaluated individually in a holistic way) and flexible. means if race taken into account along with other factors. o Rigid Quotas are not allowed. o Separate admissions tracks or procedures for minority applicants are not allowed. • Preferential admissions not allowed for secondary schools (though schools may be located and attendance zones created to maximize diversity). 3. Alienage—Requirement of U.S. Citizenship o Classifications based upon U.S. citizenship are generally suspect classifications that require a compelling interest, but two important exceptions to strict scrutiny for alienage apply: § Federal government • Congress has plenary power over citizenship, immigration, and naturalization. • Federal classifications based on U.S. citizenship do not trigger strict scrutiny. • Federal classifications are valid unless arbitrary and unreasonable. Otherwise theyre upheld if not arbitrary and unreasonable. § State and local participation in government functions • These are jobs that have a particular relevance to the role of government and non-U.S. citizens can be barred from these jobs. • States and localities may require U.S. citizenship for participation in government functions, including voting, serving on a jury, and working in any kind of government law enforcement position (including probation and parole officers), or as a public school teacher. o Rule: States and localities cannot require U.S. citizenship for access to private employment or for government benefits (That rule not found in any supreme court decision but accurately describes result of them all). - 2 contexts where alienage treated as suspect classification and law restricting private employment or govt benefits will be struck down. Example 30: Florida required U.S. citizenship for admission to the Florida bar. Admission to the bar controls private employment as an attorney and therefore triggered strict scrutiny. It was struck down. Example 31: Maryland required U.S. citizenship for Maryland's in-state tuition, a government benefit, a kind of subsidy of in-state students. Basing that subsidy on U.S. citizenship triggers strict scrutiny. The requirement was struck down. However, Maryland could and did validly require residency for in-state tuition. But Maryland was not entitled to discriminate against lawfully resident, taxpaying members of the Maryland community who were not U.S. citizens. Note 5: Constitutional rights of undocumented aliens—Plyler v. Doe: Undocumented aliens are not a suspect classification. Even so, states cannot deny undocumented children public education.

free exercise of religion

1. Religious Belief Protected absolutely (entitled to hold any belief or none at all) - absolutely entitled to have any religious belief you like or none at all. Public office cannot be limited to those with particular religious beliefs or those with religious belief of any sort. People who have religious belief, including clericks, cannot be denied public office. Simply nothing govt can do to restrict ur religious belief. Religiously motivated conduct though is a different matter. Of course you are protected against laws that prohibit religious conduct because its religious. Youre protected from laws aimed at religion. If law prohibits religious conduct because codncut is religious or prohibits religious conduct in order to suppress religion, then of course its unconstitutional but that's not something that ever really comes up. Whats not constitutionally required is accommodation of religious belief.

Right to travel/Right to vote/Gerrymandering

1. Right to travel 2. Right to vote—one person, one vote o Requires districts of approximately equal size, i.e., approximately the same number of voters in each o Applies whenever you elect representatives by district. If one district had 100k voters and one had only 50k voters, every voter in second district would have twice the impact of voter in first district. So districts ofequal size or population is how to achieve one person, one vote. That rules applies whenever u elect representatives by district including us house of reps - within each state each congressiaonal district should be approximately equal size. § Examples include: U.S. House of Representatives; both houses of a state legislature (even if one of them called state senate still must have senatorial district of approximately equal size); local governments when they elect representatives by district. • Very narrow Exception: Special purpose governments—A highly specialized government (e.g., for distribution of water rights) can have a franchise based on that special purpose (e.g., acreage or water entitlements). If highly specialized governmentt with narrow purpose, franchise can be restricted to that narrow purpose. 3. Gerrymandering - Comes in two varieties—racial and political - gerrymandering - districts of equal size with same number of people in each district but arrange them to achieve advantage for one side rather than the other. Can be different types of gerrymandering: a. Racial gerrymandering - contemplated and often required by the voting rights act. § Vote Dilution: Drawing districts to scatter minorities all around so that they are not a critical mass in any one district. If done with a discriminatory purpose, it's unconstitutional. § Voting Rights Act: Requires racial gerrymandering to ensure minority success by creating majority-minority districts. Gerrymandering in favor of minorities. More or less required by voting rights act. How to do it? Bring minorities together in a majority minority district, collect racial or ethnic minorities in a majority minority district and that's supposed to ensure minority political success. • Rule: Race may be a factor in drawing district lines so as to advantage racial minorities as voting rights act wishes to do - race may be a factor in drawing district lines, but not the predominant or only factor. • Other factors include compactness and observing/respecting local, political subdivisions. • A bizarrely shaped district which wheels goods in and out with long fingers and stretches all over state - very bizarrely shaped district will be taken as evidence that race was the predominant purpose, predominant racial purpose. But really no easy way to tell. b. Political gerrymandering (drawing districts to hurt one party) - occurs if one party controls the legislature and can screw the other party. Gerrymandering is very important in California and other states where u can draw districts with aide of computers that really advantages one political party at expense of the other. § political gerrymandering Can, in theory, violate equal protection. In practice, it is never struck down. If asked whether constitutional problem with political gerrymandering, or asked whats strongest argument that political gerrymandering is unconstitutional, equal protection is answer. In theory political gerrymandering can violate equal protection but in practice it never does because the Supreme Court has not found any judicially manageable standards for implementing that guarantee. § A political question (non-justiciable)

Timeliness (Ripeness and Mootness)

1. Ripeness concerns prematurity of a case - cant litigate hypothetical controversy, one that might arise if if if. You must show actual harm or an immediate threat of harm. 2. Mootness cases are overripe and are dismissed whenever they become moot. Cases can become moot during trial or on appeal. - Exception: Controversies capable of repetition, yet evading review, are not moot, even though they look like it. These issues are easy to spot -always there is internal time limit. Somethng about nature of problem that will not last long enough for case to make its way through the courts. - Such controversies always have an internal time limit (e.g., pregnancies and abortion). Eg: restrictions on abortion - pregnancy lasts only 9 months - no pregnancy will last long enough for a case to go all the way to sc. But pregnancy can recur and issue of abortion can come up again if case was dismissed as moot every time pregnancy was revolsed the issue would simply cycle and never be finally resolved. Capable of repetition because it will come up again but evading review because it wont last long enough to be finally decided. Those cases look moot but theyre not.

exceptions to no discrimination against out of state interests

1. State as Market Participant—When a state is buying or selling goods or services on its own or for itself, it can choose to deal with only in-state persons. 2. Subsidies—A state can always choose to subsidize only its own citizens (for example, welfare benefits or in-state college tuition). Doesn't have to provide same benefits to citizens from other states. Can restrict welfare benefits, for example, to in-state citizens. same with in state tuition costs. Why is it fine? when u attend public university ur not paying all cost of education - ur education is being subsidized by state appropriation. And when state is spending money and subsidizing activity, it can choose to subsidize only its own. 3. Federal Approval—Remember, the Dormant Commerce Clause applies only in the absence of federal action. If Congress authorizes or consents to state regulation of commerce, nothing the state does will violate the Commerce Clause, even if it discriminates against out-of-state interests.

Government Speech

1. The First Amendment restrictions basically do not apply to the government as a speaker. Its obvious - govt is free to advertise the us army. It doesn't have to give equal time to critics of military. Same is true when govt accepts a monument donated by private persons. Govt doesn't have to accept any monument just because it accepts one - it can still control what gifts it accepts. 2. Government as a speaker is free to express a point of view (e.g., advertising the military). 3. The government does not have to accept all monuments donated by a private person simply because it accepts one—when the government is controlling the message, it is entitled to say what it wants. - But one area where govt speech has been litigated is specialty license plates. These carry messages requested by the customers. Messages are requested by drivers but license plates are still govt speech so govt can refuse licenses that contain offensive messages. Govt sets up thing where u pay a little extra, u get to have a little slogan or words on ur lciense plate that mean something but theyre free to reject licenses that would be offensive to others. That's because its government speech. - But specialty license plates bearing messages requested by purchasers are still government speech, so the government can refuse to issue plates that would be offensive to other citizens.

Immunities

1. The President a. Has absolute immunity from liability for official acts (broadly construed) - does not take account of possibility of impeachment. If president impeached and convicted then president no longer president. And a non president doesnt have immunity. But impeachment and conviction necessary to overcome absolute immunity for official acts. b. Has president has no immunity for acts done prior to taking office c. Has an executive privilege not to reveal confidential communications with presidential advisers, but that privilege can be outweighed by a specifically demonstrated need in a criminal prosecution 2. Judges - Judges have absolute immunity for all judicial acts, but may be liable for non-judicial acts (e.g., employment discrimination). 3. Legislators a. United States Senators and Representatives (not state legislators) are protected by the speech or debate Clause. Speech or debate clause doesn't give congressman constitutional right to take bribes, it gives them immunity against certain kinds of evidence - Senators and Congressmen and their aides cannot be prosecuted or punished in relation to their official acts. That means the official act cannot be introduced into evidence to show that congressman was a crook. If you can prove dishonesty some other way, maybe having fake arms dealer wiretap congressman or wear wire during deal, some other weay is fine. But cannot introduce into evidence in court how rep voted or whether she introduced a bill or anything said on the floor of house or senate or anything said in an official committee hearing. All of those are official acts and they cannot be used in evidence - that's meaning of speech or debate clause. • The official acts of a federal legislator cannot be introduced into evidence.

spending power

1. The Spending Power includes spending for the general welfare. Promote general welfare is not power of congress and is always wrong answer, but spending money to promote general welfare is part of spending power and that can be right answer. 2. Congress can use the Spending Power to accomplish things it could not do by direct regulation under the Commerce Clause. Example 15: South Dakota v. Dole—The 21st Amendment repealed Prohibition and gave states the power to control the regulation and consumption of alcohol. As a result, Congress likely cannot directly regulate the drinking age in the states. Thus, in order to pass a 21-year-old drinking age law, Congress passed a "bribe." If states wanted to receive federal highway funds, they had to raise their drinking age to a minimum of 21 years of age. States all fell into line because they were bribed to do so by federal highway subsidies - that's spending power and congress can do that under spending power whether or not it could do it as regulation of interstate commerce.

FREEDOM OF RELIGION - Establishment of Religion

1. Three-Part Test of Lemon v. Kurtzman - test has never been formally overruled: o Does the law have a secular purpose - non religious purpose? o Does the law have a primary effect that neither advances nor inhibits religion? o Does the law avoid excessive government entanglement with religion?

Deficiency of Lemon Test

1. Trouble is its Exceedingly difficult to apply - hard to know what is and what is not constitutional under the test. 2. It once was interpreted to condemn aid to religious primary and secondary schools, but neutral aid is now allowed (goes to everyone). As long as neutral and goes to everyone - all private schools, all religious, all non religious schools - and govt isn't making decision about who gets aid and who doesnt - then its fine. All govt has to do today is say we will give each parent money that the parent can use to send her child to a private school, whether religious or not. That's neutral and today it passes muster. 3. The government gives aid to parents and the parents are allowed to send their children to the schools of their choice. - The parents are making the decision as to whether a religious school gets the money. - The government is not picking and choosing recipients. In recent years court has spoken more about endorsement and less about lemon test.

Vagueness and Overbreadth

1. Vagueness: Vague laws are ones that give no clear notice of what is prohibited and thus violate due process. 2. Overbreadth: Overbroad laws are ones that go too far in regulating speech. These laws burden substantially more speech than is necessary to protect a compelling interest and thus violate the First Amendment. Example 36: Laws that prohibit all nudity in drive-in movies are overbroad. Some nudity in drive in movies is legally obscene and obscenity can be banned. But much nudity in drive in movies is not legally obscene, but merely very large so law banning all nudity in drive in movies is overbroad - it goes substantially beyond a compelling interest and is therefore invalid.

fighting words

1. Words likely to provoke an immediate breach of the peace by prompting person so addressed to hit back. In other words, general vulgarity will not do. - General vulgarity is not enough. That's not fighting words, they must be aimed at someone. - Must be aimed/targeted at someone, and that person might hit back. - In theory, fighting words are not protected speech. - In fact, all fighting words statutes on the bar exam are almost always unconstitutionally vague and/or overbroad (e.g., laws against "hate speech"). All of these statutes are unconstitutionally vague and overbroad and its been years since supreme court upheld a fighting words law. 2. If you see a statute aimed at fighting words, what would it look like? Statute might say words tending to a breach of the peace are criminal or offensive language is criminal or just hate speech is criminal - right answer if u see a statute of that sort is not that the speech is constitutionally protected, that's not necessarily true but right answer is that this statute cannot be enforced because its vague and overbroad - they usually all are.

federal tax - apportionment requirement + uniformity requirement

1. apportionment - Only direct taxes such as ad valorem property taxes, are subject to the apportionment requirement (not federal sales taxes). 2. uniformity requirement - The requirement that taxes be uniform throughout the United States means only that the product must be identically taxed in every state in which it is found. Here, that requirement will be met.

5 categories of speech plus extra that can be regulated because of content

1. obscenity 2. incitement speech 3. fighting words 4. defamation 5. commercial speech 6. government speech

Doctrine of Political Question

1. political question is non-justiciable 2. A non-justiciable question when it is inappropriate for judicial resolution - why? Usually because issue is committed to another branch of govt for decision or maybe because no judicially manageable standards for adjudication. - The issue is committed to another branch of government for decision; - There are no manageable standards for adjudication. 3. Examples of famous non-justiciable political questions include: - Guarantee Clause (protecting the republican form of government) - sc said many yrs ago that guarantee clause was non justiciable - something for congress to figure out so could not be litigated; - Foreign affairs, e.g., opening or breaking off diplomatic relations with another country - presidents authority to open or break off diplomatic relations with another country, that's a political question and courts wont decide if presidents action was proper or not; - Impeachment procedures - impeachment is accusation made by house of reps by simple majority vote - one person impeached theyre tried and trial occurs in senate and requires 2/3 vote to convict and impact of impeachment or effect of it is removal from office. Once impeached then crim prosecution possible, but only remedy for impeachment is removal from office. Generally non justiciable political questions involved with impeachment like how much testimony required, etc etc - Political gerrymandering (drawing districts to establish a political advantage of one party over another). a. Political gerrymandering can violate equal protection - if question about is there constitutional problem with political gerrymandering - answer is yes it can violate equal protection; but b. on the other hand, The court has found no judicially manageable standards for determining what is and what is not acceptable in the area of drawing districts. Court has said we as courts don't know how to draw distict correctly so don't know how to remedy the wrong and therefore correction of political gerrymandering is beyond our capacity - no judicially manageable standards. So in theory political gerrymandering may be unconstitutional but in fact its tolerated and supreme courts reasoning is that political gerrymandering is a nonjusticiable political question because of lack of judicially manageable standards for resolving it.

To have a viable or constitutional time place or manner regulations.....

3 requirements: 1. Content neutral: Must be content neutral on its face and neutral as applied. If a facially neutral law is applied to favor one speaker or one point of view or one political party its not content neutral. Content neutral also means law must not allow executive discretion - if an executive officer has discretion, if exec officer has power to pick and choose who can speak risk is that the discretion will be administered in a non-neutral way. The supreme court has tried to guard against that possibility by saying that a law must be content neutral on its face, it must be content neutral as it has been applied, and it must not allow executive discretion which would threaten content neutrality. 2. Alternative channels of communication must be left open: Time, place, or manner law must be a guideline for speech, not a flat prohibition of speech. 3. Must narrowly serve a significant state interest: Under this test, most content-neutral time, place, or manner regulations are upheld.

Commercial Speech

4 part test but most imp point is first one: - Most regulations of commercial speech are struck down - generally as a rule, regulations of commercial speech are struck down. So long as the advertising is truthful and informational, it must be allowed. - Test for commercial speech is version of intermediate scrutiny: Regulation of commercial speech must directly advance a substantial government interest and be narrowly tailored to that interest. Key thing is that test is substantial state interest, not compelling which is the usual test for free speech. Not merely significant, which is the test for content neutral time place and manner - its an in between standard, substantial state interest. Also note misleading commercial speech point: - Misleading speech: Misleading commercial speech (unlike political speech) may be prohibited. If commercial advertising would cause you to think as a reasonable consumer, that the product had qualities it doesn't have, if it would cause you to believe that the product can cure disease - that its useless for - misleading commercial speech can be suppressed. Almost all nonmisleading commercial advertising must be allowed. And that includes attorney advertising.

alienage classification

A classification based on alienage is subject to a different standard depending on whether the action is taken by the state or by the federal government. Courts will generally apply the strict scrutiny test and strike down state-based laws that discriminate against resident aliens for lack of U.S. citizenship. In contrast, Congress has plenary power over aliens under Article I, and the power to expel or exclude aliens is a fundamental sovereign attribute exercised by the Government's political departments largely immune from judicial control. Therefore, a federal alienage classification is likely valid unless it is deemed arbitrary and unreasonable. Example - the government proved that the law was rationally related to a legitimate governmental interest and thus the resident alien's claim will fail.

Causation

A defendant's conduct has caused or will cause the injury.

ripeness

A federal court will not consider a claim before it has fully developed; to do so would be premature, and any potential injury would be speculative. In order for a case to be "ripe" for litigation, the plaintiff must have experienced a real injury (or imminent threat thereof).

fed govt taxing states

A federal tax that is not directly imposed on a state, but instead on payments made by the state to a private person, is constitutional. Consequently, interest payments made by the state to its bondholders may be taxed. Answer choice A is incorrect because a state does not enjoy immunity from a tax that has a substantial adverse economic impact on the state, but is not directly imposed on the state. Although there may be some protection for a state under the Tenth Amendment from federal taxation, there is clearly no immunity when the tax falls on payments made by the state to a private person, such as the payment of interest to the holders of state bonds.

Note the point about congress being able to regulate hunting due to effect on interstate commerce (answer B). In order to protect valuable state resources from over-hunting, a state department enacted regulations establishing a lottery for permits to hunt elk in the state. The regulations put a cap on the percentage of permits granted to out-of-state residents. The regulations were challenged in court, and the federal appellate court found that the regulation violated the Dormant Commerce Clause and unconstitutionally discriminated against out-of-state commerce. In response to this decision, Congress passed legislation, which was signed by the President, that it was in the public interest for each state to continue to regulate the taking for any purpose of fish and wildlife within its borders, including by means of laws and regulations that differentiate between residents and nonresidents of the state. Subsequently, another state adopted a regulation that prohibited out-of-state commercial hunters from hunting elk within the state. A group of out-of-state residents who lead elk hunting expeditions sued the state official responsible for enforcing the regulation, seeking a declaratory judgment that the regulation violated the Dormant Commerce Clause.If the court determines that hunting is an economic activity that substantially affects interstate commerce, should the court grant the requested declaratory judgment? A No, because Congress granted the states the right to discriminate against out-of-state residents. B No, because hunting is a traditional area of state regulation. C Yes, because the federal law improperly invades the province of the court. D Yes, because the regulation discriminates against out-of-state residents.

A is correct. Because Congress has exclusive authority over interstate commerce, it may explicitly permit states to act in ways that would otherwise violate the Dormant Commerce Clause. It must be unmistakably clear that Congress intended to permit the otherwise impermissible state regulation. Here, Congress has specifically provided that states may, through regulations as well as statutes, discriminate against out-of-state commercial hunters with regard to hunting and fishing within the state. Therefore, the regulation is permitted. Answer choice B is incorrect. Although hunting is an area that traditionally has been subject to regulation by the states, Congress may regulate economic activity that substantially affects interstate commerce. Consequently, the Tenth Amendment, which reserves to the states only those powers not delegated to the federal government or denied to the states, does not serve to permit this regulatory restriction on out-of-state hunters. Answer choice C is incorrect. Although the judiciary has the final say in interpreting the Constitution, here, Congress is engaged in its law-making function pursuant to the Constitution to regulate interstate commerce. Answer choice D is incorrect. Although the regulation clearly discriminates against out-of-state residents, Congress has expressly permitted this discrimination.

The State Department adopted a rule that passports issued to U.S. citizens who were born in disputed territory would list their city of birth rather than country of birth when the President had not recognized the disputed territory as belonging to a specific country. Congress passed legislation that, among other things, required the State Department to list the country indicated by passport applicants as their place of birth on their issued passport. Although the President signed the bill, he noted that this particular requirement unconstitutionally invaded his power over foreign affairs. Shortly after his birth in a disputed territory to parents who were United States citizens, a passport applicant requested through his parents that he be issued a passport that listed a specific country as his place of birth. The passport office refused. The passport applicant filed suit in federal court seeking injunctive relief that mandates the passport office to issue the applicant a passport that lists a country as his place of birth.How should the court respond? A Deny the injunction, because the President has exclusive power over the recognition of a foreign country. B Grant the injunction, because Congress has the power to override a presidential decision regarding the recognition of a foreign country.

A is correct. The President represents and acts for the United States in day-to-day international affairs. In addition to appointing and receiving ambassadors, the President has the exclusive power to recognize a foreign government. Consequently, because the President has exclusive power over the recognition of a foreign country, the court should not grant the injunction because it would require the passport office to recognize a country that the President has not yet recognized. Answer choice B is incorrect because Congress lacks the power to reverse a presidential decision with regard to the recognition of a foreign country.

A federal marshal was escorting a prisoner from federal court back to prison when the prisoner attempted to escape. The marshal, in the process of foiling the prisoner's escape, accidentally shot and killed an innocent bystander. The local district attorney has charged the marshal with involuntary manslaughter under state law.Of the following, which would provide the marshal with best constitutional defense to this charge? A The Supremacy Clause of Article VI of the U.S. Constitution. B The Due Process Clause of the Fifth Amendment to the U.S. Constitution C The Eleventh Amendment to the U.S. Constitution D The Equal Protection Clause of the Fourteenth Amendment to the U.S. Constitution

A is correct. The federal marshal can argue that, since the killing occurred when he was acting as a federal law enforcement officer, he enjoys immunity under the Supremacy Clause from state prosecution for his actions.

state operated facilities - abortions

A state may prohibit all use of public facilities and public employees in performing abortions. Although a woman has a constitutional right to have an abortion, the state is not constitutionally required to permit abortions at a state owned or operated facility.

very important rule about state discrimination against other states

A state or local regulation that, on its face or in practice, discriminates against out-of-state commerce is unconstitutional unless it serves an important local interest and no other non-discriminatory means are available to achieve that purpose.

state regulation that is not discriminatory....

A state regulation that is not discriminatory may still be struck down as unconstitutional if it imposes an undue burden on interstate commerce. The courts will balance, case by case, the objective and purpose of the state law against the burden on interstate commerce and evaluate whether there are less restrictive alternatives. If the benefits of the state law are grossly outweighed by the burdens on interstate commerce, then even nondiscriminatory regulation may be struck down. Example - The state statute requiring these checkpoints is incredibly burdensome on interstate commerce, and appears to have had only minimal effects on road safety. Therefore, even though it is not facially discriminatory against out-of-state commerce, the statute likely violates the Dormant Commerce Clause.

student dismissal - due process

A student is not entitled to a hearing with regard to dismissal for academic reasons from a public institution of higher learning. Since the architecture student was provided with notice of her academic shortcomings and an opportunity to correct those shortcomings, she was provided with adequate procedural due process.

taxpayer does not have standing for....

A taxpayer generally does not have standing to file a federal lawsuit simply because the taxpayer believes the government has allocated funds in an improper way.

FREEDOM OF EXPRESSION (CONTENT-BASED RESTRICTIONS ON SPEECH)

A. Regulation of Unprotected and Less Protected Expression There are some categories of speech that can be regulated because of content. Most content based regulations of speech are struck down. But few categroies of speech that can be regulated because of content. This is where govt is said to have a compelling interest in regulating speech. There are 5 such categories plus special case where govt itself is said to be doing the speaking. 1. Obscenity: Defined by the rule of "S" - most imp category of speech that can be regulated because of govts interest. o obscenity must be Sexy: Must be erotic; must appeal to the prurient interest. Must make you itch or long with desire. (Gore and violence are not legally obscene. May be disgusting but don't make you itch or long, theyre not sexy.) o obscenity also must make Society sick: Must be patently offensive to the average person in the society, so not to most refined and delicate amongst us but to the average person. The society may be the nation as a whole, or can be a particular state, or a major metropolitan area. But wherever you are, material must simultaneously make you itch or long and be sick. o obscenity must be defined by the proper Standards - if a law doesn't have right standards it will be invalid as vague and overbroad: Must be defined by the proper standards for determining what is obscene, not vague and/or overbroad. For example, full frontal nudity. Full frontal nudity might be obscene but some of its not the least bit sexy. So law banning al nudity is overbroad - goes too far and is unconstitutional for that reason. Example 41: Tax on films displaying frontal nudity. Not a valid law because frontal nudity is not a good enough standard. o obscenity must lack Serious value: The material must lack serious value. If material has serious value (artistic, scientific, educational, or political), it cannot be held legally obscene. This determination is made by the court, not the jury, and it must be based on a national standard, not a local one. Juries cant speculate on whether something has serious value. Example - film which was mainstream Hollywood one - maybe because of name or title of film "carnal knowledge" it was prosecuted against as being obscene in the state of Georgia. Supreme court said the film is not obscene, it was a mainstream Hollywood film, got great reviews, and it has artistic value so it has serious value. So good reviews probably means no obscenity. 4 s's - sexy, making society sick, must be defined by proper standards, lacking serious value - that defines law of obscenity but there are a few additional points to know below: o Footnotes on obscenity: § Minors: A lesser legal standard can be applied to minors, but the government cannot ban adult speech simply because it would be inappropriate for minors. A lesser legal standard can be applied to minors but only if the minors can be dealt with separately - issue cant restrict materials to everyone because it might be inappropriate to minors. But you can have special standards for matierlas distributed to minors. § Child pornography: Can be prohibited whether or not it is legally obscene, and possession can be punished even if it is in the privacy of your own home. Child porn is special case - al bets off for child porn. Child porn is sexual representation of children - it can be prohibited whether or not it is legally obscene - child porn can be prohibited even if it does have some artistic value. It can be prohibited whether or not it is legally obscene and even if u possess it in the privacy of your own home. Even in the privacy of your own home, child porn can be banned. § Land use restrictions: Narrowly drawn ordinances can regulate the zoning of adult theaters, but cannot ban them entirely. But they can regulate where they are placed. Noones quite sure whether its better to scatter them all around so no one neighborhood is overrun or whether its better to bring all the adult theaters into one neighborhood and just close it off. but whatever it is, u can zone adult theaters so long as u allow some place for them to exist. Note 6: Courts have recently begun to distinguish legally obscene speech from pornography. Merely establishing that speech constitutes pornography is generally insufficient to establish that the speech is obscene. Second category of speech that can be punished because of its content is incitement. 2. Incitement Speech is not protected if it is an incitement to immediate violence. Immediate violence - great non example of incitement - "one of these days there'll be a revolution - first well kill all the lawyers" - that's not incitement because no connection to violence here and now. Incitement must be immediate. 3. Fighting words - not protected speech o Words likely to provoke an immediate breach of the peace by prompting person so addressed to hit back. In other words, general vulgarity will not do. § General vulgarity is not enough. That's not fighting words, they must be aimed at someone. § Must be aimed/targeted at someone, and that person might hit back. § In theory, fighting words are not protected speech. § In fact, all fighting words statutes on the bar exam are almost always unconstitutionally vague and/or overbroad (e.g., laws against "hate speech"). All of these statutes are unconstitutionally vague and overbroad and its been years since supreme court upheld a fighting words law. If you see a statute aimed at fighting words, what would it look like? Statute might say words tending to a breach of the peace are criminal or offensive language is criminal or just hate speech is criminal - right answer if u see a statute of that sort is not that the speech is constitutionally protected, that's not necessarily true but right answer is that this statute cannot be enforced because its vague and overbroad - they usually all are. 4. Defamation - important category of speech that can be prohibited because of content o False statements of fact (not opinion) damaging to a person's reputation can be prohibited. Critical that defamation concern a false statement of fact. An opinion, however unpleasant, cannot be defamatory. If u said someone is worst teacher ever, that cannot be defamation because its ur opinion. If u say I saw someone stealing money from till somewhere, that's a factual statement which if not true can lead to liability for defamation. o Public officials (Self defining category - those who hold govt office) and public figures (less clear category - people who have standing and reputation and prominence in the community) can recover for defamation only on proof of knowing or reckless falsity. Reckless means newspaper or publisher consciously recognized risk that material was false and published it anyways. o Private plaintiffs (people who don't hold office and who aren't terribly prominent) can recover on proof of negligent falsity. Defamation involves false statements of fact - many contexts in which false statements of facts are punished - perjury is one example - lie under oath. Any false of statement of fact cannot be punished though. Stolen valor act made it crime ... look below. Example 42: United States v. Alvarez: The Supreme Court struck down the Stolen Valor Act. The Stolen Valor Act made it a crime to falsely claim receipt of military declaration or medals. The Act was struck down by the Supreme Court as applied to a local politician who lied about receiving the Congressional Medal of Honor. The Justices held that the fact that the statement was a blatant self-serving lie did not exempt it from First Amendment protection. Stolen valor act was content based - not one of 5 exceptions - so it was unconstitutional. The prohibition was content-based and was not supported by one of the five recognized categories of unprotected speech. The statute was unconstitutional. Not the whole category of false statements of fact that are outside the first amendment - but it's the narrower category of defamation that's outside the first amendment. 5. Commercial Speech - 4 part test but most imp ppoint is first one: o Most regulations of commercial speech are struck down - generally as a rule, regulations of commercial speech are struck down. So long as the advertising is truthful and informational, it must be allowed. o Test for commercial speech is version of intermediate scrutiny: Regulation of commercial speech must directly advance a substantial government interest and be narrowly tailored to that interest. Key thing is that test is substantial state interest, not compelling which is the usual test for free speech. Not merely significant, which is the test for content neutral time place and manner - its an in between standard, substantial state interest. Also note misleading commercial speech point: o Misleading speech: Misleading commercial speech (unlike political speech) may be prohibited. If commercial advertising would cause you to think as a reasonable consumer, that the product had qualities it doesn't have, if it would cause you to believe that the product can cure disease - that its useless for - misleading commercial speech can be suppressed. Almost all nonmisleading commercial advertising must be allowed. And that includes attorney advertising.

FREEDOM OF EXPRESSION (TIME, PLACE, AND MANNER)

A. Regulation of the Content of Expression - Content-based regulations of speech trigger strict scrutiny and are usually struck down - few categories of speech that can be regulated because of content but quite exceptional. 1. Expressive Conduct (a.k.a. Symbolic Speech) - lot of conduct may be symbolic of some expression. What to do about that? Such laws upheld if regulation furthers an important interest. o Laws regulating expressive conduct are upheld if: § They further an important interest; § That interest is unrelated to the suppression of expression; and § The burden on expression is no greater than necessary. Key is whether govts interest is unrelated to suppression of expression, in which case law will likely be upheld or whether govts law is actually aimed at suppressing speech in which case law will be struck down. Example 34: Laws against flag desecration are unconstitutional because they suppress an expressive type of speech to show disapproval. Theyre aimed at a particular message - ppl were allowed to use flag as symbol to express patrotiotism or admiration for country. They were allowed to use flag as a symbol to express affirmative ideas, but they were not allowed to use the flag to express their hatred or contempt for the country. Thus a law that prohibits any desecration of American flag was aimed at suppressing a particular kind of message and was therefore struck down. Flag desecration laws apply to your flag, obviously you have no right to do something with someone elses flag, including the govt. but laws against flag desecration apply to all flags, including those u have bought and paid for and they were struck down. Contrast public nudity - ban on public nudity is constitituional. Does ban on public nudity incidentally burden a certain kind of expression? Maybe but the law against public nudity was aimed at public order effects of nudity in public places - not at trying to suppress a particular message. Example 35: Laws against public nudity are constitutional because public order must be kept, which is unrelated to speech. o The key: If the government is trying to suppress a particular message by regulating osme kind of expressive conduct, then the law will be struck down; if the government is trying to pursue an interest unrelated to the suppression of expression, then the law will be upheld.

Regulation of the Media

A. Regulation of the Media 1. No Special Privileges The press and media have no special privileges. They have the same rights as everyone else. No more, but of course no less. If question asked about freedom of press, some law that restricts media or newspaper or television and like don't need to know a lot about that subject - just ask urself could govt do this to me? If govt could restrict ur speech they can restrict the press in the same way. Freedom of press is same rights the rest of us have - no more but no less. Exam Tip 13: When asked about a law on the media, ask whether that law could be applied to you. Example 43: Confidential sources for the media. Does a reporter have a constitutional right to refuse to answer grand jury questions about a confidential source? Think of it in terms of yourself: You will not answer questions because you promised that you would not reveal a confidential source. Can the government force you to talk? Yes, you can be put in jail (held in contempt). The same is true of the media and press. Does reporter have const right to refuse to give testimony to grand jury? Ask would I have const right to refuse to testify before grand jury? Can u go before grand jury and say sorry I promised not to tell? Of course not. U have to answer grand jury questions unless pleading fifth amendment and if u don't they can hold u in contempt. Its true of normal people and newspaper reports too. Example 44: Open trials—you have the right to attend public trails and so does the press - it has the right of access to open trials. They have same rights you do.

substantive due process

A. Standards of Review • Both substantive due process and equal protection have the same three standards of review. 1. Strict Scrutiny o Is the law necessary for a compelling government interest? o Implicit in strict scrutiny is the requirement for the least restrictive means. If some less restrictive way for govt to protect its interest the means chosen is not strictly necessary. o When strict scrutiny applies, the government bears the burden of proof. The government must show that the interest is compelling and the means chosen (or law) is necessary to that interest. o Applies when there is a suspect classification or a fundamental right strict scrutiny does require least restrictive means to be used and yes, burden of proof on govt. 2. Intermediate Scrutiny o Is the law substantially related to an important government interest? o Applies in equal protection doctrine to classifications based on legitimacy and gender. Legitimacy and gender trigger intermediate scrutiny. Note 2: While three standards of review are available in both substantive due process and equal protection claims, intermediate scrutiny has only ever been used to decide cases based on equal protection grounds. 3. Rational Basis - fall back test - "mere" rationality test o Is the law rationally related to a legitimate interest or end of government? o The challenger bears the burden of proof (burden of showing the law is not rationally related to legit interest). o Low burden of proof. - rational basis test is hard to flunk - law doesn't have to be sensible or wise or humane or appropriate - just has to be non-insane. o Applies to all other cases (residual test) govt can use one interest in passing law and then defend law on different grounds.

CHAPTER 11: EQUAL PROTECTION (ALIENAGE, GENDER, AND ONE PERSON-ONE VOTE)

A. Suspect Classifications Continued • Alienage—Requirement of U.S. Citizenship o Classifications based upon U.S. citizenship are generally suspect classifications that require a compelling interest, but two important exceptions to strict scrutiny for alienage apply: § Federal government • Congress has plenary power over citizenship, immigration, and naturalization. • Federal classifications based on U.S. citizenship do not trigger strict scrutiny. • Federal classifications are valid unless arbitrary and unreasonable. Otherwise theyre upheld if not arbitrary and unreasonable. § State and local participation in government functions • These are jobs that have a particular relevance to the role of government and non-U.S. citizens can be barred from these jobs. • States and localities may require U.S. citizenship for participation in government functions, including voting, serving on a jury, and working in any kind of government law enforcement position (including probation and parole officers), or as a public school teacher. o Rule: States and localities cannot require U.S. citizenship for access to private employment or for government benefits (That rule not found in any supreme court decision but accurately describes result of them all). - 2 contexts where alienage treated as suspect classification and law restricting private employment or govt benefits will be struck down. Example 30: Florida required U.S. citizenship for admission to the Florida bar. Admission to the bar controls private employment as an attorney and therefore triggered strict scrutiny. It was struck down. Example 31: Maryland required U.S. citizenship for Maryland's in-state tuition, a government benefit, a kind of subsidy of in-state students. Basing that subsidy on U.S. citizenship triggers strict scrutiny. The requirement was struck down. However, Maryland could and did validly require residency for in-state tuition. But Maryland was not entitled to discriminate against lawfully resident, taxpaying members of the Maryland community who were not U.S. citizens. Note 5: Constitutional rights of undocumented aliens—Plyler v. Doe: Undocumented aliens are not a suspect classification. Even so, states cannot deny undocumented children public education.

Non-Suspect Classifications

Age and wealth Age discrimination in employment is barred by statute, but it is not a suspect or quasi-suspect classification under the Equal Protection Clause. Constitutional test for age requirements is rational basis. o Triggers rational basis • Wealth is not a suspect or quasi-suspect classification, - Govt can and often does charge a price or sets price for particular license or something and usually paying price is much harder for poor people than people who are well off. govt can generally charge a price. But what happens if person is indigent and just doesn't have means and cant pay? When does govt have to waive fees for indigent? but the government has to waive filing fees for indigents when charging the fees would deny a fundamental right. Examples include: divorce (because marriage is a fundamental right and so is the escape from it - You cant be too poor to get married or to get divorced.); transcript for appeal of a criminal conviction (because appellate review is a fundamental right - Transcript for appeal of a criminal conviction - very often ppl convicted of crimes have to pay for transcript of the trial to provide basis for appellate review but crim defendant who is indigent and cant pay for transcript gets it anyways. Cant charge u for transcript for appeal of criminal conviction.); transcript for appeal of the termination of parental rights (parental rights are fundamental rights - if ur parental rights are terminated that's a judicial proceeding and if someoe found guilty of abandonment or neglect or abuse, if someones parental rights terminated after judicial proceeding, to have appeal of determination must provide transcript of the trial. if u can pay for transcript u pay. If u cant pay they have to give it to you anyways.) you cant be too poor to get divorced, cant be too poor to have a transcript to appeal criminal conviction, cant be poor to appeal termination of parental rights but u can be too poor to go bankrupt. § Bankruptcy filing fees do not have to be waived. Filing fee for bankruptcy does not have to be waived - If u cant scrape together the filing fee required for bankruptucy petition, you may be too poor to go bankrupt.

congress can and cannot....

Although Congress is permitted to tax and spend for the general welfare, it does not have the power to legislate for the general welfare. Thus, this is not a proper basis for a federal statute and would not support its enforcement against the state.

state tax on fed government contractor's purchase of materials?

Although a state may not tax the federal government or its instrumentalities, there is no similar restriction on the taxation of property owned by a private individual. In this case, the state sales taxes have only an indirect economic effect on the federal government. Therefore, the sales taxes are constitutional. Answer choice A is incorrect because the uniformity requirement of the Taxing and Spending Clause is imposed on the federal government in the exercise of its taxing power, not on a state government. Here, it is the state government that is imposing the sales tax, so this requirement is not constitutionally applicable.

one person, one vote important points

Although the "one person, one vote" principle that is based on the Equal Protection Clause of the Fourteenth Amendment does apply to state legislative districts, a state may draw its legislative districts on the basis of total population rather than eligible or registered voters. In addition, a variation of less than ten percent in the size of a state legislative district is rebuttably presumed to be a minor deviation that does not constitute a prima facie case for discrimination. Remember - While near-mathematical precision is required between United States congressional districts, when the variation between state legislative districts is less than ten percent, it is presumed to be a minor variation that does not constitute a prima facie case for discrimination.

government taking for public peril

Although the Fifth Amendment Takings Clause, which is applicable to the states through the Due Process Clause of the Fourteenth Amendment, requires the payment of just compensation when there has been a governmental taking of private property for public use, a government may destroy property in response to a public peril without the payment of compensation. Note - the Takings Clause applies to various types of property interests, including a leasehold interest so even if person is just leasing land and plants crops there and government tries to take crops, thats a taking. Also note - the Takings Clause does not require the government to directly benefit from the taking.

assignee standing

An assignee has standing under Article III to bring an assigned claim, even if the claim is mature at the time of assignment and the assignment is made for collection only.

Answer choice A is correct. In Hosanna - Tabor Evangelical Church and School v. EEOC, 132 S.Ct.694 (2012), the Supreme Court unanimously recognized the existence of a "ministerial exception" grounded in the First Amendment Religious Clauses that precluded the application of employment discrimination laws concerning the employment relationships between a religious organization and its ministers. Requiring a church to accept or retain an unwanted minister interferes with the internal operations of a religious organization in violation of the Free Exercise and Establishment Clauses of the First Amendment. Answer choice B is incorrect. The Supreme Court has rejected the argument that the ministerial exception violated the free association rights of a church under the First Amendment. The text of the First Amendment gives "special solicitude" to the rights of religious organizations. Answer C is incorrect, because even though the Americans with Disabilities Act is a neutral law of general applicability, the ministerial exception precludes the application of this law to a religious organization's minister. Answer choice D is incorrect because an employment claim by a minister is not saved by either the Due Process or Equal Protection clauses. The "ministerial exception" trumps other constitutional arguments.

An employee of a large non-denominational church served five years as the minister for the church's youth program. During the past year, the minister was diagnosed with severe chronic migraine headaches, which caused him to miss work one or two days per week. Subsequently, the senior pastor of the church and the board of deacons, who had final say on the hiring and firing of all ministerial staff, unanimously voted to terminate the minister's employment and provide him with a modest severance package. He filed suit against the church under the Americans with Disabilities Act, which, in part, protects both government and private employees against discrimination based on certain medical and physical disabilities, including chronic migraine headaches. The minister seeks reinstatement and back pay. The church filed a motion for summary judgment.Should the court grant the church's summary judgment motion? A Yes, because the Free Exercise clause of the First Amendment supports the dismissal. B Yes, because it violates the church's First Amendment association rights. C No, because the Americans with Disabilities Act is a neutral law of general applicability. D No, because the minister's due process and equal protection rights were violated.

A defendant was charged with the murder of a police officer. The murder was allegedly ordered by the kingpin of a drug cartel known to be operating in the city. The prosecution asked for a closed trial to protect the identity of witnesses, some of whom were children. The witnesses feared violent repercussions if they testified. The prosecution also sought a gag order prohibiting publication of information about the proceedings because establishing the prosecution's case against the defendant could require divulging sensitive information that might impair ongoing undercover investigations of other cartel members. The trial judge granted the prosecution's requests, ordered a closed trial, and issued a gag order. Members of the media, believing the case involved issues of significant public concern, challenged the trial judge's orders. The appellate court overturned the trial judge's order.What is the most likely reason for the appellate court's ruling? A There are less restrictive means of protecting the testifying victim witnesses and government interests in secrecy than a closed trial and gag order. B The First Amendment protects the public's right to attend criminal trials. C The First Amendment specifically protects the freedom of the press to publish information about matters of public concern. D The Sixth Amendment guarantees the defendant's right to a public trial.

Answer choice A is correct because restrictions on the right of the media to publish about matters of public concern and the right of the public to receive such information may only be restricted by governmental action that is narrowly tailored to further a compelling government interest (i.e., strict scrutiny analysis applies). If there are less restrictive means of accomplishing the government objectives, those less restrictive means should be used. Note that gag orders are almost always struck down because they are rarely the least restrictive means of protecting the defendant's right to a fair trial. Answer choice B is incorrect because the First Amendment protection of the public's right to attend criminal trials is not an absolute. Both the prosecution and defense may seek a closed trial. Public attendance at a criminal proceeding may be denied when it severely impairs the defendant's right to a fair trial or the trial judge finds an overriding prosecution interest that cannot be accommodated by less restrictive means. Answer choice C is incorrect because the media has no greater First Amendment rights than the general public. Answer choice D is incorrect because a defendant's Sixth Amendment rights to a public trial are not absolute.

Congress enacted a law establishing a federal life insurance exchange under which United States citizens and residents, regardless of age, can acquire affordable life insurance. Although United States citizens are not subject to a residency requirement in order to participate in the exchange, resident aliens are required to have lived in the United States for at least five years to be eligible. A resident alien who has lived in the United States for four years has filed suit in federal court, claiming an equal protection violation. The government has asserted only that the law is rationally related to a government interest, and the court has indicated that it agrees.Is the resident alien likely to prevail? A No, because Congress has plenary power over aliens under Article I of the Constitution. B No, because the law violates the Privileges or Immunities Clause of the Fourteenth Amendment. C Yes, because national origin is a suspect classification that triggers strict scrutiny. D Yes, because the government failed to show that the law is substantially related to an important government interest.

Answer choice A is correct. A classification based on alienage is subject to a different standard depending on whether the action is taken by the state or by the federal government. Courts will generally apply the strict scrutiny test and strike down state-based laws that discriminate against resident aliens for lack of U.S. citizenship. In contrast, Congress has plenary power over aliens under Article I, and the power to expel or exclude aliens is a fundamental sovereign attribute exercised by the Government's political departments largely immune from judicial control. Therefore, a federal alienage classification is likely valid unless it is deemed arbitrary and unreasonable. Here, the government proved that the law was rationally related to a legitimate governmental interest and thus the resident alien's claim will fail. Answer choice B is incorrect because the Privileges or Immunities Clause of the Fourteenth Amendment does not apply to aliens but only to United States citizens; further, this clause applies only to the states, not the federal government. Answer choice C is incorrect because national origin isn't at issue in this question; the issue is one of alienage. Answer choice D is incorrect because intermediate scrutiny applies to quasi-suspect classifications based on gender and nonmarital child status, but not to a federal alienage classification.

Congress enacted a law establishing a federal life insurance exchange under which United States citizens and residents, regardless of age, can acquire affordable life insurance. Although United States citizens are not subject to a residency requirement in order to participate in the exchange, resident aliens are required to have lived in the United States for at least five years to be eligible. A resident alien who has lived in the United States for four years has filed suit in federal court, claiming an equal protection violation. The government has asserted only that the law is rationally related to a government interest, and the court has indicated that it agrees.Is the resident alien likely to prevail? A No, because Congress has plenary power over aliens under Article I of the Constitution. B No, because the law violates the Privileges or Immunities Clause of the Fourteenth Amendment. C Yes, because national origin is a suspect classification that triggers strict scrutiny. D Yes, because the government failed to show that the law is substantially related to an important government interest.

Answer choice A is correct. A classification based on alienage is subject to a different standard depending on whether the action is taken by the state or by the federal government. Courts will generally apply the strict scrutiny test and strike down state-based laws that discriminate against resident aliens for lack of U.S. citizenship. In contrast, Congress has plenary power over aliens under Article I, and the power to expel or exclude aliens is a fundamental sovereign attribute exercised by the Government's political departments largely immune from judicial control. Therefore, a federal alienage classification is likely valid unless it is deemed arbitrary and unreasonable. Here, the government proved that the law was rationally related to a legitimate governmental interest and thus the resident alien's claim will fail. Answer choice B is incorrect because the Privileges or Immunities Clause of the Fourteenth Amendment does not apply to aliens but only to United States citizens; further, this clause applies only to the states, not the federal government. Answer choice C is incorrect because national origin isn't at issue in this question; the issue is one of alienage. Answer choice D is incorrect because intermediate scrutiny applies to quasi-suspect classifications based on gender and nonmarital child status, but not to a federal alienage classification.

A potential candidate for city council publicly expresses his opinion to the media that although he has not yet officially declared his candidacy, he is not optimistic about his chances of winning if he were to run for a city council position within a particular political party. Because he has an interest in both running and winning, he therefore publicly declares that he would like to be placed on the ballot as a member of "every party possible" in order to increase his chances. The elections commissioner publicly declares that if the potential candidate attempts to "spread his luck around the ballot," he will be prohibited from being placed on the ballot at all. The potential candidate sues in federal court, stating that such a prohibition is unfair.How should the court proceed? A Dismiss the case, because the commissioner has not prohibited the potential candidate's inclusion on the ballot. B Dismiss the case, because the city has the right to regulate ballot access in any way it sees fit. C Dismiss the case, because the question is a political one that the court has no standing to hear. D Rule in favor of the potential candidate because a candidate may appear on the ballot as a member of multiple parties.

Answer choice A is correct. A federal court will not consider a claim before it has fully developed; to do so would be premature, and any potential injury would be speculative. In order for a case to be "ripe" for litigation, the plaintiff must have experienced a real injury (or imminent threat thereof). Here, because the commissioner has only made a statement of his intentions but has not taken any action preventing the potential candidate from running or placing his name on the ballot (nor has the candidate even attempted to do so), the candidate has not suffered a real injury. Answer choice B is incorrect because, while it is true that a city has a legitimate interest in regulating ballot access, there are limits to this regulation. For example, a city cannot require that a candidate own property and restrictions cannot be discriminatory. Answer choice C is incorrect because this question is not a political question (i.e., one to be resolved by one or both of the other two branches of government). Courts do have jurisdiction to adjudicate the constitutionality of a state's election process. Answer choice D is incorrect because such a ruling would be unlikely and the court cannot yet even get to the merits of the case.

A potential candidate for city council publicly expresses his opinion to the media that although he has not yet officially declared his candidacy, he is not optimistic about his chances of winning if he were to run for a city council position within a particular political party. Because he has an interest in both running and winning, he therefore publicly declares that he would like to be placed on the ballot as a member of "every party possible" in order to increase his chances. The elections commissioner publicly declares that if the potential candidate attempts to "spread his luck around the ballot," he will be prohibited from being placed on the ballot at all. The potential candidate sues in federal court, stating that such a prohibition is unfair.How should the court proceed? A Dismiss the case, because the commissioner has not prohibited the potential candidate's inclusion on the ballot. B Dismiss the case, because the city has the right to regulate ballot access in any way it sees fit. C Dismiss the case, because the question is a political one that the court has no standing to hear. D Rule in favor of the potential candidate because a candidate may appear on the ballot as a member of multiple parties.

Answer choice A is correct. A federal court will not consider a claim before it has fully developed; to do so would be premature, and any potential injury would be speculative. In order for a case to be "ripe" for litigation, the plaintiff must have experienced a real injury (or imminent threat thereof). Here, because the commissioner has only made a statement of his intentions but has not taken any action preventing the potential candidate from running or placing his name on the ballot (nor has the candidate even attempted to do so), the candidate has not suffered a real injury. Answer choice B is incorrect because, while it is true that a city has a legitimate interest in regulating ballot access, there are limits to this regulation. For example, a city cannot require that a candidate own property and restrictions cannot be discriminatory. Answer choice C is incorrect because this question is not a political question (i.e., one to be resolved by one or both of the other two branches of government). Courts do have jurisdiction to adjudicate the constitutionality of a state's election process. Answer choice D is incorrect because such a ruling would be unlikely and the court cannot yet even get to the merits of the case.

A prominent broker was accused of insider trading. During his trial, he revealed the name of a colleague alleged to have provided him with the insider information. The colleague was subsequently proven to be innocent. In a ruling that was arguably incorrect per the state's court rules, the judge had allowed a live-feed of the court proceedings to occur on the Internet. As a result, the colleague's reputation was immediately ruined. His business was irrevocably damaged, and he was forced to file for bankruptcy. The colleague filed suit against the judge, alleging that had the judge not allowed court proceedings to be viewed on the Internet, his reputation and business would have remained intact.Will the colleague's lawsuit withstand a motion for summary judgment? A No, because the judge is immune from liability. B No, because it is not clear that the judge committed any error. C Yes, because the damage to the colleague's reputation occurred outside of judicial proceedings. D Yes, because the judge committed a grave procedural error.

Answer choice A is correct. A judge is absolutely immune from civil liability for judicial acts, including grave procedural errors. The judge is not immune, however, to lawsuits regarding non-judicial activities. The colleague filed a claim on the basis that the judge erroneously allowed the proceedings to be viewed by the public and should be liable for the damage that results. The judge is immune from such a suit. Answer choice B is incorrect because the judge would still be immune from civil liability even if he had committed an error, as the suit arises out of his judicial acts. Answer choice C is incorrect because while a judge is not immune from lawsuits regarding non-judicial activities, this harm resulted from the judge's decision regarding the conduct of the trial. The fact that the damage occurred outside the courtroom does not prevent the application of judicial immunity. Answer choice D is incorrect because even if the judge's action does constitute a grave procedural error, he would still be immune from civil liability.

A prominent broker was accused of insider trading. During his trial, he revealed the name of a colleague alleged to have provided him with the insider information. The colleague was subsequently proven to be innocent. In a ruling that was arguably incorrect per the state's court rules, the judge had allowed a live-feed of the court proceedings to occur on the Internet. As a result, the colleague's reputation was immediately ruined. His business was irrevocably damaged, and he was forced to file for bankruptcy. The colleague filed suit against the judge, alleging that had the judge not allowed court proceedings to be viewed on the Internet, his reputation and business would have remained intact.Will the colleague's lawsuit withstand a motion for summary judgment? A No, because the judge is immune from liability. B No, because it is not clear that the judge committed any error. C Yes, because the damage to the colleague's reputation occurred outside of judicial proceedings. D Yes, because the judge committed a grave procedural error.

Answer choice A is correct. A judge is absolutely immune from civil liability for judicial acts, including grave procedural errors. The judge is not immune, however, to lawsuits regarding non-judicial activities. The colleague filed a claim on the basis that the judge erroneously allowed the proceedings to be viewed by the public and should be liable for the damage that results. The judge is immune from such a suit. Answer choice B is incorrect because the judge would still be immune from civil liability even if he had committed an error, as the suit arises out of his judicial acts. Answer choice C is incorrect because while a judge is not immune from lawsuits regarding non-judicial activities, this harm resulted from the judge's decision regarding the conduct of the trial. The fact that the damage occurred outside the courtroom does not prevent the application of judicial immunity. Answer choice D is incorrect because even if the judge's action does constitute a grave procedural error, he would still be immune from civil liability.

A policyholder sued her casualty insurance company for failing to timely pay a claim. The case reached the highest court of the state. The sole issue before the court was whether the policyholder was entitled to punitive damages. The company timely moved to recuse one of the judges on the ground that she was pursuing a similar action against the insurance company seeking punitive damages in state court. The judge determined that state law did not require her recusal, and she cast the deciding vote in the court's punitive damages decision in favor of the policyholder. The company timely and properly filed a petition for a writ of certiorari, seeking a review of the judge's decision not to recuse herself.May the Supreme Court hear this case? A Yes, because the judge's refusal to recuse herself violated the Due Process Clause of the Fourteenth Amendment. B Yes, because the Supreme Court may correct any erroneous ruling by a state judge. C No, because the issue is moot. D No, because the judge's refusal to recuse herself was based on adequate and independent state grounds.

Answer choice A is correct. Due process entitles a person to a fair decision maker. A judge must recuse herself when she has a direct, personal, substantial, pecuniary interest in a case or there is a serious risk of actual bias. The judge's refusal to recuse herself likely violated the Due Process Clause of the Fourteenth Amendment because there was a serious risk of actual bias toward the insurance company since the judge had a similar action against the same insurance company. Answer choice B is incorrect because the Supreme Court may not correct any erroneous ruling by a state judge unless there is a federal issue. Answer choice C is incorrect because the issue is not moot. An issue is moot if further legal proceedings would have no effect, or there is no longer a controversy. Since the Supreme Court can reverse the decision and order the state court to conduct further proceedings, the issue is not moot. Answer choice D is incorrect because, although the judge's decision was based on state law, it was contrary to the Due Process Clause. Accordingly, there is a federal issue to be decided by the Supreme Court.

A policyholder sued her casualty insurance company for failing to timely pay a claim. The case reached the highest court of the state. The sole issue before the court was whether the policyholder was entitled to punitive damages. The company timely moved to recuse one of the judges on the ground that she was pursuing a similar action against the insurance company seeking punitive damages in state court. The judge determined that state law did not require her recusal, and she cast the deciding vote in the court's punitive damages decision in favor of the policyholder. The company timely and properly filed a petition for a writ of certiorari, seeking a review of the judge's decision not to recuse herself.May the Supreme Court hear this case? A Yes, because the judge's refusal to recuse herself violated the Due Process Clause of the Fourteenth Amendment. B Yes, because the Supreme Court may correct any erroneous ruling by a state judge. C No, because the issue is moot. D No, because the judge's refusal to recuse herself was based on adequate and independent state grounds.

Answer choice A is correct. Due process entitles a person to a fair decision maker. A judge must recuse herself when she has a direct, personal, substantial, pecuniary interest in a case or there is a serious risk of actual bias. The judge's refusal to recuse herself likely violated the Due Process Clause of the Fourteenth Amendment because there was a serious risk of actual bias toward the insurance company since the judge had a similar action against the same insurance company. Answer choice B is incorrect because the Supreme Court may not correct any erroneous ruling by a state judge unless there is a federal issue. Answer choice C is incorrect because the issue is not moot. An issue is moot if further legal proceedings would have no effect, or there is no longer a controversy. Since the Supreme Court can reverse the decision and order the state court to conduct further proceedings, the issue is not moot. Answer choice D is incorrect because, although the judge's decision was based on state law, it was contrary to the Due Process Clause. Accordingly, there is a federal issue to be decided by the Supreme Court.

Concerned over the increasing frequency of livestock theft, the legislature of a state with a sizable number of ranches passed legislation, which the governor signed into law, creating the position of livestock identity inspector within the state Department of Agriculture. The inspector is tasked with enforcing the state laws regarding branding of livestock. The state statute confined this position to U.S. citizens. A longtime resident of the state who was a citizen of a foreign country applied for the position and was rejected. He wants to seek injunctive relief against the state Secretary of Agriculture to prevent her from enforcing this requirement as it applies to him. Of the following, which is the best federal constitutional ground upon which the plaintiff can base this action?

Answer choice A is correct. Generally, the strict scrutiny test applies to state laws that discriminate against resident aliens. However, a growing exception exists requiring only a rational relationship to a legitimate state interest for state laws that restrict or prohibit a resident alien's participation in government functions. In this case, the position of livestock identity inspector likely falls within this category of positions because it involves enforcing the state's livestock identity laws. However, the constitutional provisions in the other answer choices either fail to protect the plaintiff as a resident alien or fail to address his concern of being denied a state job as an inspector. Therefore, although the exception regarding participation in government functions may be problematic for the plaintiff (a resident alien), the Equal Protection Clause of the Fourteenth Amendment still provides the best federal constitutional ground for his action. Answer choice B is incorrect because the Privileges or Immunities Clause of the Fourteenth Amendment protects only U.S. citizens from infringement by the states upon the privileges or immunities of national citizenship. It does not apply to resident aliens. Answer choice C is incorrect. Although the Contract Clause does apply to state legislative action, that action must affect existing contractual rights. Here, the plaintiff was not working for the state under an employment contract at the time that the statute was enacted. Instead, it was the statute itself that gave rise to the position that the plaintiff sought. Therefore, this provision will not support his cause of action. Answer choice D is incorrect because the Comity Clause of Article IV, Section 2 only protects U.S. citizens against discrimination by a state other than the state of which they are a citizen. It does not apply to resident aliens.

A state anticipates a possible reduction of its native oyster population due to a variety of factors, including non-customary weather patterns and polluted waters. To discourage excessive fishing until oyster numbers are confirmed for the season, the state enacts a statute raising the cost of oyster-fishing licenses for all out-of-state residents to twenty times that of in-state residents. The statute does not distinguish between commercial and recreational oyster fishing. An out-of-state commercial oyster fisherman files suit in federal court to have the statute struck down.Is the statute constitutional? A No, because the statute violates the Privileges and Immunities Clause of Article IV. B No, because the statute violates the Privileges or Immunities Clause of the Fourteenth Amendment. C Yes, because the statute does not discriminate against out-of-state commerce. D Yes, because the statute also applies to recreational oyster fishing.

Answer choice A is correct. Nonresident citizens are protected against discrimination with respect to fundamental rights or essential activities, including the pursuit of employment, transfer of property, access to state courts, and engaging in the political process. Discrimination against out-of-state residents in setting the fee for a commercial activity, such as the oyster-fishing license here, violates the Privileges and Immunities Clause of Article IV, but similar discrimination for a recreational activity, such as a recreational hunting license, does not, if there is a rational basis for the fee differential. Here, the state is discriminating against the out-of-state oyster fisherman by setting a much higher fee for a commercial activity. Note that this would also likely violate the Dormant Commerce Clause as well, but that answer is not provided as a choice. Answer choice B is incorrect because the Privileges or Immunities Clause of the Fourteenth Amendment applies only to a state's treatment of its own citizens, and is thus irrelevant to this case (in which a state is dealing with out-of-staters). Further, it has a limited application, and is usually only successful in cases involving the right to travel. Here, the state statute does not infringe anyone's right to travel. Answer choice C is incorrect because the statute does discriminate against out-of-state commerce, as it negatively impacts only out-of-state oyster fishermen who are in the state for commercial purposes. Answer choice D is incorrect because if the statute only impacted a recreational activity, the state might be able to meet a rational basis test. However, because commercial activity is affected here, that is irrelevant.

A state anticipates a possible reduction of its native oyster population due to a variety of factors, including non-customary weather patterns and polluted waters. To discourage excessive fishing until oyster numbers are confirmed for the season, the state enacts a statute raising the cost of oyster-fishing licenses for all out-of-state residents to twenty times that of in-state residents. The statute does not distinguish between commercial and recreational oyster fishing. An out-of-state commercial oyster fisherman files suit in federal court to have the statute struck down.Is the statute constitutional? A No, because the statute violates the Privileges and Immunities Clause of Article IV. B No, because the statute violates the Privileges or Immunities Clause of the Fourteenth Amendment. C Yes, because the statute does not discriminate against out-of-state commerce. D Yes, because the statute also applies to recreational oyster fishing.

Answer choice A is correct. Nonresident citizens are protected against discrimination with respect to fundamental rights or essential activities, including the pursuit of employment, transfer of property, access to state courts, and engaging in the political process. Discrimination against out-of-state residents in setting the fee for a commercial activity, such as the oyster-fishing license here, violates the Privileges and Immunities Clause of Article IV, but similar discrimination for a recreational activity, such as a recreational hunting license, does not, if there is a rational basis for the fee differential. Here, the state is discriminating against the out-of-state oyster fisherman by setting a much higher fee for a commercial activity. Note that this would also likely violate the Dormant Commerce Clause as well, but that answer is not provided as a choice. Answer choice B is incorrect because the Privileges or Immunities Clause of the Fourteenth Amendment applies only to a state's treatment of its own citizens, and is thus irrelevant to this case (in which a state is dealing with out-of-staters). Further, it has a limited application, and is usually only successful in cases involving the right to travel. Here, the state statute does not infringe anyone's right to travel. Answer choice C is incorrect because the statute does discriminate against out-of-state commerce, as it negatively impacts only out-of-state oyster fishermen who are in the state for commercial purposes. Answer choice D is incorrect because if the statute only impacted a recreational activity, the state might be able to meet a rational basis test. However, because commercial activity is affected here, that is irrelevant.

A municipality that owned a public swimming pool normally uses government funds to hire lifeguards and other staff and to maintain the pool. During hours when the pool closed, the municipality rents the pool out to any private group with appropriate liability insurance that also provides their own lifeguards and other staff. A private group rented the public pool in order to host an "All-American" swim competition to raise funds for veterans. The competition was open to the general public. However, because the group wanted to promote patriotism, all participants in the competition were required to be U.S. citizens. A resident alien wanted to participate in the swim competition, but was refused entry. The man has sued the group in the appropriate court for violating his rights to equal treatment under the Fourteenth Amendment.Is the man likely to prevail in his action? A No, because the group is not a state actor and is therefore not subject to the Fourteenth Amendment. B No, because resident aliens are not a protected class under the Fourteenth Amendment. C Yes, because the Fourteenth Amendment prohibits the group from discriminating against members of the public based upon their alienage. D Yes, because using a public swimming pool owned and run by the municipality subjects the group to the Fourteenth Amendment.

Answer choice A is correct. The Constitution generally only protects against wrongful conduct by the government, unless a private person's conduct constitutes "state action." State action exists when private parties carry out traditional governmental functions or when there is significant state involvement in the activities. Here, a swim competition run by a private group does not qualify as a traditional government function, even if it occurs at a facility that uses government funds during business hours. In addition, there was no significant state involvement because for the duration of the swim competition, the group was required to provide its own employees to run the competition and maintain the pool. Thus, the group is not subject to the Fourteenth Amendment, and there is not enough state entanglement to justify this action against them. Answer choice B is incorrect because aliens are a protected class. However, as stated above, this action cannot be brought against the group because the Fourteenth Amendment does not protect against discrimination by private parties. Answer choice C is incorrect because it disregards the requirement of state action necessary for this action to succeed. Answer choice D is incorrect because the group's mere use of a public swimming pool is not enough to constitute state action. Although state action may exist if there are sufficient mutual contacts between the conduct of a private party and the government to find that the government is so pervasively entwined with the private entity that constitutional standards should apply to the private actor, that requirement is not met merely by using a facility that has used government funds.

A municipality that owned a public swimming pool normally uses government funds to hire lifeguards and other staff and to maintain the pool. During hours when the pool closed, the municipality rents the pool out to any private group with appropriate liability insurance that also provides their own lifeguards and other staff. A private group rented the public pool in order to host an "All-American" swim competition to raise funds for veterans. The competition was open to the general public. However, because the group wanted to promote patriotism, all participants in the competition were required to be U.S. citizens. A resident alien wanted to participate in the swim competition, but was refused entry. The man has sued the group in the appropriate court for violating his rights to equal treatment under the Fourteenth Amendment.Is the man likely to prevail in his action? A No, because the group is not a state actor and is therefore not subject to the Fourteenth Amendment. B No, because resident aliens are not a protected class under the Fourteenth Amendment. C Yes, because the Fourteenth Amendment prohibits the group from discriminating against members of the public based upon their alienage. D Yes, because using a public swimming pool owned and run by the municipality subjects the group to the Fourteenth Amendment.

Answer choice A is correct. The Constitution generally only protects against wrongful conduct by the government, unless a private person's conduct constitutes "state action." State action exists when private parties carry out traditional governmental functions or when there is significant state involvement in the activities. Here, a swim competition run by a private group does not qualify as a traditional government function, even if it occurs at a facility that uses government funds during business hours. In addition, there was no significant state involvement because for the duration of the swim competition, the group was required to provide its own employees to run the competition and maintain the pool. Thus, the group is not subject to the Fourteenth Amendment, and there is not enough state entanglement to justify this action against them. Answer choice B is incorrect because aliens are a protected class. However, as stated above, this action cannot be brought against the group because the Fourteenth Amendment does not protect against discrimination by private parties. Answer choice C is incorrect because it disregards the requirement of state action necessary for this action to succeed. Answer choice D is incorrect because the group's mere use of a public swimming pool is not enough to constitute state action. Although state action may exist if there are sufficient mutual contacts between the conduct of a private party and the government to find that the government is so pervasively entwined with the private entity that constitutional standards should apply to the private actor, that requirement is not met merely by using a facility that has used government funds.

An individual, invoking her status as an American citizen and taxpayer, filed suit in an appropriate federal court seeking an order to remove the President from office. The individual asserted that the President was not born in the United States and was therefore ineligible to serve as President, because the Constitution provides that the President must have been born in the United States.Which doctrine would the court likely rely upon to dismiss this suit? A Standing B Ripeness C The political question doctrine D Abstention

Answer choice A is correct. The plaintiff does not have standing merely because she is a citizen or a taxpayer; she must show that she has suffered a particularized "injury in fact" different from that suffered by all other citizens, which she cannot do in this case. Answer choice B is incorrect because the injury of which the plaintiff is complaining, service as president by a person who is not eligible to do so, is presently occurring. Answer choice C is incorrect because the court will determine whether the plaintiff is legally qualified to bring the claim before considering the substance of the claim. Thus, Answer choice A is the better choice. Note, however, that courts generally will not rule on matters of impeachment under the political question doctrine because those procedures are assigned to Congress. Answer choice D is incorrect because the abstention doctrine is inapplicable; there are no strong state interests at stake.

An individual, invoking her status as an American citizen and taxpayer, filed suit in an appropriate federal court seeking an order to remove the President from office. The individual asserted that the President was not born in the United States and was therefore ineligible to serve as President, because the Constitution provides that the President must have been born in the United States.Which doctrine would the court likely rely upon to dismiss this suit? A Standing B Ripeness C The political question doctrine D Abstention

Answer choice A is correct. The plaintiff does not have standing merely because she is a citizen or a taxpayer; she must show that she has suffered a particularized "injury in fact" different from that suffered by all other citizens, which she cannot do in this case. Answer choice B is incorrect because the injury of which the plaintiff is complaining, service as president by a person who is not eligible to do so, is presently occurring. Answer choice C is incorrect because the court will determine whether the plaintiff is legally qualified to bring the claim before considering the substance of the claim. Thus, Answer choice A is the better choice. Note, however, that courts generally will not rule on matters of impeachment under the political question doctrine because those procedures are assigned to Congress. Answer choice D is incorrect because the abstention doctrine is inapplicable; there are no strong state interests at stake.

A federal statute provides that the Secretary of Education may reduce by six percent the distribution of federal education funds to a state that fails to require local elementary schools to observe February 11, the birthday of Thomas Edison, as Inventors' Day.Of the following, which provides the best support for the Congressional power to enact this statute? A The "General Welfare" Clause of Article I, Section 8 of the United States Constitution B The "Progress of Science and the Useful Arts" Clause of Article I, Section 8 of the United States Constitution C The "Necessary and Proper" Clause of Article I, Section 8 of the United States Constitution D The Tenth Amendment to the United States Constitution

Answer choice A is correct. This clause gives Congress the power to spend for the general welfare i.e., any public purpose—not just to pursue its other enumerated powers. Although Congress cannot "commandeer" state legislatures by commanding them to enact specific legislation, Congress can use its spending power to accomplish such a result indirectly by conditioning the receipt of federal funding on enacting such legislation. Congressional "encouragement" cannot exceed that point at which pressure turns into compulsion, but because this statute only limits funding by six percent, rather than eliminating it totally or by a large, coercive amount, that does not appear to be applicable on these facts. Answer choice B is incorrect because this clause is limited to the promotion of science and the useful arts through the granting of patents to inventors. Answer choice C is incorrect because this clause is not an independent source of Congressional power. Answer choice D is incorrect because the Tenth Amendment, which provides that "[t]he powers not delegated to the United States by the Constitution, nor prohibited by it to the States, are reserved to the States respectively, or to the people," would serve as a limitation on Congressional power to enact this statute, not as support for it.

As part of the redistricting of state legislative districts triggered by the decennial census, the state legislature created a "majority-minority" district for the state senate in which black voters were in the majority. In creating this district, race was the predominant factor, because the intent was to favor black voters who have been historically disenfranchised in the state. A white voter in the district has filed a challenge to the creation of this district under the Equal Protection Clause. Seventy percent of the state's population is white.Is the court required to apply the strict scrutiny standard in examining the creation of this district? A Yes, because race was the predominant factor in creating the majority-minority district. B Yes, because the plaintiff is a racial minority in the majority-minority district. C No, because seventy percent of the state's population is white. D No, because the redistricting favors voters who have been historically disenfranchised in the state.

Answer choice A is correct. Under the Equal Protection Clause, election districts for public office may not be drawn using race as the predominant factor in determining the boundary lines, unless the district plan can survive strict scrutiny. Note that the district may be able to pass the strict scrutiny test if it can be shown that the district was drawn in compliance with federal Voting Rights Act, but the call of the question only asks whether strict scrutiny should apply. Answer choice B is incorrect because, while the plaintiff's status as a voter in the challenged district gives the plaintiff standing to bring this challenge, the fact that the plaintiff is a racial minority in the district does not require the court to apply strict scrutiny in determining whether the district violates the Equal Protection Clause. Answer choice C is incorrect because the fact that the plaintiff is not a racial minority in the state does not prevent the court from applying the strict scrutiny standard in determining whether the district violates the Equal Protection Clause. Answer choice D is incorrect because the strict scrutiny standard applies even when the district is drawn to favor historically disenfranchised groups if race was the predominant factor in creating the district.

As part of the redistricting of state legislative districts triggered by the decennial census, the state legislature created a "majority-minority" district for the state senate in which black voters were in the majority. In creating this district, race was the predominant factor, because the intent was to favor black voters who have been historically disenfranchised in the state. A white voter in the district has filed a challenge to the creation of this district under the Equal Protection Clause. Seventy percent of the state's population is white.Is the court required to apply the strict scrutiny standard in examining the creation of this district? A Yes, because race was the predominant factor in creating the majority-minority district. B Yes, because the plaintiff is a racial minority in the majority-minority district. C No, because seventy percent of the state's population is white. D No, because the redistricting favors voters who have been historically disenfranchised in the state.

Answer choice A is correct. Under the Equal Protection Clause, election districts for public office may not be drawn using race as the predominant factor in determining the boundary lines, unless the district plan can survive strict scrutiny. Note that the district may be able to pass the strict scrutiny test if it can be shown that the district was drawn in compliance with federal Voting Rights Act, but the call of the question only asks whether strict scrutiny should apply. Answer choice B is incorrect because, while the plaintiff's status as a voter in the challenged district gives the plaintiff standing to bring this challenge, the fact that the plaintiff is a racial minority in the district does not require the court to apply strict scrutiny in determining whether the district violates the Equal Protection Clause. Answer choice C is incorrect because the fact that the plaintiff is not a racial minority in the state does not prevent the court from applying the strict scrutiny standard in determining whether the district violates the Equal Protection Clause. Answer choice D is incorrect because the strict scrutiny standard applies even when the district is drawn to favor historically disenfranchised groups if race was the predominant factor in creating the district.

Within the boundaries of a city was a sizeable lake that contained several uninhabited islands. A buyer purchased one of these islands as an excellent location upon which to build a quiet vacation home. Before the buyer entered into a contract to construct the home, the city passed an ordinance. The ordinance prohibited any person from erecting permanent structures upon any island in the lake in order to preserve the islands in their natural state, mainly for aesthetic reasons. The ordinance left the buyer's property without an economically viable use.Was there a taking? A Yes, because the island was without an economically viable use. B Yes, because the ordinance did not advance a legitimate governmental interest. C No, because the city did not physically occupy the property. D No, because the buyer had not yet entered the contract to build the home.

Answer choice A is correct. When a regulation results in a permanent and total loss of a property's economic value, a taking has occurred. In this case, the city ordinance prohibits any person from erecting permanent structures on the island. Because the buyer's property was left without an economically viable use as a result of the ordinance, the ordinance constituted a taking. Answer choice B is incorrect. A regulation restricting property use, such as the ordinance here, need not advance a legitimate governmental interest to be valid. In addition, aesthetic concerns can constitute a legitimate governmental interest. Answer choice C is incorrect. Physical occupation of property is not necessary for a taking to exist. A regulation resulting in a permanent total loss of the property's economic value is sufficient. Answer choice D is incorrect. The issue of the amount of compensation to which a property owner is entitled is separate from the issue of whether the governmental action has resulted in a taking.

Within the boundaries of a city was a sizeable lake that contained several uninhabited islands. A buyer purchased one of these islands as an excellent location upon which to build a quiet vacation home. Before the buyer entered into a contract to construct the home, the city passed an ordinance. The ordinance prohibited any person from erecting permanent structures upon any island in the lake in order to preserve the islands in their natural state, mainly for aesthetic reasons. The ordinance left the buyer's property without an economically viable use.Was there a taking? A Yes, because the island was without an economically viable use. B Yes, because the ordinance did not advance a legitimate governmental interest. C No, because the city did not physically occupy the property. D No, because the buyer had not yet entered the contract to build the home.

Answer choice A is correct. When a regulation results in a permanent and total loss of a property's economic value, a taking has occurred. In this case, the city ordinance prohibits any person from erecting permanent structures on the island. Because the buyer's property was left without an economically viable use as a result of the ordinance, the ordinance constituted a taking. Answer choice B is incorrect. A regulation restricting property use, such as the ordinance here, need not advance a legitimate governmental interest to be valid. In addition, aesthetic concerns can constitute a legitimate governmental interest. Answer choice C is incorrect. Physical occupation of property is not necessary for a taking to exist. A regulation resulting in a permanent total loss of the property's economic value is sufficient. Answer choice D is incorrect. The issue of the amount of compensation to which a property owner is entitled is separate from the issue of whether the governmental action has resulted in a taking.

A student joined a small national organization during her freshman year of college after several of her friends, who were active in the organization, told her about the organization's annual all-expense paid ski trip for its card-carrying members. The student attended a recruitment drive, signed a pledge of loyalty to the party, paid her annual dues, and received an organization pin. The student never attended a meeting or read the organization's literature she was given. The student later joined other members of the organization at the ski resort. During a meeting around the ski lodge fireplace, the student learned for the first time that the organization was a radical organization. The organization's members were preparing to use subversive means to achieve their objective of installing the organization's spiritual leader as Supreme Dictator of the United States. To this end, the organization was stockpiling mind control serum and planned to poison the nation's water supply. After returning home, the student consciously avoided members of the organization and never participated in the organization's activities again, but the student's name remained on the organization's active member roster. Three years later, the student neared graduation and applied for federal employment.Can the student be denied public employment based upon her membership in this organization? A No, because the student did not personally participate in the organization's subversive plot. B No, because the student did not intend to install the organization's spiritual leader as Supreme Dictator. C Yes, because the student continues to be listed as an active member of a subversive organization. D Yes, because the student has knowledge of the organization's illegal objectives and failed to report them to the authorities.

Answer choice B is correct because the student lacked the specific intent to further the organization's illegal objectives; this intent is necessary in order to punish her based upon her association with the group. The First Amendment's freedom of association protects the right to form or participate in any group, club, or other organization virtually without restriction, but the right is not absolute. The government may justifiably infringe the right of association to advance a compelling state interest, such as preventing discrimination or subversive activities designed to undermine the democratic process. A person may only be punished or deprived of public employment based on their political affiliation if that individual (i) is an active member of a subversive organization, (ii) has knowledge of the organization's illegal activities, and (iii) has a specific intent to further those illegal objectives. Answer choice A is incorrect because a person does not have to personally or physically participate in a subversive organization's overt activities to be punished if the person is a member of that organization, aware of the organization's illicit goals, and intends that those subversive objectives be furthered. Answer choice C is incorrect because her inclusion on the active member list is insufficient for purposes of denying her public employment. Answer choice D is incorrect because an individual cannot be denied public employment based simply upon knowledge of the organization's objectives. She would also need the intent to further those objectives.

A state enacted a law creating strict requirements for obtaining a license to operate a taxi within the state. The law was passed in response to a number of serious accidents involving taxi drivers in the state's major cities. The law required a person seeking a license to operate a taxi to pass three road tests, a difficult written examination, and pay a $150 licensing fee. The law also required yearly recertification. Ninety percent of the taxi drivers in the state are men. Two of these male drivers have filed suit challenging this law as a violation of the Equal Protection Clause of the Fourteenth Amendment.Is the challenge likely to be successful? A No, because it is necessary to achieve a compelling state interest. B No, because it is rationally related to a legitimate state interest. C Yes, because it places an undue burden on taxi drivers. D Yes, because it has a disparate impact on men.

Answer choice B is correct. A law that is neutral on its face and in its application may still result in a disparate impact. By itself, however, a disparate impact is not sufficient to trigger strict or intermediate scrutiny; proof of discriminatory motive or intent is required to show a violation of the Equal Protection Clause. In this case, the law has a disparate impact on men because 90% of taxi drivers in the state are men. However, there are no facts that indicate that the state had a discriminatory motive or intent in enacting the law; rather, the law was passed in response to a number of serious accidents involving taxi drivers. Because discriminatory intent cannot be shown, the law does not discriminate on the basis of gender. Therefore, the law is only subject to rational basis review, which would be satisfied here. Answer choice A is incorrect. Even if discrimination based on gender was present here, the state would only need to show that the law is substantially related to an important governmental interest. The strict scrutiny standard, articulated in this answer choice, is not applicable here. Answer choice C is incorrect. The undue burden standard of review is not applicable in assessing the constitutionality of a gender-based, Equal Protection challenge. Answer choice D is incorrect because a disparate impact is not enough to prove gender discrimination in violation of the Equal Protection Clause; there must also be discriminatory intent or motive.

A state enacted a law creating strict requirements for obtaining a license to operate a taxi within the state. The law was passed in response to a number of serious accidents involving taxi drivers in the state's major cities. The law required a person seeking a license to operate a taxi to pass three road tests, a difficult written examination, and pay a $150 licensing fee. The law also required yearly recertification. Ninety percent of the taxi drivers in the state are men. Two of these male drivers have filed suit challenging this law as a violation of the Equal Protection Clause of the Fourteenth Amendment.Is the challenge likely to be successful? A No, because it is necessary to achieve a compelling state interest. B No, because it is rationally related to a legitimate state interest. C Yes, because it places an undue burden on taxi drivers. D Yes, because it has a disparate impact on men.

Answer choice B is correct. A law that is neutral on its face and in its application may still result in a disparate impact. By itself, however, a disparate impact is not sufficient to trigger strict or intermediate scrutiny; proof of discriminatory motive or intent is required to show a violation of the Equal Protection Clause. In this case, the law has a disparate impact on men because 90% of taxi drivers in the state are men. However, there are no facts that indicate that the state had a discriminatory motive or intent in enacting the law; rather, the law was passed in response to a number of serious accidents involving taxi drivers. Because discriminatory intent cannot be shown, the law does not discriminate on the basis of gender. Therefore, the law is only subject to rational basis review, which would be satisfied here. Answer choice A is incorrect. Even if discrimination based on gender was present here, the state would only need to show that the law is substantially related to an important governmental interest. The strict scrutiny standard, articulated in this answer choice, is not applicable here. Answer choice C is incorrect. The undue burden standard of review is not applicable in assessing the constitutionality of a gender-based, Equal Protection challenge. Answer choice D is incorrect because a disparate impact is not enough to prove gender discrimination in violation of the Equal Protection Clause; there must also be discriminatory intent or motive.

A tenured professor at a state university was summarily dismissed by the university president after it came to light that she had engaged in plagiarism. Shortly following her dismissal, the professor contested her termination in a post-termination evidentiary hearing, at which her termination was upheld. The professor then filed an action in federal court, asserting that she was denied due process of law.Were the professor's due process rights violated? A Yes, because the professor was not granted a full evidentiary hearing prior to termination. B Yes, because the professor was not provided with a pre-termination opportunity to respond. C No, because the professor could be terminated without cause, so no due process was required. D No, because the professor received notice of her dismissal and a post-termination hearing.

Answer choice B is correct. A public employee who may be discharged only for cause has a property interest in her job and therefore is entitled to notice of termination and a pre-termination opportunity to respond. A formal hearing is not required, as long as there is pre-termination notice, an opportunity to respond to the decision maker, and a post-termination evidentiary hearing. In this case, the professor at the state university could only be fired for cause because she was tenured. Accordingly, due process required that the professor be provided with a pre-termination opportunity to respond. Because such an opportunity was not provided, the professor's due process rights were violated. Answer choice A is incorrect because the professor was only entitled to a pre-termination opportunity to respond, not a pre-termination evidentiary hearing. Answer choice C is incorrect because the professor's public employment at the state university could only be terminated for cause. Moreover, a public employee who may be fired with or without cause is still entitled to notice and a post-termination hearing. Answer choice D is incorrect because the professor should also have been given a pre-termination opportunity to respond.

In response to growing concerns about the overcrowding of landfills with scrap metal, Congress passed a statute requiring all unwanted vehicles to be disposed of at federally licensed auto-recycling facilities. These facilities were able to recycle more components of vehicles than most other recycling facilities. However, due to the high operating costs of these facilities, the cost of disposing the vehicles was much higher than the cost of disposing them at general purpose recycling facilities. A state wants to dispose of its fleet of decommissioned trucks at a state-operated recycling facility. However, this facility is not federally licensed.Is the state permitted to dispose of its decommissioned trucks at the state-operated facility? A No, because the federal statute was passed pursuant to Congress's power to legislate for the general welfare. B No, because the federal statute regulates interstate commerce. C Yes, because the market-participant exception applies. D Yes, because there is a presumption against preemption in an area governed by the state's police power.

Answer choice B is correct. Congress's power under the Commerce Clause extends to environmental laws such as this auto-recycling statute. As such, Congress can invoke the Necessary and Proper Clause to preempt all conflicting state laws, and the federal statute governs by virtue of the Supremacy Clause. Answer choice A is incorrect. Although Congress is permitted to tax and spend for the general welfare, it does not have the power to legislate for the general welfare. Thus, this is not a proper basis for the federal statute and would not support its enforcement against the state. Answer choice C is incorrect. The market-participant exception allows a state to behave in a discriminatory fashion against nonresident commerce if it is acting as a market participant. It does not permit a state to violate federal law regarding commerce. Answer choice D is incorrect because, although the state wants to dispose of its trucks, this disposal likely does not fall under the state's police powers. Therefore, the federal statute will govern.

In response to growing concerns about the overcrowding of landfills with scrap metal, Congress passed a statute requiring all unwanted vehicles to be disposed of at federally licensed auto-recycling facilities. These facilities were able to recycle more components of vehicles than most other recycling facilities. However, due to the high operating costs of these facilities, the cost of disposing the vehicles was much higher than the cost of disposing them at general purpose recycling facilities. A state wants to dispose of its fleet of decommissioned trucks at a state-operated recycling facility. However, this facility is not federally licensed.Is the state permitted to dispose of its decommissioned trucks at the state-operated facility? A No, because the federal statute was passed pursuant to Congress's power to legislate for the general welfare. B No, because the federal statute regulates interstate commerce. C Yes, because the market-participant exception applies. D Yes, because there is a presumption against preemption in an area governed by the state's police power.

Answer choice B is correct. Congress's power under the Commerce Clause extends to environmental laws such as this auto-recycling statute. As such, Congress can invoke the Necessary and Proper Clause to preempt all conflicting state laws, and the federal statute governs by virtue of the Supremacy Clause. Answer choice A is incorrect. Although Congress is permitted to tax and spend for the general welfare, it does not have the power to legislate for the general welfare. Thus, this is not a proper basis for the federal statute and would not support its enforcement against the state. Answer choice C is incorrect. The market-participant exception allows a state to behave in a discriminatory fashion against nonresident commerce if it is acting as a market participant. It does not permit a state to violate federal law regarding commerce. Answer choice D is incorrect because, although the state wants to dispose of its trucks, this disposal likely does not fall under the state's police powers. Therefore, the federal statute will govern.

City officials recently received a study showing a sharp increase in underage drinking. The study suggested that most purchases of alcohol by minors occurred at stores within 800 feet of a public high school, and were of a certain type of alcoholic beverage marketed to young adults. In response to the study, the city enacted a zoning ordinance that forbade the sale of that alcoholic beverage within 800 feet of a public high school. The owner of a liquor store 700 feet away from a public high school challenged the constitutionality of the zoning ordinance on the grounds that the zoning ordinance was too restrictive a solution, and that stricter enforcement of photo-ID laws would be a less restrictive and more effective solution to the problem. The owner requested an injunction preventing enforcement of this regulation against his store.If the owner fails in his challenge of the zoning ordinance, what is the most likely reason? A There is no constitutional limitation on enacting zoning ordinances that do not impact a fundamental right. B Reasonable ordinances may restrict nonfundamental rights to further a legitimate governmental interest. C There was no less restrictive regulation that would have addressed the same governmental interest more effectively. D The government may incidentally block adult access to otherwise legal substances and materials to keep them out of the hands of minors.

Answer choice B is correct. Government infringement upon nonfundamental rights—those related to social or economic interests such as business, taxation, lifestyle, or zoning—requires only a rational relationship between the law and a legitimate governmental interest. Here, the narrowly drawn zoning ordinance was constitutional because the purpose of the regulation was to reduce the sharp increase of underage high-school students from purchasing specific alcoholic beverages within 800 feet of public high schools. Limiting the sales of such beverages within this distance is rationally related to this interest. Therefore, this ordinance is most likely constitutional for this reason. Answer choice A is incorrect because the government does not have the unfettered right to enact zoning ordinances. Even ordinances that do not affect a fundamental right must still be rationally related to a legitimate government interest. Answer choice C is incorrect because this land-use restriction is tailored in response to the fact that a substantial amount of the alcohol purchased by minors was of a specific type and occurred within 800 feet of a public high school. The mere existence of an alternative does not conclusively establish that the regulation enacted by the city is not sufficiently related to a legitimate governmental interest. Therefore, this suggested alternative is insufficient to make the zoning regulation unconstitutional. Answer choice D is incorrect. It misstates the law regarding obscene materials, which states that the government cannot block an adult's access to indecent materials to keep them from reaching minors.

City officials recently received a study showing a sharp increase in underage drinking. The study suggested that most purchases of alcohol by minors occurred at stores within 800 feet of a public high school, and were of a certain type of alcoholic beverage marketed to young adults. In response to the study, the city enacted a zoning ordinance that forbade the sale of that alcoholic beverage within 800 feet of a public high school. The owner of a liquor store 700 feet away from a public high school challenged the constitutionality of the zoning ordinance on the grounds that the zoning ordinance was too restrictive a solution, and that stricter enforcement of photo-ID laws would be a less restrictive and more effective solution to the problem. The owner requested an injunction preventing enforcement of this regulation against his store.If the owner fails in his challenge of the zoning ordinance, what is the most likely reason? A There is no constitutional limitation on enacting zoning ordinances that do not impact a fundamental right. B Reasonable ordinances may restrict nonfundamental rights to further a legitimate governmental interest. C There was no less restrictive regulation that would have addressed the same governmental interest more effectively. D The government may incidentally block adult access to otherwise legal substances and materials to keep them out of the hands of minors.

Answer choice B is correct. Government infringement upon nonfundamental rights—those related to social or economic interests such as business, taxation, lifestyle, or zoning—requires only a rational relationship between the law and a legitimate governmental interest. Here, the narrowly drawn zoning ordinance was constitutional because the purpose of the regulation was to reduce the sharp increase of underage high-school students from purchasing specific alcoholic beverages within 800 feet of public high schools. Limiting the sales of such beverages within this distance is rationally related to this interest. Therefore, this ordinance is most likely constitutional for this reason. Answer choice A is incorrect because the government does not have the unfettered right to enact zoning ordinances. Even ordinances that do not affect a fundamental right must still be rationally related to a legitimate government interest. Answer choice C is incorrect because this land-use restriction is tailored in response to the fact that a substantial amount of the alcohol purchased by minors was of a specific type and occurred within 800 feet of a public high school. The mere existence of an alternative does not conclusively establish that the regulation enacted by the city is not sufficiently related to a legitimate governmental interest. Therefore, this suggested alternative is insufficient to make the zoning regulation unconstitutional. Answer choice D is incorrect. It misstates the law regarding obscene materials, which states that the government cannot block an adult's access to indecent materials to keep them from reaching minors.

GREAT EXAMPLE OF LEGISLATIVE VETO - MAKE SURE TO REVIEW! Congress enacted and the President signed the Toxic Waste Policy Act (Act). The Act delegated the primary responsibility for developing and administering a toxic waste disposal program to the Department of Energy (DOE) based on intelligible standards set forth in the Act. The Act also authorized the creation of a toxic waste fund to impose the cost of disposing of toxic waste on the creators of such waste through a statutorily-fixed fee. A provision of the Act requires the Secretary of the DOE to annually evaluate the fees collected to ensure that those fees offset the costs of the waste disposal program. If not, the Secretary is required to adjust the amount of the fee accordingly. The Act provides that the Secretary's adjusted fee becomes effective 90 days after publication unless both Houses of Congress adopt a resolution disapproving of the adjustment. This year the DOE Secretary, pursuant to the provision, proposed a fee increase. Both Houses of Congress disapproved the increase by resolution.Is a challenge to constitutionality of this provision likely to be successful? A Yes, because the Act violates the "nondelegation doctrine." B Yes, because the provision provides for a "legislative veto" of an executive action. C No, because the principle of bicameralism has been satisfied. D No, because the President approved of the provision by signing the Act.

Answer choice B is correct. It is unconstitutional for Congress to legislatively veto an executive action—that is, to retain direct control over the actions of an executive agency, rather than going through the proper channels of passing a bill. Answer choice A is incorrect. Although the "nondelegation doctrine" provides that Congress may not delegate legislative power to any other branch of government because Congress is vested by Article I with "all legislative powers," the delegation of some of Congress's authority to the executive branch has consistently been held constitutional, so long as Congress specifies an "intelligible principle" to guide the delegation. Since the Act contains intelligible standards by which the DOE is to develop and administer the toxic waste disposal program, it does not violate this doctrine. Answer choice C is incorrect because, although the passage of the resolution by both Houses of Congress does satisfy the principal of bicameralism, the provision nonetheless runs afoul of the Article I requirement that bills passed by both chambers of Congress must be presented to the President for signature or veto. Answer choice D is incorrect because the federal judiciary retains the power to declare a federal statute unconstitutional, even though the statute has received the approval of the other two branches of government (the executive as well as the legislative branches), and even though the President has agreed to the limitation on executive authority.

A federal law provides that all employees of the federal library system whose job descriptions include cataloging or disseminating materials will be required to take a reading comprehension test. The law provides that employees who do not meet a minimum standard of reading comprehension will either be fired or reassigned to a different position. The law does not provide for a hearing process for these employees, who are not at-will employees. An employee whose job description included cataloging materials failed to meet the minimum standard and was reassigned to a different position at a lower grade. The government denied his request for a hearing. He sued to enjoin the government from reassigning him, arguing that his constitutional rights were violated.Which of the following constitutional provisions provides the best support for the employee's suit? A The Privileges and Immunities Clause of Article IV. B The Due Process Clause of the Fifth Amendment. C The Takings Clause of the Fifth Amendment. D The Contract Clause of Article I, Section 10.

Answer choice B is correct. The Due Process Clause of the Fifth Amendment provides that the federal government shall not deprive any person of "life, liberty, or property, without due process of law." An employee who is not at-will has a legitimate property interest in continued public employment, and is entitled to a fair procedure to protect the employee's interests if the government seeks to discharge the employee from his position. An employee is entitled to a notice of termination and a pre-termination opportunity to respond, which the employee in this case did not receive. Answer choice A is incorrect because the Privileges and Immunities Clause prohibits one state from discriminating against the citizens of another state, and thus is not applicable in this case. Answer choice C is incorrect because the Takings Clause generally applies to the government's taking of an individual's real property, rather than an employment interest. Answer choice D is incorrect because Article I, Section 10 prohibits states from passing laws that impair the obligations of contracts; it does not apply to the federal government.

A federal law provides that all employees of the federal library system whose job descriptions include cataloging or disseminating materials will be required to take a reading comprehension test. The law provides that employees who do not meet a minimum standard of reading comprehension will either be fired or reassigned to a different position. The law does not provide for a hearing process for these employees, who are not at-will employees. An employee whose job description included cataloging materials failed to meet the minimum standard and was reassigned to a different position at a lower grade. The government denied his request for a hearing. He sued to enjoin the government from reassigning him, arguing that his constitutional rights were violated.Which of the following constitutional provisions provides the best support for the employee's suit? A The Privileges and Immunities Clause of Article IV. B The Due Process Clause of the Fifth Amendment. C The Takings Clause of the Fifth Amendment. D The Contract Clause of Article I, Section 10.

Answer choice B is correct. The Due Process Clause of the Fifth Amendment provides that the federal government shall not deprive any person of "life, liberty, or property, without due process of law." An employee who is not at-will has a legitimate property interest in continued public employment, and is entitled to a fair procedure to protect the employee's interests if the government seeks to discharge the employee from his position. An employee is entitled to a notice of termination and a pre-termination opportunity to respond, which the employee in this case did not receive. Answer choice A is incorrect because the Privileges and Immunities Clause prohibits one state from discriminating against the citizens of another state, and thus is not applicable in this case. Answer choice C is incorrect because the Takings Clause generally applies to the government's taking of an individual's real property, rather than an employment interest. Answer choice D is incorrect because Article I, Section 10 prohibits states from passing laws that impair the obligations of contracts; it does not apply to the federal government.

A small, struggling, formerly industrial city in one state was only 30 miles from the border of the neighboring state, and only 40 miles from a booming city in the neighboring state. In an effort to entice more citizens of the neighboring state to come to the small city to shop, dine, and otherwise spend money, the small city passed an ordinance relieving out-of-state citizens from paying the city's sales tax. A group of small-city citizens properly brought a suit against the city, challenging the ordinance. Of the following constitutional provisions, which would be the basis on which the citizens could most effectively challenge the ordinance?

Answer choice B is correct. The Equal Protection Clause of the Fourteenth Amendment provides that "no state shall...deny to any person within its jurisdiction the equal protection of the laws." The Equal Protection Clause is generally the constitutional safeguard that a citizen or group of citizens will use to challenge a law that is based on some kind of classification. Here, the city is classifying persons as either state citizens or out-of-state citizens, and providing the latter group with a benefit by relieving them from paying the city's sales tax. Therefore, the citizens could challenge this law on equal protection grounds. Answer choice A is incorrect because the Due Process Clause of the Fifth Amendment applies to the federal government, not to state action. Answer choice C is incorrect. The Privileges and Immunities Clause of Article IV prevents a state from treating citizens of another state in a discriminatory manner; it does not prevent a state from discriminating against its own citizens. In this case, the state is treating out-of-state citizens in a more advantageous manner than it is treating its own citizens; it is not discriminating against them. In-state residents have no claims against their own state under the Privileges and Immunities Clause. Answer choice D is incorrect because the Privileges or Immunities Clause of the Fourteenth Amendment has very limited application. This provision prevents states from infringing on the rights of national citizenship, and is usually only successfully invoked in cases involving the right to travel between the states.

Good question about supremacy clause - A federal statute that governs employer-provided health benefits specifies that health maintenance organizations (HMOs) may limit their provider networks. The statute also provides that state law that regulates insurance is not preempted by the statute. Federal courts interpreting the statute have held that in order to qualify as a law that regulates insurance, a state law must be specifically directed towards entities that engage in insurance. A state law required health insurers to acknowledge the services of any health care provider willing to abide by the insurer's plan. The HMO filed an action in federal court contending that, pursuant to federal law, it could limit its provider networks. The federal court, noting that a state court had interpreted the state law as applying to HMOs, rejected the HMOs argument that the state law did not regulate insurance. Which of the following states the reason why the state law is constitutional under the Supremacy Clause of Article VI of the U.S. Constitution?

Answer choice B is correct. The Supremacy Clause of Article VI provides that the "Constitution, and the laws of the United States" are the "supreme law of the land." Any state constitutional provision or law that directly or indirectly conflicts with a federal law, including federal regulations, is void under this clause. However, in this instance, the federal statute specifically disavowed federal preemption of a contrary state law if the state law regulated insurance. Because the federal court rejected the HMO's argument that the state law did not regulate insurance, the state law was constitutional by virtue of the federal statute's anti-preemption provision. Answer choice A is incorrect because a state law may be preempted by implied as well as express federal preemption, such as when federal law occupies an entire area of the law. Answer choice C is incorrect because the Supremacy Clause does not govern the extent to which a federal court must respect a state court's interpretation of its own laws. Here, the federal court, under principles of federalism, must defer to the state court's interpretation of state law as applying to HMOs. However, the determination of whether the state law in question regulates insurance involves interpretation of a federal, not state, statute. Answer choice D is incorrect because the Supremacy Clause applies to all state laws, even ones that regulate insurance. However, in this case, there is a federal statute that contains an express anti-preemption provision for state laws that regulate insurance.

A professional basketball player who was a citizen of one state sued an artist who was a citizen of another state. The artist had created multiple oil paintings of the player's image from which limited edition prints were created and sold without the player's consent or permission. The player sought damages in excess of $80,000 for violation of his state statutory right to publicity. While the state trial court ruled in the player's favor, the state appellate court overturned this decision on the erroneous grounds that a free speech privilege arising under the First Amendment protected the artist from this action. The appellate court decision denied the player recovery on his state-based cause of action. The state's highest court declined to hear the appeal.The player filed a petition for writ of certiorari with the U.S. Supreme Court. Can the Supreme Court grant this petition? A Yes, because diversity of citizenship exists between the player and the artist and the amount in controversy exceeds $75,000. B Yes, because the state court decision turned on whether a federal constitutional privilege existed for the artist. C No, because the state's highest court did not render a decision in this case since it declined to hear the appeal. D No, because the player sought to recover damages from the artist under a state statutory cause of action.

Answer choice B is correct. The Supreme Court's only power over a state court judgment is to correct the judgment when the state court incorrectly applies federal rights. Here, the Supreme Court can grant this petition because the state court judgment turns on the erroneous grounds that there is a federal constitutional privilege that protected the artist from a right to publicity cause of action. Answer choice A is incorrect because, while satisfaction of the diversity of citizenship requirements is one path by which a federal district court can exercise jurisdiction, it is not a requirement in order for the Supreme Court to exercise appellate jurisdiction over a state court decision. Answer choice C is incorrect because, while the Supreme Court generally reviews only decisions made by the highest court of a state, there is no constitutional requirement that the Supreme Court do so. Moreover, in this instance, the decision by the state's appellate court represented the final word on the matter by the state judiciary, since the state's highest court has refused to review that decision. Answer choice D is incorrect. While the state court decision in the case brought in state court under a state statutory cause of action generally would rest on adequate and independent state grounds that would foreclose review of the state court decision by the Supreme Court, in this case, the state court decision turns on whether a privilege based on the federal Constitution exists.

A professional basketball player who was a citizen of one state sued an artist who was a citizen of another state. The artist had created multiple oil paintings of the player's image from which limited edition prints were created and sold without the player's consent or permission. The player sought damages in excess of $80,000 for violation of his state statutory right to publicity. While the state trial court ruled in the player's favor, the state appellate court overturned this decision on the erroneous grounds that a free speech privilege arising under the First Amendment protected the artist from this action. The appellate court decision denied the player recovery on his state-based cause of action. The state's highest court declined to hear the appeal.The player filed a petition for writ of certiorari with the U.S. Supreme Court. Can the Supreme Court grant this petition? A Yes, because diversity of citizenship exists between the player and the artist and the amount in controversy exceeds $75,000. B Yes, because the state court decision turned on whether a federal constitutional privilege existed for the artist. C No, because the state's highest court did not render a decision in this case since it declined to hear the appeal. D No, because the player sought to recover damages from the artist under a state statutory cause of action.

Answer choice B is correct. The Supreme Court's only power over a state court judgment is to correct the judgment when the state court incorrectly applies federal rights. Here, the Supreme Court can grant this petition because the state court judgment turns on the erroneous grounds that there is a federal constitutional privilege that protected the artist from a right to publicity cause of action. Answer choice A is incorrect because, while satisfaction of the diversity of citizenship requirements is one path by which a federal district court can exercise jurisdiction, it is not a requirement in order for the Supreme Court to exercise appellate jurisdiction over a state court decision. Answer choice C is incorrect because, while the Supreme Court generally reviews only decisions made by the highest court of a state, there is no constitutional requirement that the Supreme Court do so. Moreover, in this instance, the decision by the state's appellate court represented the final word on the matter by the state judiciary, since the state's highest court has refused to review that decision. Answer choice D is incorrect. While the state court decision in the case brought in state court under a state statutory cause of action generally would rest on adequate and independent state grounds that would foreclose review of the state court decision by the Supreme Court, in this case, the state court decision turns on whether a privilege based on the federal Constitution exists.

Concerned with the proliferation of signs about upcoming events and the failure to remove those signs after the event, a city enacted an ordinance that limited the number of such signs that could be displayed on public property and set a time period before and after an event during which signs about that event could be displayed. A social organization wants to display signs about its monthly dinner, which is held to attract new members, in greater number and for a longer period than permitted by the ordinance. The organization has filed a lawsuit, challenging the constitutionality of the ordinance.Of the following, by which standard will this ordinance be judged? A It must be narrowly tailored to further a significant government interest, and leave open alternative channels of communication. B It must be the least restrictive means for accomplishing a compelling governmental interest. C It must be rationally related to a legitimate government interest. D It must not have a negative impact on the organization's freedom of assembly.

Answer choice B is correct. This regulation is a restriction on speech that is content-based on its face. The regulation applies to one specific topic (upcoming events) and subjects that speech to different--and more onerous--requirements than any other signs. Therefore, the city's ordinance will be judged by a strict scrutiny standard. Answer choice A is incorrect because it states the standard for time, place, and manner restrictions. However, this is a content-based restriction, so strict scrutiny applies. Answer choice C is incorrect because it states the rational basis test, which does not apply to content-based restrictions on speech. Answer choice D is incorrect. All constitutional protections, including the First Amendment protection of the freedom of assembly, are not absolute. The mere fact that governmental conduct has a negative impact on this right is not sufficient to preclude the constitutionality of an ordinance.

Concerned with the proliferation of signs about upcoming events and the failure to remove those signs after the event, a city enacted an ordinance that limited the number of such signs that could be displayed on public property and set a time period before and after an event during which signs about that event could be displayed. A social organization wants to display signs about its monthly dinner, which is held to attract new members, in greater number and for a longer period than permitted by the ordinance. The organization has filed a lawsuit, challenging the constitutionality of the ordinance. Of the following, by which standard will this ordinance be judged?

Answer choice B is correct. This regulation is a restriction on speech that is content-based on its face. The regulation applies to one specific topic (upcoming events) and subjects that speech to different--and more onerous--requirements than any other signs. Therefore, the city's ordinance will be judged by a strict scrutiny standard. Answer choice A is incorrect because it states the standard for time, place, and manner restrictions. However, this is a content-based restriction, so strict scrutiny applies. Answer choice C is incorrect because it states the rational basis test, which does not apply to content-based restrictions on speech. Answer choice D is incorrect. All constitutional protections, including the First Amendment protection of the freedom of assembly, are not absolute. The mere fact that governmental conduct has a negative impact on this right is not sufficient to preclude the constitutionality of an ordinance.

A defendant was charged with a violation of a state criminal statute. While prosecution was pending, the defendant filed a civil action under 42 U.S.C. § 1983 in federal district court, alleging that the state statute as applied to him violates the U.S. Constitution. The defendant sought an injunction against the state's prosecution of him.Should the federal district court entertain this action? A No, because the defendant lacks standing since he has not been convicted and sentenced for violation of the statute. B No, because the defendant seeks an injunction against a pending state criminal proceeding. C Yes, because a federal court may enjoin the enforcement of an unconstitutional statute. D Yes, because the defendant has alleged that the state criminal statute violates the U.S. Constitution.

Answer choice B is correct. Under the Younger doctrine, a court will not enjoin a pending state criminal case in the absence of bad faith, harassment, or a patently invalid state statute. Since the facts do not suggest that the state prosecution has been undertaken in bad faith or for the purpose of harassment, or that the criminal statute is patently invalid, the court should abstain from hearing this civil case. Answer choice A is incorrect. One of the elements that a plaintiff must establish to have standing is that the plaintiff must suffer a concrete and particularized injury in fact. An imminent threat of prosecution under an unconstitutional statute is sufficient to satisfy this element. Consequently, the defendant in the state criminal prosecution, who is the plaintiff in the federal civil action, has suffered an injury in fact even though he has not been convicted of violating the statute in question. Answer choice C is incorrect. Although a federal court has broad equitable powers in fashioning a remedy for a constitutional violation, including granting injunctive relief, a federal court will not enjoin a pending state criminal case in the absence of bad faith or harassment. Answer choice D is incorrect. Although it is clear that the federal court may determine a state statute violates the U.S. Constitution under the Supremacy Clause of Article VI, this case presents one of the rare circumstances in which the federal court should abstain from exercising its jurisdiction.

A defendant was charged with a violation of a state criminal statute. While prosecution was pending, the defendant filed a civil action under 42 U.S.C. § 1983 in federal district court, alleging that the state statute as applied to him violates the U.S. Constitution. The defendant sought an injunction against the state's prosecution of him.Should the federal district court entertain this action? A No, because the defendant lacks standing since he has not been convicted and sentenced for violation of the statute. B No, because the defendant seeks an injunction against a pending state criminal proceeding. C Yes, because a federal court may enjoin the enforcement of an unconstitutional statute. D Yes, because the defendant has alleged that the state criminal statute violates the U.S. Constitution.

Answer choice B is correct. Under the Younger doctrine, a court will not enjoin a pending state criminal case in the absence of bad faith, harassment, or a patently invalid state statute. Since the facts do not suggest that the state prosecution has been undertaken in bad faith or for the purpose of harassment, or that the criminal statute is patently invalid, the court should abstain from hearing this civil case. Answer choice A is incorrect. One of the elements that a plaintiff must establish to have standing is that the plaintiff must suffer a concrete and particularized injury in fact. An imminent threat of prosecution under an unconstitutional statute is sufficient to satisfy this element. Consequently, the defendant in the state criminal prosecution, who is the plaintiff in the federal civil action, has suffered an injury in fact even though he has not been convicted of violating the statute in question. Answer choice C is incorrect. Although a federal court has broad equitable powers in fashioning a remedy for a constitutional violation, including granting injunctive relief, a federal court will not enjoin a pending state criminal case in the absence of bad faith or harassment. Answer choice D is incorrect. Although it is clear that the federal court may determine a state statute violates the U.S. Constitution under the Supremacy Clause of Article VI, this case presents one of the rare circumstances in which the federal court should abstain from exercising its jurisdiction.

A city permitted the use of an auditorium in a city-owned building as a place of public debate and discussion. Anyone who wanted to use the auditorium for that purpose was required to reserve a specific time with a designated administrator, who was required to grant the request, subject to availability. A group of young lawyers who specialized in human rights law sought to use the auditorium to debate with members of the community about human rights violations by the U.S. military. Due to the heated controversy regarding this issue and the large number of military veterans in the city, the administrator, fearing the potential for violence, denied the group's request. The administrator pointed out that there were other city facilities in which the debate could be held.Is the administrator's denial likely to withstand a constitutional challenge by the lawyers' group? A Yes, because the denial was rationally related to a legitimate state interest in public safety. B Yes, because the city-owned auditorium was a limited public forum. C No, because the denial was based on the content of the debate. D No, because the administrator had unfettered discretion.

Answer choice C is correct. A "public forum" may be traditional or designated. A designated (or limited) public forum is one that has not historically been used for speech-related activities, but which the government has opened for such use, such as civic auditoriums. In either type of public forum, the government may impose reasonable restrictions on the time, place, or manner of protected speech, provided the restrictions: (i) are content-neutral as to both subject matter and viewpoint, (ii) are narrowly tailored to serve a significant governmental interest, and (iii) leave open ample alternative channels for communication of the information. In this case, the city designated the city-owned auditorium as a public forum by opening its use for public debate and discussion. The administrator denied the groups' request to use the public forum based on the potential for violence arising from the subject of the debate (i.e., the content of the speech). Therefore, the administrator's action was unconstitutional. Answer choice A is incorrect because it states the incorrect standard for review of the administrator's action. The action, in the form of restricting speech, must be narrowly tailored to serve a significant governmental interest. More importantly in this question, the restriction on speech must be content neutral, which this restriction was not. Answer choice B is incorrect. While it is true that the auditorium was a limited public forum, any restriction on speech in a traditional or designated (limited) forum must be content-neutral. Answer choice D is incorrect. A law or regulation that permits a governmental official to restrict speech (e.g., requires an official to issue a permit before a rally can be held) must provide definite standards as to how to apply the law in order to prevent governmental officials from having unfettered discretion over its application. Here, the administrator was required to grant a person's request to reserve an available time and could not exercise discretion. Consequently, the administrator's action is likely unconstitutional, not because she had unfettered discretion, but rather because she exercised discretion she did not have in an unconstitutional manner.

A city permitted the use of an auditorium in a city-owned building as a place of public debate and discussion. Anyone who wanted to use the auditorium for that purpose was required to reserve a specific time with a designated administrator, who was required to grant the request, subject to availability. A group of young lawyers who specialized in human rights law sought to use the auditorium to debate with members of the community about human rights violations by the U.S. military. Due to the heated controversy regarding this issue and the large number of military veterans in the city, the administrator, fearing the potential for violence, denied the group's request. The administrator pointed out that there were other city facilities in which the debate could be held.Is the administrator's denial likely to withstand a constitutional challenge by the lawyers' group? A Yes, because the denial was rationally related to a legitimate state interest in public safety. B Yes, because the city-owned auditorium was a limited public forum. C No, because the denial was based on the content of the debate. D No, because the administrator had unfettered discretion.

Answer choice C is correct. A "public forum" may be traditional or designated. A designated (or limited) public forum is one that has not historically been used for speech-related activities, but which the government has opened for such use, such as civic auditoriums. In either type of public forum, the government may impose reasonable restrictions on the time, place, or manner of protected speech, provided the restrictions: (i) are content-neutral as to both subject matter and viewpoint, (ii) are narrowly tailored to serve a significant governmental interest, and (iii) leave open ample alternative channels for communication of the information. In this case, the city designated the city-owned auditorium as a public forum by opening its use for public debate and discussion. The administrator denied the groups' request to use the public forum based on the potential for violence arising from the subject of the debate (i.e., the content of the speech). Therefore, the administrator's action was unconstitutional. Answer choice A is incorrect because it states the incorrect standard for review of the administrator's action. The action, in the form of restricting speech, must be narrowly tailored to serve a significant governmental interest. More importantly in this question, the restriction on speech must be content neutral, which this restriction was not. Answer choice B is incorrect. While it is true that the auditorium was a limited public forum, any restriction on speech in a traditional or designated (limited) forum must be content-neutral. Answer choice D is incorrect. A law or regulation that permits a governmental official to restrict speech (e.g., requires an official to issue a permit before a rally can be held) must provide definite standards as to how to apply the law in order to prevent governmental officials from having unfettered discretion over its application. Here, the administrator was required to grant a person's request to reserve an available time and could not exercise discretion. Consequently, the administrator's action is likely unconstitutional, not because she had unfettered discretion, but rather because she exercised discretion she did not have in an unconstitutional manner.

During a criminal trial in state court, the prosecutor sought and obtained a dismissal of the case. When the prosecutor subsequently attempted to try the defendant again in state court, the trial court ruled that the prosecutor was barred from doing so. The state appealed this decision. The state appellate court, noting that the Double Jeopardy provision in the state constitution was identical to the Double Jeopardy Clause in the Fifth Amendment of the U.S. Constitution, applied only case law of the U.S. Supreme Court interpreting the federal Double Jeopardy Clause in determining the meaning of the identical provision in the state constitution. Based on this federal case law, the state appellate court ruled that the federal Double Jeopardy Clause of the Fifth Amendment as applied to the state via the Fourteenth Amendment prevented the retrial of the defendant and therefore the provision in the state constitution likewise prevented retrial. When the state's highest court declined to hear an appeal of the appellate court decision, the state timely appealed the decision to the U.S. Supreme Court.Can the U.S. Supreme Court hear this appeal? A No, because the state court decision was based on adequate and independent state grounds. B No, because the state's highest court did not rule on the matter. C Yes, because the state appellate court relied on the U.S. Supreme Court's interpretation of the federal Double Jeopardy Clause in interpreting state law. D Yes, because the Supreme Court can hear appeals from all state court judgments.

Answer choice C is correct. A final state-court judgment that rests on adequate and independent state grounds may not be reviewed by the U.S. Supreme Court. The state-law grounds must fully resolve the matter (i.e., be adequate) and must not incorporate a federal standard by reference (i.e., be independent). When it is not clear whether the state court's decision rests on state or federal law, the Supreme Court may hear the case, decide the federal issue, and remand to the state court for resolution of any question of state law. Here, the Double Jeopardy provision in the state constitution was identical to the federal Double Jeopardy Clause in the Fifth Amendment, and the state appellate court applied the U.S. Supreme Court's interpretation of the federal clause in interpreting the state constitutional provision. On these facts, it appears that the state court has relied on a federal standard by reference. Therefore, the state ground was not independent, and the U.S. Supreme Court can hear the appeal. Answer choice A is incorrect because the state appellate court incorporated a federal standard by reference, and therefore its interpretation of the state Double Jeopardy provision was not independent. Answer choice B is incorrect because the U.S. Supreme can exercise jurisdiction over a final judgment of a state court, even a state trial court. The U.S. Supreme Court is not limited to hearing appeals from judgments of the state's highest court. Answer choice D is incorrect because the U.S. Supreme Court cannot hear appeals from final state-court judgments that rest on adequate and independent state grounds.

During a criminal trial in state court, the prosecutor sought and obtained a dismissal of the case. When the prosecutor subsequently attempted to try the defendant again in state court, the trial court ruled that the prosecutor was barred from doing so. The state appealed this decision. The state appellate court, noting that the Double Jeopardy provision in the state constitution was identical to the Double Jeopardy Clause in the Fifth Amendment of the U.S. Constitution, applied only case law of the U.S. Supreme Court interpreting the federal Double Jeopardy Clause in determining the meaning of the identical provision in the state constitution. Based on this federal case law, the state appellate court ruled that the federal Double Jeopardy Clause of the Fifth Amendment as applied to the state via the Fourteenth Amendment prevented the retrial of the defendant and therefore the provision in the state constitution likewise prevented retrial. When the state's highest court declined to hear an appeal of the appellate court decision, the state timely appealed the decision to the U.S. Supreme Court.Can the U.S. Supreme Court hear this appeal? A No, because the state court decision was based on adequate and independent state grounds. B No, because the state's highest court did not rule on the matter. C Yes, because the state appellate court relied on the U.S. Supreme Court's interpretation of the federal Double Jeopardy Clause in interpreting state law. D Yes, because the Supreme Court can hear appeals from all state court judgments.

Answer choice C is correct. A final state-court judgment that rests on adequate and independent state grounds may not be reviewed by the U.S. Supreme Court. The state-law grounds must fully resolve the matter (i.e., be adequate) and must not incorporate a federal standard by reference (i.e., be independent). When it is not clear whether the state court's decision rests on state or federal law, the Supreme Court may hear the case, decide the federal issue, and remand to the state court for resolution of any question of state law. Here, the Double Jeopardy provision in the state constitution was identical to the federal Double Jeopardy Clause in the Fifth Amendment, and the state appellate court applied the U.S. Supreme Court's interpretation of the federal clause in interpreting the state constitutional provision. On these facts, it appears that the state court has relied on a federal standard by reference. Therefore, the state ground was not independent, and the U.S. Supreme Court can hear the appeal. Answer choice A is incorrect because the state appellate court incorporated a federal standard by reference, and therefore its interpretation of the state Double Jeopardy provision was not independent. Answer choice B is incorrect because the U.S. Supreme can exercise jurisdiction over a final judgment of a state court, even a state trial court. The U.S. Supreme Court is not limited to hearing appeals from judgments of the state's highest court. Answer choice D is incorrect because the U.S. Supreme Court cannot hear appeals from final state-court judgments that rest on adequate and independent state grounds.

A state with heavy snowfall each winter sought to prevent injuries and accidents on the road caused by snow falling off of the roofs of large trucks. To further this interest, the state enacted a statute that requires state-employed road maintenance crews to set up random checkpoints at highway on-ramps and state border crossings during the winter months. At these checkpoints, every truck above a certain size is required to stop so the maintenance crews can make sure any snow on its roof is brushed off. The statute has caused significant traffic delays, but the effect on the rate of accidents on highways in the winter has been minimal. An out-of-state company that ships its goods across the state has started losing business because this program has made it nearly impossible to estimate delivery times due to the delays. As a result, the out-of-state company has brought an action challenging the constitutionality of the state statute.Is the company likely to succeed in its constitutional challenge to the state statute? A No, because the state statute does not discriminate against out-of-state commerce. B No, because the state statute serves an important state interest. C Yes, because the state statute imposes an unreasonable burden on interstate commerce. D Yes, because the state statute is not rationally related to a legitimate state interest.

Answer choice C is correct. A state regulation that is not discriminatory may still be struck down as unconstitutional if it imposes an undue burden on interstate commerce. The courts will balance, case by case, the objective and purpose of the state law against the burden on interstate commerce and evaluate whether there are less restrictive alternatives. If the benefits of the state law are grossly outweighed by the burdens on interstate commerce, then even nondiscriminatory regulation may be struck down. Here, the state statute requiring these checkpoints is incredibly burdensome on interstate commerce, and appears to have had only minimal effects on road safety. Therefore, even though it is not facially discriminatory against out-of-state commerce, the statute likely violates the Dormant Commerce Clause. Answer choice A is incorrect because a state regulation that is not discriminatory may still be struck down as unconstitutional if it imposes an undue burden on interstate commerce. Answer choice B is incorrect because, even if this statute serves a state interest, on these facts that interest does not outweigh the burden the statute has imposed on interstate commerce. Answer choice D is incorrect because it improperly applies rational basis scrutiny to a Dormant Commerce Clause violation.

A state with heavy snowfall each winter sought to prevent injuries and accidents on the road caused by snow falling off of the roofs of large trucks. To further this interest, the state enacted a statute that requires state-employed road maintenance crews to set up random checkpoints at highway on-ramps and state border crossings during the winter months. At these checkpoints, every truck above a certain size is required to stop so the maintenance crews can make sure any snow on its roof is brushed off. The statute has caused significant traffic delays, but the effect on the rate of accidents on highways in the winter has been minimal. An out-of-state company that ships its goods across the state has started losing business because this program has made it nearly impossible to estimate delivery times due to the delays. As a result, the out-of-state company has brought an action challenging the constitutionality of the state statute.Is the company likely to succeed in its constitutional challenge to the state statute? A No, because the state statute does not discriminate against out-of-state commerce. B No, because the state statute serves an important state interest. C Yes, because the state statute imposes an unreasonable burden on interstate commerce. D Yes, because the state statute is not rationally related to a legitimate state interest.

Answer choice C is correct. A state regulation that is not discriminatory may still be struck down as unconstitutional if it imposes an undue burden on interstate commerce. The courts will balance, case by case, the objective and purpose of the state law against the burden on interstate commerce and evaluate whether there are less restrictive alternatives. If the benefits of the state law are grossly outweighed by the burdens on interstate commerce, then even nondiscriminatory regulation may be struck down. Here, the state statute requiring these checkpoints is incredibly burdensome on interstate commerce, and appears to have had only minimal effects on road safety. Therefore, even though it is not facially discriminatory against out-of-state commerce, the statute likely violates the Dormant Commerce Clause. Answer choice A is incorrect because a state regulation that is not discriminatory may still be struck down as unconstitutional if it imposes an undue burden on interstate commerce. Answer choice B is incorrect because, even if this statute serves a state interest, on these facts that interest does not outweigh the burden the statute has imposed on interstate commerce. Answer choice D is incorrect because it improperly applies rational basis scrutiny to a Dormant Commerce Clause violation.

A long-standing state statute dictated that, if a cause of action accrued while an individual was a minor, the individual could bring a civil action within three years of turning 18 years old. The legislature later amended the statute to retroactively extend the period to seven years if the cause of action was based on sexual abuse that occurred while the individual was a minor. The legislative history for the statute showed that the legislature found that, although the change would not prevent such abuse, it would provide remedies for victims. Within a month after the enactment of this change, a defendant was sued by a 25-year-old plaintiff who alleged she had been sexually abused when she was 12 years old. The defendant contended that he could not be sued because the new legislation violated his constitutional rights.Is the defendant likely to succeed in his defense? A Yes, because the amendment constitutes an ex post facto law. B Yes, because the amendment violates the Due Process Clause of the Fourteenth Amendment. C No, because the change in the statute was remedial and its retroactive application was rational. D No, because the retroactive application of a law is subject to strict scrutiny.

Answer choice C is correct. A statute of limitations, which addresses procedural rights, is remedial in nature. Consequently, unless a change to a statute of limitations violates a fundamental right or deals with a suspect classification, the statute must simply meet the rational basis standard of review. Here, the legislature made a rational finding that the retroactive application of the change was justified. Answer choice A is incorrect because the prohibition on an ex post facto law applies only to retroactive changes that affect criminal or penal liability. Here, the change affects the defendant's civil liability. Answer choice B is incorrect because the law satisfies the rational basis test, and thus does not violate due process. Answer choice D is incorrect because, unless the retroactive application of a statute affects a fundamental right, the law is subject to the rational basis test.

A long-standing state statute dictated that, if a cause of action accrued while an individual was a minor, the individual could bring a civil action within three years of turning 18 years old. The legislature later amended the statute to retroactively extend the period to seven years if the cause of action was based on sexual abuse that occurred while the individual was a minor. The legislative history for the statute showed that the legislature found that, although the change would not prevent such abuse, it would provide remedies for victims. Within a month after the enactment of this change, a defendant was sued by a 25-year-old plaintiff who alleged she had been sexually abused when she was 12 years old. The defendant contended that he could not be sued because the new legislation violated his constitutional rights.Is the defendant likely to succeed in his defense? A Yes, because the amendment constitutes an ex post facto law. B Yes, because the amendment violates the Due Process Clause of the Fourteenth Amendment. C No, because the change in the statute was remedial and its retroactive application was rational. D No, because the retroactive application of a law is subject to strict scrutiny.

Answer choice C is correct. A statute of limitations, which addresses procedural rights, is remedial in nature. Consequently, unless a change to a statute of limitations violates a fundamental right or deals with a suspect classification, the statute must simply meet the rational basis standard of review. Here, the legislature made a rational finding that the retroactive application of the change was justified. Answer choice A is incorrect because the prohibition on an ex post facto law applies only to retroactive changes that affect criminal or penal liability. Here, the change affects the defendant's civil liability. Answer choice B is incorrect because the law satisfies the rational basis test, and thus does not violate due process. Answer choice D is incorrect because, unless the retroactive application of a statute affects a fundamental right, the law is subject to the rational basis test.

The President created an office to encourage the improvement of local communities through faith-based organizations. The office was funded from monies appropriated by Congress for the general discretionary use of the President. The office provided support only to religious organizations. A taxpayer brought suit in federal court challenging the constitutionality of this office. The federal government has moved to dismiss this suit.Should the court allow the taxpayer's suit to proceed? A Yes, because the funding of the office violates the First Amendment Establishment of Religion Clause. B Yes, because the source of the funds for the office is a congressional appropriation. C No, because the plaintiff as a taxpayer lacks standing. D No, because the First Amendment Establishment of Religion Clause does not apply to the executive branch.

Answer choice C is correct. A taxpayer generally does not have standing to file a federal lawsuit simply because the taxpayer believes the government has allocated funds in an improper way. Answer choice A is incorrect because, although there is an exception to the rule that a taxpayer lacks standing to challenge a governmental expenditure when the expenditure violates the First Amendment Establishment of Religion Clause, this exception is very narrow. The exception does not apply to the expenditure of general discretionary funds by the executive branch. Answer choice B is incorrect because the narrow exception requires that Congress authorize the funds for a specific use that violates the Establishment Clause. Here, Congress merely authorized the funds for the President's discretionary use, and not to fund the challenged office. Answer choice D is incorrect because, although the Establishment Clause reads "Congress shall make no law," this clause has been interpreted to include actions taken by the federal government in general.

The President created an office to encourage the improvement of local communities through faith-based organizations. The office was funded from monies appropriated by Congress for the general discretionary use of the President. The office provided support only to religious organizations. A taxpayer brought suit in federal court challenging the constitutionality of this office. The federal government has moved to dismiss this suit.Should the court allow the taxpayer's suit to proceed? A Yes, because the funding of the office violates the First Amendment Establishment of Religion Clause. B Yes, because the source of the funds for the office is a congressional appropriation. C No, because the plaintiff as a taxpayer lacks standing. D No, because the First Amendment Establishment of Religion Clause does not apply to the executive branch.

Answer choice C is correct. A taxpayer generally does not have standing to file a federal lawsuit simply because the taxpayer believes the government has allocated funds in an improper way. Answer choice A is incorrect because, although there is an exception to the rule that a taxpayer lacks standing to challenge a governmental expenditure when the expenditure violates the First Amendment Establishment of Religion Clause, this exception is very narrow. The exception does not apply to the expenditure of general discretionary funds by the executive branch. Answer choice B is incorrect because the narrow exception requires that Congress authorize the funds for a specific use that violates the Establishment Clause. Here, Congress merely authorized the funds for the President's discretionary use, and not to fund the challenged office. Answer choice D is incorrect because, although the Establishment Clause reads "Congress shall make no law," this clause has been interpreted to include actions taken by the federal government in general.

A state enacted a law prohibiting anyone convicted of a felony from voting in state elections. A group of released felons filed suit in federal court, alleging that the statute violated their constitutional rights.Is the court likely to uphold this statute as constitutional? A No, because it infringes upon the fundamental right to vote. B No, because it violates the Equal Protection Clause. C Yes, as permitted by Section 2 of the Fourteenth Amendment. D Yes, pursuant to the Privileges and Immunities Clause.

Answer choice C is correct. As permitted by Section 2 of the Fourteenth Amendment, a state may prohibit a felon from voting, even one who has unconditionally been released from prison. Answer choice A is incorrect because while voting is typically considered a fundamental right and would therefore have to meet the burden of strict scrutiny, a state may prohibit a felon from voting. Answer choice B is incorrect because felons are not a protected class; accordingly, only the rational basis test would apply. Answer choice D is incorrect because the Privileges and Immunities Clause prohibits discrimination against out-of-state citizens and does not apply to these facts.

A state enacted a law prohibiting anyone convicted of a felony from voting in state elections. A group of released felons filed suit in federal court, alleging that the statute violated their constitutional rights.Is the court likely to uphold this statute as constitutional? A No, because it infringes upon the fundamental right to vote. B No, because it violates the Equal Protection Clause. C Yes, as permitted by Section 2 of the Fourteenth Amendment. D Yes, pursuant to the Privileges and Immunities Clause. SUBMIT ANSWER

Answer choice C is correct. As permitted by Section 2 of the Fourteenth Amendment, a state may prohibit a felon from voting, even one who has unconditionally been released from prison. Answer choice A is incorrect because while voting is typically considered a fundamental right and would therefore have to meet the burden of strict scrutiny, a state may prohibit a felon from voting. Answer choice B is incorrect because felons are not a protected class; accordingly, only the rational basis test would apply. Answer choice D is incorrect because the Privileges and Immunities Clause prohibits discrimination against out-of-state citizens and does not apply to these facts.

Responding to stories of licensed pharmacists who lied about their educational background, Congress enacted a new law imposing additional requirements on pharmacists involved in the sale of controlled substances. The prior law was viewed as ineffective due in part to the lack of enforcement of the regulatory program set out in the law. To correct this previous problem, the new law contained a provision requiring the assistance of state health officials in conducting background checks on pharmacists.Is the provision constitutional? A Yes, under Congress's power to regulate interstate commerce. B Yes, because Congress may legislate for the general welfare. C No, because Congress cannot compel state officers to administer a federal regulatory program. D No, because Congress cannot regulate health care personnel issues, traditionally local matters.

Answer choice C is correct. Congress cannot "commandeer" state legislatures by commanding them to enact specific legislation or enforce a federal regulatory program, and it may not circumvent that restriction by conscripting a state executive officer directly. In this case, the provision of the new law required the assistance of state health officials in conducting background checks. Because the provision of the federal law required state officials' aid in administering the regulatory program, it is unconstitutional. Answer choice A is incorrect. Although the sale of controlled substances has a substantial economic effect on interstate commerce, Congress cannot compel state officers to administer the federal law. Answer choice B is incorrect. Congress may tax and spend for the general welfare. However, Congress does not have a general "police power" to legislate for the general welfare. Answer choice D is incorrect. Although the federal government lacks a general police power, Congress can regulate traditionally local matters, such as issues concerning health care, pursuant to its commerce power. Under that power, Congress can regulate matters that have a substantial economic effect on interstate commerce. Here, the provision seeks to regulate a pharmacist who is involved in the sale of controlled substances, an economic activity.

Responding to stories of licensed pharmacists who lied about their educational background, Congress enacted a new law imposing additional requirements on pharmacists involved in the sale of controlled substances. The prior law was viewed as ineffective due in part to the lack of enforcement of the regulatory program set out in the law. To correct this previous problem, the new law contained a provision requiring the assistance of state health officials in conducting background checks on pharmacists.Is the provision constitutional? A Yes, under Congress's power to regulate interstate commerce. B Yes, because Congress may legislate for the general welfare. C No, because Congress cannot compel state officers to administer a federal regulatory program. D No, because Congress cannot regulate health care personnel issues, traditionally local matters.

Answer choice C is correct. Congress cannot "commandeer" state legislatures by commanding them to enact specific legislation or enforce a federal regulatory program, and it may not circumvent that restriction by conscripting a state executive officer directly. In this case, the provision of the new law required the assistance of state health officials in conducting background checks. Because the provision of the federal law required state officials' aid in administering the regulatory program, it is unconstitutional. Answer choice A is incorrect. Although the sale of controlled substances has a substantial economic effect on interstate commerce, Congress cannot compel state officers to administer the federal law. Answer choice B is incorrect. Congress may tax and spend for the general welfare. However, Congress does not have a general "police power" to legislate for the general welfare. Answer choice D is incorrect. Although the federal government lacks a general police power, Congress can regulate traditionally local matters, such as issues concerning health care, pursuant to its commerce power. Under that power, Congress can regulate matters that have a substantial economic effect on interstate commerce. Here, the provision seeks to regulate a pharmacist who is involved in the sale of controlled substances, an economic activity.

A violent storm results in severe property damage and extreme loss of life in States A, B, and C, which all become the subjects of emergency declarations by the President. The National Guard is present for three months in each state while recovery occurs. A year later, most of the damaged property has been reconstructed, and no current threat to life or property remains. During a time of year when similar storms are common, the President instructs the National Guard to set up a presence within various points in all three states. Congress had previously enacted legislation allowing the President to instruct the National Guard in such a way. The President neither requested nor received authority from the governor of State C, the smallest state of the three, but did receive authority from the governors of States A and B.Is the President's action constitutional? A No, because the governor of State C did not authorize the presence of the National Guard. B No, because an emergency situation no longer existed at the time of the President's order. C Yes, because the President's actions are aligned with permissible Congressional authority. D Yes, because the President sent the National Guard to all three states.

Answer choice C is correct. National Guard units are under the dual control of the federal and state governments. Under the Militia Clauses (Art. I, Sec. 8, Cl. 15, 16), Congress has the power to authorize the President to call National Guard units to execute federal laws, suppress insurrections, and repel invasions. This constitutional authority extends to use of National Guard units in domestic situations and non-emergency circumstances, and is not subject the approval or veto of the governor of a state. Answer choice A is incorrect because the President retains the authority to order units of the National Guard into service within a state even without express consent from the state's governor. Answer choice B is incorrect because the President can authorize the National Guard even when no emergency exists. Answer choice D is incorrect because it is irrelevant. There is no requirement that the President order units of the National Guard into service to every state suffering from an emergency.

A violent storm results in severe property damage and extreme loss of life in States A, B, and C, which all become the subjects of emergency declarations by the President. The National Guard is present for three months in each state while recovery occurs. A year later, most of the damaged property has been reconstructed, and no current threat to life or property remains. During a time of year when similar storms are common, the President instructs the National Guard to set up a presence within various points in all three states. Congress had previously enacted legislation allowing the President to instruct the National Guard in such a way. The President neither requested nor received authority from the governor of State C, the smallest state of the three, but did receive authority from the governors of States A and B.Is the President's action constitutional? A No, because the governor of State C did not authorize the presence of the National Guard. B No, because an emergency situation no longer existed at the time of the President's order. C Yes, because the President's actions are aligned with permissible Congressional authority. D Yes, because the President sent the National Guard to all three states.

Answer choice C is correct. National Guard units are under the dual control of the federal and state governments. Under the Militia Clauses (Art. I, Sec. 8, Cl. 15, 16), Congress has the power to authorize the President to call National Guard units to execute federal laws, suppress insurrections, and repel invasions. This constitutional authority extends to use of National Guard units in domestic situations and non-emergency circumstances, and is not subject the approval or veto of the governor of a state. Answer choice A is incorrect because the President retains the authority to order units of the National Guard into service within a state even without express consent from the state's governor. Answer choice B is incorrect because the President can authorize the National Guard even when no emergency exists. Answer choice D is incorrect because it is irrelevant. There is no requirement that the President order units of the National Guard into service to every state suffering from an emergency.

The association that regulated a state's realtors organized its members into 20 regions. Four representatives were chosen in each region to join a board to advocate for the rights of all realtors in that area. The board positions were unpaid, and the association established a rule requiring that two of the four chosen representatives must be black. The stated goal of this rule was to address the history of societal discrimination against people of color in the state. In general, there are fewer black realtors in the state than white realtors.If no federal statute applies, is this rule constitutional? A Yes, because the rule is rationally related to a legitimate governmental interest. B Yes, because the rule is intended to help remedy past race-based discrimination. C No, because the rule only remedies a general societal injustice. D No, because the rule is not substantially related to an important governmental interest.

Answer choice C is correct. Programs that discriminate based on race, even if they favor racial or ethnic minorities, are subject to strict scrutiny; and for a governmental affirmative action program based on race to survive, the relevant governmental entity must show more than a history of societal discrimination. The government—whether federal, state, or local—must itself be guilty of specific past discrimination against the group it is seeking to favor, and the remedy must be narrowly tailored to end that discrimination and eliminate its effects. Here, the state board's attempt to remedy general societal racial discrimination, rather than a specific practice of discrimination, does not meet this requirement. Answer choice A is incorrect because it improperly applies the rational basis test to this rule; strict scrutiny is warranted. Answer choice B is incorrect because, to survive, a governmental affirmative action program must remedy specific past discrimination carried out by the government rather than a general societal injustice. Answer choice D is incorrect because it applies the test for intermediate scrutiny where strict scrutiny should apply.

The association that regulated all state-licensed realtors organized its members into 20 regions. Four representatives were chosen in each region to join a board to advocate for the rights of all realtors in that area. The board positions were unpaid, and the association established a rule requiring that two of the four chosen representatives must be black. The stated goal of this rule was to address the history of societal discrimination against people of color in the state. In general, there are fewer black realtors in the state than white realtors.If no federal statute applies, is this rule constitutional? A Yes, because the rule is rationally related to a legitimate governmental interest. B Yes, because the rule is intended to help remedy past race-based discrimination. C No, because the rule only remedies a general societal injustice. D No, because the rule is not substantially related to an important governmental interest.

Answer choice C is correct. Programs that discriminate based on race, even if they favor racial or ethnic minorities, are subject to strict scrutiny; and for a governmental affirmative action program based on race to survive, the relevant governmental entity must show more than a history of societal discrimination. The government—whether federal, state, or local—must itself be guilty of specific past discrimination against the group it is seeking to favor, and the remedy must be narrowly tailored to end that discrimination and eliminate its effects. Here, the state board's attempt to remedy general societal racial discrimination, rather than a specific practice of discrimination, does not meet this requirement. Answer choice A is incorrect because it improperly applies the rational basis test to this rule; strict scrutiny is warranted. Answer choice B is incorrect because, to survive, a governmental affirmative action program must remedy specific past discrimination carried out by the government rather than a general societal injustice. Answer choice D is incorrect because it applies the test for intermediate scrutiny where strict scrutiny should apply.

A state enacted an intestacy statute that provided full inheritance rights for marital children from both parents. Under the statute, nonmarital children were permitted to inherit only through their mothers. The underlying purposes of the statute were to promote family relations and establish an accurate and efficient method of disposing property at death. The father of a nonmarital child recently died intestate. The nonmarital child brought suit challenging the constitutionality of the statute.Which of the following most accurately states the appropriate standard of review for the court? A The court must determine whether the statute is the least restrictive means of achieving an important government interest. B The court must determine whether the statute is rationally related to a legitimate government interest. C The court must determine whether the statute is substantially related to an important government interest. D The court must determine whether the statute is narrowly tailored to achieve a compelling government interest.

Answer choice C is correct. The Equal Protection Clause of the Fourteenth Amendment provides that states are generally prohibited from passing legislation that treats similarly situated persons differently. Classifications on the basis of status as a nonmarital child are subject to intermediate scrutiny—they must be substantially related to an important governmental interest. The Court will closely examine the purpose behind the distinction, and it will not uphold legislation designed to punish the offspring of a nonmarital relationship. Here, a classification based on legitimacy is a quasi-suspect classification that requires intermediate scrutiny. Answer choice A is incorrect because it misstates the standard for intermediate scrutiny. Although the statute must achieve an important government interest, it need not be the least restrictive means of doing so. Answer choice B is incorrect because the rational basis test is not the correct test to apply when a statute makes a classification based on legitimacy. Answer choice D is incorrect because the strict scrutiny test is also the incorrect test to apply when the issue involves legitimacy.

Due to a rising concern for children with undisclosed food allergies, as well as preserving a controlled environment in school cafeterias, a city enacted an ordinance stating that only lunches made without a specific list of ingredients commonly causing food allergies could be prepared and served in school cafeterias. In addition, to avoid any lunches that could cause food allergies, the ordinance stated that only students who ordered lunch in the cafeteria could eat there; students who brought their lunches from home were not permitted inside of the cafeteria during lunch time. Instead, these students were required to eat their lunches in a different room. A student who only ate certain foods in accordance with his religious beliefs brought his lunch from home. The lunch did not contain any of the prohibited ingredients listed in the ordinance. All of the student's friends ate their lunch in the cafeteria, but the student was forced to eat his lunches in another room.If the student's parents challenge the constitutionality of the ordinance under the Equal Protection Clause of the Fourteenth Amendment, are they likely to succeed? A Yes, because the ordinance is not narrowly tailored to address the dangers posed by food allergies. B Yes, because the ordinance has a disparate impact on children with religious dietary restrictions. C No, because the ordinance is rationally related to a legitimate governmental interest. D No, because the ordinance applies to all students.

Answer choice C is correct. The Equal Protection Clause of the Fourteenth Amendment provides that states are generally prohibited from passing legislation that treats similarly situated persons differently. To trigger strict or intermediate scrutiny, there must be discriminatory intent on the part of the government or a classification based on a suspect or quasi-suspect class. Strict scrutiny also applies if a fundamental right is involved. The rational basis standard is used in all cases in which one of the higher standards does not apply. A law passes the rational basis standard of review if it is rationally related to a legitimate governmental interest. This is a test of minimal scrutiny. Here, students who bring their lunches from home are being treated differently from students who eat in the cafeteria because they must eat in a separate room. However, the ordinance has a rational basis that is related to the legitimate government interest of preventing complications due to food allergies. Because this ordinance does not target a suspect class or involve a fundamental right and has no discriminatory intent, it passes rational basis review. Therefore, the parents' challenge is likely to fail. Answer choice A is incorrect because this ordinance only needs to meet rational basis scrutiny, and need not be narrowly tailored to its purpose. Answer choice B is incorrect because the fact that legislation has a disparate effect on people of different races, genders, or religions, without discriminatory intent, is insufficient to trigger strict or intermediate scrutiny. Answer choice D is incorrect. The fact that the ordinance applies to all students does not mean it is constitutional on that basis because it still differentiates between students who bring their lunches from home and those who buy their lunches in the cafeteria, requiring the former to eat lunch in a separate room.

An unmarried man and woman lived together for two years in State A and had a child. Two years after the child was born, the couple ended their relationship and the woman took full custody of the child. However, the man refused to provide any financial support for the child. The woman properly sued him in State A court. The State A court issued a final judgment on the merits ordering the man to pay the woman $500 per month in child support until the child turned 18. After the judgment was issued, the man immediately moved to State B and refused to pay any child support to the woman. The woman and child both remained in State A. The woman subsequently filed an enforcement proceeding in State B to enforce the State A judgment.Which of the following would best support the enforcement of the State A judgment regarding child support in State B? A The Tenth Amendment B The Eleventh Amendment C The Full Faith and Credit Clause D The Supremacy Clause

Answer choice C is correct. The Full Faith and Credit Clause of Article IV, Section 1 provides that full faith and credit shall be given in each state to the public acts, records, and judicial proceedings of every other state. In order to be given full faith and credit, a decision must meet three requirements: (i) the court that rendered the judgment must have had jurisdiction over the parties and the subject matter; (ii) the judgment must have been on the merits rather than on a procedural issue; and (iii) the judgment must be final. Here, State A had jurisdiction over the man and woman, and the judgment was final and based upon the merits of the case. Thus, the Full Faith and Credit Clause is applicable to this situation. Answer choice A is incorrect. The Tenth Amendment provides that all powers not assigned by the Constitution to the federal government are reserved to the states or to the people. This amendment does not apply in this situation because there is no state versus federal government issue. Answer choice B is incorrect. The Eleventh Amendment is a jurisdictional bar that prohibits the citizens of one state from suing another state in federal court. Here, the woman is not suing State B in federal court, thus it does not apply. Answer choice D is incorrect. It is possible (and common) for the federal and state governments to legislate in the same area. When this happens, the Supremacy Clause (Article VI, paragraph 2) provides that federal law supersedes conflicting state law. Here, there is no conflict between federal and state legislation in the same area. Therefore, the Supremacy Clause is inapplicable.

A young mother shared joint custody of her two children with her ex-husband in State A. Three years ago, the mother was fired from her waitressing job. Without seeking court approval, the mother took her children and moved to State B to stay with her mother. The ex-husband notified the police and the mother was arrested for and convicted of parental kidnapping in State B. The father now has sole custody, the mother has completed her brief prison sentence, and both parents currently live in State A. Last month, the State A legislature passed a statute requiring registration of convicted parental kidnappers with local law enforcement. The registration information includes the convicted kidnapper's name, current address and place of employment, and this information is maintained in a public database. The mother has challenged the constitutionality of the statute.Is she likely to succeed? A Yes, because the statute is an unconstitutional ex post facto law. B Yes, because the statute is an unconstitutional bill of attainder. C No, because the statute is a non-punitive regulation enacted for the protection of the public. D No, because the parental kidnapping judgment was valid under the full faith and credit clause.

Answer choice C is correct. The constitutional prohibition on an "ex post facto" law is confined to a retroactive change to a criminal or penal law. Similarly, the constitutional prohibition on a bill of attainder applies only to criminal or penal measures. A law that is civil in purpose is treated as a criminal law only if its punitive effect clearly overrides its civil purpose. Here, the retroactive application of the statute requires registration of convicted parental kidnappers, and public notification of information about the convicts. This does not constitute an ex post facto law or a bill of attainder because the statute is a non-punitive regulation enacted for the protection of the public. Answer choice A is incorrect. A law that is civil in purpose, such as the statute in this case, will be treated as a criminal law if its punitive effect clearly overrides its civil purpose. However, in this situation, the registration requirement is not intended to punish the convicted parental kidnappers. Rather, the statute is a non-punitive program designed to protect the public. The resulting dissemination of the registration information is not significant enough to treat the statute as a criminal law. Answer choice B is incorrect. A bill of attainder is a legislative act that declares a person or group of persons guilty of some crime and punishes them without a trial. Sections 9 and 10 of Article I forbid the federal government and the states from enacting such "legislative trials." It applies only to criminal or penal measures. Here, as discussed above, the statute is not a criminal or penal measure that declares the mother guilty of a crime without a trial. The mother has already been convicted of parental kidnapping, and the statute is a non-punitive regulatory scheme designed to protect the public. Answer choice D is incorrect. The Full Faith and Credit Clause of Article IV, Section 1 provides that "[f]ull faith and credit shall be given in each state to the public acts, records, and judicial proceedings of every other state." Thus, although the parental kidnapping judgment in State B is valid under the full faith and credit clause, and it should be given effect in State A, this has no bearing on the constitutionality of the statute's registration requirement. The central issue here is whether the statute is a non-punitive regulatory scheme or whether it is so punitive in nature as to override its civil purpose. The full faith and credit clause is irrelevant to this analysis.

A young mother shared joint custody of her two children with her ex-husband in State A. Three years ago, the mother was fired from her waitressing job. Without seeking court approval, the mother took her children and moved to State B to stay with her mother. The ex-husband notified the police and the mother was arrested for and convicted of parental kidnapping in State B. The father now has sole custody, the mother has completed her brief prison sentence, and both parents currently live in State A. Last month, the State A legislature passed a statute requiring registration of convicted parental kidnappers with local law enforcement. The registration information includes the convicted kidnapper's name, current address and place of employment, and this information is maintained in a public database. The mother has challenged the constitutionality of the statute.Is she likely to succeed? A Yes, because the statute is an unconstitutional ex post facto law. B Yes, because the statute is an unconstitutional bill of attainder. C No, because the statute is a non-punitive regulation enacted for the protection of the public. D No, because the parental kidnapping judgment was valid under the full faith and credit clause.

Answer choice C is correct. The constitutional prohibition on an "ex post facto" law is confined to a retroactive change to a criminal or penal law. Similarly, the constitutional prohibition on a bill of attainder applies only to criminal or penal measures. A law that is civil in purpose is treated as a criminal law only if its punitive effect clearly overrides its civil purpose. Here, the retroactive application of the statute requires registration of convicted parental kidnappers, and public notification of information about the convicts. This does not constitute an ex post facto law or a bill of attainder because the statute is a non-punitive regulation enacted for the protection of the public. Answer choice A is incorrect. A law that is civil in purpose, such as the statute in this case, will be treated as a criminal law if its punitive effect clearly overrides its civil purpose. However, in this situation, the registration requirement is not intended to punish the convicted parental kidnappers. Rather, the statute is a non-punitive program designed to protect the public. The resulting dissemination of the registration information is not significant enough to treat the statute as a criminal law. Answer choice B is incorrect. A bill of attainder is a legislative act that declares a person or group of persons guilty of some crime and punishes them without a trial. Sections 9 and 10 of Article I forbid the federal government and the states from enacting such "legislative trials." It applies only to criminal or penal measures. Here, as discussed above, the statute is not a criminal or penal measure that declares the mother guilty of a crime without a trial. The mother has already been convicted of parental kidnapping, and the statute is a non-punitive regulatory scheme designed to protect the public. Answer choice D is incorrect. The Full Faith and Credit Clause of Article IV, Section 1 provides that "[f]ull faith and credit shall be given in each state to the public acts, records, and judicial proceedings of every other state." Thus, although the parental kidnapping judgment in State B is valid under the full faith and credit clause, and it should be given effect in State A, this has no bearing on the constitutionality of the statute's registration requirement. The central issue here is whether the statute is a non-punitive regulatory scheme or whether it is so punitive in nature as to override its civil purpose. The full faith and credit clause is irrelevant to this analysis.

The District of Columbia government has the power to levy an income tax. Congress has recently enacted a statute that prohibits the District of Columbia from imposing a tax on the income of individuals who work in the District of Columbia but live elsewhere. Congress was seeking to provide an incentive for nonresidents to do business within the District of Columbia.Is this statute likely to be found constitutional? A No, because it violates the Due Process Clause of the Fifth Amendment to the U.S. Constitution. B No, because it violates the Uniformity Clause of Article I, Section 8 of the U.S. Constitution. C Yes, under the Enclave Clause in Article I, Section 8 of the U.S. Constitution. D Yes, under the Sixteenth Amendment to the U.S. Constitution.

Answer choice C is correct. Under Article I, Section 8, Clause 17 of the U.S. Constitution, also known as the Enclave Clause, Congress has the general police and regulatory powers over the District of Columbia that a state enjoys over persons and things within its boundaries. Among those powers is the power to tax or not to tax income earned within its boundaries. Answer choice A is incorrect. There is an equal protection component of the Due Process Clause of the Fifth Amendment that corresponds to the Equal Protection Clause of the Fourteenth Amendment. However, while the statute does treat residents of the District of Columbia who earn income there differently from the nonresidents of the District of Columbia who earn income there, the residents of the District of Columbia are not members of a suspect class. Consequently, the statute needs only a rational basis. The provision of an incentive for nonresidents to do business within the District of Columbia constitutes a rational basis for the statute. Answer choice B is incorrect because the Uniformity Clause applies to indirect taxes, such as excise taxes and import duties, not to direct taxes on income. Answer choice D is incorrect because the Sixteenth Amendment gave Congress the power to impose an income tax without apportioning the tax among the states based on population. Here, the statute does not impose an income tax, but prohibits the District of Columbia from imposing one on certain income.

A state adopted a blanket primary system for choosing elected state officials. Under this system, all candidates participate in a single primary election in which all voters cast ballots. A voter is free to vote for any candidate, and a member of one party may vote for a candidate affiliated with another party. The two candidates who receive the greatest number of votes, regardless of party, advance to the general election. On the ballot, the candidates are identified only by name. Each political party is free to select its nominee by whatever method it chooses.Is this primary system likely to withstand a constitutional challenge? A No, because a state must permit a candidate to designate the candidate's party affiliation on the ballot. B No, because a blanket primary system is per se unconstitutional. C Yes, because a state is free to design and conduct a primary as it sees fit. D Yes, because the primary system doesn't determine a party's nominee.

Answer choice D is correct. A state may institute a blanket primary system in which all voters participate. However, the First Amendment freedom of expression and association prohibit states from interfering with a political party's endorsement of a candidate or selection of a candidate. Because this system does not appear to do either one, it is likely constitutional. Answer choice A is incorrect because a state is likely not required to designate the political party with which a candidate is associated. A state may refuse to list a candidate's party affiliation, despite being chosen as the party's candidate, when the candidate has also been selected by another party as its candidate. Since the refusal did not impose a severe burden on the party's rights, it does not face the strict scrutiny test. Answer choice B is incorrect because a blanket primary is not unconstitutional. Answer choice C is incorrect because while a state is generally free to design and conduct a primary as it sees fit, a state is subject to constitutional restrictions, such as the First Amendment.

A state highway administration, acting pursuant to statutory authorization by the state legislature, has promulgated rules for large electronic billboards located along roads maintained by the state. These rules are concerned with the potential for driver distraction and the ensuing adverse consequences for highway safety. Among the rules is one that bans the graphic display of violence. The producer of a movie wants to promote the movie through a short clip from the movie on billboards subject to this rule. The clip contains a graphic display of violence. The producer has filed an action in the appropriate federal court challenging the state highway administration's rule as a violation of the First Amendment as applicable to the states through the Fourteenth Amendment. Of the following standards, by which should the state highway administration's rule be judged? A As a rule that deals with a matter traditionally subject to regulation, it must be upheld unless it is arbitrary or irrational. B As a regulation of commercial speech, there must be a reasonable fit between the government's ends and the means chosen to accomplish those ends. C As a time, place, or manner restriction, it must be narrowly tailored to serve a significant governmental interest. D As a content regulation, it must be necessary to achieve a compelling governmental interest.

Answer choice D is correct. Although viewpoint neutral, the state highway administration's rule is a content-based regulation of speech because it prohibits the graphic display of violence. As such, the strict scrutiny standard of review applies. The rule must be necessary to achieve a compelling governmental interest. Answer choice A is incorrect. Although content regulation of speech that falls into certain historical categories, such as obscenity or defamation, may be permitted, the depiction of violence is not one of those categories. Answer choice B is incorrect. Although the producer is concerned with the impact of the rule on his commercial speech, the rule is not limited to commercial speech. Instead, the rule applies to any display that appears on the billboard, even those that have a noncommercial purpose, such as public service announcements. Answer choice C is incorrect. Time, place, or manner restrictions must be narrowly tailored to serve a significant governmental interest, and arguably, rules regarding electronic billboards specifically are time, place, and manner restrictions. However, the portion of the rule affecting the movie producer is a restriction of the content of expression. The more stringent standard applies.

A state highway administration, acting pursuant to statutory authorization by the state legislature, has promulgated rules for large electronic billboards located along roads maintained by the state. These rules are concerned with the potential for driver distraction and the ensuing adverse consequences for highway safety. Among the rules is one that bans the graphic display of violence. The producer of a movie wants to promote the movie through a short clip from the movie on billboards subject to this rule. The clip contains a graphic display of violence.The producer has filed an action in the appropriate federal court challenging the state highway administration's rule as a violation of the First Amendment as applicable to the states through the Fourteenth Amendment.Of the following standards, by which should the state highway administration's rule be judged? A As a rule that deals with a matter traditionally subject to regulation, it must be upheld unless it is arbitrary or irrational. B As a regulation of commercial speech, there must be a reasonable fit between the government's ends and the means chosen to accomplish those ends. C As a time, place, or manner restriction, it must be narrowly tailored to serve a significant governmental interest. D As a content regulation, it must be necessary to achieve a compelling governmental interest.

Answer choice D is correct. Although viewpoint neutral, the state highway administration's rule is a content-based regulation of speech because it prohibits the graphic display of violence. As such, the strict scrutiny standard of review applies. The rule must be necessary to achieve a compelling governmental interest. Answer choice A is incorrect. Although content regulation of speech that falls into certain historical categories, such as obscenity or defamation, may be permitted, the depiction of violence is not one of those categories. Answer choice B is incorrect. Although the producer is concerned with the impact of the rule on his commercial speech, the rule is not limited to commercial speech. Instead, the rule applies to any display that appears on the billboard, even those that have a noncommercial purpose, such as public service announcements. Answer choice C is incorrect. Time, place, or manner restrictions must be narrowly tailored to serve a significant governmental interest, and arguably, rules regarding electronic billboards specifically are time, place, and manner restrictions. However, the portion of the rule affecting the movie producer is a restriction of the content of expression. The more stringent standard applies.

In response to a highly publicized incident involving a local sonographer who failed to correctly read a sonogram, and at the urging of a state association of medical technicians, the state legislature enacted a bill that lengthened the period of education required to receive a license to be employed as a sonographer from one year to two years.Prior to the enactment of this law, an obstetrician with a private practice had promised a clerk who worked in her office that she would employ him as sonographer if he obtained the necessary state license. Three months before the passage of the law, the clerk had begun his coursework in order to satisfy the then one-year educational requirement to become a sonographer. Although discussed at the hearing that the legislative committee had held on this legislation, the statute did not contain a provision that delayed its effective date for those already enrolled in an educational program to become a sonographer.The clerk has filed suit challenging the constitutionality of the state statute as applied to him due to its failure to grant him an exemption from the two-year educational requirement.How is the court likely to rule? A For the clerk, because employment is a fundamental right and the state cannot satisfy the strict scrutiny test. B For the clerk, because the statute violates the Contract Clause of Article I, Section 10. C For the state, because the legislation is a regulatory taking of intangible personal property, and therefore the Takings Clause of the Fifth Amendment does not apply. D For the state, because the state legislature could rationally believe that the statute, as passed, was in the best interest of the public.

Answer choice D is correct. Because the statute does not discriminate against a suspect or quasi-suspect class, and does not infringe upon a fundamental right, the statute is subject to the rational basis test. Under this standard, the statute is presumed valid until the plaintiff establishes that the law is arbitrary or irrational. Here, the clerk is effectively asserting that the statute's failure to apply the one-year educational requirement to students enrolled in an educational program at the time of the enactment of the statute is arbitrary or irrational. However, the legislature did consider such an exception, and could have concluded that applying the two-year educational requirement to current students would better serve to protect the public from mistakes made by sonographers who were not adequately trained. Even if this reason had not been expressly discussed when the legislation was passed, any legitimate justification for a law, even one not demonstrably relied upon by the legislature, will suffice under the rational basis test. Answer choice A is incorrect. Although substantive due process does require the application of the strict scrutiny test to governmental action that infringes upon a fundamental right, the right to be employed in any profession is not a fundamental right. Answer choice B is incorrect. Although the Contract Clause does apply to state legislative action, it does not preclude any interference with a person's freedom to contract, but only prevents the substantial impairment of a contract between private parties if the government cannot demonstrate that the interference was reasonable and necessary to serve an important governmental interest. Here, it is unclear whether the clerk can enforce his employer's promise to employ him as a sonographer due to his detrimental reliance on the promise, but even assuming that he can, the statute does not prevent the clerk from enforcing that promise. Therefore, it does not impair a contract between private parties, and does not violate the Contract Clause. Answer choice C is incorrect because the Takings Clause does apply to a regulatory taking of intangible personal property. However, even assuming that the clerk has an enforceable contract with his employer that would give rise to an intangible personal property interest, the regulation does not take away his right to be a sonographer or permanently deprive his education of economic value, but merely delays the clerk's employment as a sonographer.

There has been a recent surge in the profitability of the agave nectar industry because more food and beverage manufacturers are utilizing agave in their food products instead of cane sugar. As a result, the cane sugar industry has been suffering economically. Agave plants are grown primarily in Mexico and South America. Congress recently enacted a statute placing an import tariff on agave imported into the United States, causing the overall price of agave to increase. An importer of agave into the United States has challenged the constitutionality of this statute because he lost significant business in the United States after having to raise the price of his imported agave products due to the import tariff.Is the importer likely to succeed in his action? A Yes, because the import tariff impedes upon the importer's constitutional right to free trade. B Yes, because the benefits of an import tariff are grossly outweighed by the burden it places on interstate commerce. C No, because the tariff does not place an undue burden on interstate commerce. D No, because Congress has the power to regulate international commerce.

Answer choice D is correct. Congress has the power to regulate international commerce. The tariff in question is a valid exercise of this power. Answer choice A is incorrect because the importer does not have a constitutional right to free trade. Although the import tariff may cause the cost of his agave to increase, it does not violate his constitutional rights. Answer choice B is incorrect. If the benefits of the state law are grossly outweighed by the burdens on interstate commerce, then even nondiscriminatory regulation may be struck down. Here, we are dealing with a federal statute, so this rule does not apply. Answer choice C is incorrect because a federal statute, not a state regulation, is involved, so the "undue burden" test for state laws does not apply.

There has been a recent surge in the profitability of the agave nectar industry because more food and beverage manufacturers are utilizing agave in their food products instead of cane sugar. As a result, the cane sugar industry has been suffering economically. Agave plants are grown primarily in Mexico and South America. Congress recently enacted a statute placing an import tariff on agave imported into the United States, causing the overall price of agave to increase. An importer of agave into the United States has challenged the constitutionality of this statute because he lost significant business in the United States after having to raise the price of his imported agave products due to the import tariff.Is the importer likely to succeed in his action? A Yes, because the import tariff impedes upon the importer's constitutional right to free trade. B Yes, because the benefits of an import tariff are grossly outweighed by the burden it places on interstate commerce. C No, because the tariff does not place an undue burden on interstate commerce. D No, because Congress has the power to regulate international commerce.

Answer choice D is correct. Congress has the power to regulate international commerce. The tariff in question is a valid exercise of this power. Answer choice A is incorrect because the importer does not have a constitutional right to free trade. Although the import tariff may cause the cost of his agave to increase, it does not violate his constitutional rights. Answer choice B is incorrect. If the benefits of the state law are grossly outweighed by the burdens on interstate commerce, then even nondiscriminatory regulation may be struck down. Here, we are dealing with a federal statute, so this rule does not apply. Answer choice C is incorrect because a federal statute, not a state regulation, is involved, so the "undue burden" test for state laws does not apply.

Due to the rising cost of water, Congress enacted a statute that allocated $20 million to fund a consortium of independent non-profit water organizations. The purpose of the consortium was to address the various issues causing a rise in the cost of water and water shortages across the United States, and to fund research into solutions. The statute provided clear guidelines about what the consortium should research and what it could spend, and specified that the remaining funds would be allocated for infrastructure improvements. In addition, the statute delegated to the Department of the Interior the power to select the water organizations to participate in the consortium.Is the statute constitutional? A No, because Congress is delegating its authority to the executive branch of the government. B No, because the statute exceeds the scope of Congress's investigative power. C Yes, because Congress has the power to enact this legislation under the Necessary and Proper Clause. D Yes, because Congress has the power to spend for the general welfare of the public.

Answer choice D is correct. Congress has the power to spend for the general welfare of the public. Here, spending money to figure out ways to cut the cost of water and reduce water shortages is a proper use of Congress's power to spend for the general welfare. ' Answer choice A is incorrect. Congress may delegate authority to the Department of the Interior so long as Congress specifies intelligible standards for the delegatee to follow. The facts indicate that the statute lays out guidelines for how the funds will be allocated. Therefore, the delegation is not unconstitutional. Answer choice B is incorrect. Congress's investigatory power may extend to any matter within a "legitimate legislative sphere," and funding research and infrastructure improvements for the conservation of water certainly falls within that sphere. Answer choice C is incorrect because the Necessary and Proper Clause is not an independent source of power; it merely gives Congress the power to execute specifically granted powers. Thus, the Necessary and Proper Clause cannot be the only source for Congress's authority to enact this statute.

Due to the rising cost of water, Congress enacted a statute that allocated $20 million to fund a consortium of independent non-profit water organizations. The purpose of the consortium was to address the various issues causing a rise in the cost of water and water shortages across the United States, and to fund research into solutions. The statute provided clear guidelines about what the consortium should research and what it could spend, and specified that the remaining funds would be allocated for infrastructure improvements. In addition, the statute delegated to the Department of the Interior the power to select the water organizations to participate in the consortium.Is the statute constitutional? A No, because Congress is delegating its authority to the executive branch of the government. B No, because the statute exceeds the scope of Congress's investigative power. C Yes, because Congress has the power to enact this legislation under the Necessary and Proper Clause. D Yes, because Congress has the power to spend for the general welfare of the public.

Answer choice D is correct. Congress has the power to spend for the general welfare of the public. Here, spending money to figure out ways to cut the cost of water and reduce water shortages is a proper use of Congress's power to spend for the general welfare. Answer choice A is incorrect. Congress may delegate authority to the Department of the Interior so long as Congress specifies intelligible standards for the delegatee to follow. The facts indicate that the statute lays out guidelines for how the funds will be allocated. Therefore, the delegation is not unconstitutional. Answer choice B is incorrect. Congress's investigatory power may extend to any matter within a "legitimate legislative sphere," and funding research and infrastructure improvements for the conservation of water certainly falls within that sphere. Answer choice C is incorrect because the Necessary and Proper Clause is not an independent source of power; it merely gives Congress the power to execute specifically granted powers. Thus, the Necessary and Proper Clause cannot be the only source for Congress's authority to enact this statute.

good example of congress power that dont exist with respect to executive branch

Answer choice D is correct. Congress may not appoint members of a body with administrative or enforcement powers. Such persons are "officers of the United States" and must be appointed by the President. Here, because the fishery committee has enforcement powers, the members must be appointed by the President, not by Congress. Therefore, any attempt by Congress to delegate this appointment authority makes the statute unconstitutional. Answer choice A is incorrect because, as stated above, only the President can appoint the members of this committee, and Congress has no authority to delegate this power to the Department of the Interior. Answer choice B is incorrect because Congress does not have the power to regulate for the general welfare. Congress only has the power to tax or spend for the general welfare under its taking or spending powers. Answer choice C is incorrect because it misstates the law. Delegation of some of Congress's authority to the executive branch has consistently been held constitutional, so long as Congress specifies an "intelligible principle" to guide the delegate. Almost any legislative delegation passes the "intelligible standards" requirement, so even broadly phrased standards have been upheld.

Faced with poorly performing public schools in a city, the state instituted a state-funded voucher program for parents with school age children who lived in the city. The state gave parents a voucher for each school-aged child that could be used either towards tuition for the child to attend a private school in the child's school district, whether parochial or not, or be presented to the child's public school, which would receive the same amount in additional funding. The choice of whether to attend a private or public school was solely left up to the parents and their child. Over 95 percent of the vouchers were used by parents towards tuition payments to parochial schools. The program contained no restrictions as to how the schools used the funds.Is this program constitutional? A No, because the primary effect of the voucher program is to provide state aid to parochial schools. B No, because the program contained no provisions to prevent the use of state aid to fund religious instruction at the parochial schools. C Yes, because governmental financial assistance that benefits religious educational institutions is constitutional so long as it does not result in excessive government entanglement with religion. D Yes, because the program is neutral with respect to religion and provides financial assistance directly to citizens who have a choice as to whether to use the voucher at a parochial or nonparochial school.

Answer choice D is correct. Giving parents tuition vouchers to assist them in paying religious-school tuition does not violate the Establishment Clause if the choice of whether to use the vouchers for religious or non-religious private school tuition lies with the parents. Therefore, because this program is neutral with respect to religion and provides financial assistance directly to citizens who have a choice as to where to use the funds, it does not violate the Establishment Clause of the First Amendment as applied to the states through the Fourteenth Amendment. Answer choice A is incorrect because, even though the primary effect of the voucher program was to provide state aid to parochial schools, the aid did not pass directly from the state to the parochial school. Instead, the aid flowed through private citizens who had a choice as to where to use the voucher. Answer choice B is incorrect because, although the program did not limit the use of state aid to funding secular instruction, the program was neutral in its design, providing benefits to a broad class of citizens without reference to religion. Because the aid was not provided directly to the parochial schools but instead passed to these schools because of the choice of private citizens, the program is constitutional. Answer choice C is incorrect. In applying the Lemon test with regard to governmental financial aid, the third element (no excessive governmental entanglement) is not a separate requirement, but instead is one factor to be considered in ascertaining whether the second element (no advancement or inhibition of religion) has been met. In any case, consideration of whether there is excessive governmental entanglement with religion is not the sole determinant of whether governmental financial aid that is received by religious institutions violates the Establishment Clause.

very good question about government involvement with private actor A patient at a government-licensed, private nursing facility was receiving financial assistance from the government for the medical care he received. The status of the patient was reviewed by a committee of physicians working at the facility to determine whether the patient's level of care was appropriate, and whether the patient's continued stay in the facility was justified. The committee determined that the patient no longer needed the level of care provided by the facility, and ordered the transfer of the patient to another nursing facility that offered a lower level of care. Since the patient was receiving financial support through a government-funded program, the committee notified the appropriate governmental official who administered the program. The official in turn contacted the patient and informed him that due to the decision of the committee of physicians, his medical financial assistance would be terminated unless he accepted the transfer. The patient was properly notified of an administrative hearing by the governmental agency who administered the program, and the hearing confirmed the official's decision to terminate his medical financial assistance unless he accepted the transfer. The patient sued the nursing facility for injunctive relief, contending that he was unconstitutionally denied his procedural due process rights with regard to the initial review of his status by the committee of physicians, having received neither notice of the review nor an opportunity to be heard with regard to the review itself.Is the court likely to rule in favor of the patient? A Yes, because the nursing facility was licensed by the government. B Yes, because the nursing facility received substantial funding from the government C No, because the patient's procedural due process rights were met by notice and the administrative hearing. D No, because the decision was made by a committee of physicians working at a private nursing facility.

Answer choice D is correct. In order for a person's procedural due process rights to be violated, the violation must be the result of an action undertaken by the government, not a private person. Although there are circumstances in which an action by a private person is deemed to be governmental action, those circumstances do not exist here. The provision of medical services is not an activity that is traditionally performed exclusively by the government. While private action may also be deemed to be governmental if there is significant government involvement, substantial governmental regulation of a business, such as regulation of a private nursing facility, is not sufficient to constitute significant government involvement. Moreover, the receipt of substantial funding from the government does not transform a private person into a governmental actor, thus the court is not likely to rule in favor of the patient against the nursing facility. Answer choice A is incorrect because the mere licensing or regulation of a private person does not transform an action undertaken by that person into government action. Answer choice B is incorrect because the receipt of substantial funds from the government by a private person does not transform an action undertaken by that person into government action. Answer choice C is incorrect because, while the patient was provided with notice and an administrative hearing with regard to the conditional termination of the financial assistance provided by the government (which satisfied his procedural due process rights with regard to such assistance), the focus of the patient's lawsuit is his right to notice and hearing with regard to the decision made by the committee of physicians, not the related decision regarding his government financial assistance.

A state legislature passed a law requiring employers to provide their employees with health insurance that covered certain prescription drugs. Violation of this statute was considered a crime that subjected the offender to fines, which were described in detail in the statute. The law was effective immediately. An employer did not provide its employees with insurance that covered the required drugs, and argued that such drugs were prohibited by the religion practiced by the employer. The employer filed a complaint in federal court asserting that the law was unconstitutional and asked for a preliminary injunction against the attorney general to prevent him from enforcing the statute while the case was heard. The attorney general filed a motion to dismiss, asserting that the federal court did not have jurisdiction to hear the case.How should the federal court rule on the attorney general's motion to dismiss? A Grant the motion based on prudential grounds. B Grant the motion based on the doctrine of abstention. C Deny the motion, because the employer has taxpayer standing due to the imposition of fines. D Deny the motion, because the employer's injury is imminent.

Answer choice D is correct. In order to bring a claim in federal court, a plaintiff must have standing and the issue must be ripe for litigation. To have standing, a plaintiff must establish (1) a particularized injury, which must have actually occurred or be imminent; (2) that defendant caused this injury; and (3) that the relief requested is likely to redress the injury. First, the employer can show an imminent injury—loss of money from the threatened fines, which can be imposed immediately. Second, the government caused this injury by forcing the employer to comply with the law or pay a fine. Third, the requested injunction will redress that injury by removing the burden of compliance unless and until a court rules that the law is constitutional. Furthermore, the case is ripe because the threat of enforcement is imminent and the legal issues are sufficiently developed for a court to decide the case. Answer choice A is incorrect because neither of the categories of prudential standing applies here: The court is not being asked to adjudicate the claims of third parties or "generalized grievances" (i.e., injuries that are widely shared, as contrasted with the employer's individualized injury here). Answer choice B is incorrect because the two abstention doctrines—the Younger abstention doctrine, which applies only to pending state criminal cases or administrative proceedings, and the Pullman abstention doctrine, in which constitutional claims depend on resolving an unsettled issue of state law—are inapplicable. Answer choice C is incorrect because the doctrine of taxpayer standing, which allows a taxpayer standing to file a federal lawsuit challenging a specific legislative appropriation for violation of the Establishment Clause, is also inapplicable.

Recently, numerous patients located in State A have become temporarily paralyzed due to severe botulism poisoning from anti-wrinkle injections. The patients that suffered from temporary paralysis were injected with a botulinum toxin called BoTween. Under federal law, the basic safety and minimum testing requirements for botulinum toxin are outlined. In addition, federal law mandates that no form of botulinum toxin may be commercially distributed for use on humans unless it has been approved by the United States Food and Drug Administration (FDA). Two types of botulinum toxin, BoTeen and BoTween, have been approved by the FDA. However, State A has enacted a new law requiring more rigorous safety standards and testing than that delineated by the FDA for botulinum toxin injections. Under the law of State A, only BoTeen may be used on humans.Which of the following statements regarding State A's law is the most accurate? A The law is invalid because Congress intended to occupy the field of botulinum toxin regulation. B The law requires conduct that is forbidden by federal law. C The law conflicts with federal law by creating an obstacle to its purpose. D State A is free to enact similar legislation despite the existence of the federal law.

Answer choice D is correct. It is possible (and common) for the federal and state governments to legislate in the same area. When this happens, the Supremacy Clause provides that federal law supersedes conflicting state law. Any state constitutional provision or law that directly or indirectly conflicts with a federal law, including federal regulations, is void under this clause. However, the Supreme Court has frequently stated that there is a presumption against preemption, especially in areas in which states have traditionally exercised police power. If federal law does not preempt state law, a state is free to enact legislation regarding the same issue. Here, the federal law did not preempt state law regarding safety standards and testing for botulinum toxin. Thus, State A is free to enact similar or more rigorous legislation. Answer choice A is incorrect. Federal preemption is implied when Congress intended for federal law to occupy the field. Intent to occupy a field can be inferred from a framework of regulation so pervasive that Congress left no room for states to supplement it or when there is a federal interest so dominant that the federal system will be assumed to preclude enforcement of state laws on the same subject. In this case, there is no indication that Congress intended to occupy the field of botulinum toxins, thus State A can legislate in this area of law. Answer choice B is incorrect. Federal preemption is implied when the state law directly conflicts with the federal law by, for example, requiring conduct that is forbidden by the federal law or making it impossible (or nearly so) to comply with both. Here, the more rigorous safety and testing standards for botulinum toxins are not forbidden by federal law, nor would it be impossible to comply with both. Answer choice C is incorrect. Federal preemption is implied when the state law indirectly conflicts with federal law by creating an obstacle to or frustrating the accomplishment of that law's purpose. Here, the state law does not create an obstacle or frustrate the accomplishment of the purpose of the federal law-to create higher standards of safety and testing for botulinum toxins.

Recently, numerous patients located in State A have become temporarily paralyzed due to severe botulism poisoning from anti-wrinkle injections. The patients that suffered from temporary paralysis were injected with a botulinum toxin called BoTween. Under federal law, the basic safety and minimum testing requirements for botulinum toxin are outlined. In addition, federal law mandates that no form of botulinum toxin may be commercially distributed for use on humans unless it has been approved by the United States Food and Drug Administration (FDA). Two types of botulinum toxin, BoTeen and BoTween, have been approved by the FDA. However, State A has enacted a new law requiring more rigorous safety standards and testing than that delineated by the FDA for botulinum toxin injections. Under the law of State A, only BoTeen may be used on humans.Which of the following statements regarding State A's law is the most accurate? A The law is invalid because Congress intended to occupy the field of botulinum toxin regulation. B The law requires conduct that is forbidden by federal law. C The law conflicts with federal law by creating an obstacle to its purpose. D State A is free to enact similar legislation despite the existence of the federal law.

Answer choice D is correct. It is possible (and common) for the federal and state governments to legislate in the same area. When this happens, the Supremacy Clause provides that federal law supersedes conflicting state law. Any state constitutional provision or law that directly or indirectly conflicts with a federal law, including federal regulations, is void under this clause. However, the Supreme Court has frequently stated that there is a presumption against preemption, especially in areas in which states have traditionally exercised police power. If federal law does not preempt state law, a state is free to enact legislation regarding the same issue. Here, the federal law did not preempt state law regarding safety standards and testing for botulinum toxin. Thus, State A is free to enact similar or more rigorous legislation. Answer choice A is incorrect. Federal preemption is implied when Congress intended for federal law to occupy the field. Intent to occupy a field can be inferred from a framework of regulation so pervasive that Congress left no room for states to supplement it or when there is a federal interest so dominant that the federal system will be assumed to preclude enforcement of state laws on the same subject. In this case, there is no indication that Congress intended to occupy the field of botulinum toxins, thus State A can legislate in this area of law. Answer choice B is incorrect. Federal preemption is implied when the state law directly conflicts with the federal law by, for example, requiring conduct that is forbidden by the federal law or making it impossible (or nearly so) to comply with both. Here, the more rigorous safety and testing standards for botulinum toxins are not forbidden by federal law, nor would it be impossible to comply with both. Answer choice C is incorrect. Federal preemption is implied when the state law indirectly conflicts with federal law by creating an obstacle to or frustrating the accomplishment of that law's purpose. Here, the state law does not create an obstacle or frustrate the accomplishment of the purpose of the federal law-to create higher standards of safety and testing for botulinum toxins.

In response to the deaths of 15 members of an extended family in a house fire, the state legislature enacted a statute that prohibited more than seven people over age 18 from living together in the same home, unless those people were members of the same immediate family. The legislature enacted the statute based on studies indicating that related individuals are more likely to be injured or killed in fire because, rather than immediately fleeing, they attempt to locate each other before escaping to safety. The study recommended the approach taken by the legislature, but acknowledged that other approaches, such as requiring attendance at fire safety classes, could also be effective. Following the report, members of an extended family of 12 individuals who lived together in the same house filed suit challenging the constitutionality of the statute.Will the court mostly likely conclude that the statute is constitutional? A Yes, because there is no fundamental right affected by the statute. B Yes, because it involves a compelling state interest. C No, because family members constitute a suspect classification. D No, because it is not the least restrictive means for reducing fire deaths of members of an extended family who live together.

Answer choice D is correct. Related persons, including extended family members, have a fundamental right to live together in a single household. Therefore, the strict scrutiny test applies to the statute. Under this test, the law must be the least restrictive means to achieve a compelling governmental interest. Here, the state does have a compelling interest in preventing loss of life due to fire, but there are less restrictive means to achieve that interest, such as requiring members of an extended family to attend fire safety classes, rather than banning such living arrangements altogether. Consequently, the court is likely to conclude that the statute is unconstitutional. Answer choice A is incorrect because related persons, including extended family members, have a fundamental right to live together in a single household. Answer choice B is incorrect because, although the state does have a compelling state interest, it does not achieve it in the least restrictive manner. Answer choice C is incorrect because extended family members do not constitute a suspect classification. Instead, the right of such individuals to live together is a fundamental right.

A state law authorized slot machines at various locations in the state and imposed a tax on revenues from the slot machines at a rate of 35 percent. In response to the financial peril in which the state riverboat industry found itself, the state lowered the tax rate on revenues from slot machines found on riverboats in the state, which represented a substantial portion of the riverboat industry's income, to 20 percent. The owners of the venues paying the 35 percent tax rate on their slot machine revenue filed an action in federal court challenging the constitutionality of the different tax rates depending upon the type of venue because it violates the Equal Protection Clause.Is the court likely to rule in favor of the plaintiffs? A Yes, because the state was favoring one intrastate venue over other types of venues. B Yes, because the law violated the geographic uniformity requirement imposed by Article I, Section 8. C No, because the subsidy exception of the Dormant Commerce Clause applies. D No, because the different tax rates were rationally related to a legitimate state interest.

Answer choice D is correct. Since the plaintiffs have challenged the differential tax rates based on equal protection grounds, and because the plaintiffs are not members of a suspect or quasi-suspect class, the discriminatory tax rate is only subject to the rational basis test. Here, the state acted to aid a state industry (the riverboat industry) that was in financial peril (a legitimate state interest) by lowering the tax rate on one form of revenue that generated a substantial portion of its income. While the state could have undertaken other actions to aid the riverboat industry, the modification of the tax rate on slot machine revenue was rationally related to the state's interest in reducing the riverboat industry's financial peril. Consequently, the court is likely to rule against the venue owners paying the 35 percent tax rate. Answer choice A is incorrect because, unlike state action that discriminates against interstate commerce, there is no specific constitutional restriction on state action that discriminates against intrastate business in favor of another intrastate business. Moreover, the plaintiffs' challenge is not based on the Dormant Commerce Clause, but on the Equal Protection Clause. Answer choice B is incorrect because the geographic uniformity requirement is a constitutional restriction imposed on the federal government, not on the states. Moreover, as stated above, the plaintiffs' challenge is based on the Equal Protection Clause. Answer choice C is incorrect because, while a state may provide a subsidy for its own citizens under the Dormant Commerce Clause (e.g., a lower tuition rate for in-state students attending college), the plaintiffs' challenge is based on the Equal Protection Clause.

A state law authorized slot machines at various locations in the state and imposed a tax on revenues from the slot machines at a rate of 35 percent. In response to the financial peril in which the state riverboat industry found itself, the state lowered the tax rate on revenues from slot machines found on riverboats in the state, which represented a substantial portion of the riverboat industry's income, to 20 percent. The owners of the venues paying the 35 percent tax rate on their slot machine revenue filed an action in federal court challenging the constitutionality of the different tax rates depending upon the type of venue because it violates the Equal Protection Clause.Is the court likely to rule in favor of the plaintiffs? A Yes, because the state was favoring one intrastate venue over other types of venues. B Yes, because the law violated the geographic uniformity requirement imposed by Article I, Section 8. C No, because the subsidy exception of the Dormant Commerce Clause applies. D No, because the different tax rates were rationally related to a legitimate state interest.

Answer choice D is correct. Since the plaintiffs have challenged the differential tax rates based on equal protection grounds, and because the plaintiffs are not members of a suspect or quasi-suspect class, the discriminatory tax rate is only subject to the rational basis test. Here, the state acted to aid a state industry (the riverboat industry) that was in financial peril (a legitimate state interest) by lowering the tax rate on one form of revenue that generated a substantial portion of its income. While the state could have undertaken other actions to aid the riverboat industry, the modification of the tax rate on slot machine revenue was rationally related to the state's interest in reducing the riverboat industry's financial peril. Consequently, the court is likely to rule against the venue owners paying the 35 percent tax rate. Answer choice A is incorrect because, unlike state action that discriminates against interstate commerce, there is no specific constitutional restriction on state action that discriminates against intrastate business in favor of another intrastate business. Moreover, the plaintiffs' challenge is not based on the Dormant Commerce Clause, but on the Equal Protection Clause. Answer choice B is incorrect because the geographic uniformity requirement is a constitutional restriction imposed on the federal government, not on the states. Moreover, as stated above, the plaintiffs' challenge is based on the Equal Protection Clause. Answer choice C is incorrect because, while a state may provide a subsidy for its own citizens under the Dormant Commerce Clause (e.g., a lower tuition rate for in-state students attending college), the plaintiffs' challenge is based on the Equal Protection Clause.

An appropriate representative of a resident in a state mental health facility sued an official of the facility in federal court. The resident, who remained a citizen of another state, sought an injunction to compel the official to comply with state law regarding the use of the least restrictive environment approach for the care of the mentally ill.The official moved to dismiss the action as unconstitutional under the Eleventh Amendment. Should the court grant the official's motion? A No, because an injunction, not damages, was sought as a remedy. B No, because the action was brought against a state official rather than the state. C Yes, because a fundamental right is not involved. D Yes, because the action sought the enforcement of state law rather than federal law.

Answer choice D is correct. The Eleventh Amendment prohibits an action in federal court by a citizen of one state against another state when the basis for the action is the violation of state law. Answer choice A is incorrect because, although the plaintiff seeks an injunction rather than damages, the Eleventh Amendment prohibits the action nonetheless, as it is based on state law. Similarly, answer choice B is incorrect because, although the action is brought against the state official rather than the state, the Eleventh Amendment prohibits an action based on state law. Answer choice C is incorrect because, although a plaintiff can pursue an action despite the Eleventh Amendment in order to prevent the enforcement of an unconstitutional state statute, the plaintiff is not advancing such an argument here. Since the plaintiff is not challenging the constitutionality of the state statute under the Due Process Clause of the Fourteenth Amendment, the issue of whether the plaintiff is asserting a fundamental right is irrelevant.

An appropriate representative of a resident in a state mental health facility sued an official of the facility in federal court. The resident, who remained a citizen of another state, sought an injunction to compel the official to comply with state law regarding the use of the least restrictive environment approach for the care of the mentally ill.The official moved to dismiss the action as unconstitutional under the Eleventh Amendment. Should the court grant the official's motion? A No, because an injunction, not damages, was sought as a remedy. B No, because the action was brought against a state official rather than the state. C Yes, because a fundamental right is not involved. D Yes, because the action sought the enforcement of state law rather than federal law.

Answer choice D is correct. The Eleventh Amendment prohibits an action in federal court by a citizen of one state against another state when the basis for the action is the violation of state law. Answer choice A is incorrect because, although the plaintiff seeks an injunction rather than damages, the Eleventh Amendment prohibits the action nonetheless, as it is based on state law. Similarly, answer choice B is incorrect because, although the action is brought against the state official rather than the state, the Eleventh Amendment prohibits an action based on state law. Answer choice C is incorrect because, although a plaintiff can pursue an action despite the Eleventh Amendment in order to prevent the enforcement of an unconstitutional state statute, the plaintiff is not advancing such an argument here. Since the plaintiff is not challenging the constitutionality of the state statute under the Due Process Clause of the Fourteenth Amendment, the issue of whether the plaintiff is asserting a fundamental right is irrelevant.

A private secular school rented out its auditorium to the public for various uses on a first come, first serve basis. A local religious group sought to rent the auditorium for weekly services. The school refused because it did not want to be seen as endorsing a particular religion. The group sued the school, arguing that, by refusing to allow it to rent the auditorium, the school was violating its constitutional rights.Is the religious group likely to succeed? A Yes, because the school's denial violated the religious group's First Amendment right of free speech. B Yes, because the religious group is entitled to rent the auditorium under the First Amendment Free Exercise Clause. C No, because permitting the religious group to use the auditorium for services would violate the First Amendment Establishment Clause. D No, because the school was a private entity.

Answer choice D is correct. The First Amendment applies only to federal or state action. A private person's action may constitute state action if a private person performs a traditional governmental function or there is significant state involvement. Here, the school is identified as a private entity. Consequently, the First Amendment does not apply to its actions. Answer choice A is incorrect. A public school that opens its facilities for use by the general public violates a religious group's First Amendment right of free speech by prohibiting the group from using the facilities because the group would discuss religious topics on the premises. However, the school in question is a private entity and therefore is not subject to First Amendment constraints. Answer choice B is incorrect because, although the school's refusal to rent to the religious organization does impact the group's ability to exercise its religious beliefs, the school in question is a private entity and therefore is not subject to First Amendment constraints. Answer choice C is incorrect because the school in question is a private entity and therefore is not subject to First Amendment constraints. Moreover, a public school that allows student groups or organizations to use its facilities does not violate the Establishment Clause by allowing a religious organization to use its facilities.

To address a shortfall in the judiciary's budget and to comply with a provision in the state constitution requiring a balanced budget, a state increased, by statute, various court fees, including the fee to file an appeal, and eliminated any waiver of these fees except for a defendant convicted of a capital crime and sentenced to death. A defendant imprisoned after his conviction of a non-capital offense filed an appeal and a request for a waiver of the fee because of his inability to pay. Citing the statute, the state appellate court ruled that the defendant was not entitled to the waiver, as he was not convicted of a capital offense and sentenced to death.The defendant properly challenged this ruling in federal court. Should the federal court uphold the state appellate court's ruling? A Yes, because the fee is rationally related to the goal of adequately funding the state's judicial system. B Yes, because state law required payment of the fee. C No, because a state may not prohibit a prisoner from exercising a fundamental right. D No, because the defendant was indigent.

Answer choice D is correct. While most state actions that discriminate against the poor are subject only to rational basis scrutiny, the availability of appeal in a criminal case cannot hinge on ability to pay a filing fee. To deny an indigent the right to appeal through the imposition of a fee violates the Due Process and Equal Protection Clauses of the Fourteenth Amendment. Answer choice A is incorrect because, although there is a rational basis for the state's increase in the various court fees, an indigent accused has a constitutional right to waiver of a fee that prevents him from enjoying a fundamental right, such as access to the courts. Answer choice B is incorrect because, even though the state court decision was based on state law, the state court cannot enforce a state law that conflicts with the federal constitution. Answer choice C is incorrect because, while a prisoner has a right of reasonable access to the court, the law in question did not deny the defendant such access on the basis of his status as a prisoner, but due to his failure to pay the necessary filing fee.

laws about same sex rights Several cities within a state passed ordinances that prohibited discrimination against gay and lesbian individuals in employment and housing. In response, the state legislature passed a statute that prohibited any city in the state from passing any ordinance that recognized gay or lesbian individuals as a protected class. A resident of the state sued the state, arguing that the statute violated the Equal Protection Clause.Is the law likely to be upheld? A No, because the Supreme Court has recognized sexual orientation as a suspect classification. B No, because the law would not satisfy the rational basis test. C Yes, because sexual orientation is not a suspect classification subject to the Equal Protection Clause. D Yes, because the law would satisfy the intermediate scrutiny test.

B is correct. The Supreme Court has struck down bans on same-sex marriage as violations of a fundamental right on both Due Process and Equal Protection grounds, but it has not resolved the issue of whether discrimination based on sexual orientation is subject to heightened scrutiny. Regardless of whether heightened scrutiny applies here, the government cannot impose a burden upon or deny a benefit to a group of persons solely based on animosity toward the class that it affects. In this case, a court would likely strike down the law because it does not satisfy the rational basis test. Note - sexual orientation is not a suspect classification. although sexual orientation is not a suspect classification, laws based on such a classification are subject to Equal Protection analysis. Also, the intermediate scrutiny test, which is used for classifications based on gender or status as a nonmarital child, has not been found to apply to sexual orientation.

government speech

B. Government Speech • The First Amendment restrictions basically do not apply to the government as a speaker. Its obvious - govt is free to advertise the us army. It doesn't have to give equal time to critics of military. Same is true when govt accepts a monument donated by private persons. Govt doesn't have to accept any monument just because it accepts one - it can still control what gifts it accepts. • Government as a speaker is free to express a point of view (e.g., advertising the military). • The government does not have to accept all monuments donated by a private person simply because it accepts one—when the government is controlling the message, it is entitled to say what it wants. But one area where govt speech has been litigated is specialty license plates. These carry messages requested by the customers. Messages are requested by drivers but license plates are still govt speech so govt can refuse licenses that contain offensive messages. Govt sets up thing where u pay a little extra, u get to have a little slogan or words on ur lciense plate that mean something but theyre free to reject licenses that would be offensive to others. That's because its government speech. • But specialty license plates bearing messages requested by purchasers are still government speech, so the government can refuse to issue plates that would be offensive to other citizens. C. Corporations - Corporations have the same First Amendment right of free speech to speak as individuals. Citizens united - supreme court held govt could not suppress polticial speech merely because speaker was a corporation.

STATE TAXATION OF INTERSTATE COMMERCE, PREEMPTION, AND RELATIONS AMONG STATES - preemption

B. Preemption • Federal law preempts (overrides) inconsistent state law. Any kind of federal law can pre-empt - can be constitution itself or can be fed statute. Can be administrative regulation or even executive order - any valid federal law overrides state law if inconsistent with or incompatible with federal law. • State law is not preempted simply because it addresses the same subject matter or topic as a federal statute. Its preempted only if inconsistent with fed statute - There must be incompatibility or conflict. Example 24: Federal law provides automobile emissions must be at least 95% pure. California adopts a more stringent requirement requiring CA emissions be 98% pure (which is automatically at least 95% pure). Is this state law preempted? No. There is no conflict—you can comply with both of those standards. • Preempting the Field: When Congress determines that there should be no state law of any sort in a particular field, then any state law in that area is inconsistent with the federal statute and is preempted. This is rare.

Common question - congress passed statute, statute authorizes states to regulate particular industry like insurance any way they like, a state then turns around and taxes out of state insurance providers at a higher rate than in state competitors - that's discrimination, it would be struck down in a heartbeat if state did it on its own but when congress authorizes or consents to it, then its okay under the commerce clause.

Common question - congress passed statute, statute authorizes states to regulate particular industry like insurance any way they like, a state then turns around and taxes out of state insurance providers at a higher rate than in state competitors - that's discrimination, it would be struck down in a heartbeat if state did it on its own but when congress authorizes or consents to it, then its okay under the commerce clause.

spending power

Congress has the power to spend for the general welfare of the public.

delegation of power to president - inferior officials

Congress may delegate the appointment of "inferior" officials to the President. "Inferior" officials are those supervised by Senate-confirmed appointees. Here, Congress can delegate to the President the power to appoint these inferior officials without Senate approval because they will be supervised by a Senate-confirmed appointee. Note - Congress may not itself appoint members of a body with administrative or enforcement powers. Such persons are "officers of the United States" and must be appointed by the President with the advice and consent of the Senate. Congress may, however, delegate the appointment of "inferior" officials to the President alone (i.e., without Senate approval), the heads of executive departments, or the courts.

commerce clause/federalism concerns

Congress's power under the Commerce Clause to regulate intrastate activity that is not obviously economic (so-called "non-economic" activity) is limited by principles of federalism, at least when the regulation involves an area of traditional state concern. Because the facts do not indicate that this regulation is an essential part of a larger regulation of interstate commerce, the fact that the legislation regulates non-economic activity in an area of traditional state concern is the strongest basis for finding this legislation unconstitutional.

procedural due process consists of

Consists of two questions: 1) Is life, liberty, or property being taken? 2) If life, liberty, or property is being taken, what process is due? Some process must be due.

Regulation of the Content of Expression

Content-based regulations of speech trigger strict scrutiny and are usually struck down - few categories of speech that can be regulated because of content but quite exceptional. 1. expressive conduct 2. vagueness and overbreadth 3. prior restraints

campaign contributions vs expenditures

Contributions versus expenditures: 1. Contributions to political campaigns and candidates can be regulated, provided that the limits are not unreasonably low. 2. Direct expenditures in support of a candidate, a campaign, or a political issue cannot be regulated. They can be made regardless of amount. - The rationale is to prevent corruption: a candidate could theoretically act in a certain way in exchange for a larger, direct contribution. Diff between contribution and expenditure? Different is in their relation to corruption. Large contributions where u give money to a campaign or candidate may create the reality or at least appearance of corruption as candidate would presumably be willing to do things to get the contributions but direct expenditure of ones own money doesn't pose same risk of corruption.

corporations

Corporations have the same First Amendment right of free speech to speak as individuals. Citizens united - supreme court held govt could not suppress polticial speech merely because speaker was a corporation.

FILL IN THE BLANKS. In violation of due process, a law that fails to give _________ of what is prohibited is void for vagueness. In violation of the Free Speech Clause, a law that burdens substantially more speech than is necessary to protect a _________ government interest is void for overbreadth. A clear notice; compelling B clear notice; significant C complete notice; compelling D complete notice; significant

Correct Answer: A

In order to establish racial discrimination under the Equal Protection Clause, which of the following must be proved? A Discriminatory purpose B Discriminatory impact C Disparate effect D Disparate impact

Correct Answer: A

Regarding federal preemption of state law, which of the following is TRUE? A A federal statute preempts an inconsistent state statute. B A state law may not set a higher standard than the federal standard. C A state law is preempted if it addresses the same topic as a federal statute. D Federal law that preempts a field does not automatically preempt a prior state law.

Correct Answer: A

Regarding governmental regulation of commercial speech, which of the following is TRUE? A The regulation must directly advance the governmental interest. B The governmental interest must be necessary. C The regulation must be the least restrictive means of advancing the governmental interest. D Unlike false commercial speech, misleading commercial speech may not be prohibited.

Correct Answer: A

Which of the following are subject to intermediate scrutiny as quasi-suspect classes? A Gender and legitimacy, but not age B Gender and age, but not legitimacy C Age and legitimacy, but not gender D Gender, legitimacy, and age

Correct Answer: A

Which of the following is TRUE regarding the Takings Clause? A A taking occurs when a regulation amounts to a permanent, total loss of economic value in the land. B Government may not restrict the use of private property without just compensation. C Any regulation that results in the decrease of economic value will be considered a taking. D Conditioning of a permit to develop property on a concession by the developer is an impermissible extraction unless just compensation is paid.

Correct Answer: A

FILL IN THE BLANKS. A law that regulates expressive conduct (i.e., symbolic speech) is upheld if it furthers an important government interest, that interest is _________, and the burden on expression is ________________. A unrelated to the suppression of expression; insignificant B unrelated to the suppression of expression; no greater than is necessary C related to the suppression of expression; insignificant D related to the suppression of expression; no greater than is necessary

Correct Answer: B

FILL IN THE BLANKS. The media has ______________________ the general public, but ______________ may be subject to greater governmental regulation. A the same rights as, print journalists B the same rights as, broadcasters C greater rights than, print journalists D greater rights than, broadcasters

Correct Answer: B

Regarding obscenity and pornography with respect to minors, which of the following is FALSE? A The government cannot ban adult speech simply because it would be obscene for minors. B The private possession of child pornography cannot be prohibited. C The sale and distribution of child pornography can be banned even if not obscene. D Neither obscenity nor child pornography enjoy the protection of the Free Speech Clause.

Correct Answer: B

Regarding state taxation of interstate commerce, which of the following is FALSE? A A tax that discriminates against out-of-state residents is unconstitutional unless Congress consents. B For a nondiscriminatory tax, there need not be a fair appointment of tax liability between the states. C For a nondiscriminatory tax, there must a substantial nexus between the taxing state and the property or activity to be taxed. D A nondiscriminatory tax may still be unconstitutional if it is unduly burdensome.

Correct Answer: B

To be constitutional, a government regulation of speech in a public forum must comply with which of the following requirements? A The regulation must be content-neutral as to viewpoint, but not subject matter. B The regulation must leave open alternative channels of communication. C The regulation must narrowly serve a compelling governmental interest. D The regulations must allow the public official charged with enforcing it reasonable discretion.

Correct Answer: B

Which of the following laws is NOT constitutionally prohibited? A A state law that criminalizes an act that was not criminal when originally committed B A federal law that impairs an existing contract between private parties C A federal law that authorizes, after an act was committed, the imposition of a more severe penalty on that act D A state law that declares a person or group of persons guilty of some crime and punishes them without a trial

Correct Answer: B

Which of the following statements is accurate under the Lemon Test regarding governmental action and religion? A The governmental action cannot have a substantial impact on religion. B The governmental action must have a secular purpose. C The governmental action cannot advance but may inhibit religion. D The governmental action must not intend, but can result in, governmental entanglement with religion.

Correct Answer: B

Which of the following statements related to wealth-based classifications is TRUE? A Classifications based on wealth are generally subject to an intermediate level of review. B Governmental fees that prevent an indigent person from exercising a fundamental right must be waived. C A government may not charge a fee for the exercise of a fundamental right. D Statutes that discriminate against the poor are generally subject to strict scrutiny.

Correct Answer: B

With regard to which of the following can a state NOT discriminate against a resident alien? A Voting B Private employment C Jury service D Public employment

Correct Answer: B

FILL IN THE BLANK. Under the Free Exercise Clause, a state's neutral laws of general applicability that have an incidental impact on religious conduct are _____________________. A subject to strict scrutiny B unconstitutional absent the showing of an important government interest C subject to rational basis review D usually struck down

Correct Answer: C

FILL IN THE BLANKS. A ______________ classification based on alienage is upheld unless the classification is ____________. A state, arbitrary and unreasonable B state, substantially related to an important government interest C federal, arbitrary and unreasonable D federal, substantially related to an important government interest

Correct Answer: C

FILL IN THE BLANKS. Campaign _________________ a candidate may be limited in order to ________________. A expenditures by, prevent corruption B expenditures by, equalize campaign resources C contributions to, prevent corruption D contributions to, equalize campaign resources

Correct Answer: C

FILL IN THE BLANKS. Non-discrimination employment laws cannot constitutionally be applied to _______________ employed by a _____________________. A religious believers, religious institution B religious believers, for-profit corporation C ministers, religious institution D ministers, for-profit corporation

Correct Answer: C

FILL IN THE BLANKS. States can enter into compacts with each other, but a compact that affects ____________ rights must secure _________________ approval. A individual, congressional B individual, presidential C federal, congressional D federal, presidential

Correct Answer: C

Regarding governmental restrictions on obscene speech, which of the following is FALSE? A To be obscene, speech must be patently offensive to the average person of the community. B To be obscene, speech must lack serious literary, artistic, scientific, or political value. C To be obscene, speech must appeal to prurient or violent interests. D In determining the lack of serious value, a national standard is applied by the court, not the jury.

Correct Answer: C

Which of the following persons cannot be tried in a military tribunal? A Enemy soldiers B Enemy civilians C U.S. civilians D U.S. soldiers

Correct Answer: C

Which of the following statements regarding racial affirmative action is FALSE? A Using race as a factor in determining admission to a public college in order to promote diversity is justified. B Using facially race-neutral criteria to determine public school attendance zones when those zones have the effect of assigning students to schools on the basis of race is a valid means of maximizing diversity. C Remedying general societal injustice through affirmative action is a compelling state interest justifying use of affirmative action. D Elimination of past discrimination in a particular governmental institution is a compelling state interest justifying use of affirmative action.

Correct Answer: C

With regard to ad valorem property taxes imposed by a state, which of the following is TRUE? A A state may tax all goods that are within its borders on a specific date. B A state may not tax the instrumentalities of commerce (e.g., trucks, airplanes). C No ad valorem property taxes are due on commodities that are merely passing through the state. D A state may not impose an ad valorem property tax on commodities that are shipped into the state from another state.

Correct Answer: C

Regarding government regulation of campaign expenditures, which of the following is FALSE? A Direct independent campaign expenditures made by unions and corporations are constitutionally protected. B Campaign expenditures that are coordinated with a candidate can be regulated as a contribution. C Independent expenditures cannot be regulated. D Direct expenditures in support of a campaign may be restricted.

Correct Answer: D

Regarding governmental regulation of content-based speech, which of the following is FALSE? A Speech that is an incitement to immediate violence is not constitutionally protected. B Laws that regulate fighting words typically run afoul of the void for vagueness and overbreadth doctrines. C A public figure can recover for defamation only if knowing or reckless falsity is proven. D A private person cannot recover for a negligent false statement of fact that damages her reputation.

Correct Answer: D

The Privileges and Immunities Clause of Article IV (i.e., the Comity Clause) deals with the privileges and immunities of ______________ citizenship and prohibits serious discrimination against out-of-state _______________. A national, citizens B national, individuals C state, corporations D state, individuals

Correct Answer: D

To be constitutional, a government regulation of speech in a limited public forum is not required to meet which of the following requirements? A The regulation must be content-neutral as to viewpoint and subject matter. B The regulation must leave open alternative channels of communication. C The regulation must narrowly serve a significant governmental interest. D The regulations must allow the public official charged with enforcing it reasonable discretion.

Correct Answer: D

Under the Establishment Clause, which of the following actions is always unconstitutional? A Aid to parochial schools B Posting the Ten Commandments on public property C Displaying a nativity scene on public grounds D Officially-sponsored school prayer

Correct Answer: D

What are the three criteria for standing? A Injury, causation, and damages B Injury, case in controversy, and causation C Injury, timeliness, and ripeness D Injury, causation, and redressability

Correct Answer: D

Which of the following does NOT accurately reflect immunities and privileges enjoyed by federal officials? A A judge has absolute immunity from civil liability for judicial acts. B The President has absolute immunity from civil liability for acts performed as part of presidential responsibilities. C The Speech or Debate Clause protects congressional aides, as well as members of Congress. D Executive privilege shields the disclosure of confidential information for any reason.

Correct Answer: D

Which of the following is not a requirement in order for a state to have to give full faith and credit to a judgment rendered by another state's court? A The court had jurisdiction over the matter and parties. B The judgment rendered was final. C The judgment was on the merits. D The judgment was adequately supported by state law.

Correct Answer: D

Which of the following statements regarding the right to vote is FALSE? A Race may be a factor, but it cannot be the predominant factor in setting the boundaries of legislative districts. B When the government establishes voting districts for the election of representatives, the number of persons in each district must be approximately equal. C Allocation of the weight given to a vote on the basis of the amount of land owned may be upheld with regard to water-district elections. D Drawing districts to scatter minorities and thus dilute their vote, even if done with a discriminatory purpose, is considered a political question and non-justiciable.

Correct Answer: D

accident - due process

Deprivation: Notice and a hearing are not required when there is an accident. Random negligence by a state employee does not constitute a deprivation of life, liberty, or property (e.g., death by a municipal garbage truck). Deprivation requires the intentional taking away of life, liberty, or property.

Due Process versus Equal Protection - - rule of thumb to distinguish between them

Due Process versus Equal Protection - - rule of thumb to distinguish between them o If a law denies a fundamental right to everyone, it violates due process. o If a law denies a fundamental right to only some but not others, it violates equal protection. But fundamental rights are the same - travel, etc.

equalization summary

Equalization is a rationale that will appeal to many in society. But under case law, equalization is not a sufficient reason to restrict campaign expenditures. What is a sufficient reason? Corruption or the appearance of corruption. Equalization is not.

Prior Restraints

Even when speech might be constitutionally punished after the fact, prior restraints are especially disfavored. Means almost impossible to get an injunction agasint publication. Really has to be extraordinarily strong case to get injunction against publication because prior restraints are disfavored. - Are especially disfavored and will be struck down even when other forms of regulation might be upheld. - Injunctions against speech are almost impossible to get.

examples

Example 34: Laws against flag desecration are unconstitutional because they suppress an expressive type of speech to show disapproval. Theyre aimed at a particular message - ppl were allowed to use flag as symbol to express patrotiotism or admiration for country. They were allowed to use flag as a symbol to express affirmative ideas, but they were not allowed to use the flag to express their hatred or contempt for the country. Thus a law that prohibits any desecration of American flag was aimed at suppressing a particular kind of message and was therefore struck down. Flag desecration laws apply to your flag, obviously you have no right to do something with someone elses flag, including the govt. but laws against flag desecration apply to all flags, including those u have bought and paid for and they were struck down. Contrast public nudity - ban on public nudity is constitituional. Does ban on public nudity incidentally burden a certain kind of expression? Maybe but the law against public nudity was aimed at public order effects of nudity in public places - not at trying to suppress a particular message. Example 35: Laws against public nudity are constitutional because public order must be kept, which is unrelated to speech. - The key: If the government is trying to suppress a particular message by regulating osme kind of expressive conduct, then the law will be struck down; if the government is trying to pursue an interest unrelated to the suppression of expression, then the law will be upheld.

Standing Examples and Commonly Tested Standing Issues

Example 3: Corporation A applies for a business license, but is turned down. Corporation B applies for the same license and gets it. Corporation A sues to enjoin the government from licensing Corporation B. Does Corporation A have standing? No. Denying the license to Corporation B would not redress the wrong to Corporation A. The wrong to Corporation A was a wrongful denial of its license. Corporation A should sue for its own license. Must have standing to seek particular relief sought - if u don't seek that particular relief youll be kicked out of court even if you have standing for some other relief. o Federal taxpayers always have standing to challenge their own tax liability. However, taxpayers do not have standing to challenge government expenditures. Govt expenditures don't directly affect ur liability, no injury. § Narrow exception under the Establishment Clause: An establishment of religion challenge to specific congressional appropriations can be raised by any taxpayer. (Note also that state courts often allow municipal taxpayers to challenge a municipality's expenditures.) extremely narrow - must be establishment clause challenge - no other const. right will do and must be specific congressional appropriation under spending clause - no other kind of govt benefit will do.

media examples

Example 43: Confidential sources for the media. Does a reporter have a constitutional right to refuse to answer grand jury questions about a confidential source? Think of it in terms of yourself: You will not answer questions because you promised that you would not reveal a confidential source. Can the government force you to talk? Yes, you can be put in jail (held in contempt). The same is true of the media and press. Does report have const right to refuse to give testimony to grand jury? Ask would I have const right to refuse to testify before grand jury? Can u go before grand jury and say sorry I promised not to tell? Of course not. U have to answer grand jury questions unless pleading fifth amendment and if u don't they can hold u in contempt. Its true of normal people and newspaper reports too. Example 44: Open trials—you have the right to attend public traisl and so does The press - it has the right of access to open trials. They have same rights you do.

examples

Example 4: Doctors who were not compensated for providing abortion services can raise the rights of women who wanted to have abortions provided. The doctor has injury because he was not paid but he wants to raise the rights of his patient to have an abortion, not his own. The doctor and the patient are involved in an exchange or transaction. Example 5: A seller of beer in a saloon can raise the rights of 18-year-old boys to buy beer at the same age as 18-year-old girls. The saloon keeper is injured because she is not making money. She wants to raise the rights of the underage boys because that is gender discrimination. She can raise the claims of those customers or would-be customers because they are parties to an exchange or transaction. so general rule is third party standing is not allowed and cannot raise rights of someone else, but can raise rights of someone else if u and someone are parties to exchange or transaction.

commerce clause cases

Example 9: Wickard v. Filburn (wheat growing case where growing crops for sale was judged in the aggregate and a substantial effect was demonstrated) - congress said it was illegal to grow wheat beyond certain amount - farmers said no im not doing interstate commerce, I don't sell the wheat. We just grow wheat, and harvest and anything done is done right here - we don't send it anywhere. How is it interstate commerce? Sc said no matter, substantial effect is determined in the aggregate - all farmers in America growing wheat will affect price of wheat nationwide and that's good enough for us. Congress can regulate production of agricultural products. Example 10: Gonzales v. Raich (where the cultivation and medical use of marijuana was judged in the aggregate and a substantial effect was demonstrated). Question about congress having power to regulate prodcutin of marijuana nationwide and congress said yes effect of sale of marijuana would have economic effect so congress can regulate it. Example 11: United States v. Lopez (a statute prohibited possession of a firearm within 1000 feet of a school—it could not be proved that it was an economic or commercial activity, and it could not be demonstrated that there was a substantial effect on interstate commerce). This is non economic activity - sc said no one showed that possessing firearm within 1000 feet of school has substantial effect on interstate commerce. Example 12: United States v. Morrison (a statute provided a federal remedy for gender-motivated violence—it could not be shown that, in the aggregate, the activity had a substantial effect on interstate commerce) - sc said gender violence is not commercial or economic activity - its not buying or selling stuff so gender violence may have substantial effect insome areas but not sufficiently proved. Example 13: 2012 Affordable Health Care case (the individual mandate required individuals to buy health insurance or to pay a penalty for not doing so—the Court said that forcing individuals who are not engaged in commercial or economic activities to buy health insurance they do not want cannot be sustained as a regulation of interstate commerce.) - key question concerned so called individual mandate - what mandate does is forces individuals to buy health insurance or pay penalty if failing to do so. Sc said that forcing individuals not engaged in commercial activities to buy health insurance they didn't want couldn't be sustained as regulation of interstate commerce. Most imp thing is to remember way to think of it - for economic and commercial activity, substantial effect on interstate commerce is easy, presumed, and always found. For noneconomic and noncommercial activity, must actually demonstrate substantial effect on interstate commerce. sometimes can be done and sometimes not.

Lot of examples for religion

Examples: 1. Officially-sponsored school prayer is unconstitutional. - Officially-sponsored graduation prayer is unconstitutional. However its phrased, there will be someone who doesn't want to say it - and that person may feel in the context of graduation ceremony, coerced. But truly voluntary student prayer - one not sponsored by govt - but simply volunteered by individual - that's okay. 2. Bible reading is permissible, but cannot be inspirational (e.g., as literature or poetry). In public schools can read bible as literature and poetry - as maybe extended genology. As maybe fouhdnataionl document of western culture. Can read bible for all those purposes but cannot read it for inspiration. The minute kids begin to take bible seriously as religious text, u have to cut out rreading in public schools. Its likehearsay rule - bible can be admitted to classroom for any purpose other than its truth. 3. Display of the Ten Commandments is sometimes okay. It can be displayed for secular purposes (e.g., historical or promoting morals), but not to inspire religious belief. - Lot of cases about whether displays of ten commandments are endorsements of religion. Ppl argue theyre not because they have historical or secular purpose to promote morals rather than advance religion. Some uses of ten commandments have been upheld and others have not. You could teach ten commandments though as early legal code. You can say look how short they were then - no ban on having ten commandments as topic of discussion. But Cannot post the Ten Commandments in a classroom and leave it there every day of the school year— that's designed to inspire religious belief in ten commandments and would be struck down. 4. Can teach the Ten Commandments in school as an example of an early legal code. 5. Cannot post the Ten Commandments in a classroom and leave it there every day of the school year—designed to inspire religious belief. - Cannot post the Ten Commandments in a courthouse if the context makes plain that the purpose is to endorse religious belief. Main idea is that - if ten commandments are posted to endorse their religious content, that's an establishment of religion - posted by the government to endorse their religious content -thats an establishment of religion. If theyre posted for some other secular purpose, that's okay. 6. Laws prohibiting teaching evolution have been struck down. Probably because they fail test of mere rationality. 7. Legislative prayer is okay for historical practices. Supreme court said its okay for state legislature to have a chaplain say a prayer at the beginning of each session - very commonly done in state legislatures and in US congress. Its been done forever and court said historical practice seemed to matter. 8. Nativity scenes are okay on public property if there is something else there to dilute the religious message (e.g., Hanukkah symbols, Rudolph the red-nosed reindeer). Suprem court has wrestled with this - first time sc had cresh case, they said nativity scene okay because Christmas used to be religiously significant but no logner is - itd becomg shopping season. Even court grew ashamed of saying that and backed up to try again. - Second time Court said u can have cresh on public property at Christmas with all the religious symbolism it includes if u have something else there with it - some other thigns there that dilute religious message. so if just nativity scene with jesus and mary and shephards, that would be struck down as endorsement of chritsianity. But if nativity scene and additions like Rudolph the rednose raindeer, the augmented nativity scene augmented by non Christian elements would be allowed.

Advisory Opinions

Federal courts cannot issue advisory opinions. - Federal courts cannot rule on the constitutionality of proposed legislation. If seeing any scheme where court asked to predict constitutionality of law not yet applied, if court asked to say prospectively whether some law would be constitutional if enacted, that's unconstitutional advisory opinion.

takings clause

Fifth Amendment Takings Clause, which is applicable to state and local governments through the Due Process Clause of the Fourteenth Amendment, requires the payment of just compensation. Private property shall not be taken for public use without just compensation. simply property govt doesn't own yet. just compensation - fair market value at the time of the taking. Physical occupation—This is the key question. If the government physically occupies a private owner's property, then a taking has occurred and it owes just compensation. Generally, no physical occupation means that no taking has occurred. Even if they restrict use of ur property in way making it less valuable. Zoning - govt doesnt physically occupy property or move in but they pass zoning laws restricting use—generally no compensation required for zoning decisions. the Takings Clause applies both when the property taken by the government is given to a private person and when the government retains the property. Example - The placement of the seismographic equipment in the building would result in the permanent physical occupation of the building by the city. Consequently, the placement would constitute a taking. Since the owner would not be compensated for the taking, it is unconstitutional. The fact that the economic impact on the building owner would be slight would affect the amount of compensation, but not the requirement that compensation be paid.

Fighting words

Fighting words - not protected speech o Words likely to provoke an immediate breach of the peace by prompting person so addressed to hit back. In other words, general vulgarity will not do. § General vulgarity is not enough. That's not fighting words, they must be aimed at someone. § Must be aimed/targeted at someone, and that person might hit back. § In theory, fighting words are not protected speech. § In fact, all fighting words statutes on the bar exam are almost always unconstitutionally vague and/or overbroad (e.g., laws against "hate speech"). All of these statutes are unconstitutionally vague and overbroad and its been years since supreme court upheld a fighting words law. If you see a statute aimed at fighting words, what would it look like? Statute might say words tending to a breach of the peace are criminal or offensive language is criminal or just hate speech is criminal - right answer if u see a statute of that sort is not that the speech is constitutionally protected, that's not necessarily true but right answer is that this statute cannot be enforced because its vague and overbroad - they usually all are.

Ad Valorem Property Taxes (value-based property taxes)

For ad valorem taxes on personal property key is to distinguish between commodities in interstate commerce and the instrumentalities of interstate commerce - they are taxed differently. Commodities are goods moving from state to state - instrumentalities are transportation moving goods like railroads, trucks, airplanes, etc. o Levied on personal property o Distinguish between two kinds of personal property because they are taxed differently: § Commodities are the goods that move from state to state. • States usually tax all commodities within their borders on a specified date (called tax day), but not goods that are merely in transit—the commodities have to come to rest in the state. Cannot tax goods merely in transit - merely on their way through the state to another destination. • Rule: for commodities in interstate commerce, pay the full tax to every state where goods are stopped for a business purpose on tax day. No taxes are due where they are merely passing through. § Instrumentalities are the transportation equipment that moves commodities (railroads, trucks, airplanes, etc.). for instrumentalities, each state can tax its fair share of an instrumentality used in that state.taxpayers domicile can tax entire value of instrumentality but domicile then has to give credit for taxes paid elsewhere. So end up with scheme of fair apportionment of tax liability among those states with a substantial nexus to the instrumentality of interstate commerce. • Fair apportionment of tax liability among states with a substantial nexus to the instrumentality. • Each state in which an instrumentality is used can tax the value of that instrumentality. Example 23: A long-haul trucker drives his rig from Maine to California. Every state through which the truck passes can tax the truck itself but each state can only tax a portion of the truck's value, roughly equal to the portion of in-state use. The state of the taxpayer's domicile taxes the full value of the truck, then turns around and gives a credit to every state that asserts taxation when the truck uses its highways. You end up with a scheme of fair apportionment of tax liability among those states with a substantial nexus to the truck. The commodities in the truck cannot be taxed if they are merely passing through; they must stop for a business purpose on tax day.

Speech by Government Employees

Generally public employees have free speech rights. That means below govt employees cannot be hired or fired based on... • General rule: Government employees generally cannot be hired or fired based on political party, political philosophy, or any act of expression such as letter to the editor. This is true for everyone including run of the mill govt workers like police offices, public school teachers, transit workers, etc. - they can be fired for not doing their jobs - fact that u disagree with govt policy doesn't give u the right not to follow it. That is u can be fired if u disrupt the work place. You can be fired if you dont carry out your employment instructions. But cant be fired simply because you belong to the wrong party or endorse the wrong views. None of this applies to confidential or policy making employees - these are upper level employees - policy makers are like cabinet officials, agency heads - confidential employees are their immediate advisors. And confidential and policy making employees can be hired and fired based on loyalty and politics. o Can be fired for disrupting the workplace or not doing their jobs. • Exception: This general rule does not apply to confidential advisors or policy-making employees (e.g., the President's cabinet officers). President trump can choose, as all of his successors did, cabinet officers who are loyal to his points of view and he can fire people who disagree with him. That's true at level of cabinet officer, at level of agency head, at level of special asst to president. But run of the mill govt employees like school teachers and clerical workers and uni professors - people who pave streets, transportation workers - ordinary run of the mill workers cannot be hired or fired based on political party, political philosophy, or any act of expression.

A state statute provides for a reduction in the property tax assessed against any residential dwelling located in a particular economically depressed area of the largest city in the state. The purpose of the statute is to encourage homeownership in the designated area and thereby improve the economic wellbeing of the residents living there. The statute provides that, in order to qualify for this tax break, the homeowner must be a citizen of the United States.Is the statute likely constitutional? A Yes, because the state can offer tax breaks to specific citizens under the Tenth Amendment. B Yes, because the state has a rational basis for providing for the tax break to specific citizens. C No, because the tax break offered to specific citizens by the state violates the Equal Protection Clause. D No, because the state cannot legislate with regard to alienage.

Generally, a state law that discriminates against aliens is subject to strict scrutiny under the Equal Protection Clause and will be struck down. While a state law that restricts or prohibits an alien's participation in government functions is subject only to a rational basis review, this statute does not fall within that category. For this reason, answer choice B is incorrect. Answer choice A is incorrect because, while the Tenth Amendment reserves to the states powers not delegated to the federal government nor prohibited by the constitution, the limitation of a tax break to citizens of the United States violates the Equal Protection Clause. Answer choice B is incorrect. Although the state had a rational basis for enacting the statute, the rational basis standard only applies when a state law prohibits an alien's participation in government functions. As discussed above, this statute is subject to strict scrutiny because it discriminates against aliens regarding the tax break in violation of the Equal Protection Clause. Answer choice D is incorrect because a state may legislate with regard to alienage if Congress has explicitly authorized it.

procedural due process - real property/personal property

Generally, the government is required to provide the owner of real property with notice and a hearing prior to seizure of the property pursuant to a forfeiture statute. However, the government does not need to provide notice prior to the seizure of personal property. Here, the city provided no notice or hearing for either the home or the vehicle. The city was required only to provide notice and a hearing for the seizure of the home but did not do so. Accordingly, the seizure of the home is improper though the seizure of the vehicle is not.

State Taxation of Interstate Commerce

Generally, the requirements are the same as for any other state regulation of commerce: Discriminatory taxation will be struck down unless Congress consents to it, and non-discriminatory taxation (simply means in state and out of state taxpayers are treated alike) will be upheld unless it is unduly burdensome which is rare. Non-discriminatory taxation is valid if the following two requirements are met: 1. Must be Substantial nexus between the taxing state and the property or activity to be taxed - Substantial nexus no longer requires physical presence. At least where the out of state seller does some nontrivial amount of business in state - it can be required to collect a sales tax on in state purchases even if all the physical facilities are located elsewhere. • "Substantial nexus" does not require any kind of physical presence of seller in state. In general interstate sales by internet marketers can be subjected to sales tax whether or not theres physical activity in the state. 2. Must be a fair apportionment of tax liability among states. Imagine international corporation doing business all over US and all over world. - A state may tax the in-state portion of a corporation's revenue because theres a substantial nexus between taxing state and corporation doing business there but it cannot tax the entire world-wide activity of the corporation - that would be piggy and would interfere with taxing interest of other jx. Instead the state must adopt some form of fair apportionment of tax liability among states - fair apportionment of tax liability among states - can tax portion of corps revenue that is equivalent to the in-state activity of revenue.

Defamation

Important category of speech that can be prohibited because of content o False statements of fact (not opinion) damaging to a person's reputation can be prohibited. Critical that defamation concern a false statement of fact. An opinion, however unpleasant, cannot be defamatory. If u said someone is worst teacher ever, that cannot be defamation because its ur opinion. If u say I saw someone stealing money from till somewhere, that's a factual statement which if not true can lead to liability for defamation. o Public officials (Self defining category - those who hold govt office) and public figures (less clear category - people who have standing and reputation and prominence in the community) can recover for defamation only on proof of knowing or reckless falsity. Reckless means newspaper or publisher consciously recognized risk that material was false and published it anyways. o Private plaintiffs (people who don't hold office and who aren't terribly prominent) can recover on proof of negligent falsity. Defamation involves false statements of fact - many contexts in which false statements of facts are punished - perjury is one example - lie under oath. Any false of statement of fact cannot be punished though. Stolen valor act made it crime ... look below. Example 42: United States v. Alvarez: The Supreme Court struck down the Stolen Valor Act. The Stolen Valor Act made it a crime to falsely claim receipt of military declaration or medals. The Act was struck down by the Supreme Court as applied to a local politician who lied about receiving the Congressional Medal of Honor. The Justices held that the fact that the statement was a blatant self-serving lie did not exempt it from First Amendment protection. Stolen valor act was content based - not one of 5 exceptions - so it was unconstitutional. The prohibition was content-based and was not supported by one of the five recognized categories of unprotected speech. The statute was unconstitutional. Not the whole category of false statements of fact that are outside the first amendment - but it's the narrower category of defamation that's outside the first amendment.

regulatory taking factors

In determining whether a regulation constitutes a taking, the court will consider: (1) the economic impact of the regulation on the property owner; (2) the extent to which the regulation interferes with the owner's reasonable, investment-backed expectations regarding his use of the property; and (3) the character of the regulation, including the degree to which it will benefit society, how the regulation distributes the burdens and benefits among property owners, and whether the regulation violates any of the owner's essential attributes of property ownership, such as the right to exclude others from the property.

traditional public forum rule

In traditionally public forums, like sidewalks, the government may impose reasonable restrictions on the time, place, or manner of protected speech, provided the restrictions are content-neutral as to both subject matter and viewpoint, are narrowly tailored to serve a significant governmental interest, and leave open ample alternative channels for communication of the information. Although the city may have an important interest in preventing the breaking and entering into the animal testing facilities, the ordinance preventing all picketing on the subject of animal testing is not narrowly tailored to further this interest. Rather, the regulation suppresses all picketing on this subject. Additionally, this regulation is not content-neutral, since it would appear to permit picketing about other issues on the same sidewalk. furthering a governmental interest does not justify a content-based restriction of speech in a public forum. The ordinance forbids all picketing on this subject matter, making no attempt to limit its restriction to picketing that could reasonably incite criminal activity or violence. Therefore, regardless of the intended purpose, the ordinance impermissibly limits protected speech and is unconstitutional.

Incitement Speech

Incitement Speech is not protected if it is an incitement to immediate violence. Immediate violence - great non example of incitement - "one of these days there'll be a revolution - first well kill all the lawyers" - that's not incitement because no connection to violence here and now. Incitement must be immediate.

incitement speech

Incitement Speech is not protected if it is an incitement to immediate violence. Immediate violence - great non example of incitement - "one of these days there'll be a revolution - first well kill all the lawyers" - that's not incitement because no connection to violence here and now. Incitement must be immediate.

limited public forum

Limited public forum - idea is its some place that's not public forum but govt has chosen voluntarily to open to all comers. - Describes a place that is not a traditional public forum, but that the government chooses to open to all comers (e.g., a municipal theater that anyone can rent). Municipal theater - govt doesn't have to build it or have to have one but if it does have one and it rents it out to all kinds of plays, then it can only enforce content neutral regulations on what happens there. Cant disapprove a particular play because of its point of view. - limited public forum is a very limited idea. School auditorium for example is not a limited public forum. Now schools have auditoriums - they open them to various speech activities but only on appropriate topics. So a student who makes a profane or sexually explicit speech in a school assembly absolutely may be disciplined because the speech was inppropriate in the school auditotorium. Student could have gone outside to the public sidewalk, down the street to the public park, or student could have gone to public forum and said exactly the same profane and sexually explicit things and been home free. Difference is a public forum vs other property. - In such areas, only time, place, or manner regulations are allowed, but this is a narrow category. Example 40: A school auditorium is not a public forum, limited or otherwise. Just because a school auditorium is open to some speech does not mean it has to be open to all.

zoning laws review standard

Most legislation related to zoning is reviewed under the rational basis standard, and a law meets that standard if it is rationally related to a legitimate state interest. Laws are presumed valid under this standard, so the burden is on the challenger to overcome this presumption by establishing that the law is arbitrary or irrational. In court, the government's stated interest in enacting the law need not be one that it offered when the law was passed, and here, the city's stated interest constitutes a legitimate interest. The ordinances, while not necessarily the most direct way to accomplish that interest, are rationally related to it. REMEMBER - the government interest need not be stated when the law was passed, as long as the stated interest is legitimate.

Content neutral:

Must be content neutral on its face and neutral as applied. If a facially neutral law is applied to favor one speaker or one point of view or one political party its not content neutral. Content neutral also means law must not allow executive discretion - if an executive officer has discretion, if exec officer has power to pick and choose who can speak risk is that the discretion will be administered in a non-neutral way. The supreme court has tried to guard against that possibility by saying that a law must be content neutral on its face, it must be content neutral as it has been applied, and it must not allow executive discretion which would threaten content neutrality. Example 37: A local D.C. ordinance prohibited picketing within 500 feet of a foreign embassy (but it didn't just stop there - it didn't say all picketing within 500 feet of a forein embassy, it said only if...) if the picketing sign brought the foreign government into odium or disrepute (that's not time place or manner regulation, that's regulation based on content - whether signs were hostile to foreign govt.). This is plainly content-based and is thus, unconstitutional. Example 38: Parade permit laws (getting permit before parading on street or using public property) giving permits to first come, first served (like first group to apply for a permit gets one) are constitutional because content neutral. Parade permit laws giving the chief of police discretion to decide whether it was in public interest to grant someones permit are unconstitutional because the law may not be applied in a content-neutral way - if chief of police has discretion to pick and choose among permit applicants, the risk is that he will grant the applicaiotns for those he agrees with. And deny permits to those he finds reprehensible. That risk of content based administration makes the law unconstitutional.

Necessary and Proper Clause is not....

Necessary and Proper Clause is not an independent source of power; it merely gives Congress the power to execute specifically granted powers. Thus, the Necessary and Proper Clause cannot be the only source for Congress's authority to enact a statute.

Regulation of the Media

No Special Privileges The press and media have no special privileges. They have the same rights as everyone else. No more, but of course no less. If question asked about freedom of press, some law that restricts media or newspaper or television and like don't need to know a lot about that subject - just ask urself could govt do this to me? If govt could restrict ur speech they can restrict the press in the same way. Freedom of press is same rights the rest of us have - no more but no less.

privileges and immunities clause of article iv

Nonresident citizens are protected against discrimination with respect to fundamental rights or essential activities, including the pursuit of employment, transfer of property, access to state courts, and engaging in the political process. Discrimination against out-of-state residents in setting the fee for a commercial activity, such as an oyster-fishing license, violates the Privileges and Immunities Clause of Article IV, but similar discrimination for a recreational activity, such as a recreational hunting license, does not, if there is a rational basis for the fee differential. Example - the state is discriminating against the out-of-state oyster fisherman by setting a much higher fee for a commercial activity. Note that this would also likely violate the Dormant Commerce Clause as well.

A manufacturer entered into a contract with the United States Forest Service, an agency of the federal government, to construct a special plane to be used by the Forest Service in fighting forest fires. During the construction of the plane, as the Forest Service made progress payments to the manufacturer, the supplier of materials obtained, under state law, a valid lien upon the plane for the cost of these materials. Shortly before construction was completed, the manufacturer ceased operation and the Forest Service, as permitted by the terms of the contract, took title to the plane. The supplier's otherwise valid lien is now unenforceable against the Forest Service because of the doctrine of sovereign immunity.Has there been an unconstitutional taking of the supplier's property? Yes, because the Forest Service's action effectively eliminated the supplier's lien on the plane.

One way in which a taking may occur is through the destruction of property or the elimination of a property right. Here, while the supplier continued to possess a valid lien on the plane, the lien was worthless because the supplier could not enforce it against the U.S. Forest Service due to sovereign immunity. Because this action effectively destroyed the value of the supplier's property lien, it constitutes a taking. The Takings Clause is not limited to possessory interests in property, but extends to property rights, such as an easement or a lien.

Public employees who can be fired only "for cause" must be given....

Public employees who can be fired only "for cause" must be given some opportunity to be heard prior to discharge, unless there is a signficiant reason not to keep the employee on the job. Public employees who can be fired only for cause have tenure - doesn't apply to just university profesors, but to every public employee who has job security. People who have job security typically msut be given some oppornity to heard - so not necessarily full scale trial hearing but some opportunity to be heard prior to discharge. However if signficiant reason not to keep employee on job employee can be immediately suspended without pay. Signficiant reason not to keep employee on job - employee can be immediately suspended without pay, so long as theres a prompt post suspension hearing and so long as there is reinstatement with back pay to make employee whole. If there is a significant reason not to keep a person on the job, then the discharge can come first with a subsequent hearing that is prompt and provides reinstatement with back pay (fair).

The restriction of voting to a class of persons - constitutional?

Pursuant to the Equal Protection Clause, the principle of "one person, one vote" requires that one person's vote must be essentially equal to any other person's vote. The "one person, one vote" principle applies to local elections of entities that perform governmental functions, even when the functions are specialized rather than general in nature, such as a local school board. The restriction of voting to a class of persons (e.g., landowners) has generally been found to violate the "one person, one vote" principle, except in the case of water-district elections.

PRIVILEGES & IMMUNITIES, TAKINGS, AND PROHIBITED LEGISLATION - Privileges & Immunities Clauses Distinguished

Recall that the Privileges & Immunities Clause of state citizenship under Article IV prohibits serious discrimination against out-of-state individuals, chiefly regarding employment. Almost all cases involve state discrimination against out of state indiivduals wrt private job market - narrow but important. Exam Tip 11: The Privileges or Immunities of national citizenship under the 14th Amendment has no modern content and is never a strong answer that something is unconstitutional on the bar exam. Never a strong argument for anything.

Privileges & Immunities Clauses Distinguished

Recall that the Privileges & Immunities Clause of state citizenship under Article IV prohibits serious discrimination against out-of-state individuals, chiefly regarding employment. Almost all cases involve state discrimination against out of state indiivduals wrt private job market - narrow but important. Exam Tip 11: The Privileges or Immunities of national citizenship under the 14th Amendment has no modern content and is never a strong answer that something is unconstitutional on the bar exam. Never a strong argument for anything.

Third-party standing

Refers to the question of whether you can raise the rights of someone else. - Generally, the answer is no. generally even if u have standing and show injury, causation, and redressability u can generally only raise ur own rights. Generally cant claim statute violates rights of someone else. - Exception: Parties to an exchange or transaction can raise the rights of other parties to that exchange or transaction. if 2 people involved in some exchange or transaction one of them can raise the rights of the other.

commodities in interstate commerce taxing

Rule: for commodities in interstate commerce, pay the full tax to every state where goods are stopped for a business purpose on tax day. No taxes are due where they are merely passing through. - Instrumentalities are the transportation equipment that moves commodities (railroads, trucks, airplanes, etc.). for instrumentalities, each state can tax its fair share of an instrumentality used in that state.taxpayers domicile can tax entire value of instrumentality but domicile then has to give credit for taxes paid elsewhere. So end up with scheme of fair apportionment of tax liability among those states with a substantial nexus to the instrumentality of interstate commerce. - Fair apportionment of tax liability among states with a substantial nexus to the instrumentality. - Each state in which an instrumentality is used can tax the value of that instrumentality.

Fundamental Rights

Some fundamental rights almost always come up under equal protection. Fundamental rights trigger same strict scrutiny test under both due process and equal protection. 1. right to travel 2. right to vote - one person, one vote (Requires districts of approximately equal size, i.e., approximately the same number of voters in each) 3. Gerrymandering - Comes in two varieties—racial and political - gerrymandering - districts of equal size with same number of people in each district but arrange them to achieve advantage for one side rather than the other a. racial gerrymandering - Rule: Race may be a factor in drawing district lines so as to advantage racial minorities as voting rights act wishes to do - race may be a factor in drawing district lines, but not the predominant or only factor. b. Political gerrymandering (drawing districts to hurt one party) - occurs if one party controls the legislature and can screw the other party. political gerrymandering Can, in theory, violate equal protection. In practice, it is never struck down. If asked whether constitutional problem with political gerrymandering, or asked whats strongest argument that political gerrymandering is unconstitutional, equal protection is answer. In theory political gerrymandering can violate equal protection but in practice it never does because the Supreme Court has not found any judicially manageable standards for implementing that guarantee.

Fundamental Rights

Some fundamental rights almost always come up under equal protection. Fundamental rights trigger same strict scrutiny test under both due process and equal protection. General rule of thumb - law denying fundamental right to everyone violates due process. law denying fundamental right to some people and not to others violates equal protection but some fundamental rights arise in the context of equal protection, including: 1. Right to travel 2. Right to vote 3. Gerrymandering

EQUAL PROTECTION (RACE, ETHNICITY, AND NATIONAL ORIGIN) - Standards of Review

Standards of Review—Same for Due Process and Equal Protection • Strict scrutiny, i.e., is the law necessary for a compelling interest? • Intermediate scrutiny, i.e., is the law substantially related to an important government interest? • Rational basis, i.e., is the law rationally related to a legitimate government interest? o Rational basis test is generally easily passed. Almost everything survives review for mere rationality. However, recent Supreme Court cases have made exceptions in the field of sexual orientation. § In the 2003 Lawrence case, the Supreme Court struck down laws criminalizing homosexual sodomy using language suggesting the laws did not pass the rational basis test as they reflected prejudice, not a legitimate reason for the laws. § In the 2013 Windsor case, the Court used the same approach in striking down the federal Defense of Marriage Act (DOMA). The Court applied a version of the rational basis test "with teeth" suggesting that because the motivation behind the federal DOMA was merely animus and prejudice, the law was not a rational plan of government. o Laws against sexual orientation might be struck down as irrational because they are not supported by reasons other than mere prejudice. Rational basis test in general is hard to flunk - in general almost anything goes under the test but there is an exception in area of sexual orientation where court has now repeatedly applied rational basis with teeth - rational basis with rigor - rational basis which begins to look something like strict scrutiny.

State Taxation of Interstate Commerce

State Taxation of Interstate Commerce • Generally, the requirements are the same as for any other state regulation of commerce: Discriminatory taxation will be struck down unless Congress consents to it, and non-discriminatory taxation (simply means in state and out of state taxpayers are treated alike) will be upheld unless it is unduly burdensome which is rare. o Sometimes not obvious what constitutes discrimination in field of taxation - so two particular requirements for non-discirminatory taxation of out of state interests. Non-discriminatory taxation is valid if the following two requirements are met: 1) Must be Substantial nexus between the taxing state and the property or activity to be taxed Substantial nexus previously was interpreted to require the physical presence of the seller in state. Until recently a state could levy a sales tax, one that the seller is forced to collect, only if the seller had some physical presence in the state beyond merely delivering goods there. That meant interstate marketers, like amazon, unless happening to have physical facilities in the state, could not be forced to collect sales tax on goods sold over the internet. Supreme court changed its mind and said old rule was invalid because court did not then have before it the present realities of the interstate marketplace. Because court had not foreseen in 1992 just how large internet based commerce would become. Under new rule substantial nexus still required for imposing sales tax collected by seller but substantial nexus no longer requires physical presence. At least where the out of state seller does some nontrivial amount of business in state - it can be required to collect a sales tax on in state purchases even if all the physical facilities are located elsewhere. • "Substantial nexus" does not require any kind of physical presence of seller in state. In general interstate sales by internet marketers can be subjected to sales tax whether or not theres physical activity in the state. 2) other requirement - Must be a fair apportionment of tax liability among states. Imagine international corporation doing business all over US and all over world. Example 22: A state may tax the in-state portion of a corporation's revenue because theres a substantial nexus between taxing state and corporation doing business there but it cannot tax the entire world-wide activity of the corporation - that would be piggy and would interfere with taxing interest of other jx. Instead the state must adopt some form of fair apportionment of tax liability among states - fair apportionment of tax liability among states - can tax portion of corps revenue that is equivalent to the in-state activity of revenue. Now technically in real world state taxation of out of state businesses can become very complicated because of exactly how u administer idea of fair apportionment. But on bar exam - simple - just substantial nexus which now includes delivering goods in state and some scheme of fair apportionment among taxing states.

Taking versus regulation

Taking versus regulation—If there is a taking of property, compensation is required; if there is a mere regulation on property, compensation is not required, even if the regulation reduces the value of the property.

13th Amendment

The 13th Amendment (outlawing slavery and involuntary servitude) applies directly to private parties and individuals, but it is narrow in its focus. Protections only against slavery and involuntary servitude.

13th amendment

The 13th Amendment (outlawing slavery and involuntary servitude) applies directly to private parties and individuals, but it is narrow in its focus. Protections only against slavery and involuntary servitude.

state law regulating federal elections

The Elections Clause explicitly empowers Congress to override state laws concerning federal elections. Here, the legislation attempts to regulate the federal election process by limiting voting times and polling sites as applied to federal elections, and Congress has the power to override such legislation. Answer choice A is incorrect because even if the voters are able to cast their votes, the limitations in the election process apply to federal elections. Congress has authority to override such a regulation. Answer choice B is incorrect because, even though the changes are unrelated to the suppression of ideas, Congress can nevertheless override state laws concerning federal elections. This power is not restricted to state election laws that suppress ideas. Answer choice C is incorrect because Congress does not have blanket authority to override a state election law that applies to elections for state offices.

equal protection modes of analysis

The Equal Protection Clause of the Fourteenth Amendment provides that states are generally prohibited from passing legislation that treats similarly situated persons differently. To trigger strict or intermediate scrutiny, there must be discriminatory intent on the part of the government or a classification based on a suspect or quasi-suspect class. Strict scrutiny also applies if a fundamental right is involved. The rational basis standard is used in all cases in which one of the higher standards does not apply. A law passes the rational basis standard of review if it is rationally related to a legitimate governmental interest. This is a test of minimal scrutiny. Example - Students who bring their lunches from home are being treated differently from students who eat in the cafeteria because they must eat in a separate room. However, the ordinance has a rational basis that is related to the legitimate government interest of preventing complications due to food allergies. Because this ordinance does not target a suspect class or involve a fundamental right and has no discriminatory intent, it passes rational basis review. Therefore, the parents' challenge is likely to fail.

First Amendment media

The First Amendment shields the media from liability for publication of a lawfully obtained private fact, e.g., the identity of a rape victim, so long as the news story involves a matter of public concern. Therefore, neither the newspaper nor the reporter are liable for legally gathering and publishing these private facts concerning a matter of public concern. Note: The First Amendment also shields the media from liability for publishing information that was obtained illegally by a third party as long as the information involves a matter of public concern, and the publisher neither obtained it unlawfully nor knows who did.)

media liability for publishing information obtained illegally by a third party

The First Amendment shields the media from liability for publishing information that was obtained illegally by a third party as long as the information involves a matter of public concern and the publisher neither obtained it unlawfully nor knows who did. Here, the newspaper likely knew the information was not lawfully obtained and certainly knows who obtained the information. Therefore, the corporation can recover against both the reporter and the newspaper.

freedom of association vs. free exercise clauses note answer choice D

The Free Exercise Clause protects a religious organization's right to shape its own faith and mission through its choice of ministers. The Establishment Clause prevents the government from imposing its choice of a minister on the organization. Together, these clauses create a ministerial exception that operates as an affirmative defense to an employment discrimination claim brought by a minister. Answer choice A is incorrect because the First Amendment Free Exercise Clause ensures that a religious organization has the authority to control who serves as a minister; it does not only protect the organization's decision when it is made for a religious reason. Answer choice B is incorrect. Although a neutral law of general applicability is generally subject to the rational basis test under the Free Exercise Clause of the First Amendment, this clause also requires that a religious organization be free to select its ministers without governmental interference. Answer choice D is incorrect. Although the Freedom of Association Clause of the First Amendment may protect a religious organization's freedom to choose with whom it associates, this freedom is not absolute. It is subject to a compelling state interest. Moreover, this freedom is linked to the organization's purpose. Although a religious organization might be able to refuse to retain a minister who does not adhere to the tenets of its faith, a rejection of a minister on the basis of personal animus could not be justified under this clause. Therefore, the religious organization must rely instead on the ministerial exception created by the Free Exercise and Establishment Clauses.

privileges and immunities clauses distinguished

The Privileges & Immunities Clause of state citizenship under Article IV prohibits serious discrimination against out-of-state individuals, chiefly regarding employment. Almost all cases involve state discrimination against out of state indiivduals wrt private job market - narrow but important. However, the Privileges or Immunities of national citizenship under the 14th Amendment has no modern content and is never a strong answer that something is unconstitutional on the bar exam. Never a strong argument for anything.

Property Clause of Article IV, Section 3

The Property Clause of Article IV, Section 3 states, "The Congress shall have Power to dispose of and make all needful Rules and Regulations respecting the Territory or other Property belonging to the United States." This power includes not only the power to dispose of property owned by the United States and to make incidental rules regarding its use, but also the power to protect the property. Although a state generally has the same right as any property owner to construct a fence on its property, this right must give way to federal regulation. Here, Congress has the power to protect federal land from a nuisance erected on adjoining property, and this power likely extends to property that is owned by the state. Answer choice A is incorrect. Although the Necessary and Proper Clause of Article I, Section 8 permits Congress to fully exercise its authority enumerated elsewhere in the Constitution, this clause does not serve as an independent source of congressional power. Answer choice C is incorrect because the Tenth Amendment only reserves for the states those powers not already given to the federal government. The power exercised here by Congress was granted to the federal government by the Property Clause of Article IV, Section 3 of the U.S. Constitution.

taxing power

The Taxing and Spending Power - Think of these as separate powers of Congress even though they are lumped together in one clause. taxing clause: - The Taxing Clause is the right answer whenever Congress imposes a tax, even when the tax is actually used to prohibit the good or activity in question. Taxing is right even if tax plainly designed to suppress or prohibit good or activity involved. - The tax need only be rationally related to raising revenue. Example 14: 2012 Affordable Health Care case—the individual mandate was not sustained as a regulation of interstate commerce, but it was held to be valid under the taxing power. Why valid under taxing power? The individual who chooses not to buy health insurance must pay a tax, and the taxing power supports anything that is rationally related to raising revenue.

13th amendment

The Thirteenth Amendment gives Congress the power to adopt legislation rationally related to eliminating involuntary servitude. This power has been broadly interpreted to allow Congress to regulate both private and government action and is the only Amendment that authorizes Congress to regulate purely private conduct. Therefore, the proposed statute would likely be constitutional as it addresses forcing anyone to work against his or her will.

Twenty-First Amendment

The Twenty-First Amendment, in addition to repealing prohibition, specifically gives states the authority to prohibit the transportation or importation of alcoholic beverages into the state for delivery or use within the state.

Economic impact

The adverse economic impact of the government's action does not necessarily mean there has been a taking (e.g., a new prison built next door to a beautiful, countryside home - value of ur property plummets but no taking so they don't owe you any compensation). Many regulations can dramatically affect the value of property but that does not trigger a right to compensation.

federal government/state taxation

The federal government and its instrumentalities (such as a national bank chartered by the federal government) are immune from taxation by the states. In this case, the Department of Defense, part of the federal government, is immune from taxation by the state for any purchases made pursuant to the defense contract. The Department of Defense did not consent to state taxation merely by entering into a contract for defense equipment from a state-owned manufacturer. It is immune from state taxation regardless of whether the other party to the contract was the state or a private entity.

Intergovernmental Immunities

The federal government is generally immune from direct state regulation or taxation. However, states can tax indirectly (e.g., taxing the income of federal employees). Fine as long as they don't discriminate against fed employees. States are not immune from direct federal regulation or taxation (e.g., pollution regulations, employment laws, etc.).

market-participant exception

The market-participant exception allows a state to behave in a discriminatory fashion against nonresident commerce if it is acting as a market participant. It does not permit a state to violate federal law regarding commerce.

the press/first amendment rights to publish

The press has no greater First Amendment rights than does the general public. There is no special privilege allowing the press to invade the rights of others. Members of the press are not immune from the application of generally applicable laws, even if the application of such laws has a negative incidental effect on the ability to gather and report the news.

congress taking away citizenship

The protection of national citizenship in Clause I of the Fourteenth Amendment prevents Congress from taking away a person's citizenship without her consent, unless that citizenship was obtained by fraud or in bad faith. Under these facts, plaintiff did not consent to the taking away of her citizenship. In addition, the facts do not indicate that the plaintiff obtained her citizenship by fraud or in bad faith. Answer choice B is incorrect because the Privileges and Immunities Clause of Article IV is irrelevant; it applies to discrimination by a state against out-of-state citizens. Answer choice C is incorrect because although Article I, Section 8, Clause 4 of the Constitution does grant Congress exclusive authority over naturalization and gives Congress the power to set the rules for determining how a person can become a citizen, the provision does not grant Congress the power to take away the citizenship of a person. Answer choice D is incorrect because although an alien does not have a right to enter the United States, the plaintiff is a citizen. The statutory attempt to strip the plaintiff of her citizenship is unconstitutional.

Campaign Finance

The use of money to support a political campaign is political speech and the regulation of campaign finance raises First Amendment issues. as regulations have become more elaborate constitutional restictions on those regulations have also become more complicated and to some extent unstable

Regulation of Unprotected and Less Protected Expression

There are some categories of speech that can be regulated because of content. Most content based regulations of speech are struck down. But few categroies of speech that can be regulated because of content. This is where govt is said to have a compelling interest in regulating speech. There are 5 such categories plus special case where govt itself is said to be doing the speaking.

Regulation of Unprotected and Less Protected Expression

There are some categories of speech that can be regulated because of content. Most content based regulations of speech are struck down. But few categroies of speech that can be regulated because of content. This is where govt is said to have a compelling interest in regulating speech. There are 5 such categories plus special case where govt itself is said to be doing the speaking. 1. Obscenity 2. Incitement Speech 3. Fighting words 4. Defamation 5. Commercial Speech

After a trademarked cartoon character was almost elected as mayor of a small town as a write-in candidate, the state in which the town was located enacted a statute banning all write-in candidates. Instead, candidates must obtain a set number of signatures on a petition to appear on a ballot. The petition requirement has allowed some independent candidates to appear on ballots in the state, but not all candidates who set out to achieve the required number of signatures have succeeded. A candidate has brought an action challenging the constitutionality of the new rules.Are the state's new rules regarding candidates constitutional?

There is no fundamental right to have one's name on an election ballot or to hold office through election or appointment. A state may ban all write-in candidates in both primary and general elections, as long as the state provides other reasonable means by which a candidate can get on the ballot. For example, a candidate for elected public office can be required to obtain the signatures of voters on a petition in order to appear on the ballot, as long as the requirement does not deny independent candidates ballot access. Therefore, on these facts, the ban is likely constitutional because the petition signature requirement is a reasonable alternative that allows a candidate to get the candidate's name on the ballot. The petition signature requirement is not unconstitutional unless it denies independent candidates ballot access or imposes undue burdens on placing new or small parties on the state ballots. This does not appear to be the case here. Therefore, the petition requirement is likely a reasonable alternative to permitting write-in candidates.

Thirteenth Amendment

Thirteenth Amendment • Congress has broad power to legislate against racial discrimination, whether public or private. • Includes purely private racial discrimination. This can be reached by congress under 13th amendment power.

Nonpublic forum

This includes all kinds of government property that is not a public forum and not traiditonally open to speech activities (e.g., government offices, jails, power plants, military bases, etc. - not typically places where you can go and parade and protest and speak). Here, the government has great power over what u do in nonpublic forums. Basically, any reasonable regulation of speech will be upheld. Some things that are not reasonable - it would not be reasonable to favor republicans over democrats, etc. viewpoint discrimination is invalid - but absent that kind of viewpoint discrimination which u rarely see, govt can regulate speech activities in a nonpublic forum almost any way it likes. You cant go to govt office building and march up and down inside corridors shouting slogans - that would disrupt govt functions. but where can u go? You can go to public sidewalk outside govt building as long as u obey neutral time place and manner regulations and protest to your hearts content. That's difference between a public forum like streets, sidewalks and public parks, and other govt property such as an office building. - Viewpoint discrimination is invalid: One clearly unreasonable kind of regulation would be to discriminate based on viewpoint (e.g., between members of different political parties). - Disruption of the functions of government: One should go outside to the public sidewalk surrounding the building since that is a public forum.

Alternative channels of communication must be left open

Time, place, or manner law must be a guideline for speech, not a flat prohibition of speech. Example 39: Laws against amplified sound trucks during the night-time hours are fine - its valid, content neutral, and leaves lot of other times and places where u can sound. Laws against amplified sound during all times and all places would be struck down. Whats the difference? One is a guideline for speech leaving alternative channels open, other is a prohibition of speech attempting to shut down a certain method of speaking.

timing of hearing required

Timing of the hearings - Sometimes, a hearing must occur before the deprivation or something taken away. So pre-deprivation hearing. But sometimes evidentiary hearings may be allowed after something taken away so long as they are prompt and fair. Pre deprivation hearings apply to terminating welfare benefits, and to non emergency revocations of drivers licenses. But for many other govt actions, hearings can come after action taken so long as hearing is prompt and fair. - Examples include: Terminating welfare benefits; non-emergency revocations of driver's licenses. - Sometimes, the hearing can occur after the action, so long as the hearing is prompt and fair. Usually okay to act first and give hearing later but hearing must be prompt and fair - cant be hearing down the road several months and cant be hearing not giving u fair shot at prevailing. - Examples of major actions for which post deprivation hearings are allowed so long as they are prompt and fair include: Terminating disability benefits; disciplinary suspension from a public secondary school. Question of pre or post deprivation hearing is resolved by same 3 factor balancing test -weigh individual interest at stake, and the value of a pre termination hearing - value of having hearing before termination - against govts interest in efficiency.

Suspension Clause of Article I, Section 9, Clause 2

Under the Suspension Clause of Article I, Section 9, Clause 2, a detainee retains the privilege to file a habeas corpus petition unless this privilege is suspended. This clause applies to individuals detained in a territory over which the United States has sovereign control, even though such territory is outside the United States. While Congress may limit the jurisdiction of the federal judiciary, Congress may not exercise this power in a manner that violates the Constitution. Accordingly, Congress may not limit the constitutional right to habeas corpus by removing jurisdiction from federal courts.

property power - Article IV, Section 3 of the U.S. Constitution

Under the property power found in Article IV, Section 3 of the U.S. Constitution, Congress has the power to act as a proprietor as well as a legislature with regard to federal lands. This includes the power to dispose of federal land as Congress sees fit, but does not authorize the acquisition of such lands.

Must narrowly serve a significant state interest:

Under this test, most content-neutral time, place, or manner regulations are upheld. - Does not require a compelling interest - Note absence of buzz word compelling. Content neutral regulation of time place and manner doesn't require a compelling state interest. Doesn't require a national emergency we cant live without it kind of justification. It requires a signficiant state interest which is basically anything that makes sense to a judge and for that reason most content neutral regulations of time place and manner are upheld - not all by nay menas - but most content neutral regulations of time place and manner are upheld - Theyre extremely common in American cities and theyre routinely enforced.

government employee contends that his free speech rights have been violated by his employer

When a government employee contends that his rights under the Free Speech Clause of the First Amendment, as made applicable to state and local government action through the Fourteenth Amendment, have been violated by his employer, the employee must show that he was speaking as a citizen on a matter of public concern. Even when an employee is speaking as a citizen on a matter of public concern, the First Amendment interest of the employee must be balanced against the interest of the state, as an employer, in effective and efficient management of its internal affairs. Here, although police behavior is a matter of public concern, there is no indication that this sexually explicit video actually addresses that matter of public concern in any meaningful way. Even if the sexually explicit video does have some tangential relation to the public concern of police conduct, the police department's interests in promoting the efficiency of the public services it performs through its employees likely prevails over the officer's interests in profiting from a sexually explicit video.

sales tax favoring religious organizations

When a governmental program shows preference to one religion over another, or to religion over nonreligion, strict scrutiny applies. This sales tax exemption fails under strict scrutiny because it does not appear to be the least restrictive means available to achieve any compelling state interest. Consequently, it violates the Establishment Clause of the First Amendment. Note - the Constitution does not prevent a religious organization from qualifying for a tax exemption if the exemption has a secular purpose and its primary effect is secular. On the contrary, strict scrutiny applies when the government purposely denies a religious entity access to an otherwise available public benefit purely on account of its religious status. Also, sales tax exemption is unconstitutional if it benefits only religious organizations and has the purpose and effect of advancing religion.

parents - right to educate children

While parents do enjoy a fundamental right to make decisions regarding the care, custody, and control of one's children, including the right to privately educate one's child outside the public school system, that right is subject to reasonable educational standards imposed by the state. Consequently, the court must strike down any requirements that the court finds impose unreasonable educational requirements on the homeschooling of children.

future injury/standing

While the threat of future injury can suffice, it cannot be merely hypothetical or conjectural, but must be actual and imminent. Here, the cereal manufacturer has not suffered a direct injury, and because it does not manufacture children's cereal, any threat of a future injury is purely hypothetical. Therefore, the cereal manufacturer lacks standing, and the federal court cannot determine the merits of the suit. An imminent threat of injury may be sufficient to create standing and ripeness.

Can Congress restrict supreme court's appellate review?

Yes. Pursuant to Article III, Section 2 of the U.S. Constitution, the U.S. Supreme Court's appellate jurisdiction is subject to "such exceptions, and under such regulations as the Congress shall make." Congress can restrict the U.S. Supreme Court's appellate review. Ex: While the U.S. Supreme Court can constitutionally hear an appeal on the grounds asserted by the defendant (i.e., a violation of his right to freedom of expression, first amendment), the Court cannot hear this appeal because Congress has restricted the Court's appellate jurisdiction to a final judgment rendered by the highest state court that could have ruled on the issue. The U.S. Supreme Court has appellate jurisdiction to review a state criminal conviction if the defendant is challenging the conviction as a violation of his federal rights. However, the U.S. Supreme Court's appellate jurisdiction is subject to "such exceptions, and under such regulations as the Congress shall make."

IMPORTANT ANALYSIS - alienage classification

alienage classification -A classification based on alienage is subject to a different standard depending on whether the action is taken by the state or by the federal government. Courts will generally apply the strict scrutiny test and strike down state-based laws that discriminate against resident aliens for lack of U.S. citizenship. In contrast, Congress has plenary power over aliens under Article I, and the power to expel or exclude aliens is a fundamental sovereign attribute exercised by the Government's political departments largely immune from judicial control. Therefore, a federal alienage classification is likely valid unless it is deemed arbitrary and unreasonable. Here, the government proved that the law was rationally related to a legitimate governmental interest and thus the resident alien's claim will fail.

Congress's investigatory power may extend to....

any matter within a "legitimate legislative sphere," Example - funding research and infrastructure improvements for the conservation of water fall within that sphere.

IMPORTANT ANALYSIS - equal protection modes of analysis

equal protection modes of analysis - The Equal Protection Clause of the Fourteenth Amendment provides that states are generally prohibited from passing legislation that treats similarly situated persons differently. To trigger strict or intermediate scrutiny, there must be discriminatory intent on the part of the government or a classification based on a suspect or quasi-suspect class. Strict scrutiny also applies if a fundamental right is involved. The rational basis standard is used in all cases in which one of the higher standards does not apply. A law passes the rational basis standard of review if it is rationally related to a legitimate governmental interest. This is a test of minimal scrutiny. Here, students who bring their lunches from home are being treated differently from students who eat in the cafeteria because they must eat in a separate room. However, the ordinance has a rational basis that is related to the legitimate government interest of preventing complications due to food allergies. Because this ordinance does not target a suspect class or involve a fundamental right and has no discriminatory intent, it passes rational basis review. Therefore, the parents' challenge is likely to fail.

exception to the rule that a taxpayer lacks standing to challenge a governmental expenditure

exception to the rule that a taxpayer lacks standing to challenge a governmental expenditure when the expenditure violates the First Amendment Establishment of Religion Clause - but this exception is very narrow. The exception does not apply to the expenditure of general discretionary funds by the executive branch. the narrow exception requires that Congress authorize the funds for a specific use that violates the Establishment Clause. Example - Congress merely authorized the funds for the President's discretionary use, and not to fund the challenged office so not violation.

quasi suspect classifications

gender and legitimacy Trigger intermediate scrutiny—is the law substantially related to an important government interest? a. Gender classifications are almost always invalid (e.g., Oklahoma had a law permitting women to legally drink alcohol at a younger age than men). Almost no gender distinctions are ever upheld anymore. b. Legitimacy (i.e., something depends on whether parents were married at the time of one's birth) laws are almost always invalid, especially if punitive in nature. Under this test most legitimacy classifications are invalid - most of older laws struck down seemed to impose some kind of disability on illegitimate as way of punishing the parents for having had an illegitimate child. Plainly not imp govt interest and laws of that character that have punitive motivation are struck down.

Injury

o Almost anything can be injury, especially if Congress says so (either past or future). o Must be concrete (not abstract), particular in some sense, but need not be economic. § If your freedom of movement or enjoyment of public space is impaired, that constitutes injury. Even if no economic consequence to you at all. o Mere ideological objection is not injury. Cannot sue to stop govt from doing something because you don't like it. Must harm you in some concrete way. o An organization has standing if its members have standing.

Anti-Commandeering

o Congress cannot force states to adopt or enforce regulatory programs. It cannot commandeer state and local agencies to implement federal programs. Example 16: The Brady Gun-Control Act required background checks for people wanting to purchase firearms, and it required that state and local law enforcement carry out those background checks. The Supreme Court said this was unconstitutional. Congress cant force states to adopt federal regulatory programs. o What can Congress do to enforce regulatory programs? § It can bribe states through use of the spending power. Congress can say were going to make a lot available money for law enforcement and u get it only if you do the thing we want about background checks. § It can adopt its own regulatory program and enforce it with federal officers. All congress cannot do is force the state to carry out federal programs.

Two Due Process Clauses

o Fifth Amendment applies to the federal government. o Fourteenth Amendment due process clause applies to localities and states.

Privileges and Immunities of State Citizenship under Article IV (Comity Clause)

o Forbids serious discrimination against out of state individuals, absent substantial justification. - Does not protect out-of-state corporations but do protect nonresident individuals against serious discrimination. - "Serious discrimination" typically involves employment. o Rule: There can be no legal requirement of residency for private employment. States cannot require that you live or reside in the state to work in the state. However, public employment can require residency requirements.

Due Process versus Equal Protection - - rule of thumb to distinguish between 2

o If a law denies a fundamental right to everyone, it violates due process. o If a law denies a fundamental right to only some but not others, it violates equal protection. But fundamental rights are the same - travel, etc and others below.

Endorsement

o It is a violation of the Establishment Clause for the government to endorse one religion over another and also to endorse religion over non-religion. You not only have freedom of religion - freedom to choose any one you want without govt pressure - you also have freedom from religion, freedom to choose none of the above if that's your choice. endorsement idea sounds good but number of very obvious endorsements that we tolerate without much concern. Like in god we trust below. § But, many endorsements are upheld, such as "In God We Trust" on currency. o The Supreme Court really mainly wants to prevent coercive endorsement of religion (one that might override individual choice). that's the real nub of the problem - coercive endorsement of religion. Whats unconstitutional is govt endorsement of religion in a context that might prove coercive on an individuals conscience. That explains why school prayer is unconstitutional - any officially sponsored school prayer is unconstitutional, no matter how general the prayer or how nondenominational the phrasing. There will be some kids who don't believe it, and they may feel intimidated or coerced when everyone else stands up to pray. Officially sponsored school prayer is a coercive endorsement of religion. § The Establishment Clause prohibits government endorsement of religion in a context that might prove coercive on an individual's conscience. § Examples: • Officially-sponsored school prayer is unconstitutional. • Officially-sponsored graduation prayer is unconstitutional. However its phrased, there will be someone who doesn't want to say it - and that person may feel in the context of graduation ceremony, coerced. But truly voluntary student prayer - one not sponsored by govt - but simply volunteered by individual - that's okay. • Bible reading is permissible, but cannot be inspirational (e.g., as literature or poetry). In public schools can read bible as literature and poetry - as maybe extended genology. As maybe foundational document of western culture. Can read bible for all those purposes but cannot read it for inspiration. The minute kids begin to take bible seriously as religious text, u have to cut out reading in public schools. Its like hearsay rule - bible can be admitted to classroom for any purpose other than its truth. • Display of the Ten Commandments is sometimes okay. It can be displayed for secular purposes (e.g., historical or promoting morals), but not to inspire religious belief. Lot of cases about whether displays of ten commandments are endorsements of religion. Ppl argue theyre not because they have historical or secular purpose to promote morals rather than advance religion. Some uses of ten commandments have been upheld and others have not. You could teach ten commandments though as early legal code. You can say look how short they were then - no ban on having ten commandments as topic of discussion. But Cannot post the Ten Commandments in a classroom and leave it there every day of the school year— that's designed to inspire religious belief in ten commandments and would be struck down. o Can teach the Ten Commandments in school as an example of an early legal code. o Cannot post the Ten Commandments in a classroom and leave it there every day of the school year—designed to inspire religious belief. o Cannot post the Ten Commandments in a courthouse if the context makes plain that the purpose is to endorse religious belief. Main idea is that - if ten commandments are posted to endorse their religious content, that's an establishment of religion - posted by the government to endorse their religious content -thats an establishment of religion. If theyre posted for some other secular purpose, that's okay. • Laws prohibiting teaching evolution have been struck down. Probably because they fail test of mere rationality. • Legislative prayer is okay for historical practices. Supreme court said its okay for state legislature to have a chaplain say a prayer at the beginning of each session - very commonly done in state legislatures and in US congress. Its been done forever and court said historical practice seemed to matter. • Nativity scenes are okay on public property if there is something else there to dilute the religious message (e.g., Hanukkah symbols, Rudolph the red-nosed reindeer). Supreme court has wrestled with this - first time sc had cresh case, they said nativity scene okay because Christmas used to be religiously significant but no longer is - its become shopping season. Even court grew ashamed of saying that and backed up to try again. Second time Court said u can have cresh on public property at Christmas with all the religious symbolism it includes if u have something else there with it - some other things there that dilute religious message. so if just nativity scene with jesus and mary and shephards, that would be struck down as endorsement of christianity. But if nativity scene and additions like Rudolph the rednose raindeer, the augmented nativity scene augmented by non Christian elements would be allowed.

If life, liberty, or property is being taken by the government, what process is due?

o Procedural due process is variable, and the types of hearings can range from casual to very elaborate. Due process can range from nothing so very minimal and casual procedures all the way to the elaborate procedures of criminal trial. procedural due process can mean a little or a lot. o To decide what kind of process is due, the courts balance three factors: 1) The individual interest at stake (life, liberty, property) - what kind of interest will it be? Life, liberty, or property- theyre the only ones that count.; 2) The value of the procedure in protecting that interest - whatever procedure being claimed or whatever person wants to have, u assess individual interest at stake and value of procedure in protecting that interest against the governments interest in efficiency; and 3) The governments interest in efficiency and cost - which usually means cost. Sometimes balance requires highly elaborate procedures as in criminal trials or imposing a fine. Sometimes it allows minimal procedures like disciplining a school child - not much in terms of a hearing required.

Nonpublic forum

o This includes all kinds of government property that is not a public forum and not traditionally open to speech activities (e.g., government offices, jails, power plants, military bases, etc. - not typically places where you can go and parade and protest and speak). Here, the government has great power over what u do in nonpublic forums. Basically, any reasonable regulation of speech will be upheld. Some things that are not reasonable - it would not be reasonable to favor republicans over democrats, etc. viewpoint discrimination is invalid - but absent that kind of viewpoint discrimination which u rarely see, govt can regulate speech activities in a nonpublic forum almost any way it likes. You cant go to govt office building and march up and down inside corridors shouting slogans - that would disrupt govt functions. but where can u go? You can go to public sidewalk outside govt building as long as u obey neutral time place and manner regulations and protest to your hearts content. That's difference between a public forum like streets, sidewalks and public parks, and other govt property such as an office building. § Viewpoint discrimination is invalid: One clearly unreasonable kind of regulation would be to discriminate based on viewpoint (e.g., between members of different political parties). § Disruption of the functions of government: One should go outside to the public sidewalk surrounding the building since that is a public forum. • Limited public forum - idea is its some place that's not public forum but govt has chosen voluntarily to open to all comers. o Describes a place that is not a traditional public forum, but that the government chooses to open to all comers (e.g., a municipal theater that anyone can rent). Municipal theater - govt doesn't have to build it or have to have one but if it does have one and it rents it out to all kinds of plays, then it can only enforce content neutral regulations on what happens there. Cant disapprove a particular play because of its point of view. Don't get carried away with limited public forum - it's a very limited idea. School auditorium for example is not a limited public forum. Now schools have auditoriums - they open them to various speech activities but only on appropriate topics. So a student who makes a profane or sexually explicit speech in a school assembly absolutely may be disciplined because the speech was inppropriate in the school auditotorium. Student could have gone outside to the public sidewalk, down the street to the public park, or student could have gone to public forum and said exactly the same profane and sexually explicit things and been home free. Difference is a public forum vs other property. o In such areas, only time, place, or manner regulations are allowed, but this is a narrow category. Example 40: A school auditorium is not a public forum, limited or otherwise. Just because a school auditorium is open to some speech does not mean it has to be open to all.

Deficiency of Lemon Test

o Trouble is its Exceedingly difficult to apply - hard to know what is and what is not constitutional under the test. o It once was interpreted to condemn aid to religious primary and secondary schools, but neutral aid is now allowed (goes to everyone). As long as neutral and goes to everyone - all private schools, all religious, all non religious schools - and govt isn't making decision about who gets aid and who doesnt - then its fine. All govt has to do today is say we will give each parent money that the parent can use to send her child to a private school, whether religious or not. That's neutral and today it passes muster. § The government gives aid to parents and the parents are allowed to send their children to the schools of their choice. § The parents are making the decision as to whether a religious school gets the money. • The government is not picking and choosing recipients. In recent years court has spoken more about endorsement and less about lemon test.

Vagueness and Overbreadth

o Vagueness: Vague laws are ones that give no clear notice of what is prohibited and thus violate due process. o Overbreadth: Overbroad laws are ones that go too far in regulating speech. These laws burden substantially more speech than is necessary to protect a compelling interest and thus violate the First Amendment. Example 36: Laws that prohibit all nudity in drive-in movies are overbroad. Some nudity in drive in movies is legally obscene and obscenity can be banned. But much nudity in drive in movies is not legally obscene, but merely very large so law banning all nudity in drive in movies is overbroad - it goes substantially beyond a compelling interest and is therefore invalid. Exam Tip 12: Vagueness and overbreadth are almost always seen together on the bar exam. That is its almost inconceivable that you'll be required to distinguish between an answer saying vague and an answer saying overbroad. More likely one of 4 answers will say law Is vague and overbroad - in fact they overlap. On ground and in real world, vagueness and overbreadth overlap and on bar exam typically seen together. 3. Prior Restraints - even when speech might be constitutionally punished after the fact, prior restraints are especially disfavored. Means almost impossible to get an injunction agasint publication. Really has to be extraordinarily strong case to get injunction against publication because prior restraints are disfavored. o Are especially disfavored and will be struck down even when other forms of regulation might be upheld. o Injunctions against speech are almost impossible to get.

Voting and Ballot Access

o Voting is a fundamental right to all citizens age 18 and over. Not right for noncitizens though. § Poll taxes are unconstitutional because they burden the fundamental right to vote. Taxes on exercise of ballot - tax paid to vote. § Short-term (e.g., 30 days) residency requirements are permitted. Anything over 90 days runs into trouble. § Congress controls the residency requirements for presidential elections. States control residency requirements for all other elections. o Ballot access - has looser rules. § States can impose residency requirements for candidates to be listed on a ballot, such as longer residency, filing fees, and nomination petitions, so long as serious candidates can reasonably comply. Don't want ballot to be so long or confusing or with so many fake names that no one knows what it means. So states can do what they want as long as requirements allow reasonable, serious candidates to comply. § If the requirements become so onerous that they effectively bar access to the ballot, then they are unconstitutional.

political question

one to be resolved by one or both of the other two branches of government. Example - not a political question - Courts do have jurisdiction to adjudicate the constitutionality of a state's election process so thats not a political question.

exclusively federal powers

power over foreign relations and the power to coin money. States don't have foreign relations and cant enter into treaties with foreign powers. They cant freelance in international arena just because they disagree with washingtons policies. And states also cannot coin money. But generally state and fed powers are concurrent.

substantive due process

same standards of review as equal protection. Strict Scrutiny o Is the law necessary for a compelling government interest? o Implicit in strict scrutiny is the requirement for the least restrictive means. If some less restrictive way for govt to protect its interest the means chosen is not strictly necessary. o When strict scrutiny applies, the government bears the burden of proof. The government must show that the interest is compelling and the means chosen (or law) is necessary to that interest. o Applies when there is a suspect classification or a fundamental right strict scrutiny does require least restrictive means to be used and yes, burden of proof on govt. 2. Intermediate Scrutiny o Is the law substantially related to an important government interest? o Applies in equal protection doctrine to classifications based on legitimacy and gender. Legitimacy and gender trigger intermediate scrutiny. Note 2: While three standards of review are available in both substantive due process and equal protection claims, intermediate scrutiny has only ever been used to decide cases based on equal protection grounds. 3. Rational Basis - fall back test - "mere" rationality test o Is the law rationally related to a legitimate interest or end of government? o The challenger bears the burden of proof (burden of showing the law is not rationally related to legit interest). o Low burden of proof. - rational basis test is hard to flunk - law doesn't have to be sensible or wise or humane or appropriate - just has to be non-insane. o Applies to all other cases (residual test)

the Privileges or Immunities Clause of the Fourteenth Amendment

the Privileges or Immunities Clause of the Fourteenth Amendment does not apply to aliens but only to United States citizens; further, this clause applies only to the states, not the federal government.

Adequate and Independent State Grounds (AISG)

§ Arises only in the U.S. Supreme Court, and it arises only when the Supreme Court reviews a state-court judgment. Very narrow issue since issue only arises in supreme court. Exam Tip 2: Frequently tested topic § Rule: The Supreme Court can review a state court judgment only if it turned on federal grounds. The court has no jurisdiction if the judgment below rested on an adequate and independent state ground. • Adequate: The state ground must control the decision no matter how a federal issue is decided. When does this happen? When the federal claimant (i.e., the party asserting a federal right) wins anyway under state law. Example 1: A criminal defendant objects to the legality of a search and seeks to suppress the evidence under both the Fourth Amendment and under the analogous search and seizure provision of the state's constitution - every state constitution has some version of search and seizure provision. Suppose the state court says it is unsure about the Fourth Amendment, but under the state law, the search was illegal and the evidence must be suppressed. This kind of MBE case cannot be reviewed by the U.S. Supreme Court—the federal claimant wins anyway under state law. Whether the evidence would also be suppressed under federal law simply does not matter. Note 1: Remember this key point: The U.S. Constitution is a floor, not a ceiling, for individual rights. A state court interpreting a state constitution can always give you more protection or more due process or more protection against death penalty, or generally more protection. But a state can never give you less than the federal constitution requires. • Independent: The state law does not depend on an interpretation of federal law. No AISG if state law adopts or follows federal law. • When a state court decision is unclear as to the basis of the decision (i.e., whether it is based on the state constitution or the federal constitution), the Supreme Court can review the federal issue. Situation that shouldn't happen - state courts should not be unclear - should say clearly whether relying on state constitution or not. But sometimes state courts invoke both state and fed constitution without making clear whether state is independent ground of decision or whether state is forced tofollow fed rule. When state is unclear as to basis of decision, supreme court can review the federal issue.

Third-party standing" refers to the question of whether you can raise the rights of someone else.

§ Generally, the answer is no. generally even if u have standing and show injury, causation, and redressability u can generally only raise ur own rights. Generally cant claim statute violates rights of someone else. § Exception: Parties to an exchange or transaction can raise the rights of other parties to that exchange or transaction. if 2 people involved in some exchange or transaction one of them can raise the rights of the other. Example 4: Doctors who were not compensated for providing abortion services can raise the rights of women who wanted to have abortions provided. The doctor has injury because he was not paid but he wants to raise the rights of his patient to have an abortion, not his own. The doctor and the patient are involved in an exchange or transaction. Example 5: A seller of beer in a saloon can raise the rights of 18-year-old boys to buy beer at the same age as 18-year-old girls. The saloon keeper is injured because she is not making money. She wants to raise the rights of the underage boys because that is gender discrimination. She can raise the claims of those customers or would-be customers because they are parties to an exchange or transaction. so general rule is third party standing is not allowed and cannot raise rights of someone else, but can raise rights of someone else if u and someone are parties to exchange or transaction.

Legislative standing

§ Legislators do not have standing to challenge laws that they voted against. When legislator votes against law, that's merely ideological objection - voting against it because you don't like it - so ideological objection doesn't give standing to anyone. § In some circumstances, the legislature itself as a body may have standing, if the claim concerns its institutional functions. Supreme court occasionally allowed legislature standing, but not legislator standing. Occasional legislature standing, but no legislator standing.

Non-Suspect Classifications (Age and Wealth)

• Age discrimination in employment is barred by statute, but it is not a suspect or quasi-suspect classification under the Equal Protection Clause. Constitutional test for age requirements is rational basis. o Triggers rational basis • Wealth is not a suspect or quasi-suspect classification, - Govt can and often does charge a price or sets price for particular license or something and usually paying price is much harder for poor people than people who are well off. govt can generally charge a price. But what happens if person is indigent and just doesn't have means and cant pay? When does govt have to waive fees for indigent? but the government has to waive filing fees for indigents when charging the fees would deny a fundamental right. Examples include: divorce (because marriage is a fundamental right and so is the escape from it - You cant be too poor to get married or to get divorced.); transcript for appeal of a criminal conviction (because appellate review is a fundamental right - Transcript for appeal of a criminal conviction - very often ppl convicted of crimes have to pay for transcript of the trial to provide basis for appellate review but crim defendant who is indigent and cant pay for transcript gets it anyways. Cant charge u for transcript for appeal of criminal conviction.); transcript for appeal of the termination of parental rights (parental rights are fundamental rights - if ur parental rights are terminated that's a judicial proceeding and if someoe found guilty of abandonment or neglect or abuse, if someones parental rights terminated after judicial proceeding, to have appeal of determination must provide transcript of the trial. if u can pay for transcript u pay. If u cant pay they have to give it to you anyways.) you cant be too poor to get divorced, cant be too poor to have a transcript to appeal criminal conviction, cant be poor to appeal termination of parental rights but u can be too poor to go bankrupt. § Bankruptcy filing fees do not have to be waived. Filing fee for bankruptcy does not have to be waived - If u cant scrape together the filing fee required for bankruptucy petition, you may be too poor to go bankrupt.

prohibited legislation

• Bill of Attainder—A bill of attainder is a legislative punishment imposed without judicial trial and is unconstitutional. For example law declaring jane doe a felon would be a bill of attainer - attempt to legislate criminal guilt and unconstitutional. • Ex Post Facto Laws—Unconstitutional to expand criminal liability retroactively, either by creating a new crime that applies retroactively to past conduct or by increasing the penalty for past conduct. If u either create new crime retroactively or increase penalty retroactively, that's ex post facto and unconstitutional. Ex post facto is doctrine limited to criminal statutes - has no application to civil legislation. • Contract Clause—Bars states from legislative impairment of existing contracts, unless there is an overriding need (something like an emergency). Legislation can c hange rules for future - u can say all contracts made from this day forward shall include or not include some features - u can always change rules for future. U can even change rules for already exising contracts if theres an overriding need.overriding need means something like emergency. Ex: great depression where value of money collapsed and people owed amounts on mortgages they could never hope to repay in depression era dollars. That was sudden, unanticipated event and made it reasonable to alter the terms of existing contracts.

Prohibited Legislation

• Bill of Attainder—A bill of attainder is a legislative punishment imposed without judicial trial and is unconstitutional. For example law declaring jane doe a felon would be a bill of attainer - attempt to legislate criminal guilt and unconstitutional. • Ex Post Facto Laws—Unconstitutional to expand criminal liability retroactively, either by creating a new crime that applies retroactively to past conduct or by increasing the penalty for past conduct. If u either create new crime retroactively or increase penalty retroactively, that's ex post facto and unconstitutional. Ex post facto is doctrine limited to criminal statutes - has no application to civil legislation. • Contract Clause—Bars states from legislative impairment of existing contracts, unless there is an overriding need (something like an emergency). Legislation can change rules for future - u can say all contracts made from this day forward shall include or not include some features - u can always change rules for future. U can even change rules for already exising contracts if theres an overriding need.overriding need means something like emergency. Ex: great depression where value of money collapsed and people owed amounts on mortgages they could never hope to repay in depression era dollars. That was sudden, unanticipated event and made it reasonable to alter the terms of existing contracts.

The Powers of the President—Foreign Affairs

• Commander in Chief: The President has control over military decisions, although Congress has exclusive power to declare war. o Congress can cut off funding to military actions. Congress can reign in foreign involvement by president by cutting off funding for military activities. • Treaties and Executive Agreements o Treaties are negotiated by the President, but require approval by a 2/3 vote of the Senate. Once a treaty is ratified (approved), it has the same authority as a statute. o treaties should be distinguished from executive agreements - Executive agreements are presidential negotiations not submitted for approval by the Senate. They can be authorized, precluded, or overridden by statute, but they take precedence over conflicting state laws. They do not have the binding status of a treaty.

Advisory Opinions

• Federal courts cannot issue advisory opinions. • Federal courts cannot rule on the constitutionality of proposed legislation. If seeing any scheme where court asked to predict constitutionality of law not yet applied, if court asked to say prospectively whether some law would be constitutional if enacted, that's unconstitutional advisory opinion. Exam Tip 4: Commonly tested Example 6: Disputes about federal grants and aid projects were to be litigated in federal court, but the statute also provided that the court's judgment would be given effect only if the head of the relevant federal agency agreed with it. This is an unconstitutional advisory opinion. Federal courts decide cases; they do not make preliminary recommendations for decisions by executive officers.

Preemption

• Federal law preempts (overrides) inconsistent state law. Any kind of federal law can pre-empt - can be constitution itself or can be fed statute. Can be administrative regulation or even executive order - any valid federal law overrides state law if inconsistent with or incompatible with federal law. • State law is not preempted simply because it addresses the same subject matter or topic as a federal statute. Its preempted only if inconsistent with fed statute - There must be incompatibility or conflict. Example 24: Federal law provides automobile emissions must be at least 95% pure. California adopts a more stringent requirement requiring CA emissions be 98% pure (which is automatically at least 95% pure). Is this state law preempted? No. There is no conflict—you can comply with both of those standards. • Preempting the Field: When Congress determines that there should be no state law of any sort in a particular field, then any state law in that area is inconsistent with the federal statute and is preempted. This is rare.

Regulation of Association

• Freedom of association: generally means persons cannot be punished or disadvantaged because of political associations. • Loyalty oaths: Public employees can be required to take a loyalty oath to the Constitution, but most loyalty oaths that say anything else besides constitituion are struck down as vague and/or overbroad. Loyalty oaths are very likely invalid because of vagueness and overbreadth. • Bar membership: State bar can investigate good character, but they cannot deny admission based on political affiliations. Cant deny because ur member of communist party or because ur member of some alt right organization. Ur political views cannot be used to deny bar membership. But they can refuse bar membership if u refuse to answer relevant questions or give them false information. • Political parties: political parties have rights of asociation too as well as individuals. Political parties have rights of association that limit how states can regulate them - specifically States cannot require open primaries (i.e., one in which you do not have to be registered as a member of the particular party to participate in the primary). In open primary all voters can participate in either primary or whatever primary they like - lot of states have open primaries now but state cannot demand open primaries - party itself gets to decide how it chooses its nominee.

regulation of association

• Freedom of association: generally means persons cannot be punished or disadvantaged because of political associations. • Loyalty oaths: Public employees can be required to take a loyalty oath to the Constitution, but most loyalty oaths that say anything else besides constitituion are struck down as vague and/or overbroad. Loyalty oaths are very likely is invalid because of vagueness and overbreadth. • Bar membership: State bar can investigate good character, but they cannot deny admission based on political affiliations. Cant deny because ur member of communist party or because ur member of some alt right organization. Ur political views cannot be used to deny bar membership. But they can refuse bar membership if u refuse to answer relevant questions or give them false information. • Political parties: political parties have rights of asociaiton too as well as indiivduals. Political aprties have rights of association that limit how states can regulate them - specifically States cannot require open primaries (i.e., one in which you do not have to be registered as a member of the particular party to participate in the primary). In open primary all voters can participate in either primary or whatever primary they like - lot of states have open primaries now but state cannot demand open primaries - party itself gets to decide how it chooses its nominee.

Quasi-Suspect Classifications

• Gender and Legitimacy trigger intermediate scrutiny—is the law substantially related to an important government interest? • Gender classifications are almost always invalid (e.g., Oklahoma had a law permitting women to legally drink alcohol at a younger age than men). Almost no gender distinctions are ever upheld anymore. o really only Permissible examples of gender classifications: § Statutory rape can be gender-specific (historically) - most statutory rape laws are gender neutral nowadays though but they don't have to be and supreme court said reason is because far greater risk of physical imposition or injury if underaged person is female; and § The draft - ok to draft men but not women for military service. This also slightly outdated now because women becoming more involved in the military. • Legitimacy (i.e., something depends on whether parents were married at the time of one's birth) laws are almost always invalid, especially if punitive in nature. Under this test most legitimacy classifications are invalid - most of older laws struck down seemed to impose some kind of disability on illegitimate as way of punishing the parents for having had an illegitimate child. Plainly not imp govt interest and laws of that character that have punitive motivation are struck down.

speech by government employees

• General rule: Government employees generally cannot be hired or fired based on political party, political philosophy, or any act of expression such as letter to the editor. This is true for everyone including run of the mill govt workers like police offices, public school teahcers, transit workers, etc. - they can be fired for not doing their jobs - fact that u disagree with govt policy doesn't give u the right not to follow it. That is u can be fired if u disrupt the work place. You can be fired if you dotn carry out your emplokymetn instructions. But cant be fired simply because you belong to the wrong party or endorse the wrong views. None of this applies to confidential or policy making employees - these are upper level employees - policy makers are like cabinet officials, agency heads - confiddental employees are their immediate advsors. And confidential and policy making employees can be hired and fired based on loyalty and politics. o Can be fired for disrupting the workplace or not doing their jobs. • Exception: This general rule does not apply to confidential advisors or policy-making employees (e.g., the President's cabinet officers). President trump can choose, as all of his successors did, cabinet officers who are loyal to his points of view and he can fire people who disagree with him. That's true at level of cabinet officer, at level of agency head, at level of special asst to president. But run of the mill govt employees like school teachers and clerical workers and uni professors - people who pave streets, transportioan workers - ordinary run of the mill workers cannot be hired or fired based on political party, political philosophy, or any act of expression.

EQUAL PROTECTION (RACE, ETHNICITY, AND NATIONAL ORIGIN) - General Considerations

• Privileges or Immunities of National Citizenship under the 14th Amendment—means nothing today (so it is never the correct answer on the MBE) Distinguish privileges and immunities of national citizenship under 14th amendment - means nothing, always wrong answer vs. privileges and immunities of state citizenship under article 4 - sometimes called comity clause. Note 3: Privileges and Immunities of State Citizenship Clause in Article IV (Comity Clause) has a narrow but important meaning. It prohibits serious discrimination against out-of-state individuals, especially in the context of access to the private job market, i.e. prohibits requiring those who work in the state to live in the state. • Two Due Process Clauses o Fifth Amendment applies to the federal government. o Fourteenth Amendment due process clause applies to localities and states. • One Equal Protection Clause—found in the 14th Amendment o Applies to localities and states Technically no equal protection clause applicable to the federal government. So do feds get free ride for equal protection? No they don't. they have to follow equal protection concepts and principles and equal protection law but they apply to fed govt under 5th amendment due process clause. That is, equal protection for fed government is called due process - same standards and content, but a different label. Note 4: Though no Equal Protection Clause guarantee technically applies to the federal government, equal protection concepts are applied to the federal government via the Due Process Clause of the Fifth Amendment. Therefore, states and localities have both equal protection and due process; for the federal government, equal protection and due process are called Fifth Amendment Due Process. For national govt only have one clause - due process - and everything applies to national govt as due process. for states and localities have 2 clauses - due process clause and equal protection clause. How to distinguish the two? Overlap concerns fundamental rights - if law denies a fundamental right to everyone in a state or a locality, it violate due process. if law denies fundamental right to some persons but not others, it violates equal protection. Distinction made at level of state legislation and local ordinance. For national govt, its all due process.

Standards of Review—Same for Due Process and Equal Protection

• Strict scrutiny, i.e., is the law necessary for a compelling interest? • Intermediate scrutiny, i.e., is the law substantially related to an important government interest? • Rational basis, i.e., is the law rationally related to a legitimate government interest?

State Action

• The 13th Amendment (outlawing slavery and involuntary servitude) applies directly to private parties and individuals, but it is narrow in its focus. Protections only against slavery and involuntary servitude. • Other individual rights apply to states and localities through the 14th Amendment, which requires state action. If state violates free speech, correct thing is to say they've violated ur 1st and 14th amendment rights. If city conducts illegal search theyve violated ur 4th and 14th amendment rights. Virtually all individual rights enforced through the 14th amendment, which requires state action. • "State action" means government action, whether state or local (e.g., a county firing a sheriff is state action; a city denying a permit is state action). • Most state action is clear, but not always. Government cannot be significantly involved in private discrimination but what does that mean? Means govt cannot ... o Significant state involvement: § Government cannot facilitate private discrimination. § Government cannot profit (or be financial partner profiting from) from private discrimination. § Government cannot enforce a private agreement to discriminate. § But, government is not required to prevent private discrimination. o Generally, the government acts constitutionally so long as its own conduct is neutral and even-handed (e.g., trespass laws, liquor licenses for private clubs). Trespass laws - these protect landowners of all races, creeds, colors, religions, and political views against trespassers of all races, colors, creed, religions, and political views. Landowner might be prejudiced in asserting who can come on his property but enforcement of trespass laws is fine as long as govts conduct is neutral and even handed - so long as they enforce laws equally for everyone. Moose lodge - discriminated on basis of race In its membership. Liquor license - moose lodge is private entity and not govt so no state action but p claimed because moose lodge got liquor license it made govt responsible for discrimination by moose lodge. Turns out liquor license is not incidental feature of what happens at moose lodge but very central. So argument that when govt gave liquor license enabling private club to thrive, govt became responsible for racial discrimination. But this is wrong argument. So long as govt gives liquor licenses to all white/black clubs, all male/female clubs, so long as it treats al liquor licensees the same, govts conduct is constitutional. Govt acts constitutionally so long as its own conduct is neutral and even handed - no constitutional obligation to prevent discrimination by private parties. • Anti-Discrimination Statutes o State action is required to show a violation of the Constitution. o State action is irrelevant if there is anti-discrimination legislation. Why? Legislation isn't limited to controlling what govt itself does - legislature most of time has power to regulate purely private conduct. If legislation says u cant discriminate on basis of race and private employment, u cant discriminate on basis of race and private employment. Not because its unconstitutional - theres no state action, but because private discrimination is illegal because anti discrimination statute says so and u have to obey the law. State action is a general requirement for all rights except 13th amendment right against slavery - all other rights enforced by 14th amendment and require state action.

Government Speech

• The First Amendment restrictions basically do not apply to the government as a speaker. Its obvious - govt is free to advertise the us army. It doesn't have to give equal time to critics of military. Same is true when govt accepts a monument donated by private persons. Govt doesn't have to accept any monument just because it accepts one - it can still control what gifts it accepts. • Government as a speaker is free to express a point of view (e.g., advertising the military). • The government does not have to accept all monuments donated by a private person simply because it accepts one—when the government is controlling the message, it is entitled to say what it wants. But one area where govt speech has been litigated is specialty license plates. These carry messages requested by the customers. Messages are requested by drivers but license plates are still govt speech so govt can refuse licenses that contain offensive messages. Govt sets up thing where u pay a little extra, u get to have a little slogan or words on ur lciense plate that mean something but theyre free to reject licenses that would be offensive to others. That's because its government speech. • But specialty license plates bearing messages requested by purchasers are still government speech, so the government can refuse to issue plates that would be offensive to other citizens.

Intergovernmental Immunities

• The federal government is generally immune from direct state regulation or taxation. However, states can tax indirectly (e.g., taxing the income of federal employees). Fine as long as they don't discriminate against fed employees. • States are not immune from direct federal regulation or taxation (e.g., pollution regulations, employment laws, etc.). • States cannot shield state officers from federal liability for violating federal constitutional rights. • Exception: The anti-commandeering principle covered in Chapter 3. States cannot be directly forced to carry out federal programs - state officers cannot be conscripted for federal duties - violates anti commandeering principle. The federal government can always use the spending power to bribe states to comply though, by making money available only if you do something and fed govt can adopt direct federal regulations administered by federal agents.

Campaign Finance

• The use of money to support a political campaign is political speech and the regulation of campaign finance raises First Amendment issues. as regulations have become more elaborate constitutional restrictions on those regulations have also become more complicated and to some extent unstable. Key points - distinguish campaign contributions from campaign expenditures. • Contributions versus expenditures: o Contributions to political campaigns and candidates can be regulated, provided that the limits are not unreasonably low. o Direct expenditures in support of a candidate, a campaign, or a political issue cannot be regulated. They can be made regardless of amount. § The rationale is to prevent corruption: a candidate could theoretically act in a certain way in exchange for a larger, direct contribution. Diff between contribution and expenditure? Different is in their relation to corruption. Large contributions where u give money to a campaign or candidate may create the reality or at least appearance of corruption as candidate would presumably be willing to do things to get the contributions but direct expenditure of ones own money doesn't pose same risk of corruption. Now lots of people who doubt whether the distinction between contributions and expenditures makes much sense on the ground - but its deeply embedded in constitutional doctrine. Campaign contributions can be limited, campaign expenditures cannot be. If you distinguish between contributions and expenditures, must also distinguish between independent expenditures and coordinated expenditures. • Independent expenditures (person spends her own money to support a political candidate maybe by buying tv ads, that's fine) versus coordinated expenditures (a disguised campaign contribution - when a supporter spends money as the campaign directs, for example campaign says pls pay bills for tv ads and they do it. May be structured to look like an expenditure by you but its actually a contribution): So expenditures coordinated with or controlled by the campaign - these are treated as contributions and can be regulated as all other contributions can be regulated. Expenditures that are coordinated with or controlled by camapgin - they tell u how to spend ur money to support them. Coordinated expenditures are treated as contributions and can be regulated as all contributions can be. o Independent expenditures cannot be regulated. o A coordinated expenditure is a disguised contribution (the campaign is in control) and be regulated as contributions can be regulated. Law banning judicial candidates from personally soliciting campaign contributions - in many states, judges are elected and there was statute prohibiting candidates for judgeships from personally soliciting campaign contributions - the law was upheld based on the appearance of corruption. Citizens united case - The constitutional protection of direct independent expenditures (that is you can make direct expenditures to support your point of view or the people who agree with you in amounts that are unlimited) applies not only to individuals, but also to corporations, including nonprofits, and unions. Citizens united case involved ideological group organized as a corporation that promoted mocumentary against a presidential candidate at the time - overtly political debate and court said it was constitutionally protected. • The Supreme Court has consistently rejected equalization of campaign resources as a valid rationale for restricting campaign expenditures. Most of people who want to put limits on campaign expenditures have as their essential reason - equalization - preventing person with lot of money from drowning out those who have less. Court has considered that and time and time again, court has rejected equalization of campaign resources as a rationale for restricting campaign expenditures. Idea that you want to restrict wealthy speakers in order to increase relative voice of others - that's consistently been a losing argument. Can see that in the corporate free speech case and it temps to give advantages to poorer candidates. Example 45: Congress passed the Millionaires' Amendment, which basically said that if your opponent spends a lot of his or her own money, then your contribution limit is automatically raised. So telling rich people if you spend your own money above a certain amount, then the rules for your opponent are changed. If you spend your own money above a certain figure, then contribution limits that are normally applicable don't apply to your opponent. That's restricting how much you can spend or making something depend on how much you spend in an effort to equalize the voice of the rich and poor. The Supreme Court struck it down. Court also struck down another scheme that said that public candidates who accept public financing can get extra money if their opponents outspend them. Point is equalization is a rationale that will appeal to many in society. But under case law, equalization is not a sufficient reason to restrict campaign expenditures. What is a sufficient reason? Corruption or the appearance of corruption. Equalization is not.

Fundamental Rights

• Triggers the strict scrutiny test under both due process and equal protection

Fundamental Rights

• Triggers the strict scrutiny test under both due process and equal protection 1. Due Process versus Equal Protection - - rule of thumb to distinguish between 2 o If a law denies a fundamental right to everyone, it violates due process. o If a law denies a fundamental right to only some but not others, it violates equal protection. But fundamental rights are the same - travel, etc and others below. 2. Travel o We have a fundamental right of interstate travel and settlement. Right to go to other states in country and right to settle there and become full fledged citizen. o States can impose reasonable residency requirements for political participation (requiring u live there for 30 days for example before you vote) or for government benefits. § Most reasonable residency requirements are 30-90 days § One year is too long for everything except in-state tuition and jurisdiction to issue a divorce. Once you become resident after 30-90 days, you have right to be treated same as all other residents. All residents have a right to be treated equally. A state cannot have a tax scheme that favors long-term residents over recently arrived residents. If youre resident of a state you have right to be treated the same as other residents of the state, without regard to your longevity in that state. 3. Voting and Ballot Access o Voting is a fundamental right to all citizens age 18 and over. Not right for noncitizens though. § Poll taxes are unconstitutional because they burden the fundamental right to vote. Taxes on exercise of ballot - tax paid to vote. § Short-term (e.g., 30 days) residency requirements are permitted. Anything over 90 days runs into trouble. § Congress controls the residency requirements for presidential elections. States control residency requirements for all other elections. o Ballot access - has looser rules. § States can impose residency requirements for candidates to be listed on a ballot, such as longer residency, filing fees, and nomination petitions, so long as serious candidates can reasonably comply. Don't want ballot to be so long or confusing or with so many fake names that no one knows what it means. So states can do what they want as long as requiremnets allow reasonable, serious candidates can comply. § If the requirements become so onerous that they effectively bar access to the ballot, then they are unconstitutional. 4. Privacy - amorphous concept - includes whats below: a. Marriage - There are all sorts of requirements for marriage (e.g., age or restriction on marrying close relatives), but substantial interference with the marriage—including same-sex marriage—is unconstitutional. b. Contraception - It is a fundamental right for everyone, whether married or not, to purchase contraceptives. Everyone has fundamental right to buy contraceptives - true for unmarried and married persons, and old and young people. c. Sexual intimacy - Perhaps not technically a fundamental right—the Supreme Court found that the government has no legitimate interest in regulating non-commercial sexual intimacy between consenting adults, including same-sex couples. Not quite same as sayig it's a fundamental right, but saying even if not fundamental right, govt regulation fails because govt has no legitimate interest in regulating non-commercial sexual intimacy between consenting adults and that includes same sex couples. d. Abortion § Roe v. Wade: A woman has a right to terminate her pregnancy until viability of the fetus. Viability means the fetus can live outside the womb. After that stage, restrictions can apply so long as there are exceptions to preserve the life and health of the mother. § States can regulate abortion in a variety of reasonable ways, but they cannot impose an undue burden on the woman's right to terminate her pregnancy. • Informed-consent requirements are allowed. That is you can require before person has abortion, that person be informed about opportunities available for carrying child to term, and having it adopted. Long lines of people who wish to adopt babies, etc. you can require certain info to be communicated, the informed consent requirement is allowed. • Twenty-four hour waiting periods are allowed - we don't want abortions to be performed upon spur of moment so can have a system requiring woman sign consent form and wait 24 hours for procedure incase she changes her mind. • Parental notification requirements (for minors) are allowed. Obviously not for adult females, but for minors. • Parental consent requirements are generally not allowed. Govt can require underaged minor to notify her parent of intention to have abortion - maybe gives parent opportunity to voice an opinion. But parent cant be given veto over girls decision to have abortion. Parents consent cannot be required. o Narrow exception for requirement that an underage female get the consent of a parent or a judge, but requires the judge to give consent if the underage female understands the nature of the act. Very elaborate way of allowing judicial override of parental veto. So Parental notification can be required, but parental consent cannot be required - must be way around a parental no. • Spousal consent requirements are not allowed. No law requiring consent of spouse has been upheld. • Government financing of abortion is not required. Govt never has to pay for abortions, even if it provides funding for live births. Govt can refuse to pay for abortions and can refuse to allow abortions to be performed in facilities it funds. Govt has to allow abortion but doesn't have to pay for it. e. Parental rights § Parents have a fundamental right to raise their children as they see fit, including the choice of religious or private schools. § Can lose their rights through abandonment, abuse, or neglect - until parents' rights are severed because of some kind of misconduct, parents have a fundamental right to raise kids as they see fit. f. Family relations - fundamental right to live with close relatives if they want you to. § Includes the right to live together with close relatives City adopted an extremely narrow single family zoning ordinance - nothing odd about having zoning ordinance limiting habitation to single family - most places have law like that. But city adopted a single family zoning ordinance so narrow that if you wanted to take ur nephew into household, u had to kick your grandmother out - they both couldnt fit in single family. Supreme court said that violates due process, u have a right to live with close relatives if they want you to. g. Obscene material § Fundamental right to read obscene material in the privacy of one's own home. § However, no fundamental right to purchase, sell, import, or distribute such material. § Right to possess obscenity in ones home does not apply to child porn. For child porn completely banned. h. Refusal of medical treatment § Not clear whether this is a fundamental right, but there is a liberty interest in refusing medical treatment § No fundamental right to commit suicide

quasi suspect classifications

• gender and legitimacy Trigger intermediate scrutiny—is the law substantially related to an important government interest? • Gender classifications are almost always invalid (e.g., Oklahoma had a law permitting women to legally drink alcohol at a younger age than men). Almost no gender distinctions are ever upheld anymore. o really only Permissible examples of gender classifications: § Statutory rape can be gender-specific (historically) - most statutory rape laws are gender neutral nowadays though but they don't have to be and supreme court said reason is because far greater risk of physical imposition or injury if underaged person is female; and § The draft - ok to draft men but not women for military service. This also slightly outdated now because women becoming more involved in the military. • Legitimacy (i.e., something depends on whether parents were married at the time of one's birth) laws are almost always invalid, especially if punitive in nature. Under this test most legitimacy classifications are invalid - most of older laws struck down seemed to impose some kind of disability on illegitimate as way of punishing the parents for having had an illegitimate child. Plainly not imp govt interest and laws of that character that have punitive motivation are struck down.

FILL IN THE BLANK. Congress has the power to regulate _______________ of interstate commerce. A the channels and the instrumentalities B the instrumentalities, but not the channels C the channels, but not the instrumentalities D neither the channels nor the instrumentalities

Correct Answer: A

FILL IN THE BLANK. Under the _____________ Amendment, Congress has broad power to legislate against private as well as public racial discrimination. A Thirteenth B Fourteenth C Fifteenth D Sixteenth

Correct Answer: A

FILL IN THE BLANKS. Congress ______________ the power to spend for the general welfare. Congress ____________ pressure states into acting through the conditioning of federal funding on taking such action. A has; can B has; cannot C does not have; can D does not have; cannot

Correct Answer: A

FILL IN THE BLANKS. Procedural due process applies only to ____________ governmental action and only when ________________ has been taken. A intentional; life, liberty, or property B intentional; a fundamental right C state; life, liberty, or property D state; a fundamental right

Correct Answer: A

Procedural Due Process—the Right to Notice and a Hearing

1. Consists of two questions: 1) Is life, liberty, or property being taken? 2) If life, liberty, or property is being taken, what process is due? Some process must be due.

Privacy - amorphous concept - includes whats below:

1. Marriage 2. Contraception 3. Sexual intimacy 4. Abortion 5. Parental rights 6. Obscene material 7. Refusal of medical treatment

Jurisdiction of the Supreme Court

1. Original Jurisdiction A case may be filed first in the Supreme Court (controversies between states, mostly and can be filed originally in sc.). 2. Appellate Jurisdiction Mostly, the jurisdiction of the Supreme Court is appellate. This jx includes certiorari. a. Certiorari § Almost all cases come to the Supreme Court by way of writ of certiorari. § The key factor is that granting a writ of certiorari is discretionary with the court. Internally they operate on rule of 4 - if 4 justices agree its enough itll be heard by the court. by that judgment can be ad hoc, sometimes practical, sometimes even political discretionary judgment. § The Supreme Court is the only federal court that exercises discretionary jurisdiction. Limitations on the Supreme Court's appellate jurisdiction § Congress can make exceptions to the supreme Court's appellate jurisdiction (i.e., Congress can effectively control the Supreme Court's docket (its appellate jurisdiction) by legislating exceptions to its appellate jurisdiction).=

FILL IN THE BLANK. The Fifteenth Amendment specifically gives Congress the power to eliminate racial discrimination in _____________. A Housing B Employment C Education D Voting

Correct Answer: D

exception to the judicial power

A major exception to the judicial power is created by the eleventh Amendment and by the preexisting concept of state sovereign immunity. § Rule: You cannot sue a state for money damages in either state or federal court unless the state consents or the U.S. Congress expressly says so to enforce fourteenth Amendment rights. • Note that the Eleventh Amendment protects states and state agencies, not local governments (i.e., no immunity for cities, counties, or towns). • Note also that a state's sovereign immunity applies in both federal and in state court, unless the state consents (state can waive immunity), or Congress expressly says so to enforce Fourteenth Amendment rights. o When enforcing individual rights, Congress can override state sovereign immunity. o It can force states to pay money damages for violating individual rights, but it must say so expressly. o Any lack of clarity in congressional command will preclude damages.

THE JUDICIAL POWER

A. Source, Scope, and Limitations • Source: Article III • Scope: The jurisdiction of federal courts is limited to cases or controversies. • Limitations: The Eleventh Amendment and State Sovereign Immunit

Interbranch Relations

Congressional Limits on the Executive a. Impeachment § Applies to executive officers (e.g., President, Vice President, cabinet officers, federal judges) can be removed from office by impeachment § An accusation passed by the House of Representatives requiring a majority vote. § Once impeached, the person is tried in the Senate. § Conviction requires a two-thirds vote of the Senate. § Impeachment and conviction lead to removal from office. No other penalty applies. b. Impoundment § If a statute gives the President discretion to spend or withhold funds, he may do so. § But, if a statute unambiguously requires that certain funds be spent on certain purposes, then the President cannot refuse to do so. There is no power to impound (withhold) funds simply because he disagrees with congress's decisions. c. Legislative Veto § It is unconstitutional; § It arises when Congress passes a law reserving to itself the right to disapprove future executive actions by simple resolution. That's unconstitutional - why? § If Congress wants to override executive actions, it must change the law (so that the President has an opportunity to veto the new legislation). To change the law, it must enact a new statute. And how to enact new statute? Must be passed by house of reps and passed by senate and presented to president for his veto. And if president doesn't like the new law he can veto it and congress has to override by 2/3 of each house. So problem with so called legislative veto - problem with congress trying to set up scheme where it can override executive actions by simple resolution, problem is it cuts out constitutionally guaranteed veto opportunity of the president. And avoiding constitutionally guaranteed veto opportunity of president makes it unconstitutional. § Congress cannot evade the President's guaranteed veto opportunity by passing a law saying that in the future it plans to govern by resolution.

FILL IN THE BLANKS. Under the Commerce Clause,Congress may regulate intrastate activities that have _____________ effect on interstate commerce. This effect is judged ______________. A any; individually B any; in the aggregate C a substantial; individually D a substantial; in the aggregate

Correct Answer: D

In applying the "adequate and independent state grounds" limitation on the U.S. Supreme Court's review of a state court decision, which of the following is TRUE? A The U.S. Supreme Court will not review a state court decision when the party claiming a federal right is successful under state law. B The U.S. Supreme Court will not review a state court decision if the state court judgment turns only on federal grounds. C The U.S. Supreme Court will not review a state court decision that turns on state law if state law adopts or follows federal law. D When a state court decision is unclear as to whether it rests on federal grounds or state grounds, the Supreme Court cannot review the federal issue.

Correct Answer: A

When a party obtains an unfavorable judgment from a state's highest court, that party is unable to obtain review of that judgment by the U. S. Supreme Court if the judgment rests on which of the following? A Adequate and independent state grounds B Adequate and independent federal grounds C Necessary and proper state grounds D Necessary and proper federal grounds

Correct Answer: A

Which of the following accurately describes the relationship between Congress and the Executive? A Congress can delegate its power to administrative agencies so long as there are intelligible standards governing the exercise of that delegated power. B By a two-thirds majority vote, the House of Representatives can impeach the President. C Congress can retain a legislative veto over executive action. D Congress cannot specifically require the President to spend funds.

Correct Answer: A

For which of the following is the government required to provide a pre-deprivation hearing? A Termination of disability benefits B Termination of welfare benefits C Terminating a government employee who is employed "at will" D Dismissing students for academic reasons

Correct Answer: B

In which of the following does a person NOT have a liberty interest? A Loss of parental rights B Reputation C Commitment to a mental institution D Free speech

Correct Answer: B

Regarding Congressional power under the Fourteenth Amendment to remedy violations of individual rights by the government, there must be a _________________________ between the injury to be remedied and the means adopted to achieve that end. A federal connection B congruence and proportionality C necessary and proper link D perfect fit

Correct Answer: B

Regarding standing, which of the following is FALSE? A The type of injury suffered by the plaintiff need not be economic. B The threat of future injury can give a plaintiff standing to seek damages. C The type of injury must be concrete. D Congress can create new interests, the injury to which may establish standing.

Correct Answer: B

Regarding the right to travel, which of the following is TRUE? A The right to travel from one state to another is not a fundamental right. B The right to travel internationally is not a fundamental right. C A state may not impose residency restrictions on the entitlement to governmental benefits. D In its tax scheme, a state may distinguish between residents on the basis of the length of their residency.

Correct Answer: B

The imposition of a penalty on individuals who fail to buy health insurance is supported by which of the following powers of Congress? A Spending Power B Taxing Power C Commerce Power D General Police Power

Correct Answer: B

Which of the following statements correctly describes the application of the Equal Protection Clause of the Fourteenth Amendment? A Directly to the states and the federal government B Directly to the states and indirectly to the federal government through the Fifth Amendment Due Process Clause C Directly to the federal government, and indirectly to the states through the Fifth Amendment Due Process Clause D Indirectly through the Fifth Amendment Due Process Clause to both the states and the federal government

Correct Answer: B

With regard to the Eleventh Amendment prohibition, which of the following is TRUE? A The prohibition against bringing suit may not be waived by a state. B The prohibition can be overridden by Congress in enforcing rights created by the Fourteenth Amendment. C The prohibition applies only in state court. D The prohibition extends to suits against a local government.

Correct Answer: B

With regard to the _____________ of certain high- level federal officials (e.g., cabinet officers, judges) by the president, the approval of the _______________ is required. A appointment, Congress B appointment, Senate C appointment or removal, Congress D appointment or removal, Senate

Correct Answer: B

FILL IN THE BLANKS. In order for a case to be ripe, the controversy must involve _________. Mootness generally results in the dismissal of a case unless the controversy is _________. A actual harm only; capable of repetition yet evading review B actual harm only; subject to a statute of limitations C actual harm or the threat of immediate harm; capable of repetition yet evading review D actual harm or the threat of immediate harm; subject to a statute of limitations

Correct Answer: C

FILL IN THE BLANKS. Under the Fourteenth Amendment,Congress __________________ redefine or create new individual rights. Thisamendment _________________ Congress the power to remedy violations of individual rights. A can, gives B can, denies C cannot, gives D cannot, denies

Correct Answer: C

Regarding the Comity Clause of Article IV, which deals with privileges and immunities of state citizenship, which of the following is TRUE? A This clause protects all out-of-state individuals and entities against discrimination by a state. B This clause prevents discrimination with respect to any type of license issued by the state. C This clause prevents a state from imposing a residency requirement for private employment. D This clause protects against any form of discrimination by a state against out-of-state individuals.

Correct Answer: C

Regarding the strict scrutiny standard of review for substantive due process cases, which of the following is FALSE? A The government bears the burden of proof. B The governmental interest must be compelling. C The law must be substantially related to the governmental interest. D The government must be using the least restrictive means necessary to protect the governmental interest.

Correct Answer: C

The President may NOT do which of the following? A Veto congressionally approved legislation B Appoint executive officers C Pardon persons convicted of state crimes D Remove executive officers

Correct Answer: C

In determining the procedures to which a person who is entitled to due process should receive, which of the following is NOT considered? A The importance of the individual interest B The cost to the government in providing the procedures C The value of the procedure in protecting the interest D Whether the governmental actor is the federal government or a state

Correct Answer: D

In which of the following circumstances may a state NOT discriminate against out-of-state commerce under the Dormant Commerce Clause? A The state is buying or selling goods as a market participant. B The state is subsidizing its own citizens. C The state is regulating a particular area of commerce with Congressional consent D The state is regulating conduct that occurs wholly beyond its borders.

Correct Answer: D

Regarding the powers of the federal and state governments, which of the following is FALSE? A Under the Supremacy Clause, federal law supersedes state law when the laws conflict. B The Constitution grants some powers, such as the power to coin money, exclusively to the federal government. C A state may not shield state officials from federal liability. D Federal and state governments are generally immune from direct regulation by the other.

Correct Answer: D

Regarding the right to vote, which of the following statements is FALSE? A Residency requirements on the right to vote are permissible. B Age restrictions on the right to vote are permissible. C A fee may not be imposed on the right to vote. D Restrictions on ballot access can be no greater than those imposed on the right to vote.

Correct Answer: D

Regarding treaties and executive agreements, which of the following statements is FALSE? A Once ratified by the Senate, a treaty has the same authority as a federal statute. B A treaty must be ratified by a 2/3 majority of the Senate. C Executive agreements do not require Senate approval. D Executive agreements do not take precedence over conflicting state laws.

Correct Answer: D

Which of the following is NOT a fundamental right for purposes of substantive due process? A The right to travel B The right to vote C The right to privacy D The right to citizenship

Correct Answer: D

Which of the following statements regarding justiciability is FALSE? A A federal court will not rule on a matter in controversy if the matter is a political question. B Federal courts may not render advisory opinions. C A federal court will not rule on the President's conduct of foreign affairs. D A federal court can issue a declaratory judgment on the constitutionality of proposed legislation.

Correct Answer: D

Examples of entitlement as property right

Example 25: You are hired as the city planner and the boss says, "This job has a term of five years. You will not be fired during that five-year term except for cause." You have a property right in that job for five years. You have entitlement to job for 5 yr term. But if u go for job interview and boss says this is political appointment - you'll be my special assistant and u serve at will. If they tell you that you can be fired at any time and for any reason, u have no entitlement to hold position, u can be fired any time and for any reason. Example 26: Barnes was an assistant professor at a state community college. He had a three-year teaching contract and under state law, five years was required for tenure. At the end of three years, his contract was not renewed, and he sued. What is his strongest argument that he should get notice and a hearing? A. He was the best teacher in the school. (no ur not entitled to job just because youre good at it) B. He was the only teacher not rehired. (so what? Youre not entitled to job just because your friends have one. If barnes can show though that he wasn't rehired because of race or religion or gender, then he woud have claim. U have constitutional right not to be discriminated against based on race, religion, or gender, but if just that other teachers were rehired and he was not, that doesn't do it.) C. He was so sure of success that he moved his elderly parents to town to care for them. (that's not property, doesn't create entitlement) D. He had an oral promise of re-employment from the president of the college. (maybe not winning argument, but right answer because strongest argument) Answer choice D is the correct answer because the president is acting as the government, and arguably Barnes had an entitlement to continued employment. President, someone in charge, told him hed be reemployed and that would tend to show legitimate entitlement. What is the strongest argument against Barnes receiving notice and a hearing? Answer: He had a three-year contract that ended. School said 5 years required for tenure, you've only been here for 3, so you cant be entitled before you've been here for 5 years.

One Equal Protection Clause

Found in the 14th Amendment o Applies to localities and states Technically no equal protection clause applicable to the federal government. So do feds get free ride for equal protection? No they don't. they have to follow equal protection concepts and principles and equal protection law but they apply to fed govt under 5th amendment due process clause. That is, equal protection for fed government is called due process - same standards and content, but a different label. Note 4: Though no Equal Protection Clause guarantee technically applies to the federal government, equal protection concepts are applied to the federal government via the Due Process Clause of the Fifth Amendment. Therefore, states and localities have both equal protection and due process; for the federal government, equal protection and due process are called Fifth Amendment Due Process. For national govt only have one clause - due process - and everything applies to national govt as due process. for states and localities have 2 clauses - due process clause and equal protection clause. How to distinguish the two? Overlap concerns fundamental rights - if law denies a fundamental right to everyone in a state or a locality, it violate due process. if law denies fundamental right to some persons but not others, it violates equal protection. Distinction made at level of state legislation and local ordinance. For national govt, its all due process.

In theory, the federal government is..... but.....

In theory, the federal government is a government of limited powers. In fact, the federal government can do almost anything it likes, so long as it does not violate individual rights

No undue burdens on interstate commerce

Non discriminatory regulation of commerce simply means in state interests and out of state interests are treated alike - that's non discriminatory. § Non-discriminatory state regulation of commerce is almost always upheld. § Only when it is so outrageously costly relative to the benefits of the regulation is a non-discriminatory state regulation struck down as an undue burden on interstate commerce. § This is a balancing test, but non-discriminatory regulations are rarely struck down.

Deprivation

Notice and a hearing are not required when there is an accident. Random negligence by a state employee does not constitute a deprivation of life, liberty, or property (e.g., death by a municipal garbage truck). Deprivation requires the intentional taking away of life, liberty, or property. What is deprivation? Maybe youre run over by city owned garbage truck - Garbage truck might kill you in which case you lost life, garbage truck might injure you in which case you've lost liberty, garbage truck might just tear up clothes in which case you lost property. Are you entitled to notice and hearing to protect against accidents with garbage truck? Defs not, random negligence by a state employee does not constitute a deprivation under due process. if accidentally injured by govt employee may have a tort claim. But you don't have constitutional violation unless govt intentionally takes life, liberty, or property from you.

key errors to avoid for fed statute question

On the Multistate exam, you'll be given a federal statute, and you'll be asked to identify the strongest argument in favor of congressional power for passing that legislation. Three wrong answers - pick answer that fits legislation in question. Usually very easy question. But three key errors to avoid: i) Promoting the general welfare is not a power of Congress - never pick this answer, its always wrong; ii) The federal government, unlike states, does not have a general police power - doenst refer narrowly to law enforcement but refers broadly to any govt action serving the public good. But states have a general police power which means they have unlimited legislative power but the national government doesn't have general police power; iii) also necessary and proper is not a free-standing power of Congress. Its an addon - addition to some other legislative power. It works only as an add-on to some other legislative power. Never pick necessary and proper by itself. Right answer might by necessary and porper to regulate interstate commerce or necessary and proper to protet individual rights or to coin money or to maintain post office or to have army and navy, but never just necessary and proper by itself.

14th Amendment

Other individual rights apply to states and localities through the 14th Amendment, which requires state action. If state violates free speech, correct thing is to say they've violated ur 1st and 14th amendment rights. If city conducts illegal search theyve violated ur 4th and 14th amendment rights. Virtually all individual rights enforced through the 14th amendment, which requires state action. • "State action" means government action, whether state or local (e.g., a county firing a sheriff is state action; a city denying a permit is state action).

EQUAL PROTECTION - General Considerations

Privileges or Immunities of National Citizenship under the 14th Amendment—means nothing today (so it is never the correct answer on the MBE) Distinguish privileges and immunities of national citizenship under 14th amendment - means nothing, always wrong answer vs. privileges and immunities of state citizenship under article 4 - sometimes called comity clause. Note 3: Privileges and Immunities of State Citizenship Clause in Article IV (Comity Clause) has a narrow but important meaning. It prohibits serious discrimination against out-of-state individuals, especially in the context of access to the private job market, i.e. prohibits requiring those who work in the state to live in the state.

Standards of Review - substantive due process and equal protection

Standards of Review • Both substantive due process and equal protection have the same three standards of review. 1. Strict Scrutiny o Is the law necessary for a compelling government interest? o Implicit in strict scrutiny is the requirement for the least restrictive means. If some less restrictive way for govt to protect its interest the means chosen is not strictly necessary. o When strict scrutiny applies, the government bears the burden of proof. The government must show that the interest is compelling and the means chosen (or law) is necessary to that interest. o Applies when there is a suspect classification or a fundamental right strict scrutiny does require least restrictive means to be used and yes, burden of proof on govt. 2. Intermediate Scrutiny o Is the law substantially related to an important government interest? o Applies in equal protection doctrine to classifications based on legitimacy and gender. Legitimacy and gender trigger intermediate scrutiny. Note 2: While three standards of review are available in both substantive due process and equal protection claims, intermediate scrutiny has only ever been used to decide cases based on equal protection grounds. 3. Rational Basis - fall back test - "mere" rationality test o Is the law rationally related to a legitimate interest or end of government? o The challenger bears the burden of proof (burden of showing the law is not rationally related to legit interest). o Low burden of proof. - rational basis test is hard to flunk - law doesn't have to be sensible or wise or humane or appropriate - just has to be non-insane. o Applies to all other cases (residual test) govt can use one interest in passing law and then defend law on different grounds.

3 big powers of congress

Taxing, spending, and commerce. The general rule for the bar exam: i) Pick the taxing power when the law involves a tax - if fed tax and asking why they can do it, yes they can do it because they have the taxing power; ii) Pick the spending power whenever federal money is spent or disbursed - if fed spending appropriation or fed expenditure of fed money, spending is the right argument; iii) When in doubt, pick the commerce power - if no tax and no money pick commerce. Example 8: There is a federal law that states that every state that receives federal highway funds has to limit its speed limit to 65 mph. What is the strongest argument Congress has to accomplish this? Answer: The taxing and spending clause. Why? Congress said every state receiving highway funds. What if the law simply caps the highway speeds at 65 mph without saying anything about federal highway funds? Answer: doesn't involve tax and any money so probably Regulation of interstate commerce.

The Taxing and Spending Power

Think of these as separate powers of Congress even though they are lumped together in one clause. a. Taxing § The Taxing Clause is the right answer whenever Congress imposes a tax, even when the tax is actually used to prohibit the good or activity in question. Taxing is right even if tax plainly designed to suppress or prohibit good or activity involved. § The tax need only be rationally related to raising revenue. Example 14: 2012 Affordable Health Care case—the individual mandate was not sustained as a regulation of interstate commerce, but it was held to be valid under the taxing power. Why valid under taxing power? The individual who chooses not to buy health insurance must pay a tax, and the taxing power supports anything that is rationally related to raising revenue. b. Spending § The Spending Power includes spending for the general welfare. Promote general welfare is not power of congress and is always wrong answer, but spending money to promote general welfare is part of spending power and that can be right answer. § Congress can use the Spending Power to accomplish things it could not do by direct regulation under the Commerce Clause. Example 15: South Dakota v. Dole—The 21st Amendment repealed Prohibition and gave states the power to control the regulation and consumption of alcohol. As a result, Congress likely cannot directly regulate the drinking age in the states. Thus, in order to pass a 21-year-old drinking age law, Congress passed a "bribe." If states wanted to receive federal highway funds, they had to raise their drinking age to a minimum of 21 years of age. States all fell into line because they were bribed to do so by federal highway subsidies - that's spending power and congress can do that under spending power whether or not it could do it as regulation of interstate commerce.

Immunities

a. The President § Has absolute immunity from liability for official acts (broadly construed) - does not take account of possibility of impeachment. If president impeached and convicted then president no longer president. And a non president doesnt have immunity. But impeachment and conviction necessary to overcome absolute immunity for official acts. § President has no immunity for acts done prior to taking office § Has an executive privilege not to reveal confidential communications with presidential advisers, but that privilege can be outweighed by a specifically demonstrated need in a criminal prosecution (U.S. v. Nixon) - supreme court said yes Nixon had executive privilege not to reveal things that were confidential within the executive branch but specifically demonstrated need in an ongoing criminal prosecution outweighed that privilege. b. Judges - Judges have absolute immunity for all judicial acts, but may be liable for non-judicial acts (e.g., employment discrimination). c. Legislators § United States Senators and Representatives (not state legislators) are protected by the speech or debate Clause. Speech or debate clause doesn't give congressman constitutional right to take bribes, it gives them immunity against certain kinds of evidence - rule below. § Senators and Congressmen and their aides cannot be prosecuted or punished in relation to their official acts. That means the official act cannot be introduced into evidence to show that congressman was a crook. If you can prove dishonesty some other way, maybe having fake arms dealer wiretap congressman or wear wire during deal, some other way is fine. But cannot introduce into evidence in court how rep voted or whether she introduced a bill or anything said on the floor of house or senate or anything said in an official committee hearing. All of those are official acts and they cannot be used in evidence - that's meaning of speech or debate clause. • The official acts of a federal legislator cannot be introduced into evidence.

How does Supreme Court respond after taking up case?

o If the Supreme Court agrees with the state court's decision of federal law, it affirms the decision. o If the Supreme Court disagrees with the state court's understanding of the federal issue, it remands the case to state court, so that the state court can reconsider state law. Sc can take case and review federal search and seizure issue. if court decides search violated fed law, it simply affirms decision of state court suppressing evidence. But if supreme court finds fourth amendment doesn't require suppression of evidence and search is ok under federal law, it remands the case to the Michigan supreme court. If supreme court disagrees with state court, it remands for further proceedings. Why is that true? Because state court then has opportunity to reconsider state law question. Once fed issue clarified, remand to state court so it can consider state law. Example 2: Michigan v. Long: The defendant moved to suppress evidence under both the Fourth Amendment and the analogous search and seizure provision of the Michigan Constitution. The highest state court ruled in the defendant's favor, holding that the evidence must be suppressed, but cited state cases and federal cases indiscriminately (i.e., it did not make clear whether it was illegal as a matter of state law or federal law, or both). The Supreme Court took the case, reviewed the federal issue, found that the search was valid, that the evidence could lawfully be used under the Fourth Amendment, and remanded the case to the Michigan Supreme Court to reconsider whether the search was lawful under the state constitution. Why does it remand? Now that fed issue is clarfieid still upto state court to determine whether search violated state law. Exam Tip 3: Again, remember that the U.S. Constitution is a floor, not a ceiling. State law can always provide more rights than fed law requires, but can never give less.

Breakdown of loss of life, liberty, and property

o Life: Death penalty requires procedural due process o Liberty: Physical confinement, probation and parole, physical injury (such as a spanking in school), any restriction on legal rights (including being punished for free speech). Even someone on parole or probation has limitations on where they can go, etc. so limitations on them so procedural due process is required. Physical injury is also loss of liberty - if school child is spanked he has loss of liberty - not much procedural due process but physical injury is still loss of liberty. Any restriction on ur legal righs is also loss of liberty - if ur punished for exercising free speech, if some adverse action taken against u by govt because u belong to wrong party or u wrote letter to edior or some other exercise of free speech, uve lost liberty. Injury to reputation is not a loss of liberty. Without more, it's not loss of liberty. If prosecutor says were really keeping our eye on you because we think ur engaged in illict activities, that statement may damage ur reputation, conceivably maybe defamation claim, but its not loss of liberty and doesn't trigger procedural due process. o Property Exam Tip 10: Out of the three, property is the most heavily tested. Most of the questions in this area concern government jobs or benefits. Most property is clear - owning house or car, etc. if they take that stuff, they must give u procedural due process. what about govt jobs and benefits? Can u have property right in govt employment or some kind of govt benefit? Answer is yes. § You have a property interest in your government job or benefit whenever you have a legitimate entitlement to continued enjoyment of the job or benefit. § A mere expectation of continued employment or benefit does not suffice. § Most government benefits, such as welfare, Medicaid, social security, are entitlements, and hence property. Statutes say if ur this old or this income level , or meet this criteria ur entitled to this benefit - that's property interest and due process is required if they take it away. Jobs however are different - u may have entitlement to job in which case u ave property interest but mere expectation of continued employment - like ur expectation that u keep working for govt isn't enough. How to tell whether its entitlement to govt job or mere expectation? Entitlement is property, mere expectation is nothing. § How can you tell whether you have an entitlement to a government job or whether you have a mere expectation of government employment? • Government jobs are entitlements only when the government says so—such as by providing a contractual term or discharge only "for cause."

Dormant Commerce Clause

o More important than Privileges and Immunities of Article IV because it protects out-of-state businesses as well as out-of-state individuals. Therefore has much broader application. o "Dormant" describes what the Commerce Clause means in the absence of federal regulation (when the federal commerce power is unexercised). A state has no power to regulate commerce in the face of federal regulation - state has no power to regulate commerce in a way that congress has forbidden. Question is what happens when congress does not act? What happens when no federal law on point? What happens in absence of fed regulation? o Rule: In the absence of federal regulation, state regulation of commerce is valid so long as: 1) There is no discrimination against out-of-state interests; 2) The regulation does not unduly burden interstate commerce; and 3) The regulation does not apply to wholly extraterritorial activity. State regulation of commerce in the absence of federal regulation is okay so long as it meets requirements above.

Rational basis test

o Rational basis test is generally easily passed. Almost everything survives review for mere rationality. However, recent Supreme Court cases have made exceptions in the field of sexual orientation. § In the 2003 Lawrence case, the Supreme Court struck down laws criminalizing homosexual sodomy using language suggesting the laws did not pass the rational basis test as they reflected prejudice, not a legitimate reason for the laws. § In the 2013 Windsor case, the Court used the same approach in striking down the federal Defense of Marriage Act (DOMA). The Court applied a version of the rational basis test "with teeth" suggesting that because the motivation behind the federal DOMA was merely animus and prejudice, the law was not a rational plan of government. o Laws against sexual orientation might be struck down as irrational because they are not supported by reasons other than mere prejudice. Rational basis test in general is hard to flunk - in general almost anything goes under the test but there is an exception in area of sexual orientation where court has now repeatedly applied rational basis with teeth - rational basis with rigor - rational basis which begins to look something like strict scrutiny

Travel

o We have a fundamental right of interstate travel and settlement. Right to go to other states in country and right to settle there and become full fledged citizen. o States can impose reasonable residency requirements for political participation (requiring u live there for 30 days for example before you vote) or for government benefits. § Most reasonable residency requirements are 30-90 days § One year is too long for everything except in-state tuition and jurisdiction to issue a divorce. Once you become resident after 30-90 days, you have right to be treated same as all other residents. All residents have a right to be treated equally. A state cannot have a tax scheme that favors long-term residents over recently arrived residents. If youre resident of a state you have right to be treated the same as other residents of the state, without regard to your longevity in that state.

Doctrine of Political Question

political question is non-justiciable • A non-justiciable question is when it is inappropriate for judicial resolution - why? Usually because issue is committed to another branch of govt for decision or maybe because no judicially manageable standards for adjudication. o The issue is committed to another branch of government for decision; o There are no manageable standards for adjudication. • Examples of famous non-justiciable political questions include: o Guarantee Clause (protecting the republican form of government) - sc said many yrs ago that guarantee clause was non justiciable - something for congress to figure out so could not be litigated; o Foreign affairs, e.g., opening or breaking off diplomatic relations with another country - presidents authority to open or break off diplomatic relations with another country, that's a political question and courts wont decide if presidents action was proper or not; Example 7: Congress enacted a law stating that American citizens born in Jerusalem could list Israel as their place of birth on their passports, which the executive branch challenged. The Court held that a dispute over the regulation of passports is somewhat related to foreign affairs but is nonetheless justiciable. o Impeachment procedures - impeachment is accusation made by house of reps by simple majority vote - one person impeached theyre tried and trial occurs in senate and requires 2/3 vote to convict and impact of impeachment or effect of it is removal from office. Once impeached then crim prosecution possible, but only remedy for impeachment is removal from office. Generally non justiciable political questions involved with impeachment like how much testimony required, etc etc o Political gerrymandering (drawing districts to establish a political advantage of one party over another). § Political gerrymandering can violate equal protection - if question about is there constitutional problem with political gerrymandering - answer is yes it can violate equal protection; but § on the other hand, The court has found no judicially manageable standards for determining what is and what is not acceptable in the area of drawing districts. Court has said we as courts don't know how to draw distict correctly so don't know how to remedy the wrong and therefore correction of political gerrymandering is beyond our capacity - no judicially manageable standards. So in theory political gerrymandering may be unconstitutional but in fact its tolerated and supreme courts reasoning is that political gerrymandering is a nonjusticiable political question because of lack of judicially manageable standards for resolving it.

Timing of the hearings

§ Sometimes, a hearing must occur before the deprivation or something taken away. So pre-deprivation hearing. But sometimes evidetinary hearings may be allowed after something taken away so long as they are prompt and fair. Pre deprivation hearings apply to terminating welfare benefits, and to non emergency revocations of drivers licenses. But for many other govt actions, hearings can come after action taken so long as hearing is prompt and fair. • Examples include: Terminating welfare benefits; non-emergency revocations of driver's licenses. § Sometimes, the hearing can occur after the action, so long as the hearing is prompt and fair. Usually okay to act first and give hearing later but hearing must be prompt and fair - cant be hearing down the road several months and cant be hearing not giving u fair shot at prevailing. • Examples of major actions for which post deprivation hearings are allowed so long as they are prompt and fair include: Terminating disability benefits; disciplinary suspension from a public secondary school. Question of pre or post deprivation hearing is resolved by same 3 factor balancing test above - weigh individual interest at stake, and the value of a pre termination hearing - value of having hearing before termination - against govts interest in efficiency. o Public employees who can be fired only "for cause" must be given some opportunity to be heard prior to discharge, unless there is a signficiant reason not to keep the employee on the job. Public employees who can be fired only for cause have tenure - doesn't apply to just university professors, but to every public employee who has job security. People who have job security typically msut be given some oppornity to heard - so not necessarily full scale trial hearing but some opportunity to be heard prior to discharge. However if signficiant reason not to keep employee on job employee can be immediately suspended without pay. Signficiant reason not to keep employee on job - employee can be immediately suspended without pay, so long as theres a prompt post suspension hearing and so long as there is reinstatement with back pay to make employee whole. If there is a significant reason not to keep a person on the job, then the discharge can come first with a subsequent hearing that is prompt and provides reinstatement with back pay (fair). Example 27: A police officer charged with a crime can be suspended immediately because there is a significant reason not to keep criminally charged officer on the job. The hearing can come afterwards, but it must be prompt and provide reinstatement with back pay if the charges are ultimately unfounded. May be union agreements that limit police department freedom of action but only about constitutional law for purposes of the problem.

Fourteenth Amendment

• Congress has the power to remedy violations of individual rights by the government, but only as those rights have been defined by the courts. o Does not enable Congress to redefine constitutional rights by legislation o Only permits Congress to adopt remedies reasonably designed to enforce individual rights as they have been defined by the courts. • To be properly remedial, the legislation must have "congruence" and "proportionality." That is, there has to be a reasonable fit or reasonable closeness between the remedial law enacted by Congress and the constitutional right as defined by the Supreme Court. this is subtle concept and not either simple nor obvious - best way to explain is with example in next slide.

Federal and State Powers

• Even though federal powers are superior, most federal powers are concurrent with those of the states. (On most topics, both Congress and the states have regulatory powers. If there is a conflict, Congress wins.) concurrent means most things congress can regulate can also be regulated by states. If conflict congress wins, but if no conflict, both state and national govt have concurrent power over most matters. • Some powers are exclusively federal. They include the power over foreign relations and the power to coin money. States don't have foreign relations and cant enter into treaties with foreign powers. They cant freelance in international arena just because they disagree with washingtons policies. And states also cannot coin money. But generally state and fed powers are concurrent.

Powers of the President—Domestic

• Has the power to enforce the law, not to make it or break it • The power to enforce is greatest when authorized by statute. Generally speaking, the President's powers are subject to control by statute. • Few powers that are exclusively executive and, therefore, not subject to statutory control: o Pardon Power: The President can pardon or commute punishment for any and all federal offenses. (Governors have a similar power for state crimes.) This power cannot be limited by Congress. Congress cannot deny or overrule effect of presidential pardon. o Veto Power: The President has 10 days to veto legislation. The President can veto for any reason or no reason, but cannot veto specific provisions in the legislation and accept others. Veto power is all or nothing. § Overriding a veto requires a 2/3 majority vote of each house for congress to override veto. o Appointment and Removal of Executive Officers: Only the President (or his appointees) can hire or fire executive officers. Some senior officers (e.g., cabinet officers, ambassadors, federal judges) require the advice and consent of the Senate to be appointed. The Senate has a power of rejection though - can refuse to appoint or confirm a fed judge whos been nominated or refuse to confirm cabinet nominee but it cannot appoint judges itself nor can it fire executive officers save by elaborate process of impeachment. The Senate's approval power does not translate into a power of appointment. § Who are executive officers? Anyone who takes action on behalf of the U.S. executive officers must be hired and fired within executive branch. Who does that not include? Whom can congress hire and fire? People who don't act. Congress can hire and fire People who investigate, advise, and recommend. All kinds of commissions and monitors, people who do studies and recommend action - but if person takes action binding the U.S., that person is executive officer and must be hired and fired (impeachment aside) within executive branch. § Just as Congress cannot hire or fire an executive officer, it cannot give executive power to any officer it can hire or fire. Exam Tip 8: The question tells you that a certain officer is appointed by Congress (e.g., the Speaker of the House), and is therefore under legislative control. Then, someone proposes legislation that gives the officer executive power to act on behalf of US. Once the officer is under legislative control, any legislation that attempts to give the officer executive power is unconstitutional. Congress cannot hire or fire executive officers, also cannot give executive power to any officer it can hire or fire.

State Action Discrimination

• Most state action is clear, but not always. Government cannot be significantly involved in private discrimination but what does that mean? Means govt cannot ... o Significant state involvement: § Government cannot facilitate private discrimination. § Government cannot profit (or be financial partner profiting from) from private discrimination. § Government cannot enforce a private agreement to discriminate. § But, government is not required to prevent private discrimination. o Generally, the government acts constitutionally so long as its own conduct is neutral and even-handed (e.g., trespass laws, liquor licenses for private clubs). Trespass laws - these protect landowners of all races, creeds, colors, religions, and political views against trespassers of all races, colors, creed, religions, and political views. Landowner might be prejudiced in asserting who can come on his property but enforcement of trespass laws is fine as long as govts conduct is neutral and even handed - so long as they enforce laws equally for everyone. Moose lodge - discriminated on basis of race In its membership. Liquor license - moose lodge is private entity and not govt so no state action but p claimed because moose lodge got liquor license it made govt responsible for discrimination by moose lodge. Turns out liquor license is not incidental feature of what happens at moose lodge but very central. So argument that when govt gave liquor license enabling private club to thrive, govt became responsible for racial discrimination. But this is wrong argument. So long as govt gives liquor licenses to all white/black clubs, all male/female clubs, so long as it treats al liquor licensees the same, govts conduct is constitutional. Govt acts constitutionally so long as its own conduct is neutral and even handed - no constitutional obligation to prevent discrimination by private parties. • Anti-Discrimination Statutes o State action is required to show a violation of the Constitution. o State action is irrelevant if there is anti-discrimination legislation. Why? Legislation isn't limited to controlling what govt itself does - legislature most of time has power to regulate purely private conduct. If legislation says u cant discriminate on basis of race and private employment, u cant discriminate on basis of race and private employment. Not because its unconstitutional - theres no state action, but because private discrimination is illegal because anti discrimination statute says so and u have to obey the law. State action is a general requirement for all rights except 13th amendment right against slavery - all other rights enforced by 14th amendment and require state action.

Timeliness (Ripeness and Mootness)

• Ripeness concerns prematurity of a case - cant litigate hypothetical controversy, one that might arise if if if. You must show actual harm or an immediate threat of harm. • Mootness cases are overripe and are dismissed whenever they become moot. Cases can become moot during trial or on appeal. o Exception: Controversies capable of repetition, yet evading review, are not moot, even though they look like it. These issues are easy to spot -always there is internal time limit. Something about nature of problem that will not last long enough for case to make its way through the courts. § Such controversies always have an internal time limit (e.g., pregnancies and abortion). Eg: restrictions on abortion - pregnancy lasts only 9 months - no pregnancy will last long enough for a case to go all the way to sc. But pregnancy can recur and issue of abortion can come up again - if case was dismissed as moot every time pregnancy was resolved the issue would simply cycle and never be finally resolved. Capable of repetition because it will come up again but evading review because it wont last long enough to be finally decided. Those cases look moot but theyre not.


Kaugnay na mga set ng pag-aaral

Chapter 44: Loss, Grief, and Dying

View Set

Post Test: Western and World Art Appreciation

View Set

AP Comparative Final Exam Review

View Set

Milady Esthetics Chapter 10, 11, 12 Exam Review

View Set

MJ Civics End-of-Course Practice Exam

View Set

Excel Chapter 3: End-of-Chapter Quiz

View Set

Algebra I: Modeling With Quadratic Functions

View Set